Elektrodynamik Schöll: Unterschied zwischen den Versionen

Aus PhysikWiki
Zur Navigation springen Zur Suche springen
K Die Seite wurde neu angelegt: „<font size = "6"><u>'''Theorie III - Elektrodynamik'''</u></font> <font size = "6"><u>'''Skript zur Vorlesung'''</u></font> <font size = "6"><u>'''von Prof. Dr.…“
 
 
(10 dazwischenliegende Versionen von 2 Benutzern werden nicht angezeigt)
Zeile 1: Zeile 1:
<font size = "6"><u>'''Theorie III - Elektrodynamik'''</u></font>
<noinclude>{{Scripthinweis|Elektrodynamik|0|0}}</noinclude>


<font size = "6"><u>'''Skript zur Vorlesung'''</u></font>
=== Elektrodynamik ===
 
<font size = "6"><u>'''von Prof. Dr. Schoell'''</u></font>
 
<font size = "6"><u>'''erweitert um eine kurze Abhandlung zur Holografie'''</u></font>
 
<font size = "6"><u>'''Verfasser:'''</u></font>
 
<font size = "6"><u>'''Franz- Josef Schmitt'''</u></font>
 
----
 
<u>'''Elektrodynamik'''</u>


Klassische elektrische und magnetische Erscheinungen
Klassische elektrische und magnetische Erscheinungen


* Elektrodynamik ist relativistisch invariant
* Elektrodynamik ist relativistisch invariant
* Feldtheorie ( Nahwirkungstheorie, Kontinuumstheorie, endliche Ausbreitungsgeschwindigkeit von Wirkungen)
* Feldtheorie (Nahwirkungstheorie, Kontinuumstheorie, endliche Ausbreitungsgeschwindigkeit von Wirkungen)
* lokale Theorie:
* lokale Theorie:
* <math>\bar{E}(\bar{r},t),\bar{B}(\bar{r},t)</math>
* <math>\bar{E}(\bar{r}, t),\bar{B}(\bar{r}, t)</math>
*
* quantentheoretische Erweiterung: Quantenelektrodynamik (nicht behandelt)
* quantentheoretische Erweiterung: Quantenelektrodynamik ( nicht behandelt)
 
Vereinheitlichung der elektromagnetischen und schwachen Wechselwirkung zur elektroschwachen WW in den 70- er Jahren ( Weinberg)


Starke WW: Quantenchromodynamik ( nach dem Vorbild der Quantenelektrodynamik)
Vereinheitlichung der elektromagnetischen und schwachen Wechselwirkung zur elektroschwachen WW in den 70- er Jahren (Weinberg)


GUT ( Grand unified): Vereinheitlichung der Elektroschwachen Theorie mit der starken Kernkraft + Gravitationswechselwirkung ( nichtlinear, allgemein- relativistisch).
Starke WW: Quantenchromodynamik (nach dem Vorbild der Quantenelektrodynamik)


grundlegende Theorie: elektrische und magnetische Felder im Vakuum, erzeugt durch lokalisierte Ladungs- und Stromverteilungen
GUT (Grand unified): Vereinheitlichung der Elektroschwachen Theorie mit der starken Kernkraft + Gravitationswechselwirkung (nichtlinear, allgemein- relativistisch).


'''elektromagnetische Felder in Materie:'''
* grundlegende Theorie:
** elektrische und magnetische Felder im Vakuum, erzeugt durch lokalisierte Ladungs- und Stromverteilungen
* elektromagnetische Felder in Materie
** freie und gebundene Ladungen in Festkörpern/ Gasen, Materie im Allgemeinen
** Zusammenfassung des Beitrags der mikroskopisch gebundenen Ladungen in phänomenologischen Materialkonstanten: Dielektrizitätskonstante, Permeabilität
** phänomenologische Theorie elektromagnetische Felder in Materie


freie und gebundene Ladungen in Festkörpern/ Gasen, Materie im Allgemeinen
(Theorie der Materialkonstanten → Quantentheorie der Festkörper, Flüssigkeiten, Gase)


* Zusammenfassung des Beitrags der mikroskopisch gebundenen Ladungen in phänomenologischen Materialkonstanten: Dielektrizitätskonstante, Permeabilität
* phänomenologische Theorie elektromagnetische Felder in Materie


( Theorie der Materialkonstanten -> Quantentheorie der Festkörper, Flüssigkeiten, Gase )
== Stoff der Vorlesung ==


<u>'''Stoff der Vorlesung'''</u>
* Elektrodynamik im Vakuum


Elektrodynamik im Vakuum
* Elektrodynamik in materie


Elektrodynamik in materie
* Relativistische Formulierung


Relativistische Formulierung


<u>'''Literatur'''</u>
=== Literatur ===


* H. Mitter: Elektrodynamik,. besonders gute relativistisch Formulierung
* H. Mitter: Elektrodynamik,. besonders gute relativistisch Formulierung
* Stumpf, H.: Elektrodynamik Vieweg 1973
* Stumpf, H.: Elektrodynamik Vieweg 1973
* J.D. Jackson
* J. D. Jackson
* R. Becker, Sauter: Theorie der Elektrizität
* R. Becker, Sauter: Theorie der Elektrizität
* Landau- Lifschitz Band II und VIII
* Landau- Lifschitz Band II und VIII


<u>'''1. Elektrostatik'''</u>
[[Kategorie:Vorlesungsstartseite]]
 
*# <u>'''Coulomb- Wechselwirkung'''</u>
 
<u>'''Experimentelle Grundtatsachen'''</u>
 
* Materie trägt als skalare Eigenschaften Masse und elektrische Ladung
 
'''Masse:'''
 
* Gravitations- Wechselwirkung ( Newton: 1643 - 1727 )
 
Kraft auf Masse
 
<math>{{m}_{2}}</math>
 
bei
 
<math>{{\bar{r}}_{2}}</math>
 
, ausgeübt von Masse
 
<math>{{m}_{1}}</math>
 
bei
 
<math>{{\bar{r}}_{1}}</math>
 
:
 
<math>\begin{align}
 
& {{{\bar{F}}}_{g}}^{(2)}=-\gamma \frac{{{m}_{1}}{{m}_{2}}}{{{\left| {{{\bar{r}}}_{1}}-{{{\bar{r}}}_{2}} \right|}^{2}}}{{{\bar{e}}}_{12}} \\
& {{{\bar{e}}}_{12}}:=\frac{{{{\bar{r}}}_{2}}-{{{\bar{r}}}_{1}}}{\left| {{{\bar{r}}}_{1}}-{{{\bar{r}}}_{2}} \right|} \\
\end{align}</math>
 
Wegen:
<math>\gamma ,{{m}_{1}},{{m}_{2}}>0</math>
wird dem Phänomen Rechnung getragen, dass Gravitation stets anziehend wirkt.
Festlegung von
<math>\gamma </math>
durch Wahl einer willkürlichen Einheit kg für Masse:
 
<math>\gamma =6,67\cdot {{10}^{-11}}\frac{N{{m}^{2}}}{k{{g}^{2}}}</math>
 
schwere Masse = träge Masse:
 
<math>\Rightarrow 1N=1\frac{kg\cdot m}{{{s}^{2}}}</math>
 
'''Coulomb- Wechselwirkung ( C. Coulomb 1736-1806)'''
 
Kraft auf Ladung
<math>{{q}_{2}}</math>
bei
<math>{{\bar{r}}_{2}}</math>
, ausgeübt von Masse
<math>{{q}_{1}}</math>
bei
<math>{{\bar{r}}_{1}}</math>
:
 
<math>\begin{align}
& {{{\bar{F}}}_{e}}^{(2)}=k\frac{{{q}_{1}}{{q}_{2}}}{{{\left| {{{\bar{r}}}_{1}}-{{{\bar{r}}}_{2}} \right|}^{2}}}{{{\bar{e}}}_{12}} \\
& {{{\bar{e}}}_{12}}:=\frac{{{{\bar{r}}}_{2}}-{{{\bar{r}}}_{1}}}{\left| {{{\bar{r}}}_{1}}-{{{\bar{r}}}_{2}} \right|} \\
\end{align}</math>
 
<math>\begin{align}
& \gamma >0 \\
& {{q}_{1}},{{q}_{2}}_{>}^{<}0 \\
\end{align}</math>
 
<math>{{q}_{1}}{{q}_{2}}>0</math>
-> Abstoßung
 
<math>{{q}_{1}}{{q}_{2}}<0</math>
-> Anziehung
 
Festlegung von k durch Wahl einer willkürlichen Einheit Coulomb [C] für die elektrische Ladung:
 
<math>k=8,988\cdot {{10}^{9}}\frac{N{{m}^{2}}}{{{C}^{2}}}</math>
 
<math>\Rightarrow </math>
Einheit des elektrischen Stromes: 1 Ampere
<math>\left[ A \right]=1\frac{C}{s}</math>
 
'''Bemerkungen'''
 
* je nach Wahl von k ergeben sich verschiedene Einheitssysteme ( Maßsysteme):
 
# '''SI'''
 
System International d´ Unites  , seit 1.1.1978 verbindlich
m, kg, s, A -> MKSA
K
mol
cd ( Candela) -> Lichtstärke
 
historisch bedingte Schreibweise:
<math>k=\frac{1}{4\pi {{\varepsilon }_{0}}}</math>
 
mit der absoluten dielektrischen Konstanten
<math>{{\varepsilon }_{0}}=8,854\cdot {{10}^{-12}}\frac{{{C}^{2}}{{s}^{2}}}{kg{{m}^{3}}}</math>
 
# '''Gauß:  k=1  ( Miller) CGS- System'''
 
<math>{{F}_{e}}=\frac{{{q}_{1}}{{q}_{2}}}{{{r}^{2}}}</math>
 
Elektrostatische Ladungseinheit:
<math>\begin{align}
& 1ESE=1\sqrt{dyn}\cdot cm \\
& 1C=3\cdot {{10}^{9}}ESE \\
\end{align}</math>
 
# Ladungen e1 = e2 = 1 ESE  im Abstand r = 1cm üben die Kraft
# <math>1dyn=1\frac{g\cdot cm}{{{s}^{2}}}</math>
# aufeinander aus
* Sehr zweckmäßig bei mikroskopischen Rechnungen, da Coulombgesetz einfacher
* unzweckmäßig in der phänomenologischen Elektrodynamik, da Ladungseinheit
* <math>1ESE=1\sqrt{dyn}\cdot cm</math>
*
 
'''Gute Umrechungstabellen: Vergl. Jackson'''
 
'''Weitere Bemerkungen'''
 
# Das Coulombgesetz gilt bis zu Abständen
# <math>r>{{10}^{-11}}cm</math>
#
Bei kleineren Abständen sind quantenelektrodynamische Korrekturen nötig
 
# Die gesamte Ladung eines abgeschlossenen Systems ist konstant.  Aber: Paarerzeugung von positiver und negativer Ladung und lokale Ladungstrennung ist möglich.
# Ladung tritt quantisiert auf:
Elementarladung:
<math>e=1,6\cdot {{10}^{-19}}C</math>
 
Schwere Elementarteilchen ( Hadronen)sind aus Quarks mit Ladungen
<math>-\frac{1}{3}e</math>
oder
<math>+\frac{2}{3}e</math>
zusammengesetzt , aber Quarks wurden bisher nicht als freie Teilchen beobachtet
 
# Die Ausdehnung der geladenen Elementarteilchen ist
# <math><{{10}^{-13}}cm</math>
# . Also erfolgt die makroskopische Beschreibung mit dem Punktladungsmodell.
 
<u>'''1.2 Elektrisches Feld und Potenziale'''</u>
 
Lineare Superposition ( 4. Newtonsches Prinzip) der Kräfte der Ladungen
<math>{{q}_{i}}</math>
bei
<math>{{\bar{r}}_{i}}</math>
,i=1,2,... auf die Ladung
<math>{{q}_{{}}}</math>
bei
<math>{{\bar{r}}_{{}}}</math>
:
 
<math>{{\bar{F}}_{e}}^{(2)}=\frac{1}{4\pi {{\varepsilon }_{0}}}\sum\limits_{i}{{}}\frac{q{{q}_{i}}}{{{\left| \bar{r}-{{{\bar{r}}}_{i}} \right|}^{2}}}\frac{\bar{r}-{{{\bar{r}}}_{i}}}{\left| \bar{r}-{{{\bar{r}}}_{i}} \right|}</math>
 
Darüber wird das elektrische Feld definiert:
 
<math>q\cdot \bar{E}\equiv {{\bar{F}}_{e}}^{(2)}=\frac{1}{4\pi {{\varepsilon }_{0}}}\sum\limits_{i}{{}}\frac{q{{q}_{i}}}{{{\left| \bar{r}-{{{\bar{r}}}_{i}} \right|}^{2}}}\frac{\bar{r}-{{{\bar{r}}}_{i}}}{\left| \bar{r}-{{{\bar{r}}}_{i}} \right|}</math>
 
Also:
 
<math>\bar{E}=\frac{1}{4\pi {{\varepsilon }_{0}}}\sum\limits_{i}{{}}\frac{{{q}_{i}}}{{{\left| \bar{r}-{{{\bar{r}}}_{i}} \right|}^{2}}}\frac{\bar{r}-{{{\bar{r}}}_{i}}}{\left| \bar{r}-{{{\bar{r}}}_{i}} \right|}</math>
 
Warum ist die Elektrodynamik eine Feldtheorie ?
 
* Die endliche Ausbreitungsgeschwindigkeit von physikalischen Wechselwirkungen ( maximal mit c) ist universell. Das Feld als Medium für die Übertragung physikalischer Wechselwirkungen ersetzt ein Modell des Austauschs im Sinne einer Nahwirkung statt einem Austauschmodell.
* Das Feld
* <math>\bar{E}(\bar{r})</math>
* ist der PHYSIKALISCHE Zustand des leeren Raumes bei
* <math>\bar{r}</math>
* .
* Eigenständige FELDDYNAMIK ( partielle Diffgl.) zur Beschreibung der endlich schnellen Ausbreitung ( Retardierungseffekte)
* Feld muss IMPULS, DREHIMPULS und ENERGIE aufnehmen und abgeben können.
 
Einheit:
 
<math>\begin{align}
& \left[ E \right]=\frac{N}{C}=\frac{kgm}{C{{s}^{2}}}=\frac{V}{m} \\
& 1V:=1\frac{kg{{m}^{2}}}{C{{s}^{2}}} \\
\end{align}</math>
 
Das Volt ist benannt nach A. Volta ( 1745 - 1887)
 
Die Messung des elektrischen Feldes erfolgt durch Einbringung einer Probeladung:
Dabei sollte q-> 0, damit keine Rückwirkung auf
<math>{{q}_{i}}</math>
erfolgt.
 
Unter Berücksichtigung des Selbstkonsistenzproblems müsste man also schreiben:
 
<math>\left[ \bar{E}(\bar{r}) \right]=\begin{matrix}
\lim  \\
q\to 0  \\
\end{matrix}\frac{1}{q}\bar{F}(\bar{r})</math>
 
<u>'''Das Elektrostatische Potenzial'''</u>
Mit
<math>\begin{align}
& \nabla \frac{1}{r\acute{\ }}=-\frac{1}{r{{\acute{\ }}^{3}}}\bar{r}\acute{\ } \\
& r\acute{\ }:=\left| \bar{r}-{{{\bar{r}}}_{i}} \right| \\
\end{align}</math>
 
Läßt sich schreiben:
 
<math>\begin{align}
& \bar{E}(\bar{r})=\frac{1}{4\pi {{\varepsilon }_{0}}}\sum\limits_{i}^{{}}{{}}\frac{{{q}_{i}}}{|\bar{r}-{{{\bar{r}}}_{i}}{{|}^{3}}}\left( \bar{r}-{{{\bar{r}}}_{i}} \right)=-\nabla \Phi (\bar{r}) \\
& \Phi (\bar{r}):=\frac{1}{4\pi {{\varepsilon }_{0}}}\sum\limits_{i}^{{}}{{}}\frac{{{q}_{i}}}{|\bar{r}-{{{\bar{r}}}_{i}}|} \\
\end{align}</math>
 
Mit dem elektrostatischen Potenzial
<math>\Phi (\bar{r}):=\frac{1}{4\pi {{\varepsilon }_{0}}}\sum\limits_{i}^{{}}{{}}\frac{{{q}_{i}}}{|\bar{r}-{{{\bar{r}}}_{i}}|}</math>
, Einheit : 1 V
 
<u>'''Kontinuierliche Ladungsverteilung'''</u>
 
<math>\begin{align}
& {{q}_{i}}\to {{d}^{3}}r\acute{\ }\rho (\bar{r}\acute{\ }) \\
& \sum\limits_{i}^{{}}{{}}{{q}_{i}}\to \int_{{}}^{{}}{{{d}^{3}}r\acute{\ }\rho }(\bar{r}\acute{\ }) \\
\end{align}</math>
 
Mit der Ladungsdichte
<math>\rho (\bar{r}\acute{\ })</math>
. Diese muss beschränkt sein und
<math>O\left( {{r}^{-3-\varepsilon }} \right),\varepsilon >0</math>
für
<math>r\to \infty </math>
.
 
Es wird
 
 
Bei Verteilung von Punktladungen:
 
<math>\rho (\bar{r}\acute{\ })=\sum\limits_{i}^{{}}{{}}{{q}_{i}}\delta (\bar{r}\acute{\ }-{{\bar{r}}_{i}})=\sum\limits_{i}^{{}}{{}}{{q}_{i}}\prod\limits_{j=1}^{3}{{}}\delta ({{x}_{j}}\acute{\ }-{{x}_{j}}_{i})</math>
 
'''Quellen des elektrischen Feldes:'''
 
Bei Punktladung q bei
<math>\bar{r}\acute{\ }=0\Rightarrow \bar{E}\left( {\bar{r}} \right)=\frac{1}{4\pi {{\varepsilon }_{0}}}\frac{q}{{{r}^{2}}}\cdot \frac{{\bar{r}}}{r}</math>
 
Legt man eine geschlossene Oberfläche S um q, so beobachtet man einen elektrischen Kraftfluss:
 
 
<math>{{\Phi }_{e}}=\oint_{S}{d\bar{f}\cdot }\bar{E}\left( {\bar{r}} \right)=\frac{q}{4\pi {{\varepsilon }_{0}}}\cdot \oint_{S}{{}}\frac{d\bar{f}\cdot \bar{r}}{{{r}^{3}}}=\oint_{S}{df}{{E}_{n}}(\bar{r})</math>
als geschl. Flächenintegral über die Normalkomponenten des austretenden elektrischen Feldes
 
<math>{{\Phi }_{e}}=\oint_{S}{df}\left| \bar{E}(\bar{r}) \right|\cos \Theta </math>
 
 
<math>d\bar{f}</math>
entspricht einem Raumwinkel
<math>d\Omega :d\bar{f}\cdot \bar{r}=df\cdot r\cdot \cos \Theta ={{r}^{3}}d\Omega </math>
 
<math>{{\Phi }_{e}}=\oint_{S}{d\bar{f}\cdot }\bar{E}\left( {\bar{r}} \right)=\frac{q}{4\pi {{\varepsilon }_{0}}}\cdot \oint_{S}{{}}d\Omega =\frac{q}{{{\varepsilon }_{0}}}</math>
 
<math>\Rightarrow {{\varepsilon }_{0}}\oint_{S}{d\bar{f}\cdot }\bar{E}\left( {\bar{r}} \right)=q</math>
 
Dies kann leicht auf kontinuierliche Ladungsverteilungen verallgemeinert werden:
 
<math>\Rightarrow {{\varepsilon }_{0}}\oint_{\partial V}{d\bar{f}\cdot }\bar{E}\left( {\bar{r}} \right)=\int_{V}^{{}}{{{d}^{3}}r\acute{\ }\rho \left( \bar{r}\acute{\ } \right)}</math>
 
Der Fluß des elektrischen Feldes einer von
<math>S=\partial V</math>
eingeschlossenen Gesamtladung
 
'''Integralform '''des Coulomb- Gesetzes
 
<u>'''Der Gaußsche Integralsatz'''</u>
 
<math>\oint_{\partial V}{d\bar{f}\cdot }\bar{E}\left( {\bar{r}} \right)=\int_{V}^{{}}{{{d}^{3}}rdiv\bar{E}\left( {\bar{r}} \right)}=\int_{V}^{{}}{{{d}^{3}}r\nabla \cdot \bar{E}\left( {\bar{r}} \right)}</math>
 
wichtig: einfach zusammenhängendes Gebiet !
 
<math>\begin{align}
& \Rightarrow \int_{V}^{{}}{{{d}^{3}}r\rho \left( {\bar{r}} \right)}={{\varepsilon }_{0}}\int_{V}^{{}}{{{d}^{3}}r\nabla \cdot }\bar{E}\left( {\bar{r}} \right) \\
& \Rightarrow {{\varepsilon }_{0}}\nabla \cdot \bar{E}\left( {\bar{r}} \right)=\rho \left( {\bar{r}} \right) \\
\end{align}</math>
 
Die untere, differenzielle Form gilt deshalb, da die obere, integral Form für beliebige Volumina V gilt.
 
<math>{{\varepsilon }_{0}}\nabla \cdot \bar{E}\left( {\bar{r}} \right)=\rho \left( {\bar{r}} \right)</math>
 
sagt jedoch nichts anderes als dass die Ladungen die Quellen des elektrischen Feldes sind.
Dies ist allgemeingültig uns gilt insbesondere auch für nichtstationäre
<math>\bar{E}\left( {\bar{r}} \right),\rho \left( {\bar{r}} \right)</math>
 
<u>'''Äquivalente Aussagen der Elektrostatik'''</u>
#
# <math>\bar{E}\left( {\bar{r}} \right)</math>
#  besitzt ein skalares Potenzial
# <math>\bar{E}\left( {\bar{r}} \right)=-\nabla \Phi (\bar{r})</math>
#
#
# <math>\int_{1}^{2}{d\bar{s}}\bar{E}\left( {\bar{r}} \right)</math>
# , also gerade die Arbeit, eine Ladung q=1 von 1 nach 2 zu bringen  ist wegunabhängig
#
# <math>\nabla \times \bar{E}\left( {\bar{r}} \right)=0</math>
#  : Das statische elektrische Feld ist wirbelfrei
 
Es gilt:
 
<math>1)\Leftrightarrow 2)\Leftrightarrow 3)</math>
 
'''Beweis:'''
<math>1)\Leftrightarrow 3)</math>
<u>'''Stokescher Satz:'''</u>
 
<math>0=\oint_{\partial F}{d\bar{s}\cdot }\bar{E}\left( {\bar{r}} \right)=\int_{F}^{{}}{\nabla \times \bar{E}\left( {\bar{r}} \right)d\bar{f}}</math>
für beliebige Flächen F mit einer Umrandung
<math>\partial F</math>
.
 
<u>'''1.3 Poisson- Gleichung und Greensche Funktion'''</u>
 
<math>\bar{E}\left( {\bar{r}} \right)=-\nabla \Phi (\bar{r})</math>
in
<math>\nabla \cdot \bar{E}\left( {\bar{r}} \right)=\frac{\rho \left( {\bar{r}} \right)}{{{\varepsilon }_{0}}}</math>
liefert:
 
<math>\Delta \Phi (\bar{r})=-\frac{\rho \left( {\bar{r}} \right)}{{{\varepsilon }_{0}}}</math>
 
Dies ist nicht anderes als die berühmte Poisson- Gleichung
 
Eine partielle DGL zur Berechnung des elektrischen Potenzials für eine vorgegebene Ladungsverteilung.
 
'''Die Eindeutigkeit kommt aus den Randbedingungen:'''
 
'''Entweder:'''
1)
<math>\Phi (\bar{r})\to 0</math>
hinreichend rasch für
<math>r\to \infty </math>
 
oder
2)
<math>\Phi (\bar{r})</math>
sei gegeben auf Flächen im Endlichen, Beispielsweise Leiteroberflächen
 
'''Lösung zu 1):'''
 
<math>\Phi (\bar{r})=\frac{1}{4\pi {{\varepsilon }_{0}}}\int_{{{R}^{3}}}^{{}}{{{d}^{3}}r\acute{\ }}\frac{\rho \left( \bar{r}\acute{\ } \right)}{|\bar{r}-\bar{r}\acute{\ }|}</math>
für hinreichend rasch abfallendes
<math>\rho \left( \bar{r}\acute{\ } \right)</math>
 
'''Einsetzen in Poisson- Gleichung:'''
 
<math>\Delta \Phi (\bar{r})=\frac{1}{4\pi {{\varepsilon }_{0}}}{{\Delta }_{r}}\int_{{{R}^{3}}}^{{}}{{{d}^{3}}r\acute{\ }}\frac{\rho \left( \bar{r}\acute{\ } \right)}{|\bar{r}-\bar{r}\acute{\ }|}=\frac{1}{4\pi {{\varepsilon }_{0}}}\int_{{{R}^{3}}}^{{}}{{{d}^{3}}r\acute{\ }}{{\Delta }_{r}}\frac{\rho \left( \bar{r}\acute{\ } \right)}{|\bar{r}-\bar{r}\acute{\ }|}</math>
, falls Integration und Differenziation vertauschbar, also über verschiedene Koordinaten ausgeführt wird.
 
Man definiere für ein festes
<math>\bar{r}\acute{\ }</math>
, dass
<math>\begin{align}
& \bar{s}:=\bar{r}-\bar{r}\acute{\ } \\
& {{\nabla }_{r}}={{\nabla }_{s}} \\
\end{align}</math>
:
Also:
 
<math>\begin{align}
& {{\Delta }_{r}}\frac{1}{|\bar{r}-\bar{r}\acute{\ }|}={{\nabla }_{S}}\left( {{\nabla }_{S}}\frac{1}{s} \right)=-{{\nabla }_{S}}\frac{1}{{{s}^{2}}}\frac{{\bar{s}}}{s}=-\frac{1}{{{s}^{3}}}{{\nabla }_{S}}\bar{s}-\bar{s}{{\nabla }_{S}}\frac{1}{{{s}^{3}}} \\
& {{\nabla }_{S}}\bar{s}=3 \\
& \Rightarrow {{\Delta }_{r}}\frac{1}{|\bar{r}-\bar{r}\acute{\ }|}=-\frac{1}{{{s}^{3}}}{{\nabla }_{S}}\bar{s}-\bar{s}{{\nabla }_{S}}\frac{1}{{{s}^{3}}}=-\frac{3}{{{s}^{3}}}+\frac{1}{{{s}^{3}}}=0 \\
\end{align}</math>
 
Dies ist aber ein Widerspruch zu
<math>\Delta \Phi (\bar{r})=-\frac{\rho \left( {\bar{r}} \right)}{{{\varepsilon }_{0}}}</math>
 
Grund ist , dass die Vertauschung von
<math>{{\Delta }_{r}}</math>
und
<math>\int_{{{R}^{3}}}^{{}}{{{d}^{3}}r\acute{\ }}</math>
sowie auch die obige Umformung nicht erlaubt ist für
<math>\bar{r}=\bar{r}\acute{\ }</math>
, also s=0  ( Singularität!!)
 
Stattdessen für beliebige V:
 
 
Nun kann man
<math>\oint\limits_{\partial V}{d\bar{f}\cdot {{\nabla }_{r}}}</math>
mit
<math>\int_{{{R}^{3}}}^{{}}{{{d}^{3}}r\acute{\ }}</math>
vertauschen.
Dies ist erlaubt, falls der Integrand von
<math>\int_{{{R}^{3}}}^{{}}{{{d}^{3}}r\acute{\ }}</math>
nach der Vertauschung stetig ist !:
 
<math>\begin{align}
& \int_{V}^{{}}{{{d}^{3}}}r\Delta \Phi (\bar{r})=\frac{1}{4\pi {{\varepsilon }_{0}}}\int_{{{R}^{3}}}^{{}}{{{d}^{3}}r\acute{\ }}\rho \left( \bar{r}\acute{\ } \right)\oint\limits_{\partial V}{d\bar{f}\cdot {{\nabla }_{r}}}\frac{1}{|\bar{r}-\bar{r}\acute{\ }|} \\
& {{\nabla }_{r}}\frac{1}{|\bar{r}-\bar{r}\acute{\ }|}=-\frac{\left( \bar{r}-\bar{r}\acute{\ } \right)}{|\bar{r}-\bar{r}\acute{\ }{{|}^{3}}} \\
\end{align}</math>
 
Somit:
 
<math>\begin{align}
& \int_{V}^{{}}{{{d}^{3}}}r\Delta \Phi (\bar{r})=\frac{1}{4\pi {{\varepsilon }_{0}}}\int_{{{R}^{3}}}^{{}}{{{d}^{3}}r\acute{\ }}\rho \left( \bar{r}\acute{\ } \right)\oint\limits_{\partial V}{d\bar{f}\cdot {{\nabla }_{r}}}\frac{1}{|\bar{r}-\bar{r}\acute{\ }|}=-\frac{1}{4\pi {{\varepsilon }_{0}}}\int_{{{R}^{3}}}^{{}}{{{d}^{3}}r\acute{\ }}\rho \left( \bar{r}\acute{\ } \right)\oint\limits_{\partial V}{d\bar{f}}\frac{\bar{r}-\bar{r}\acute{\ }}{|\bar{r}-\bar{r}\acute{\ }{{|}^{3}}} \\
& \oint\limits_{\partial V}{d\bar{f}}\frac{\bar{r}-\bar{r}\acute{\ }}{|\bar{r}-\bar{r}\acute{\ }{{|}^{3}}}=\int_{{}}^{{}}{d\Omega } \\
\end{align}</math>
 
aber:
 
<math>\oint\limits_{\partial V}{d\bar{f}}\frac{\bar{r}-\bar{r}\acute{\ }}{|\bar{r}-\bar{r}\acute{\ }{{|}^{3}}}=\int_{{}}^{{}}{d\Omega }=4\pi </math>
, falls
<math>\bar{r}\acute{\ }\in V</math>
 
<math>\oint\limits_{\partial V}{d\bar{f}}\frac{\bar{r}-\bar{r}\acute{\ }}{|\bar{r}-\bar{r}\acute{\ }{{|}^{3}}}=\int_{{}}^{{}}{d\Omega }=0</math>
falls
<math>\bar{r}\acute{\ }\notin V</math>
 
Somit:
 
<math>\int_{V}^{{}}{{{d}^{3}}}r\Delta \Phi (\bar{r})=\frac{1}{4\pi {{\varepsilon }_{0}}}\int_{{{R}^{3}}}^{{}}{{{d}^{3}}r\acute{\ }}\rho \left( \bar{r}\acute{\ } \right)\oint\limits_{\partial V}{d\bar{f}\cdot {{\nabla }_{r}}}\frac{1}{|\bar{r}-\bar{r}\acute{\ }|}=-\frac{1}{{{\varepsilon }_{0}}}\int_{V}^{{}}{{{d}^{3}}r\acute{\ }}\rho \left( \bar{r}\acute{\ } \right)</math>
 
Mathematisch streng gilt im Distributionen- Sinn:
 
<math>{{\Delta }_{r}}\frac{1}{|\bar{r}-\bar{r}\acute{\ }|}=-4\pi \delta (\bar{r}-\bar{r}\acute{\ })</math>
 
Mit Hilfe der delta- Distribution, die auf eine Testfunktion anzuwenden ist !
 
<u>'''Greensche Funktion der Poisson- Gleichung'''</u>
 
<math>\Delta \Phi (\bar{r})=-\frac{\rho \left( \bar{r}\acute{\ } \right)}{{{\varepsilon }_{0}}}\Rightarrow </math>
Invertierung
<math>\Rightarrow \Phi (\bar{r})=\hat{G}\rho \left( \bar{r}\acute{\ } \right)</math>
 
Mit dem Greenschen Operator
<math>\hat{G}</math>
:
 
Eine Fourier- Transformation von
<math>\Delta \Phi (\bar{r})=-\frac{\rho \left( \bar{r}\acute{\ } \right)}{{{\varepsilon }_{0}}}\Rightarrow </math>
liefert
<math>-{{k}^{2}}\tilde{\Phi }=-\frac{{\tilde{\rho }}}{{{\varepsilon }_{0}}}\Rightarrow </math>
 
Man kann schreiben:
 
<math>\begin{align}
& \tilde{\Phi }=\tilde{\hat{G}}\tilde{\rho } \\
& \tilde{\hat{G}}:=\frac{1}{{{\varepsilon }_{0}}{{k}^{2}}} \\
\end{align}</math>
 
Die einfache Fourier- Transformierte Form von
<math>\Rightarrow \Phi (\bar{r})=\hat{G}\rho \left( \bar{r}\acute{\ } \right)</math>
, nur dass der Fourier- transformierte Greens- Operator angegeben werden kann.
 
Die Rücktransformation löst dann die Poiisson-gleichung:
 
<math>\Rightarrow \Phi (\bar{r})=\int_{{}}^{{}}{{{d}^{3}}r\acute{\ }}\hat{G}(\bar{r}-\bar{r}\acute{\ })\rho \left( \bar{r}\acute{\ } \right)</math>
 
Es gilt:
 
<math>{{\Delta }_{r}}\hat{G}(\bar{r}-\bar{r}\acute{\ })=-\frac{1}{{{\varepsilon }_{0}}}\delta \left( \bar{r}-\bar{r}\acute{\ } \right)</math>
 
Das heißt, die Greensfunktion ist eine Lösung der Poissongleichung für eine Punktladung q=1 an
<math>\bar{r}\acute{\ }</math>
:
Insbesondere bei speziellen Randbedingungen
 
<math>\begin{matrix}
\lim  \\
\bar{r}\to \infty  \\
\end{matrix}\Phi (\bar{r})=0</math>
 
ist die Greensfunktion dann:
 
<math>G(\bar{r}-\bar{r}\acute{\ })=\frac{1}{4\pi {{\varepsilon }_{0}}}\frac{1}{|\bar{r}-\bar{r}\acute{\ }|}</math>
 
Denn
 
<math>{{\Delta }_{r}}G=\Delta \frac{1}{4\pi {{\varepsilon }_{0}}}\frac{1}{|\bar{r}-\bar{r}\acute{\ }|}=-\frac{1}{{{\varepsilon }_{0}}}\delta \left( \bar{r}-\bar{r}\acute{\ } \right)</math>
 
Für eine beliebige Ladungsverteilung
<math>\rho </math>
ist also die Lösung der Poissongleichung
 
<math>\Phi (\bar{r})=\int_{-\infty }^{\infty }{{}}\frac{1}{4\pi {{\varepsilon }_{0}}}\frac{\rho (\bar{r}\acute{\ })}{|\bar{r}-\bar{r}\acute{\ }|}{{d}^{3}}r\acute{\ }=\int_{-\infty }^{\infty }{{}}G(\bar{r}-\bar{r}\acute{\ })\rho (\bar{r}\acute{\ }){{d}^{3}}r\acute{\ }</math>
 
wobei die Identität insgesamt nur für die Randbedingungen
<math>\begin{matrix}
\lim  \\
\bar{r}\to \infty  \\
\end{matrix}\Phi (\bar{r})=0</math>
gilt, ansonsten ist G durch die andern Randbdingungen festgelegt.
 
<u>'''1.4 Elektrische Multipolentwicklung'''</u>
 
Betrachtet man räumlich begrenzte Ladungsverteilungen
<math>\rho (\bar{r}\acute{\ })</math>
in der Nähe des Ursprungs
<math>\bar{r}\acute{\ }=0</math>
, so kann man sich Gedanken machen um das asymptotische Verhalten  von
<math>\Phi (\bar{r})=\int_{-\infty }^{\infty }{{}}\frac{1}{4\pi {{\varepsilon }_{0}}}\frac{\rho (\bar{r}\acute{\ })}{|\bar{r}-\bar{r}\acute{\ }|}{{d}^{3}}r\acute{\ }</math>
für
<math>r\to \infty </math>
:
Methode: Der Integrand wird als Taylorreihe entwickelt für
<math>r>>r\acute{\ }</math>
:
 
<math>G(\bar{r}-\bar{r}\acute{\ })=\sum\limits_{l=0}^{\infty }{{}}\frac{{{\left( -1 \right)}^{l}}}{l!}{{\left( \bar{r}\acute{\ }\cdot {{\nabla }_{r}} \right)}^{l}}G(\bar{r})</math>
 
Also
 
<math>\Phi (\bar{r})=\int_{-\infty }^{\infty }{{}}G(\bar{r}-\bar{r}\acute{\ })\rho (\bar{r}\acute{\ }){{d}^{3}}r\acute{\ }=\sum\limits_{l=0}^{\infty }{{}}\frac{{{\left( -1 \right)}^{l}}}{l!}\int_{{}}^{{}}{d_{^{{}}}^{3}r\acute{\ }}{{\left( \bar{r}\acute{\ }\cdot {{\nabla }_{r}} \right)}^{l}}G(\bar{r})\rho (\bar{r}\acute{\ })</math>
 
explizit für unsere Situation:
 
<math>G(\bar{r})=\frac{1}{4\pi {{\varepsilon }_{0}}}\frac{1}{|\bar{r}|}</math>
 
<math>\frac{1}{|\bar{r}-\bar{r}\acute{\ }|}={{\left( {{r}^{2}}-2rr\acute{\ }\cos \vartheta +r{{\acute{\ }}^{2}} \right)}^{-\frac{1}{2}}}=\frac{1}{r}{{\left( 1-2\frac{r\acute{\ }}{r}\cos \vartheta +{{\left( \frac{r\acute{\ }}{r} \right)}^{2}} \right)}^{-\frac{1}{2}}}</math>
 
Wobei
<math>\vartheta </math>
den Winkel zwischen
<math>\bar{r}</math>
und
<math>\bar{r}\acute{\ }</math>
bezeichnet.
Betrachtet man die hierbei entstehende Reihe, die für
<math>r\acute{\ }<r</math>
und
<math>\left| \cos \vartheta  \right|=\left| \xi  \right|<1</math>
konvergiert, so definiert diese Reihe gerade die sogenannten Kugelfunktionen ( Legendre- Polynome):
<math>{{P}_{l}}(\xi )</math>
:
 
<math>{{\left( 1-2\frac{r\acute{\ }}{r}\xi +{{\left( \frac{r\acute{\ }}{r} \right)}^{2}} \right)}^{-\frac{1}{2}}}=\sum\limits_{l=0}^{\infty }{{}}{{\left( \frac{r\acute{\ }}{r} \right)}^{l}}{{P}_{l}}(\xi )</math>
 
Also sind die Legendre- Polynome gerade definiert über ihre Eigenschaft, als Entwicklungsfunktionen einer Potenzreihe ( Taylorreihe) multipliziert mit
<math>{{\left( \frac{r\acute{\ }}{r} \right)}^{l}}</math>
in jeweils l-ter Ordnung die Funktion
<math>{{\left( 1-2\frac{r\acute{\ }}{r}\xi +{{\left( \frac{r\acute{\ }}{r} \right)}^{2}} \right)}^{-\frac{1}{2}}}</math>
zu ergeben, die wiederum das r- Fache von
<math>\frac{1}{|\bar{r}-\bar{r}\acute{\ }|}=\frac{1}{r}{{\left( 1-2\frac{r\acute{\ }}{r}\cos \vartheta +{{\left( \frac{r\acute{\ }}{r} \right)}^{2}} \right)}^{-\frac{1}{2}}}</math>
ist.
Also:
 
<math>{{P}_{l}}(\xi )=\frac{1}{l!}\left( \frac{{{\partial }^{l}}}{\partial {{t}^{l}}}{{\left( 1-2t\xi +{{t}^{2}} \right)}^{-\frac{1}{2}}} \right)</math>
 
Insbesondere folgt damit:
 
<math>{{P}_{l}}(\xi )=\frac{1}{l!}\left( \frac{{{\partial }^{l}}}{\partial {{t}^{l}}}{{\left( 1-2t\xi +{{t}^{2}} \right)}^{-\frac{1}{2}}} \right)</math>
 
und speziell:
 
<math>\begin{align}
& {{P}_{0}}(\xi )=1 \\
& {{P}_{1}}(\xi )=\xi =\cos \vartheta  \\
& {{P}_{2}}(\xi )=\frac{1}{2}\left( 3{{\xi }^{2}}-1 \right)==\frac{1}{4}\left( 3{{\cos }^{2}}\vartheta +1 \right) \\
\end{align}</math>
 
Also:
 
<math>\Phi (\bar{r})=\frac{1}{4\pi {{\varepsilon }_{0}}}\frac{1}{r}\int_{-\infty }^{\infty }{{}}{{d}^{3}}r\acute{\ }\rho (\bar{r}\acute{\ })\sum\limits_{l=0}^{\infty }{{}}{{\left( \frac{r\acute{\ }}{r} \right)}^{l}}{{P}_{l}}(\cos \vartheta )=\frac{1}{4\pi {{\varepsilon }_{0}}}\sum\limits_{l=0}^{\infty }{{}}{{Q}_{l}}{{r}^{-l-1}}</math>
 
Mit
 
<math>{{Q}_{l}}=\int_{-\infty }^{\infty }{{}}{{d}^{3}}r\acute{\ }r{{\acute{\ }}^{l}}\rho (\bar{r}\acute{\ }){{P}_{l}}(\cos \vartheta )</math>
als 2<sup>l</sup>- Pol
Die Multipolentwicklung ( Entwicklung nach 2<sup>l</sup>- Polen) entspricht also einer Entwicklung ach Potenzen von r !!
 
Für stark lokalisierte Ladungsverteilungen ( r´<<r) konvergiert die Reihe jedoch sehr schnell !
Man erhält sehr schnelle Konvergenz für dezentrale Ladungsverteilungen, wenn man für
* Punktladungen bis zum Monopol entwickelt
* Ladungsverteilungen entlang einer Geraden bis zum Dipol entwickelt
* Ladungsverteilungen in einem Rechteck bis zum Quadrupol entwickelt usw...
 
<math>l=0</math>
:
 
<math>{{\Phi }^{(0)}}(\bar{r})=\frac{1}{4\pi {{\varepsilon }_{0}}}\frac{{{Q}_{0}}}{r}</math>
 
<math>{{Q}_{0}}=\int_{-\infty }^{\infty }{{}}{{d}^{3}}r\acute{\ }\rho (\bar{r}\acute{\ })</math>
sogenannter Monopol ( die Gesamtladung).
Der Monopol entspricht dem Feld einer Gesamtladung, die im Ursprung zentriert ist. Er fällt am langsamsten ab, die Ladungsverteilung wirkt in großer Entfernung wie eine Punktladung
 
'''l=1:'''
 
<math>{{\Phi }^{(1)}}(\bar{r})=\frac{1}{4\pi {{\varepsilon }_{0}}}\frac{\bar{p}\cdot \bar{r}}{{{r}^{3}}}</math>
 
<math>{{Q}_{1}}=\int_{-\infty }^{\infty }{{}}{{d}^{3}}r\acute{\ }\rho (\bar{r}\acute{\ })r\acute{\ }\cos \vartheta =\frac{\bar{p}\cdot \bar{r}}{r}</math>
 
Mit dem Dipolmoment
 
<math>\bar{p}:=\int_{-\infty }^{\infty }{{}}{{d}^{3}}r\acute{\ }\rho (\bar{r}\acute{\ })\bar{r}\acute{\ }</math>
 
Das Dipolpotenzial fällt also
<math>\tilde{\ }\frac{1}{{{r}^{2}}}</math>
ab.
Das Dipolmoment ist der wichtigste Term für insgesamt neutrale Körper (
<math>{{Q}_{0}}=0</math>
).
 
<u>'''Beispiel: 2 Punktladungen q, -q '''</u> bei
<math>{{\bar{r}}_{1}},{{\bar{r}}_{2}}</math>
:
 
<math>\begin{align}
& \rho (\bar{r}\acute{\ })=q\left[ \delta \left( \bar{r}\acute{\ }-{{{\bar{r}}}_{1}} \right)-\delta \left( \bar{r}\acute{\ }-{{{\bar{r}}}_{2}} \right) \right] \\
& {{Q}_{0}}=0 \\
& \bar{p}=q\left( {{{\bar{r}}}_{1}}-{{{\bar{r}}}_{2}} \right)=q\cdot \bar{a} \\
\end{align}</math>
 
'''Feld des Dipolpotenzials:'''
 
<math>{{E}_{i}}=-\frac{1}{4\pi {{\varepsilon }_{0}}}\frac{\partial }{\partial {{x}_{i}}}\frac{{{p}_{k}}\cdot {{x}_{k}}}{{{r}^{3}}}=\frac{1}{4\pi {{\varepsilon }_{0}}}\left[ \frac{3{{x}_{i}}\cdot {{p}_{k}}\cdot {{x}_{k}}}{r5}-{{\delta }_{ik}}\frac{{{p}_{k}}}{{{r}^{3}}} \right]</math>
 
<math>\Rightarrow E(\bar{r})=\frac{1}{4\pi {{\varepsilon }_{0}}}\frac{1}{{{r}^{5}}}\left[ 3\left( \bar{p}\cdot \bar{r} \right)\bar{r}-{{r}^{2}}\bar{p} \right]</math>
 
Im Fernfeld für r gegen unendlich gilt damit:
 
<math>\Rightarrow E(\bar{r})\tilde{\ }\frac{1}{{{r}^{3}}}</math>
 
'''l=2:'''
 
<math>{{\Phi }^{(2)}}(\bar{r})=\frac{1}{4\pi {{\varepsilon }_{0}}}\frac{{{Q}_{2}}}{{{r}^{3}}}</math>
 
<math>\begin{align}
& {{Q}_{2}}=\frac{1}{2}\int_{-\infty }^{\infty }{{}}{{d}^{3}}r\acute{\ }\rho (\bar{r}\acute{\ })r{{\acute{\ }}^{2}}\left( 3{{\cos }^{2}}\vartheta -1 \right)=\frac{1}{2}\int_{-\infty }^{\infty }{{}}{{d}^{3}}r\acute{\ }\rho (\bar{r}\acute{\ })\left( 3\frac{\bar{r}\acute{\ }\cdot \bar{r}}{r}\frac{\bar{r}\acute{\ }\cdot \bar{r}}{r}-\bar{r}{{\acute{\ }}^{2}} \right) \\
& \frac{\bar{r}\acute{\ }\cdot \bar{r}}{r}\frac{\bar{r}\acute{\ }\cdot \bar{r}}{r}=\frac{{{x}_{k}}\acute{\ }{{x}_{k}}{{x}_{l}}\acute{\ }{{x}_{l}}}{{{r}^{2}}} \\
& \Rightarrow {{Q}_{2}}=\frac{1}{2{{r}^{2}}}\int_{-\infty }^{\infty }{{}}{{d}^{3}}r\acute{\ }\rho (\bar{r}\acute{\ })\left( 3{{x}_{k}}\acute{\ }{{x}_{l}}\acute{\ }-\bar{r}{{\acute{\ }}^{2}}{{\delta }_{kl}} \right) \\
\end{align}</math>
 
Dieses Objekt ist jedoch ein Tensor zweiter Stufe. Demnach erhalten wir einen Tenor zweiter Stufe als Quadrupolmoment:
 
<math>\begin{align}
& {{Q}_{2}}=\frac{1}{2{{r}^{2}}}{{Q}_{kl}} \\
& \int_{-\infty }^{\infty }{{}}{{d}^{3}}r\acute{\ }\rho (\bar{r}\acute{\ })\left( 3{{x}_{k}}\acute{\ }{{x}_{l}}\acute{\ }-\bar{r}{{\acute{\ }}^{2}}{{\delta }_{kl}} \right)={{Q}_{kl}} \\
\end{align}</math>
 
<math>{{Q}_{kl}}</math>
ist ein spurfreier und symmetrischer Tensor:
 
<math>\sum\limits_{i=1}^{3}{{}}{{Q}_{ii}}=\sum\limits_{i=1}^{3}{{}}\int_{-\infty }^{\infty }{{}}{{d}^{3}}r\acute{\ }\rho (\bar{r}\acute{\ })\left( 3{{x}_{i}}\acute{\ }{{x}_{i}}\acute{\ }-\bar{r}{{\acute{\ }}^{2}}{{\delta }_{ii}} \right)=\int_{-\infty }^{\infty }{{}}{{d}^{3}}r\acute{\ }\rho (\bar{r}\acute{\ })\left( 3\bar{r}{{\acute{\ }}^{2}}-3\bar{r}{{\acute{\ }}^{2}} \right)=0</math>
 
Dies ist jedoch gerade damit gleichbedeutend, dass eine orthogonale Transformation auf Diagonalform existiert:
 
<math>\begin{align}
& {{Q}_{kl}}=0f\ddot{u}r\quad k\ne l \\
& {{Q}_{11}}+{{Q}_{22}}+{{Q}_{33}}=0 \\
\end{align}</math>
 
Somit existieren nur noch 2 unabhängige Komponenten des Quadrupolmoments. Der Rest ergibt sich durch die Spurfreiheit !
 
Für das Potenzial ergibt sich:
 
<math>{{\Phi }^{(2)}}(\bar{r})=\frac{1}{4\pi {{\varepsilon }_{0}}}\frac{1}{2{{r}^{5}}}{{Q}_{kl}}{{x}_{k}}{{x}_{l}}=\frac{1}{4\pi {{\varepsilon }_{0}}}\frac{\bar{r}\cdot \bar{\bar{Q}}\cdot \bar{r}}{2{{r}^{5}}}\tilde{\ }\frac{1}{{{r}^{3}}}</math>
 
<u>'''Beispiel: 2 entgegengesetzte Dipole:'''</u>
 
<u>'''1.5 Die elektrostatische Feldenergie'''</u>
 
Kraft:
<math>\begin{align}
& \bar{F}(\bar{r})=q\bar{E}(\bar{r})=-q\nabla \Phi (\bar{r}) \\
& \Rightarrow V(\bar{r})=\Phi (\bar{r}) \\
\end{align}</math>
 
ist die potenzielle Energie ! einer Ladung im Feld
<math>\bar{E}(\bar{r})</math>
 
Also:
 
<math>{{W}_{ij}}={{q}_{i}}\frac{1}{4\pi {{\varepsilon }_{0}}}\frac{{{q}_{j}}}{|{{{\bar{r}}}_{i}}-{{{\bar{r}}}_{j}}|}={{W}_{ji}}</math>
 
ist die Energie der Ladung
<math>{{q}_{i}}</math>
an
<math>{{\bar{r}}_{i}}</math>
im Feld der Ladung
<math>{{q}_{j}}</math>
an
<math>{{\bar{r}}_{j}}</math>
. ( In ihrem Potenzial)
Die gesamte potenzielle Energie eines Systems von Ladungen q1.... ergibt sich demnach durch Summation:
 
<math>W=\frac{1}{2}\sum\limits_{\begin{smallmatrix}
i,j \\
i\ne j
\end{smallmatrix}}^{{}}{{}}{{W}_{ij}}=\frac{1}{8\pi {{\varepsilon }_{0}}}\sum\limits_{\begin{smallmatrix}
i,j \\
i\ne j
\end{smallmatrix}}^{{}}{{}}\frac{{{q}_{i}}{{q}_{j}}}{|{{{\bar{r}}}_{i}}-{{{\bar{r}}}_{j}}|}={{W}_{ji}}</math>
 
und bei einer kontinuierlichen Ladungsverteilung:
 
<math>W=\frac{1}{2}\int_{{}}^{{}}{\Phi (\bar{r})\rho (\bar{r}){{d}^{3}}r}=\frac{1}{8\pi {{\varepsilon }_{0}}}\int_{{}}^{{}}{{}}{{d}^{3}}r\int_{{}}^{{}}{{}}{{d}^{3}}r\acute{\ }\frac{\rho (\bar{r})\rho (\bar{r}\acute{\ })}{|\bar{r}-\bar{r}\acute{\ }|}</math>
 
<math>W=\frac{1}{2}\int_{{}}^{{}}{\Phi (\bar{r})\rho (\bar{r}){{d}^{3}}r}</math>
 
Mit
<math>\rho (\bar{r})={{\varepsilon }_{0}}\nabla \cdot \bar{E}</math>
 
folgt:
 
<math>W=\frac{{{\varepsilon }_{0}}}{2}\int_{{{R}^{3}}}^{{}}{\Phi (\bar{r})\nabla \cdot \bar{E}{{d}^{3}}r}=\frac{{{\varepsilon }_{0}}}{2}\left[ \int_{{{R}^{3}}}^{{}}{\nabla \cdot \left( \Phi (\bar{r})\bar{E} \right){{d}^{3}}r}-\int_{{{R}^{3}}}^{{}}{\left( \nabla \Phi (\bar{r}) \right)\cdot \bar{E}{{d}^{3}}r} \right]</math>
 
Mit Hilfe des Gaußschen Satz folgt dann:
 
<math>\begin{align}
& W=\frac{{{\varepsilon }_{0}}}{2}\left[ \int_{{{S}_{\infty }}}^{{}}{\left( \Phi (\bar{r})\bar{E} \right)d\bar{f}}+\int_{{{R}^{3}}}^{{}}{{{{\bar{E}}}^{2}}(\bar{r}){{d}^{3}}r} \right] \\
& \begin{matrix}
\lim  \\
r\to \infty  \\
\end{matrix}\left( \Phi (\bar{r})\bar{E} \right)=0 \\
&  \\
\end{align}</math>
 
da die Größen ~ 1/3 bzw. 1/r² gehen
 
Also:
 
<math>W=\int_{{{R}^{3}}}^{{}}{\frac{{{\varepsilon }_{0}}}{2}{{{\bar{E}}}^{2}}(\bar{r}){{d}^{3}}r}=\int_{{{R}^{3}}}^{{}}{{{d}^{3}}rw(\bar{r})}</math>
 
Somit folgt für die Energiedichte des elektromagnetischen Feldes:
 
<math>w(\bar{r})=\frac{{{\varepsilon }_{0}}}{2}{{\bar{E}}^{2}}(\bar{r})</math>
 
Die Selbstenergie einer Punktladung ergibt sich zu
 
<math>\begin{align}
& \left| \bar{E}(\bar{r}) \right|=\frac{q}{4\pi {{\varepsilon }_{0}}{{r}^{2}}} \\
& w(\bar{r})=\frac{{{\varepsilon }_{0}}}{2}{{\left( \frac{q}{4\pi {{\varepsilon }_{0}}} \right)}^{2}}\frac{1}{{{r}^{4}}} \\
\end{align}</math>
 
und die Gesamtenergie ist folglich:
 
<math>\begin{align}
& W=\int_{{}}^{{}}{{{d}^{3}}r}w(\bar{r})=\frac{{{\varepsilon }_{0}}}{2}{{\left( \frac{q}{4\pi {{\varepsilon }_{0}}} \right)}^{2}}4\pi \int_{0}^{\infty }{{{r}^{2}}dr}\frac{1}{{{r}^{4}}} \\
& \int_{0}^{\infty }{{{r}^{2}}dr}\frac{1}{{{r}^{4}}}=\int_{0}^{\infty }{dr}\frac{1}{{{r}^{2}}}=\left[ \frac{1}{r} \right]_{0}^{\infty }\to \infty  \\
\end{align}</math>
 
Dies divergiert jedoch !!
Beim Übergang von Punktladungen zu kontinuierlichen Ladungsverteilungen wird
<math>i\ne j</math>
nicht mehr ausgeschlossen. Das bede4utet, zusätzlich zur Wechselwirkungsenergie wird die Energie berücksichtigt, die Zum Aufbau der Punktladung durch Zusammenführung aus dem Unendlichen benötigt wird, mitgenommen.
 
Dies ist jedoch bei einer Punktladung unendlich viel. Also ist der begriff der P8unktladung in einem Widerspruch zum feldtheoretischen Begriff der Energiedichte ( wen wunderts ?)
 
<u>'''1.6 Leiter in der Elektrostatik'''</u>
 
Elektrischer Leiter = Materie, mit quasi frei beweglichen Elektronen. Ein elektrisches Feld im Inneren eines Leiters übt dann eine Kraft auf die frei beweglichen Elektronen aus:
 
<math>\bar{F}(\bar{r})=q\bar{E}(\bar{r})</math>
 
 
Dadurch werden die Ladungen verschoben.
Es folgt, dass ein kompensierendes Feld
<math>\bar{E}\acute{\ }(\bar{r})</math>
aufgebaut wird, bis
<math>\bar{F}=0</math>
, also
<math>\bar{E}\acute{\ }-\bar{E}=0</math>
:
 
Anfangssituation:
 
Endsituation:
 
Für das Innere des Leiters folgt:
 
<math>\begin{align}
& {{{\bar{E}}}^{res.}}(\bar{r})=0 \\
& {{{\bar{E}}}^{res.}}(\bar{r})=-\nabla \Phi (\bar{r})=0 \\
& \Rightarrow \Phi (\bar{r})=const \\
\end{align}</math>
 
im Inneren des Leiters.
 
Man sagt: die Leiteroberfläche ist eine sogenannte Äquipotenzialfläche !
 
Allgemein gilt:
 
<math>\bar{E}(\bar{r})\bot \Phi (\bar{r})=const</math>
 
Somit steht das elektrische Feld immer senkrecht auf der Leiteroberfläche !
Vor allem beim Übergang zwischen einem Leiter und dem Vakuum !
 
Allgemein gilt:
 
<math>{{\varepsilon }_{0}}\nabla \cdot \bar{E}(\bar{r})=\rho (\bar{r})</math>
 
Hier:
 
<math>\begin{align}
& \bar{E}(\bar{r})=0 \\
& \Rightarrow \rho (\bar{r})=0 \\
\end{align}</math>
 
Das heißt: es existieren keine elektrischen Ladungen im Inneren eine Leiters !
 
'''Flächenladungsdichte'''
auf Leiteroberflächen:
 
 
<math>\begin{align}
& {{\varepsilon }_{0}}\int_{V}^{{}}{{{d}^{3}}r}\nabla \cdot \bar{E}(\bar{r})=\int_{V}^{{}}{{{d}^{3}}r}\rho (\bar{r})=\int_{\partial V}^{{}}{d\bar{f}}\bar{E}(\bar{r}) \\
& V=d\bar{f}\cdot \Delta s \\
\end{align}</math>
 
Mit
 
<math>\begin{align}
& d\bar{f}\to 0 \\
& \Delta s\to 0 \\
\end{align}</math>
 
folgt:
 
<math>\begin{align}
& \int_{\partial V}^{{}}{d\bar{f}}\bar{E}(\bar{r})\to df\bar{n}\cdot \bar{E} \\
& \bar{n}\cdot \bar{E}=E,da \\
& \bar{n}\quad Normalenvektor\ \bar{n}||\bar{E} \\
\end{align}</math>
 
Also:
 
<math>\begin{align}
& \int_{V}^{{}}{{{d}^{3}}r}\rho (\bar{r})\to df\rho (\bar{r})\Delta s \\
& \rho (\bar{r})\Delta s=\sigma (\bar{r}) \\
\end{align}</math>
 
= Flächenladungsdichte !!
 
Also gilt für das elektrische Feld auf der Leiteroberfläche:
 
<math>\begin{align}
& \int_{V}^{{}}{{{d}^{3}}r}\rho (\bar{r})\to df\rho (\bar{r})\Delta s \\
& E(\bar{r})=\frac{1}{{{\varepsilon }_{0}}}\sigma (\bar{r})\bar{n} \\
\end{align}</math>
 
Allgemein gilt für Flächenladungen:
 
 
<math>En\acute{\ }\acute{\ }-En\acute{\ }=\frac{1}{{{\varepsilon }_{0}}}\sigma (s)</math>
 
Man bezeichnet
<math>En\acute{\ }\acute{\ }-En\acute{\ }</math>
als Flächendivergenz analog zur "Volumendivergenz"
<math>\nabla \cdot \bar{E}=\frac{1}{{{\varepsilon }_{0}}}\rho (\bar{r})</math>
 
Dies ist ein Sprung der Normalkomponente von
<math>\bar{E}</math>
beim Durchgang durch eine geladene Fläche
 
Die Tangentialkompoente von E dagegen ist stetig beim Durchgang durch geladene Flächen
 
'''Beweis:'''
 
 
 
<math>\oint\limits_{\partial F}{{}}\bar{E}d\bar{s}=\int_{F}^{{}}{\nabla \times }\bar{E}d\bar{f}=0</math>
 
<math>\begin{align}
& F=dl\cdot dh \\
& dl\to 0 \\
& dh\to 0 \\
\end{align}</math>
 
<math>\begin{align}
& \Rightarrow \left( Et\acute{\ }\acute{\ }-Et\acute{\ } \right)dl=0 \\
& \Rightarrow Et\acute{\ }\acute{\ }-Et\acute{\ }=0 \\
\end{align}</math>
 
<math>En\acute{\ }\acute{\ }-En\acute{\ }=\frac{1}{{{\varepsilon }_{0}}}\sigma (s)</math>
 
<u>'''Randwertaufgaben der Elektrostatik mit Leitern'''</u>
 
# <u>'''Grundaufgabe:'''</u>
 
Gegeben sind Leiter
<math>{{L}_{\alpha }}</math>
mit den Oberflächen
<math>{{S}_{\alpha }}</math>
 
<math>\alpha =1,2,..,n</math>
, die auf den Potenzialen
<math>{{\Phi }_{\alpha }}</math>
liegen.
Die Raumladungsdichte im Außenraum V ist
<math>\rho (\bar{r})</math>
.
Gesucht ist
<math>\Phi (\bar{r})</math>
als Lösung der Poissongleichung
<math>\Delta \Phi (\bar{r})=-\frac{1}{{{\varepsilon }_{0}}}\rho (\bar{r})</math>
 
zu den gegebenen Randbedingungen
 
<math>\begin{align}
& \Phi (\bar{r}){{\left. {} \right|}_{S\alpha }}={{\Phi }_{\alpha }} \\
& \begin{matrix}
\lim  \\
r\to \infty  \\
\end{matrix}\Phi (\bar{r})=0 \\
\end{align}</math>
 
außerdem: Gesamtladungen
<math>{{Q}_{\alpha }}</math>
auf den Leitern.
Dies ist das Dirichletsche Randwertproblem
Beispiel: 2 Leiterschleifen mit Potenzial Phi1/ Phi 2 auf den Oberflächen S1 und S2, die im Außenraum V mit der Ladungsdichte
<math>\rho (\bar{r})</math>
liegen.
 
'''Formale Lösung:'''
 
<math>\Phi (\bar{r})=\int_{V}^{{}}{{{d}^{3}}r\acute{\ }}G\left( \bar{r}-\bar{r}\acute{\ } \right)\rho \left( \bar{r}\acute{\ } \right)+{{\varepsilon }_{0}}\sum\limits_{\alpha =1}^{n}{{{\Phi }_{\alpha }}\int_{S\alpha }^{{}}{{}}d\bar{f}\acute{\ }\cdot {{\nabla }_{r\acute{\ }}}G\left( \bar{r}-\bar{r}\acute{\ } \right)}</math>
 
Dabei ist die Greensche Funktion
<math>G\left( \bar{r}-\bar{r}\acute{\ } \right)</math>
die Lösung von
<math>\Delta G\left( \bar{r}-\bar{r}\acute{\ } \right)=-\frac{1}{{{\varepsilon }_{0}}}\delta (\bar{r}-\bar{r}\acute{\ })</math>
zu den Randbedingungen
 
<math>G\left( \bar{r}-\bar{r}\acute{\ } \right){{\left. {} \right|}_{\begin{smallmatrix}
\bar{r}\in S\alpha  \\
\bar{r}\acute{\ }\in V
\end{smallmatrix}}}=0</math>
 
<math>\begin{matrix}
\lim  \\
r\to \infty  \\
\end{matrix}G\left( \bar{r}-\bar{r}\acute{\ } \right)=0</math>
 
Somit ist
<math>G\left( \bar{r}-\bar{r}\acute{\ } \right)</math>
das Potenzial am Ort
<math>\bar{r}</math>
einer Punktladung am Ort
<math>\bar{r}\acute{\ }</math>
.
 
Beweis:
 
Aus dem Gaußschen Satz
 
<math>\int_{\partial V}^{{}}{d\bar{f}\cdot \bar{v}=}\int_{V}^{{}}{{{d}^{3}}r\nabla \cdot \bar{v}}</math>
 
folgt mittels der Funktion
 
<math>\bar{v}=\phi \nabla \psi </math>
:
 
<math>\int_{\partial V}^{{}}{d\bar{f}\cdot \left( \phi \nabla \psi  \right)=}\int_{V}^{{}}{{{d}^{3}}r\nabla \cdot \left( \phi \nabla \psi  \right)}=\int_{V}^{{}}{{{d}^{3}}r\nabla \phi \nabla \psi +\phi \Delta \psi }</math>
 
<math>\bar{v}=\psi \nabla \phi </math>
 
<math>\int_{\partial V}^{{}}{d\bar{f}\cdot \left( \psi \nabla \phi  \right)}=\int_{V}^{{}}{{{d}^{3}}r\nabla \phi \nabla \psi +\psi \Delta \phi }</math>
 
Also:
 
'''Greenscher Satz:'''
 
<math>\int_{\partial V}^{{}}{d\bar{f}\cdot \left( \phi \nabla \psi -\psi \nabla \phi  \right)}=\int_{V}^{{}}{{{d}^{3}}r\phi \Delta \psi -\psi \Delta \phi }</math>
 
Nun kann man einsetzen:
 
<math>\begin{align}
& \phi \left( {\bar{r}} \right):=G(\bar{r}-\bar{r}\acute{\ }) \\
& \psi \left( {\bar{r}} \right):=\Phi (\bar{r}) \\
& \partial V=\bigcup\limits_{\alpha =1}^{n}{{{S}_{\alpha }}} \\
\end{align}</math>
 
Bleibt zu zeigen:
 
<math>\begin{align}
& \Delta \Phi (\bar{r})=-\frac{1}{{{\varepsilon }_{0}}}\Rightarrow  \\
& \Phi (\bar{r})=\int_{V}^{{}}{{{d}^{3}}r\acute{\ }}G\left( \bar{r}-\bar{r}\acute{\ } \right)\rho \left( \bar{r}\acute{\ } \right)+{{\varepsilon }_{0}}\sum\limits_{\alpha =1}^{n}{{{\Phi }_{\alpha }}\int_{S\alpha }^{{}}{{}}d\bar{f}\acute{\ }\cdot {{\nabla }_{r\acute{\ }}}G\left( \bar{r}-\bar{r}\acute{\ } \right)} \\
\end{align}</math>
 
 
<math>\int_{\partial V}^{{}}{d\bar{f}\cdot \Phi (\bar{r}){{\nabla }_{r}}G\left( \bar{r}-\bar{r}\acute{\ } \right)-}\int_{\partial V}^{{}}{d\bar{f}\cdot G\left( \bar{r}-\bar{r}\acute{\ } \right){{\nabla }_{r}}\Phi (\bar{r})=-\frac{1}{{{\varepsilon }_{0}}}\left[ \int_{V}^{{}}{{{d}^{3}}r\Phi (\bar{r})\delta \left( \bar{r}-\bar{r}\acute{\ } \right)-}\int_{V}^{{}}{{{d}^{3}}rG\left( \bar{r}-\bar{r}\acute{\ } \right)\rho (\bar{r})} \right]}</math>
 
<math>\begin{align}
& \int_{\partial V}^{{}}{d\bar{f}\cdot G\left( \bar{r}-\bar{r}\acute{\ } \right){{\nabla }_{r}}\Phi (\bar{r})=0\quad wegen\quad G{{\left. {} \right|}_{\bar{r}\in S\alpha }}=0} \\
& \int_{V}^{{}}{{{d}^{3}}r\Phi (\bar{r})\delta \left( \bar{r}-\bar{r}\acute{\ } \right)=\Phi (\bar{r}\acute{\ })} \\
\end{align}</math>
 
'''Für'''
<math>\int_{\partial V}^{{}}{d\bar{f}\cdot \Phi (\bar{r}){{\nabla }_{r}}G\left( \bar{r}-\bar{r}\acute{\ } \right)}</math>
setzen wir
<math>-\int_{\bigcup\limits_{\alpha =1}^{n}{{{S}_{\alpha }}}}^{{}}{d\bar{f}\cdot \Phi (\bar{r}){{\nabla }_{r}}G\left( \bar{r}-\bar{r}\acute{\ } \right)}</math>
 
Dies führt deshalb zu einem Vorzeichenwechsel, da
<math>d\bar{f}</math>
stets nach außen zeigt .
 
Also:
 
<math>\Phi (\bar{r}\acute{\ })=\int_{V}^{{}}{{{d}^{3}}r}G\left( \bar{r}-\bar{r}\acute{\ } \right)\rho \left( {\bar{r}} \right)+{{\varepsilon }_{0}}\sum\limits_{\alpha =1}^{n}{{{\Phi }_{\alpha }}\oint\limits_{S\alpha }{{}}d\bar{f}\cdot {{\nabla }_{r}}G\left( \bar{r}-\bar{r}\acute{\ } \right)}</math>
 
Zeige:
 
<math>\begin{align}
& \Phi (\bar{r})=\int_{V}^{{}}{{{d}^{3}}r\acute{\ }}G\left( \bar{r}-\bar{r}\acute{\ } \right)\rho \left( \bar{r}\acute{\ } \right)+{{\varepsilon }_{0}}\sum\limits_{\alpha =1}^{n}{{{\Phi }_{\alpha }}\int_{S\alpha }^{{}}{{}}d\bar{f}\acute{\ }\cdot {{\nabla }_{r\acute{\ }}}G\left( \bar{r}-\bar{r}\acute{\ } \right)} \\
& \Rightarrow \Delta \Phi (\bar{r})=-\frac{1}{{{\varepsilon }_{0}}}\rho  \\
\end{align}</math>
 
im Inneren von V und
 
<math>\Phi (\bar{r}){{\left. {} \right|}_{S\alpha }}={{\Phi }_{\alpha }}</math>
, erfüllt also die Randbedingungen.
 
<math>\begin{align}
& {{\Delta }_{r\acute{\ }}}\Phi (\bar{r}\acute{\ })=\int_{V}^{{}}{{{d}^{3}}r{{\Delta }_{r\acute{\ }}}}G\left( \bar{r}-\bar{r}\acute{\ } \right)\rho \left( {\bar{r}} \right)+{{\varepsilon }_{0}}\sum\limits_{\alpha =1}^{n}{{{\Phi }_{\alpha }}\oint\limits_{S\alpha }{{}}d\bar{f}\cdot {{\nabla }_{r}}}{{\Delta }_{r\acute{\ }}}G\left( \bar{r}-\bar{r}\acute{\ } \right) \\
& {{\Delta }_{r\acute{\ }}}G\left( \bar{r}-\bar{r}\acute{\ } \right)=-\frac{1}{{{\varepsilon }_{0}}}\delta \left( \bar{r}-\bar{r}\acute{\ } \right) \\
& \delta \left( \bar{r}-\bar{r}\acute{\ } \right)=0,da\ \bar{r}\in S\alpha ,\bar{r}\acute{\ }\in V-\partial V \\
& \Rightarrow {{\varepsilon }_{0}}\sum\limits_{\alpha =1}^{n}{{{\Phi }_{\alpha }}\oint\limits_{S\alpha }{{}}d\bar{f}\cdot {{\nabla }_{r}}}{{\Delta }_{r\acute{\ }}}G\left( \bar{r}-\bar{r}\acute{\ } \right)=0 \\
& {{\Delta }_{r\acute{\ }}}\Phi (\bar{r}\acute{\ })=\int_{V}^{{}}{{{d}^{3}}r{{\Delta }_{r\acute{\ }}}}G\left( \bar{r}-\bar{r}\acute{\ } \right)\rho \left( {\bar{r}} \right)=-\int_{V}^{{}}{{{d}^{3}}r\frac{1}{{{\varepsilon }_{0}}}\delta \left( \bar{r}-\bar{r}\acute{\ } \right)}\rho \left( {\bar{r}} \right)=-\frac{\rho \left( \bar{r}\acute{\ } \right)}{{{\varepsilon }_{0}}} \\
\end{align}</math>
 
Dabei
 
<math>{{\varepsilon }_{0}}\sum\limits_{\alpha =1}^{n}{{{\Phi }_{\alpha }}\oint\limits_{S\alpha }{{}}d\bar{f}\cdot {{\nabla }_{r}}}{{\Delta }_{r\acute{\ }}}G\left( \bar{r}-\bar{r}\acute{\ } \right)</math>
als Anteil der Lösung, die die homogene Poissongleichung lösen, ohne Ladungsdichte
 
<math>\int_{V}^{{}}{{{d}^{3}}r{{\Delta }_{r\acute{\ }}}}G\left( \bar{r}-\bar{r}\acute{\ } \right)\rho \left( {\bar{r}} \right)</math>
dagegen löst gerade die inhomogene Poisson- Gleichung
 
'''Randbedingungen:'''
 
<math>\begin{align}
& \Phi (\bar{r}\acute{\ }){{\left. {} \right|}_{\bar{r}\acute{\ }\in S\beta }}=\int_{V}^{{}}{{{d}^{3}}r}G\left( \bar{r}-\bar{r}\acute{\ } \right){{\left. {} \right|}_{\bar{r}\acute{\ }\in S\beta }}\rho \left( {\bar{r}} \right)+{{\varepsilon }_{0}}\sum\limits_{\alpha =1}^{n}{{{\Phi }_{\alpha }}\oint\limits_{S\alpha }{{}}d\bar{f}\cdot {{\nabla }_{r}}}G\left( \bar{r}-\bar{r}\acute{\ } \right){{\left. {} \right|}_{\bar{r}\acute{\ }\in S\beta }} \\
& G\left( \bar{r}-\bar{r}\acute{\ } \right){{\left. {} \right|}_{\bar{r}\acute{\ }\in S\beta }}\rho \left( {\bar{r}} \right)=0 \\
& \Rightarrow \Phi (\bar{r}\acute{\ }){{\left. {} \right|}_{\bar{r}\acute{\ }\in S\beta }}={{\varepsilon }_{0}}\sum\limits_{\alpha =1}^{n}{{{\Phi }_{\alpha }}\oint\limits_{S\alpha }{{}}d\bar{f}\cdot {{\nabla }_{r}}}G\left( \bar{r}-\bar{r}\acute{\ } \right){{\left. {} \right|}_{\bar{r}\acute{\ }\in S\beta }}=-{{\varepsilon }_{0}}\int_{\partial V}^{{}}{{}}d\bar{f}\cdot \Phi (\bar{r}){{\nabla }_{r}}G\left( \bar{r}-\bar{r}\acute{\ } \right){{\left. {} \right|}_{\bar{r}\acute{\ }\in S\beta }} \\
\end{align}</math>
 
Auch hier: Vorzeichenwechsel, da df nach außen zeigt:
 
 
Das Innere der Ellipse ist die Leiterfläche , die vom Leiter
<math>{{L}_{\alpha }}</math>
eingeschlossene Fläche .
Mit dem Gaußschen Satz folgt:
 
<math>\begin{align}
& \Phi (\bar{r}\acute{\ }){{\left. {} \right|}_{\bar{r}\acute{\ }\in S\beta }}=-{{\varepsilon }_{0}}\int_{\partial V}^{{}}{{}}d\bar{f}\cdot \Phi (\bar{r}){{\nabla }_{r}}G\left( \bar{r}-\bar{r}\acute{\ } \right){{\left. {} \right|}_{\bar{r}\acute{\ }\in S\beta }} \\
& =-{{\varepsilon }_{0}}\left[ \int_{\partial V}^{{}}{{}}d\bar{f}G\left( \bar{r}-\bar{r}\acute{\ } \right){{\left. {} \right|}_{\bar{r}\acute{\ }\in S\beta }}\cdot {{\nabla }_{r}}\Phi (\bar{r})+\int_{V}^{{}}{{{d}^{3}}r\left( \Phi (\bar{r}){{\left. {} \right|}_{\bar{r}\acute{\ }\in S\beta }}{{\Delta }_{r}}G\left( \bar{r}-\bar{r}\acute{\ } \right)-G\left( \bar{r}-\bar{r}\acute{\ } \right){{\left. {} \right|}_{\bar{r}\acute{\ }\in S\beta }}{{\Delta }_{r}}\Phi (\bar{r}) \right)} \right] \\
& G\left( \bar{r}-\bar{r}\acute{\ } \right){{\left. {} \right|}_{\bar{r}\acute{\ }\in S\beta }}=0 \\
& \Rightarrow \Phi (\bar{r}\acute{\ }){{\left. {} \right|}_{\bar{r}\acute{\ }\in S\beta }}=-{{\varepsilon }_{0}}\int_{V}^{{}}{{{d}^{3}}r\left( \Phi (\bar{r}){{\left. {} \right|}_{\bar{r}\acute{\ }\in S\beta }}{{\Delta }_{r}}G\left( \bar{r}-\bar{r}\acute{\ } \right) \right)=}\int_{V}^{{}}{{{d}^{3}}r\left( \Phi (\bar{r}){{\left. {} \right|}_{\bar{r}\acute{\ }\in S\beta }}\left( -\frac{1}{{{\varepsilon }_{0}}}\delta \left( \bar{r}-\bar{r}\acute{\ } \right) \right) \right)} \\
& =\int_{V}^{{}}{{{d}^{3}}r\left( \Phi (\bar{r}){{\left. {} \right|}_{\bar{r}\acute{\ }\in S\beta }}\delta \left( \bar{r}-\bar{r}\acute{\ } \right) \right)}=\Phi (\bar{r}\acute{\ }){{\left. {} \right|}_{\bar{r}\acute{\ }\in S\beta }}={{\Phi }_{\beta }} \\
\end{align}</math>
 
'''Ladung:'''
 
<math>\begin{align}
& Q=\oint\limits_{S\alpha }{df\sigma ={{\varepsilon }_{0}}}\oint\limits_{S\alpha }{df\bar{n}\cdot \bar{E}} \\
& df\bar{n}=d\bar{f} \\
& \Rightarrow Q={{\varepsilon }_{0}}\oint\limits_{S\alpha }{d\bar{f}\cdot \bar{E}}=-{{\varepsilon }_{0}}\oint\limits_{S\alpha }{d\bar{f}\cdot \nabla \Phi } \\
\end{align}</math>
 
<u>'''Konstruktion der Greenschen Funktion'''</u>
 
Für Leiteroberflächen mit hoher Symmetrie bietet sich die Methode der Bildladungen an ! ( Spiegelladungsmethode).
Dabei wählt man eine fiktive Bildladung q´ bei
<math>\bar{r}\acute{\ }\acute{\ }</math>
im Leiter, so dass das Potenzial beider Ladungen auf der Leiteroberfläche verschwindet: q´=-q
 
<math>G\left( \bar{r}-\bar{r}\acute{\ } \right)=\frac{1}{4\pi {{\varepsilon }_{0}}}\left( \frac{1}{\left| r-r\acute{\ } \right|}-\frac{1}{\left| r-r\acute{\ }\acute{\ } \right|} \right)</math>
 
 
 
 
# <u>'''Grundaufgabe'''</u>
 
Gegeben: Gegeben sind Leiter
<math>{{L}_{\alpha }}</math>
mit den Oberflächen
<math>{{S}_{\alpha }}</math>
 
<math>\alpha =1,2,..,n</math>
, die mit
<math>{{Q}_{\alpha }}</math>
geladen sind.
Die Raumladungsdichte im Außenraum V ist
<math>\rho (\bar{r})</math>
.
Gesucht:
Gesucht ist
<math>\Phi (\bar{r})</math>
als Lösung der Poissongleichung
<math>\Delta \Phi (\bar{r})=-\frac{1}{{{\varepsilon }_{0}}}\rho (\bar{r})</math>
 
und
<math>{{\Phi }_{\alpha }}</math>
.
Lösung:
 
Das Problem kann auf die erste Grundaufgabe zurückgeführt werden durch Ausnutzung eines Zusammenhangs zwischen
<math>{{\Phi }_{\alpha }}</math>
und
<math>{{Q}_{\alpha }}</math>
:
 
Es gilt:
 
<math>{{Q}_{\alpha }}=\sum\limits_{\beta =1}^{n}{{}}{{C}_{\alpha \beta }}{{\Phi }_{\beta }}\quad \alpha =1,..,n</math>
 
Mit den Kapazitätskoeffizienten
<math>{{C}_{\alpha \beta }}</math>
.
 
'''Beweis:'''
 
<math>{{Q}_{\alpha }}=-{{\varepsilon }_{0}}\oint\limits_{S\alpha }{{}}d\bar{f}\cdot \nabla \Phi (\bar{r})</math>
 
<math>\begin{align}
& {{Q}_{\alpha }}=-{{\varepsilon }_{0}}\oint\limits_{S\alpha }{{}}d\bar{f}\cdot {{\nabla }_{r}}\int_{V}^{{}}{{{d}^{3}}r}G\left( \bar{r}-\bar{r}\acute{\ } \right)\rho \left( \bar{r}\acute{\ } \right)=-{{\varepsilon }_{0}}^{2}\oint\limits_{S\alpha }{{}}d\bar{f}\cdot {{\nabla }_{r}}\sum\limits_{\beta =1}^{n}{{{\Phi }_{\beta }}\oint\limits_{{{S}_{\beta }}}{{}}d\bar{f}\acute{\ }\cdot {{\nabla }_{r\acute{\ }}}}G\left( \bar{r}-\bar{r}\acute{\ } \right) \\
& {{Q}_{\alpha }}=-{{\varepsilon }_{0}}\int_{L\alpha }^{{}}{{}}{{d}^{3}}r\int_{V}^{{}}{{}}{{d}^{3}}r\acute{\ }{{\Delta }_{r}}G\left( \bar{r}-\bar{r}\acute{\ } \right)\rho \left( \bar{r}\acute{\ } \right)-\sum\limits_{\beta =1}^{n}{{{\Phi }_{\beta }}{{\varepsilon }_{0}}^{2}\oint\limits_{{{S}_{\alpha }}}{{}}d\bar{f}\cdot {{\nabla }_{r}}}\oint\limits_{{{S}_{\beta }}}{{}}d\bar{f}\acute{\ }\cdot {{\nabla }_{r\acute{\ }}}G\left( \bar{r}-\bar{r}\acute{\ } \right) \\
& {{\Delta }_{r}}G\left( \bar{r}-\bar{r}\acute{\ } \right)=-\frac{1}{{{\varepsilon }_{0}}}\delta \left( \bar{r}-\bar{r}\acute{\ } \right)=0\quad f\ddot{u}r\quad \bar{r}\in {{L}_{\alpha }},\bar{r}\acute{\ }\in V, \\
& {{\varepsilon }_{0}}^{2}\oint\limits_{{{S}_{\alpha }}}{{}}d\bar{f}\cdot {{\nabla }_{r}}\oint\limits_{{{S}_{\beta }}}{{}}d\bar{f}\acute{\ }\cdot {{\nabla }_{r\acute{\ }}}G\left( \bar{r}-\bar{r}\acute{\ } \right)=:-{{C}_{\alpha \beta }} \\
& \Rightarrow {{Q}_{\alpha }}=-\sum\limits_{\beta =1}^{n}{{{\Phi }_{\beta }}{{\varepsilon }_{0}}^{2}\oint\limits_{{{S}_{\alpha }}}{{}}d\bar{f}\cdot {{\nabla }_{r}}}\oint\limits_{{{S}_{\beta }}}{{}}d\bar{f}\acute{\ }\cdot {{\nabla }_{r\acute{\ }}}G\left( \bar{r}-\bar{r}\acute{\ } \right)=\sum\limits_{\beta =1}^{n}{{{C}_{\alpha \beta }}{{\Phi }_{\beta }}} \\
\end{align}</math>
 
Aus der Symmetrie
 
<math>G\left( \bar{r}-\bar{r}\acute{\ } \right)=G\left( \bar{r}\acute{\ }-\bar{r} \right)</math>
 
was aus der Greenschen Formel folgt mit
<math>\begin{align}
& \psi =G\left( \bar{r}-\bar{r}\acute{\ } \right) \\
& \phi =G\left( \bar{r}\acute{\ }-\bar{r} \right) \\
\end{align}</math>
 
folgt
 
<math>{{C}_{\alpha \beta }}={{C}_{\beta \alpha }}</math>
 
Einheit der Kapazität ist
 
<math>1F=1\frac{C}{V}=1Farad</math>
 
nach M. Faraday , 1791-1867
 
Betrachte speziell einen einzelnen Leiter mit Potenzial
<math>{{\Phi }_{l}}</math>
:
Für die Kapazität des Leiters gilt dann:
 
<math>C=\frac{Q}{{{\Phi }_{l}}}</math>
 
'''Beispiel: Plattenkondensator:'''
 
Zwei Kondensatorplatten befinden sich auf dem Potenzial
<math>{{\Phi }_{1}},{{\Phi }_{2}}</math>
:
 
 
Es gilt:
 
<math>\begin{align}
& {{Q}_{1}}={{C}_{11}}{{\Phi }_{1}}+{{C}_{12}}{{\Phi }_{2}} \\
& {{Q}_{1}}={{C}_{21}}{{\Phi }_{1}}+{{C}_{22}}{{\Phi }_{2}} \\
& {{C}_{12}}={{C}_{21}}=C\acute{\ } \\
& 1\leftrightarrow 2Symmetrie \\
& \Rightarrow {{C}_{11}}={{C}_{22}}=C \\
\end{align}</math>
 
Spezialfall: Q1+Q2=0
 
<math>\begin{align}
& \Rightarrow Q=C{{\Phi }_{1}}+C\acute{\ }{{\Phi }_{2}} \\
& -Q=C\acute{\ }{{\Phi }_{1}}+C{{\Phi }_{2}} \\
& \Rightarrow 0=(C+C\acute{\ })({{\Phi }_{1}}+{{\Phi }_{2}})\Rightarrow C=-C\acute{\ }=\frac{Q}{{{\Phi }_{1}}-{{\Phi }_{2}}}\quad (1) \\
\end{align}</math>
 
Das E-Feld existiert fast nur zwischen den Platten
Also:
 
<math>\begin{align}
& \sigma =\frac{Q}{F}={{\varepsilon }_{0}}E=const.\quad (2) \\
& \Rightarrow \Phi (x)=-Ex+{{\Phi }_{0}} \\
& \Rightarrow {{\Phi }_{1}}-{{\Phi }_{2}}=E({{x}_{2}}-{{x}_{1}})\quad (3) \\
& ({{x}_{2}}-{{x}_{1}}):=a \\
& \Rightarrow C=-C\acute{\ }=\frac{Q}{{{\Phi }_{1}}-{{\Phi }_{2}}}=\frac{Q}{Ea}={{\varepsilon }_{0}}\frac{F}{a} \\
\end{align}</math>
 
Betrachten wir nun die Lösung der zweiten Grundaufgabe:
 
<math>{{C}_{\alpha \beta }}={{C}_{\beta \alpha }}</math>
ist eine positiv definite Matrix und damit nicht singulär. Also können wir die Inverse suchen:
 
<math>{{\Phi }_{\alpha }}=\sum\limits_{\beta =1}^{n}{{{C}_{\alpha \beta }}^{-1}{{Q}_{\beta }}}</math>
 
Eingesetzt in die Lösung der ersten Grundaufgabe liefert dies
<math>\Phi (\bar{r})</math>
für gegebene
<math>{{Q}_{\beta }},\rho (\bar{r})</math>
 
Damit ist dann die zweite Grundaufgabe gelöst !
 
'''Energie '''des Feldes im Außenraum:
 
für
<math>\rho (\bar{r})=0</math>
:
 
<math>W=\frac{{{\varepsilon }_{0}}}{2}\int_{V}^{{}}{{{d}^{3}}r{{(\bar{E}(\bar{r}))}^{2}}}</math>
 
Betrachten wir nun eine differenzielle Änderung der Randbedingungen auf den
<math>{{L}_{\alpha }}</math>
:
 
<math>\begin{align}
& {{Q}_{\alpha }}\to {{Q}_{\alpha }}+\delta {{Q}_{\alpha }} \\
& {{\Phi }_{\alpha }}\to {{\Phi }_{\alpha }}+\delta {{\Phi }_{\alpha }} \\
\end{align}</math>
 
Lösung
 
<math>\Phi (\bar{r})\to \Phi (\bar{r})+\delta \Phi (\bar{r})</math>
 
Räumliche Anordnung ungeändert ermöglicht die Vertauschung von
 
<math>\delta ,\nabla </math>
''':'''
 
<math>\Delta \Phi (\bar{r})=0\Rightarrow \Delta \delta \Phi (\bar{r})=0</math>
in V ( Außenraum)
 
<math>\bar{E}(\bar{r})=-\nabla \Phi (\bar{r})\Rightarrow \delta \bar{E}(\bar{r})=-\nabla \delta \Phi (\bar{r})</math>
 
<math>\begin{align}
& \Rightarrow \delta W=\frac{{{\varepsilon }_{0}}}{2}\int_{V}^{{}}{{{d}^{3}}r(2\bar{E}(\bar{r})\delta \bar{E}(\bar{r}))=-}{{\varepsilon }_{0}}\int_{V}^{{}}{{{d}^{3}}r(\nabla \Phi (\bar{r})\delta \bar{E}(\bar{r}))} \\
& (\nabla \Phi (\bar{r})\delta \bar{E}(\bar{r}))=\nabla \left( \Phi (\bar{r})\delta \bar{E}(\bar{r}) \right)-\Phi (\bar{r})\nabla \delta \bar{E}(\bar{r}) \\
& \nabla \cdot \delta \bar{E}(\bar{r})=\delta \nabla \cdot \bar{E}(\bar{r})=0,da\ \rho =0 \\
\end{align}</math>
 
im Außenraum !
 
<math>\Rightarrow \delta W=-{{\varepsilon }_{0}}\int_{V}^{{}}{{{d}^{3}}r\nabla \left( \Phi (\bar{r})\delta \bar{E}(\bar{r}) \right)=}{{\varepsilon }_{0}}\sum\limits_{\alpha }^{{}}{{}}\oint\limits_{S\alpha }{{}}d\bar{f}\left( \Phi (\bar{r})\delta \bar{E}(\bar{r}) \right)</math>
 
Als Umformung mit dem Gaußschen Satz
 
Auch hier: Vorzeichenwechsel, da df an allen
<math>S\alpha </math>
in den Außenraum nach außen zeigt:
 
 
Wegen
<math>\Phi (\bar{r}){{\left. {} \right|}_{S\alpha }}={{\Phi }_{\alpha }}</math>
 
<math>\Rightarrow \delta W={{\varepsilon }_{0}}\sum\limits_{\alpha }^{{}}{{{\Phi }_{\alpha }}}\oint\limits_{S\alpha }{{}}d\bar{f}\delta \bar{E}(\bar{r})=\sum\limits_{\alpha }^{{}}{{{\Phi }_{\alpha }}}\delta {{Q}_{\alpha }}</math>
 
'''Mit'''
<math>{{Q}_{\alpha }}=\sum\limits_{\beta =1}^{n}{{}}{{C}_{\alpha \beta }}{{\Phi }_{\beta }}\quad \alpha =1,..,n</math>
 
<math>\begin{align}
& \Rightarrow \delta W={{\varepsilon }_{0}}\sum\limits_{\alpha }^{{}}{{{\Phi }_{\alpha }}}\oint\limits_{S\alpha }{{}}d\bar{f}\delta \bar{E}(\bar{r})=\sum\limits_{\alpha }^{{}}{{{\Phi }_{\alpha }}}\delta \sum\limits_{\beta =1}^{n}{{}}{{C}_{\alpha \beta }}{{\Phi }_{\beta }}=\sum\limits_{\alpha ,\beta =1}^{n}{{}}{{\Phi }_{\alpha }}{{C}_{\alpha \beta }}\delta {{\Phi }_{\beta }} \\
& \sum\limits_{\alpha ,\beta =1}^{n}{{}}{{\Phi }_{\alpha }}{{C}_{\alpha \beta }}\delta {{\Phi }_{\beta }}=\frac{1}{2}\left\{ \sum\limits_{\alpha ,\beta =1}^{n}{{}}{{\Phi }_{\beta }}{{C}_{\beta \alpha }}\delta {{\Phi }_{\alpha }}+\sum\limits_{\alpha ,\beta =1}^{n}{{}}{{\Phi }_{\alpha }}{{C}_{\alpha \beta }}\delta {{\Phi }_{\beta }} \right\} \\
& {{C}_{\beta \alpha }}={{C}_{\alpha \beta }} \\
& \Rightarrow \delta W=\frac{1}{2}\sum\limits_{\alpha ,\beta =1}^{n}{{}}{{C}_{\alpha \beta }}\left\{ {{\Phi }_{\beta }}\delta {{\Phi }_{\alpha }}+{{\Phi }_{\alpha }}\delta {{\Phi }_{\beta }} \right\}=\delta \left\{ \frac{1}{2}\sum\limits_{\alpha ,\beta =1}^{n}{{}}{{C}_{\alpha \beta }}{{\Phi }_{\alpha }}{{\Phi }_{\beta }} \right\} \\
\end{align}</math>
 
Damit ist jedoch die Feldenergie gefunden als
 
<math>W=\left\{ \frac{1}{2}\sum\limits_{\alpha ,\beta =1}^{n}{{}}{{C}_{\alpha \beta }}{{\Phi }_{\alpha }}{{\Phi }_{\beta }} \right\}</math>
 
2. <u>'''Stationäre Ströme und Magnetfeld'''</u>
 
<u>'''2.1 Kontinuitätsgleichung'''</u>
 
Bewegte Ladungen entsprechen elektrischem Strom I
 
Experimentelle Erfahrung:  Die Ladung bleibt erhalten:
 
<math>Q(t)=\int_{V}^{{}}{{}}{{d}^{3}}r\rho (\bar{r},t)</math>
 
Damit folgt ein globaler Erhaltungssatz:
 
<math>\frac{d}{dt}Q(t)=\frac{d}{dt}\int_{V}^{{}}{{}}{{d}^{3}}r\rho (\bar{r},t)=-\oint\limits_{\partial V}{{}}\delta I</math>
 
 
<math>\delta I=\frac{\rho dV}{dt}=\frac{\rho \left| v \right|dt\left| df \right|\cos \alpha }{dt}=\rho \bar{v}d\bar{f}</math>
 
Also gerade die Ladung, die durch
<math>d\bar{f}</math>
pro zeit aus V herausströmt
Als eine lokale Größe findet man die elektrische Stromdichte:
 
<math>\bar{j}(\bar{r},t):=\rho (\bar{r},t)\bar{v}(\bar{r},t)</math>
 
<math>\Rightarrow \frac{d}{dt}\int_{V}^{{}}{{}}{{d}^{3}}r\rho (\bar{r},t)=-\oint\limits_{\partial V}{{}}d\bar{f}\bar{j}(\bar{r},t)=-\int_{V}^{{}}{{{d}^{3}}r}\nabla \cdot \bar{j}(\bar{r},t)</math>
( Gauß !)  für alle Volumina V ( einfach zusammenhängend)
 
Somit folgt die Kontinuitätsgleichung als LOKALER Erhaltungssatz:
 
<math>\Rightarrow \frac{\partial }{\partial t}\rho (\bar{r},t)+\nabla \cdot \bar{j}(\bar{r},t)=0</math>
 
Speziell bei stationären Ladungsverteilungen gilt die Divergenzfreiheit  des Stroms:
 
<math>\nabla \cdot \bar{j}(\bar{r},t)=0</math>
 
Aber : natürlich muss deswegen nicht
<math>\bar{j}(\bar{r},t)=0</math>
gelten. Der Strom muss räumlich lediglich stationär sein !
 
<u>'''2.2 Magnetische Induktion'''</u>
 
<u>'''Experimentelle Erfahrung:'''</u>
 
Es existieren Wechselwirkungen zwischen den Ladungen: Eine Kraft wirkt auf Ladungen q, die sich mit v bewegen:
 
<math>\bar{F}=q\bar{v}\times \bar{B}(\bar{r})</math>
 
Die sogenannte Lorentz- Kraft !
 
<math>\bar{B}(\bar{r})</math>
ist die magnetische Induktion am Ort
<math>\bar{r}</math>
, die erzeugt wird von den anderen Ladungen mit einer zugeordneten Stromdichte
<math>\bar{j}(\bar{r}\acute{\ })</math>
.
 
Die Erzeugung dieser Magnetischen Induktion erfolgt gemäß des Ampereschen Gesetzes:
 
<math>\bar{B}(\bar{r})=\frac{{{\mu }_{0}}}{4\pi }\int_{{}}^{{}}{{}}{{d}^{3}}r\acute{\ }\bar{j}(\bar{r}\acute{\ })\times \frac{\bar{r}-\bar{r}\acute{\ }}{{{\left| \bar{r}-\bar{r}\acute{\ } \right|}^{3}}}</math>
 
Dies läuft völlig analog zur Coulomb- Wechselwirkung in der Elektrostatik:
 
<math>\begin{align}
& \bar{F}=q\bar{E}(\bar{r}) \\
& \bar{E}(\bar{r})=\frac{1}{4\pi {{\varepsilon }_{0}}}\int_{{}}^{{}}{{{d}^{3}}r\acute{\ }}\rho (\bar{r}\acute{\ })\frac{\bar{r}-\bar{r}\acute{\ }}{\left| \bar{r}-\bar{r}\acute{\ } \right|} \\
\end{align}</math>
 
Die Einheiten im SI- System lauten:
 
<math>\left[ B \right]=\frac{1Ns}{Cm}=\frac{1kg{{m}^{2}}}{C{{s}^{2}}}\cdot \frac{s}{{{m}^{2}}}=1V\frac{s}{{{m}^{2}}}=1T</math>
 
Mit diesen Einheiten ist dann
<math>{{\mu }_{0}}=1,26\cdot {{10}^{-6}}\frac{Vs}{Am}</math>
festgelegt, wie die Dielektrizitätskonstante jedoch frei wählbar !!
Die magnetische Induktion beschreibt keine neue, von der Coulomb- Wechselwirkung unabhängige WW: Man betrachte dazu lediglich die Transformation auf das lokale Ruhesystem einer bewegten Ladung:
 
Im Gauß System:
 
<math>\bar{F}=\frac{q}{c}\bar{v}\times \bar{B}(\bar{r})</math>
 
<math>\begin{align}
& \bar{B}(\bar{r})=\frac{1}{c}\int_{{}}^{{}}{{}}{{d}^{3}}r\acute{\ }\bar{j}(\bar{r}\acute{\ })\times \frac{\bar{r}-\bar{r}\acute{\ }}{{{\left| \bar{r}-\bar{r}\acute{\ } \right|}^{3}}} \\
&  \\
\end{align}</math>
 
<u>'''Die Kraft zwischen 2 stromdurchflossenen Leitern:'''</u>
 
Betrachten wir zwei infinit. dünne Leiter L, L´, die mit konstanten Strömen I und I´ durchflossen werden:
 
Der Strom durch L´:
 
<math>\begin{align}
& \bar{j}(\bar{r}\acute{\ }){{d}^{3}}r\acute{\ }=\rho {{d}^{3}}r\acute{\ }\bar{v}\acute{\ }=\frac{d}{dt}\rho {{d}^{3}}r\acute{\ }d\bar{r}\acute{\ } \\
& \frac{d}{dt}\rho {{d}^{3}}r\acute{\ }=I\acute{\ } \\
& \Rightarrow \bar{j}(\bar{r}\acute{\ }){{d}^{3}}r\acute{\ }=I\acute{\ }d\bar{r}\acute{\ } \\
\end{align}</math>
 
Somit folgt das Biot- Savartsche Gesetz für unendlich lange Leiter L´:
 
Die magnetische Induktion ist gerade:
 
<math>\begin{align}
& \bar{B}(\bar{r})=\frac{{{\mu }_{0}}}{4\pi }I\acute{\ }\int_{L\acute{\ }}^{{}}{{}}d\bar{r}\acute{\ }\times \frac{\bar{r}-\bar{r}\acute{\ }}{{{\left| \bar{r}-\bar{r}\acute{\ } \right|}^{3}}} \\
&  \\
\end{align}</math>
 
Die Kraft auf eine Ladung im Volumenelement d³r  von L ist damit gerade:
 
<math>d\bar{F}=\rho \bar{v}\times \bar{B}(\bar{r}){{d}^{3}}r=\bar{j}\times \bar{B}{{d}^{3}}r=Id\bar{r}\times \bar{B}</math>
 
Also:
 
<math>\bar{F}=\frac{{{\mu }_{0}}}{4\pi }II\acute{\ }\int_{L}^{{}}{{}}d\bar{r}\times \int_{L\acute{\ }}^{{}}{{}}d\bar{r}\acute{\ }\times \frac{\bar{r}-\bar{r}\acute{\ }}{{{\left| \bar{r}-\bar{r}\acute{\ } \right|}^{3}}}</math>
 
Dies ist dann die gesamte Kraft von L´ auf L
 
mit
 
<math>\begin{align}
& d\bar{r}\times \left( d\bar{r}\acute{\ }\times \left( \bar{r}-\bar{r} \right) \right)=\left( d\bar{r}\left( \bar{r}-\bar{r} \right) \right)d\bar{r}\acute{\ }-\left( d\bar{r}d\bar{r}\acute{\ } \right)\left( \bar{r}-\bar{r} \right) \\
& und \\
& \int_{L}^{{}}{{}}d\bar{r}\frac{\bar{r}-\bar{r}\acute{\ }}{{{\left| \bar{r}-\bar{r}\acute{\ } \right|}^{3}}}=-\left. \frac{1}{\left| \bar{r}-\bar{r}\acute{\ } \right|} \right|_{L-ANfang}^{L-Ende}=0 \\
\end{align}</math>
 
( Der Leiter ist entweder geschlossen oder die Enden liegen im Unendlichen)
folgt:
 
<math>\bar{F}=\frac{{{\mu }_{0}}}{4\pi }II\acute{\ }\int_{L}^{{}}{{}}\int_{L\acute{\ }}^{{}}{{}}\left( d\bar{r}d\bar{r}\acute{\ } \right)\frac{\bar{r}-\bar{r}\acute{\ }}{{{\left| \bar{r}-\bar{r}\acute{\ } \right|}^{3}}}</math>
 
für parallele Ströme:
 
<math>Id\bar{r}I\acute{\ }d\bar{r}\acute{\ }>0</math>
folgt Anziehung
für antiparallele Ströme:
 
<math>Id\bar{r}I\acute{\ }d\bar{r}\acute{\ }<0</math>
dagegen Abstoßung
 
Man sieht außerdem das dritte Newtonsche Gesetz:
 
<math>\begin{align}
& \bar{r}\leftrightarrow \bar{r}\acute{\ } \\
& d\bar{r}\leftrightarrow d\bar{r}\acute{\ } \\
& I\leftrightarrow I\acute{\ } \\
\end{align}</math>
 
Somit:
 
<math>\bar{F}\leftrightarrow -\bar{F}</math>
( actio gleich reactio)
 
<u>'''2.3 Die magnetostatischen feldgleichungen:'''</u>
 
Sie gelten auch in quasistaischer Näherung: Die zeitliche Änderung muss viel kleiner sein als die räumliche !!
 
Mit dem Vektorpotenzial
 
<math>\bar{A}(\bar{r})=\frac{{{\mu }_{0}}}{4\pi }\int_{{{R}^{3}}}^{{}}{{}}{{d}^{3}}r\acute{\ }\frac{\bar{j}(\bar{r}\acute{\ })}{\left| \bar{r}-\bar{r}\acute{\ } \right|}</math>
 
Welches nicht eindeutig ist, sondern beliebig gemäß
<math>\bar{A}(\bar{r})\to \bar{A}+\nabla \Psi </math>
umgeeicht werden kann.
(
<math>\Psi (\bar{r})</math>
beliebig möglich, da
<math>\nabla \times \nabla \Psi =0</math>
)
 
Mit diesem Vektorpotenzial also kann man schreiben:
 
<math>\bar{B}=rot\bar{A}(\bar{r})=\nabla \times \frac{{{\mu }_{0}}}{4\pi }\int_{{{R}^{3}}}^{{}}{{}}{{d}^{3}}r\acute{\ }\frac{\bar{j}(\bar{r}\acute{\ })}{\left| \bar{r}-\bar{r}\acute{\ } \right|}</math>
 
Beweis:
 
<math>\begin{align}
& rot\bar{A}(\bar{r})=\nabla \times \frac{{{\mu }_{0}}}{4\pi }\int_{{{R}^{3}}}^{{}}{{}}{{d}^{3}}r\acute{\ }\frac{\bar{j}(\bar{r}\acute{\ })}{\left| \bar{r}-\bar{r}\acute{\ } \right|}=\frac{{{\mu }_{0}}}{4\pi }\int_{{{R}^{3}}}^{{}}{{}}{{d}^{3}}r\acute{\ }{{\nabla }_{r}}\frac{1}{\left| \bar{r}-\bar{r}\acute{\ } \right|}\times \bar{j}(\bar{r}\acute{\ }) \\
& {{\nabla }_{r}}\frac{1}{\left| \bar{r}-\bar{r}\acute{\ } \right|}=-\frac{\bar{r}-\bar{r}\acute{\ }}{{{\left| \bar{r}-\bar{r}\acute{\ } \right|}^{3}}} \\
& \Rightarrow rot\bar{A}(\bar{r})=\frac{{{\mu }_{0}}}{4\pi }\int_{{{R}^{3}}}^{{}}{{}}{{d}^{3}}r\acute{\ }\bar{j}(\bar{r}\acute{\ })\times \frac{\bar{r}-\bar{r}\acute{\ }}{{{\left| \bar{r}-\bar{r}\acute{\ } \right|}^{3}}}=\bar{B}(\bar{r}) \\
\end{align}</math>
 
Folgende Aussagen sind äquivalent:
Es existiert ein Vektorpotenzial mit
 
<math>\begin{align}
& \bar{B}=rot\bar{A}(\bar{r}) \\
& \Leftrightarrow  \\
\end{align}</math>
 
<math>div\bar{B}=0</math>
 
Beweis:
 
<math>div(rot\bar{A}(\bar{r}))=0</math>
 
es gibt keine Quellen der magnetischen Induktion ( es existieren keine "magnetischen Ladungen".
 
Aber: Magnetische Monopole wurden 1936 von Dirac postuliert, um die Quantelung der Ladung zu erklären. ( aus der quantenmechanischen Quantisierung des Drehimpulses !)
Dies wurde durch die vereinheitlichte Feldtheori4e wieder aufgenommen !
Es wurden extrem schwere magnetische Monopole postuliert, die beim Urknall in den ersten
<math>{{10}^{-35}}s</math>
erzeugt worden sein sollen.
 
Sehr umstritten ist ein angeblicher experimenteller Nachweis von 1982 ( Spektrum der Wissenschaft, Juni 1982, S. 78 ff.)
'''Der Zusammenhang zwischen'''
 
<math>\bar{B}(\bar{r})</math>
und
<math>\bar{j}(\bar{r})</math>
:
 
<math>\begin{align}
& \nabla \times \bar{B}(\bar{r})=\nabla \times \left( \nabla \times \bar{A}(\bar{r}) \right)=\nabla \left( \nabla \cdot \bar{A}(\bar{r}) \right)-\Delta \bar{A}(\bar{r}) \\
& \nabla \cdot \bar{A}(\bar{r})=\nabla \cdot \frac{{{\mu }_{0}}}{4\pi }\int_{{{R}^{3}}}^{{}}{{}}{{d}^{3}}r\acute{\ }\frac{\bar{j}(\bar{r}\acute{\ })}{\left| \bar{r}-\bar{r}\acute{\ } \right|}=\frac{{{\mu }_{0}}}{4\pi }\int_{{{R}^{3}}}^{{}}{{}}{{d}^{3}}r\acute{\ }{{\nabla }_{r}}\cdot \left( \frac{\bar{j}(\bar{r}\acute{\ })}{\left| \bar{r}-\bar{r}\acute{\ } \right|} \right)=\frac{{{\mu }_{0}}}{4\pi }\int_{{{R}^{3}}}^{{}}{{}}{{d}^{3}}r\acute{\ }\bar{j}(\bar{r}\acute{\ }){{\nabla }_{r}}\cdot \frac{1}{\left| \bar{r}-\bar{r}\acute{\ } \right|} \\
& {{\nabla }_{r}}\cdot \frac{1}{\left| \bar{r}-\bar{r}\acute{\ } \right|}=-{{\nabla }_{r\acute{\ }}}\cdot \frac{1}{\left| \bar{r}-\bar{r}\acute{\ } \right|} \\
& \Rightarrow \nabla \cdot \bar{A}(\bar{r})=-\frac{{{\mu }_{0}}}{4\pi }\int_{{{R}^{3}}}^{{}}{{}}{{d}^{3}}r\acute{\ }\left[ {{\nabla }_{r\acute{\ }}}\cdot \left( \frac{\bar{j}(\bar{r}\acute{\ })}{\left| \bar{r}-\bar{r}\acute{\ } \right|} \right)+\frac{1}{\left| \bar{r}-\bar{r}\acute{\ } \right|}{{\nabla }_{r\acute{\ }}}\cdot \bar{j}(\bar{r}\acute{\ }) \right] \\
& {{\nabla }_{r\acute{\ }}}\cdot \bar{j}(\bar{r}\acute{\ })=-\frac{\partial }{\partial t}\rho =0 \\
& \Rightarrow \nabla \cdot \bar{A}(\bar{r})=-\frac{{{\mu }_{0}}}{4\pi }\int_{{{R}^{3}}}^{{}}{{}}{{d}^{3}}r\acute{\ }{{\nabla }_{r\acute{\ }}}\cdot \left( \frac{\bar{j}(\bar{r}\acute{\ })}{\left| \bar{r}-\bar{r}\acute{\ } \right|} \right) \\
\end{align}</math>
 
Wobei die verwendete Kontinuitätsgleichung natürlich nur für statische Ladungsverteilungen gilt !
 
Im Allgemeinen Fall gilt dagegen:
 
<math>\begin{align}
& \Rightarrow \nabla \cdot \bar{A}(\bar{r})=-\frac{{{\mu }_{0}}}{4\pi }\int_{{{R}^{3}}}^{{}}{{}}{{d}^{3}}r\acute{\ }{{\nabla }_{r\acute{\ }}}\cdot \left( \frac{\bar{j}(\bar{r}\acute{\ })}{\left| \bar{r}-\bar{r}\acute{\ } \right|} \right)-\frac{\partial }{\partial t}\frac{{{\mu }_{0}}}{4\pi }\int_{{{R}^{3}}}^{{}}{{}}{{d}^{3}}r\acute{\ }\frac{\rho (\bar{r}\acute{\ },t)}{\left| \bar{r}-\bar{r}\acute{\ } \right|} \\
& \frac{{{\mu }_{0}}}{4\pi }\int_{{{R}^{3}}}^{{}}{{}}{{d}^{3}}r\acute{\ }\frac{\rho (\bar{r}\acute{\ },t)}{\left| \bar{r}-\bar{r}\acute{\ } \right|}={{\mu }_{0}}{{\varepsilon }_{0}}\Phi (\bar{r},t) \\
& \Rightarrow \nabla \cdot \bar{A}(\bar{r})=-\frac{{{\mu }_{0}}}{4\pi }\oint\limits_{S\infty }{{}}{{d}^{3}}\bar{f}\acute{\ }\left( \frac{\bar{j}(\bar{r}\acute{\ })}{\left| \bar{r}-\bar{r}\acute{\ } \right|} \right)-{{\mu }_{0}}{{\varepsilon }_{0}}\frac{\partial }{\partial t}\Phi (\bar{r},t) \\
\end{align}</math>
 
Mit dem Gaußschen Satz.
Wenn das Potenzial jedoch ins unendliche hinreichend rasch abfällt, so gilt:
 
<math>\oint\limits_{S\infty }{{}}{{d}^{3}}\bar{f}\acute{\ }\left( \frac{\bar{j}(\bar{r}\acute{\ })}{\left| \bar{r}-\bar{r}\acute{\ } \right|} \right)=0</math>
 
Also:
 
<math>\nabla \cdot \bar{A}(\bar{r})=-{{\mu }_{0}}{{\varepsilon }_{0}}\frac{\partial }{\partial t}\Phi (\bar{r},t)</math>
 
Also:
 
<math>\nabla \left( \nabla \cdot \bar{A}(\bar{r}) \right)={{\mu }_{0}}{{\varepsilon }_{0}}\frac{\partial }{\partial t}\bar{E}(\bar{r},t)</math>
 
Auf der anderen Seite ergibt sich ganz einfach
 
<math>\begin{align}
& \Delta \bar{A}(\bar{r})=\frac{{{\mu }_{0}}}{4\pi }\int_{{{R}^{3}}}^{{}}{{}}{{d}^{3}}r\acute{\ }{{\Delta }_{r}}\cdot \left( \frac{\bar{j}(\bar{r}\acute{\ })}{\left| \bar{r}-\bar{r}\acute{\ } \right|} \right)=\frac{{{\mu }_{0}}}{4\pi }\int_{{{R}^{3}}}^{{}}{{}}{{d}^{3}}r\acute{\ }\bar{j}(\bar{r}\acute{\ }){{\Delta }_{r}}\cdot \left( \frac{1}{\left| \bar{r}-\bar{r}\acute{\ } \right|} \right) \\
& =\frac{{{\mu }_{0}}}{4\pi }\int_{{{R}^{3}}}^{{}}{{}}{{d}^{3}}r\acute{\ }\bar{j}(\bar{r}\acute{\ })\delta \left( \bar{r}-\bar{r}\acute{\ } \right)=-{{\mu }_{0}}\bar{j}(\bar{r}) \\
\end{align}</math>
 
wegen
 
<math>{{\Delta }_{r}}\cdot \left( \frac{1}{\left| \bar{r}-\bar{r}\acute{\ } \right|} \right)=4\pi \delta \left( \bar{r}-\bar{r}\acute{\ } \right)</math>
 
Also:
 
<math>\nabla \times \bar{B}(\bar{r})=\nabla \left( \nabla \cdot \bar{A}(\bar{r}) \right)-\Delta \bar{A}(\bar{r})={{\mu }_{0}}\bar{j}(\bar{r})+{{\mu }_{0}}{{\varepsilon }_{0}}\frac{\partial }{\partial t}\bar{E}(\bar{r},t)</math>
 
Für stationäre Ströme, die gerade bei stationären Ladungsverteilungen vorliegen, folgt:
 
<math>\begin{align}
& \nabla \times \bar{B}(\bar{r})={{\mu }_{0}}\bar{j}(\bar{r}) \\
& {{\mu }_{0}}{{\varepsilon }_{0}}\frac{\partial }{\partial t}\bar{E}(\bar{r},t)=0 \\
\end{align}</math>
 
Dies ist die differenzielle Form des Ampereschen Gesetzes
Die Ströme sind die Wirbel der magnetischen Induktion !!
 
Integration über eine Fläche F mit Rand
<math>\partial F</math>
liefert die Intgralform:
 
<math>\begin{align}
& \int_{{}}^{{}}{d\bar{f}\cdot }\nabla \times \bar{B}(\bar{r})=\oint\limits_{\partial F}{{}}d\bar{s}\bar{B}(\bar{r})=\int_{{}}^{{}}{d\bar{f}\cdot }{{\mu }_{0}}\bar{j}(\bar{r})={{\mu }_{0}}I \\
& \oint\limits_{\partial F}{{}}d\bar{s}\bar{B}(\bar{r})={{\mu }_{0}}I \\
\end{align}</math>
 
Mit dem Satz von Stokes
Das sogenannte Durchflutungsgesetz !
 
<u>'''Zusammenfassung:'''</u>
 
<u>'''Magnetostatik:'''</u>
 
<math>div\bar{B}=0\Leftrightarrow \bar{B}=rot\bar{A}</math>
( quellenfreiheit)
 
<math>\begin{align}
& rot\bar{B}={{\mu }_{0}}\bar{j}(\bar{r})\Leftrightarrow \oint\limits_{\partial F}{{}}d\bar{s}\cdot \bar{B}={{\mu }_{0}}I \\
& \Rightarrow \Delta \bar{A}=-{{\mu }_{0}}\bar{j}(\bar{r}) \\
\end{align}</math>
 
Gilt jedoch nur im Falle der Coulomb- Eichung:
 
<math>\nabla \cdot \bar{A}=0</math>
 
Dies geschieht durch die Umeichung
 
<math>\begin{align}
& \bar{A}\acute{\ }(\bar{r})\to \bar{A}+\nabla \Psi  \\
& \nabla \times \bar{A}\acute{\ }(\bar{r})\to \nabla \times \bar{A}+\nabla \times \nabla \Psi  \\
& \nabla \times \nabla \Psi =0\Rightarrow \nabla \times \bar{A}\acute{\ }(\bar{r})\to \nabla \times \bar{A} \\
& \Rightarrow \nabla \times \left( \nabla \times \bar{A}\acute{\ }(\bar{r}) \right)=\nabla \times \bar{B}(\bar{r})={{\mu }_{0}}\bar{j} \\
& \nabla \times \left( \nabla \times \bar{A}\acute{\ }(\bar{r}) \right)=\nabla \left( \nabla \cdot \bar{A}\acute{\ }(\bar{r}) \right)-\Delta \bar{A}\acute{\ }(\bar{r}) \\
\end{align}</math>
 
<u>'''Elektrostatik:'''</u>
 
<math>rot\bar{E}=0\Leftrightarrow \bar{E}=-\nabla \Phi </math>
( Wirbelfreiheit)
 
<math>\begin{align}
& {{\varepsilon }_{0}}\nabla \cdot \bar{E}=\rho  \\
& \Leftrightarrow {{\varepsilon }_{0}}\oint\limits_{\partial V}{d\bar{f}\cdot }\bar{E}=Q \\
\end{align}</math>
differenzielle Form / integrale Form
 
<math>\Rightarrow \Delta \Phi =-\frac{1}{{{\varepsilon }_{0}}}\rho \left( {\bar{r}} \right)</math>
( Poissongleichung)
 
<u>'''Magnetische Multipole ( stationär)'''</u>
 
Ausgangspunkt ist
<math>\bar{A}(\bar{r})=\frac{{{\mu }_{0}}}{4\pi }\int_{{{R}^{3}}}^{{}}{{}}{{d}^{3}}r\acute{\ }\frac{\bar{j}(\bar{r}\acute{\ })}{\left| \bar{r}-\bar{r}\acute{\ } \right|}</math>
(mit der Coulomb- Eichung
<math>\nabla \cdot \bar{A}(\bar{r})=0</math>
)
 
mit den Randbedingungen
<math>\bar{A}(\bar{r})\to 0</math>
für r-> unendlich
 
Taylorentwicklung nach
<math>\frac{1}{\left| \bar{r}-\bar{r}\acute{\ } \right|}</math>
von analog zum elektrischen Fall:
Die Stromverteilung
<math>\bar{j}(\bar{r}\acute{\ })</math>
sei stationär für
<math>r>>r\acute{\ }</math>
 
<math>\frac{1}{\left| \bar{r}-\bar{r}\acute{\ } \right|}=\frac{1}{r}+\frac{1}{{{r}^{3}}}\left( \bar{r}\cdot \bar{r}\acute{\ } \right)+...</math>
 
<math>\bar{A}(\bar{r})=\frac{{{\mu }_{0}}}{4\pi r}\int_{{{R}^{3}}}^{{}}{{}}{{d}^{3}}r\acute{\ }\bar{j}(\bar{r}\acute{\ })+\frac{{{\mu }_{0}}}{4\pi {{r}^{3}}}\int_{{{R}^{3}}}^{{}}{{}}{{d}^{3}}r\acute{\ }\bar{j}(\bar{r}\acute{\ })\left( \bar{r}\cdot \bar{r}\acute{\ } \right)+...</math>
 
'''Monopol- Term'''
 
'''Mit'''
 
<math>{{\nabla }_{r\acute{\ }}}\cdot \left[ {{x}_{k}}\acute{\ }\bar{j}(\bar{r}\acute{\ }) \right]={{x}_{k}}\acute{\ }\left( {{\nabla }_{r\acute{\ }}}\cdot \bar{j}(\bar{r}\acute{\ }) \right)+\bar{j}(\bar{r}\acute{\ })\cdot \left( {{\nabla }_{r\acute{\ }}}{{x}_{k}}\acute{\ } \right)</math>
 
Im stationären Fall folgt aus der Kontinuitätsgleichung:
 
<math>{{\nabla }_{r\acute{\ }}}\cdot \bar{j}(\bar{r}\acute{\ })=0</math>
 
<math>{{\nabla }_{r\acute{\ }}}\cdot \left[ {{x}_{k}}\acute{\ }\bar{j}(\bar{r}\acute{\ }) \right]=\bar{j}(\bar{r}\acute{\ })\cdot \left( {{\nabla }_{r\acute{\ }}}{{x}_{k}}\acute{\ } \right)={{j}_{l}}{{\delta }_{kl}}={{j}_{k}}</math>
 
Mit
<math>{{\nabla }_{r\acute{\ }}}\cdot \left[ {{x}_{k}}\acute{\ }\bar{j}(\bar{r}\acute{\ }) \right]={{j}_{k}}</math>
folgt dann:
 
<math>\int_{{}}^{{}}{{{d}^{3}}r\acute{\ }}{{j}_{k}}(\bar{r}\acute{\ })=\int_{{}}^{{}}{{{d}^{3}}r\acute{\ }}{{\nabla }_{r\acute{\ }}}\cdot \left[ {{x}_{k}}\acute{\ }\bar{j}(\bar{r}\acute{\ }) \right]=\oint\limits_{S\infty }{d\bar{f}}\left[ {{x}_{k}}\acute{\ }\bar{j}(\bar{r}\acute{\ }) \right]=0</math>
 
Somit verschwindet der Monopolterm in der Theorie
 
'''Dipol- Term'''
 
mit
 
<math>\left[ \bar{r}\acute{\ }\times \bar{j}(\bar{r}\acute{\ }) \right]\times \bar{r}=\left( \bar{r}\bar{r}\acute{\ } \right)\bar{j}-\left( \bar{r}\bar{j} \right)\bar{r}\acute{\ }=2\left( \bar{r}\bar{r}\acute{\ } \right)\bar{j}-\left[ \left( \bar{r}\bar{r}\acute{\ } \right)\bar{j}+\left( \bar{r}\bar{j} \right)\bar{r}\acute{\ } \right]</math>
 
und mit
 
<math>\begin{align}
& {{\nabla }_{r\acute{\ }}}\left[ {{x}_{k}}\acute{\ }\left( \bar{r}\bar{r}\acute{\ } \right)\bar{j} \right]=\left[ \left( \bar{r}\bar{r}\acute{\ } \right){{j}_{k}}+{{x}_{k}}\acute{\ }\left( \bar{r}\bar{j} \right)+{{x}_{k\acute{\ }}}\left( \bar{r}\bar{r}\acute{\ } \right){{\nabla }_{r\acute{\ }}}\cdot \bar{j} \right] \\
& {{\nabla }_{r\acute{\ }}}\cdot \bar{j}=0 \\
& \Rightarrow {{\nabla }_{r\acute{\ }}}\left[ {{x}_{k}}\acute{\ }\left( \bar{r}\bar{r}\acute{\ } \right)\bar{j} \right]=\left[ \left( \bar{r}\bar{r}\acute{\ } \right){{j}_{k}}+{{x}_{k}}\acute{\ }\left( \bar{r}\bar{j} \right) \right] \\
\end{align}</math>
 
Folgt:
 
<math>\int_{{{R}^{3}}}^{{}}{{}}{{d}^{3}}r\acute{\ }{{\nabla }_{r\acute{\ }}}\left[ {{x}_{k}}\acute{\ }\left( \bar{r}\bar{r}\acute{\ } \right)\bar{j} \right]=\int_{{{R}^{3}}}^{{}}{{}}{{d}^{3}}r\acute{\ }\left[ \left( \bar{r}\bar{r}\acute{\ } \right){{j}_{k}}+{{x}_{k}}\acute{\ }\left( \bar{r}\bar{j} \right) \right]=0</math>
 
Da
 
<math>\int_{{{R}^{3}}}^{{}}{{}}{{d}^{3}}r\acute{\ }{{\nabla }_{r\acute{\ }}}\left[ {{x}_{k}}\acute{\ }\left( \bar{r}\bar{r}\acute{\ } \right)\bar{j} \right]=\oint\limits_{S\infty }{d\bar{f}}\left[ {{x}_{k}}\acute{\ }\left( \bar{r}\bar{r}\acute{\ } \right)\bar{j} \right]=0</math>
weil der Strom verschwindet !
Somit gibt der Term
 
<math>\left[ \left( \bar{r}\bar{r}\acute{\ } \right)\bar{j}+\left( \bar{r}\bar{j} \right)\bar{r}\acute{\ } \right]</math>
 
keinen Beitrag zum
 
<math>\frac{{{\mu }_{0}}}{4\pi {{r}^{3}}}\int_{{{R}^{3}}}^{{}}{{}}{{d}^{3}}r\acute{\ }\bar{j}(\bar{r}\acute{\ })\left( \bar{r}\cdot \bar{r}\acute{\ } \right)</math>
 
Also:
 
<math>\bar{A}(\bar{r})=\frac{{{\mu }_{0}}}{4\pi {{r}^{3}}}\frac{1}{2}\int_{{{R}^{3}}}^{{}}{{}}{{d}^{3}}r\acute{\ }\left( \bar{r}\acute{\ }\times \bar{j}(\bar{r}\acute{\ }) \right)\times \bar{r}</math>
 
Als DIPOLPOTENZIAL !!
 
<math>\begin{align}
& \bar{A}(\bar{r}):=\frac{{{\mu }_{0}}}{4\pi {{r}^{3}}}\bar{m}\times \bar{r} \\
& \bar{m}=\frac{1}{2}\int_{{{R}^{3}}}^{{}}{{}}{{d}^{3}}r\acute{\ }\left( \bar{r}\acute{\ }\times \bar{j}(\bar{r}\acute{\ }) \right) \\
\end{align}</math>
 
das magnetische Dipolmoment !
 
Analog zu
 
<math>\begin{align}
& \Phi (\bar{r}):=\frac{1}{4\pi {{\varepsilon }_{0}}{{r}^{3}}}\bar{p}\cdot \bar{r} \\
& \bar{p}:=\int_{{{R}^{3}}}^{{}}{{}}{{d}^{3}}r\acute{\ }\bar{r}\acute{\ }\rho (\bar{r}\acute{\ }) \\
\end{align}</math>
 
dem elektrischen Dipolmoment
 
Die magnetische Induktion des Dipolmomentes ergibt sich als:
 
<math>\bar{B}(\bar{r}):=\nabla \times \frac{{{\mu }_{0}}}{4\pi {{r}^{3}}}\bar{m}\times \bar{r}=\frac{{{\mu }_{0}}}{4\pi {{r}^{5}}}\left[ 3\left( \bar{m}\cdot \bar{r} \right)\bar{r}-{{r}^{2}}\bar{m} \right]</math>
 
Wegen:
 
<math>\nabla \times \left( \bar{a}\times \bar{b} \right)=\left( \bar{b}\cdot \nabla  \right)\bar{a}-\left( \bar{a}\cdot \nabla  \right)\bar{b}+\bar{a}\left( \nabla \cdot \bar{b} \right)-\bar{b}\left( \nabla \cdot \bar{a} \right)</math>
 
mit
 
<math>\begin{align}
& \bar{a}=\frac{{\bar{m}}}{{{r}^{3}}} \\
& \bar{b}=\bar{r} \\
& \Rightarrow div\bar{a}=-3\frac{\bar{m}\cdot \bar{r}}{{{r}^{5}}} \\
& div\bar{b}=3 \\
& \left( \bar{b}\cdot \nabla  \right)\bar{a}=-3\frac{\bar{m}\cdot {{r}^{2}}}{{{r}^{5}}} \\
& \left( \bar{a}\cdot \nabla  \right)\bar{b}=\frac{{\bar{m}}}{{{r}^{3}}} \\
\end{align}</math>
 
Analog ergab sich als elektrisches Dipolfeld:
 
<math>\bar{E}(\bar{r}):=\frac{1}{4\pi {{\varepsilon }_{0}}{{r}^{5}}}\left[ 3\left( \bar{p}\cdot \bar{r} \right)-{{r}^{2}}\bar{p} \right]</math>
 
<u>'''Beispiel: Ebene Leiterschleife L:'''</u>
 
 
 
<math>\begin{align}
& d\bar{f}\acute{\ }=\frac{1}{2}\bar{r}\acute{\ }\times d\bar{s}\acute{\ } \\
& {{d}^{3}}\bar{r}\acute{\ }j(\bar{r}\acute{\ })=d\bar{s}\acute{\ }I \\
\end{align}</math>
 
Mit I = Strom durch den Leiter
 
<math>\Rightarrow \bar{m}=\frac{1}{2}\oint\limits_{L}{{}}{{d}^{3}}r\acute{\ }\left( \bar{r}\acute{\ }\times \bar{j}(\bar{r}\acute{\ }) \right)=\frac{I}{2}\oint\limits_{L}{{}}\bar{r}\acute{\ }\times d\bar{s}\acute{\ }=I\int_{F}^{{}}{{}}d\bar{f}\acute{\ }=IF\bar{n}</math>
 
Dabei ist
 
<math>\bar{n}</math>
die Normale auf der von L eingeschlossenen Fläche F
 
Also: Ein Ringstrom bedingt ein magnetisches Dipolmoment
<math>\bar{m}</math>
 
 
analog: 2 Punktladungen bedingen ein elektrisches Dipolmoment
<math>\bar{p}=q\bar{a}</math>
, welches von der positiven zur negativen Ladung zeigt.
 
<u>'''Bewegte Ladungen'''</u>
N Teilchen mit den Massen mi und den Ladungen qi bewegen sich.
 
Dabei sei die spezifische Ladung
 
<math>\frac{{{q}_{i}}}{{{m}_{i}}}=\frac{q}{m}</math>
konstant:
 
<math>\begin{align}
& \rho (\bar{r})=\sum\limits_{i}{{}}{{q}_{i}}\delta \left( \bar{r}-{{{\bar{r}}}_{i}} \right) \\
& \bar{j}(\bar{r})=\sum\limits_{i}{{}}{{q}_{i}}{{{\bar{v}}}_{i}}\delta \left( \bar{r}-{{{\bar{r}}}_{i}} \right) \\
& {{{\bar{v}}}_{i}}=\frac{d{{{\bar{r}}}_{i}}}{dt} \\
\end{align}</math>
 
Das magnetische Dipolmoment beträgt:
 
<math>\begin{align}
& \bar{m}=\frac{1}{2}\oint\limits_{L}{{}}{{d}^{3}}r\acute{\ }\left( \bar{r}\acute{\ }\times \bar{j}(\bar{r}\acute{\ }) \right)=\frac{1}{2}\sum\limits_{i}{{}}{{q}_{i}}\int_{{}}^{{}}{{}}{{d}^{3}}r\acute{\ }\bar{r}\acute{\ }\times {{{\bar{v}}}_{i}}\delta \left( \bar{r}\acute{\ }-{{{\bar{r}}}_{i}} \right)=\frac{1}{2}\sum\limits_{i}{{}}{{q}_{i}}{{{\bar{r}}}_{i}}\times {{{\bar{v}}}_{i}}=\frac{1}{2}\sum\limits_{i}{{}}\frac{{{q}_{i}}}{{{m}_{i}}}{{m}_{i}}{{{\bar{r}}}_{i}}\times {{{\bar{v}}}_{i}} \\
& \frac{{{q}_{i}}}{{{m}_{i}}}=\frac{q}{m} \\
& \Rightarrow \bar{m}=\frac{q}{2m}\bar{L} \\
\end{align}</math>
 
Mit dem Bahndrehimpuls
<math>\bar{L}</math>
:
 
<math>\bar{m}=\frac{q}{2m}\bar{L}</math>
gilt aber auch für starre Körper !
* Allgemeines Gesetz !
 
Jedoch gilt dies nicht für den Spin eines Elektrons !!!
 
<math>\begin{align}
& \bar{m}=g\frac{e}{2m}\bar{S} \\
& g\approx 2 \\
\end{align}</math>
 
Somit ist der Spin nicht vollständig durch die Vorstellung von einer rotierenden Ladungsverteilung zu verstehen !
 
'''Kraft auf eine Stromverteilung:'''
 
<math>\bar{j}(\bar{r}\acute{\ })={{\rho }_{i}}(\bar{r}\acute{\ })\bar{v}(\bar{r}\acute{\ })</math>
 
im Feld einer externen magnetischen Induktion
<math>\bar{B}(\bar{r}\acute{\ })</math>
:
 
Spürt die Lorentzkraft
 
<math>\bar{F}=\int_{{}}^{{}}{{}}{{d}^{3}}r\acute{\ }\bar{j}(\bar{r}\acute{\ })\times \bar{B}(\bar{r}\acute{\ })</math>
 
Talyorentwicklung liefert:
 
<math>\begin{align}
& \bar{B}(\bar{r}\acute{\ })=\bar{B}(\bar{r})+\left[ \left( \bar{r}\acute{\ }-\bar{r} \right)\nabla  \right]\bar{B}(\bar{r})+.... \\
& \Rightarrow \bar{F}=\left[ \int_{{}}^{{}}{{}}{{d}^{3}}r\acute{\ }\bar{j}(\bar{r}\acute{\ }) \right]\times \bar{B}(\bar{r}\acute{\ })+\int_{{}}^{{}}{{}}{{d}^{3}}r\acute{\ }\bar{j}(\bar{r}\acute{\ })\times \left[ \left( \bar{r}\acute{\ }-\bar{r} \right)\nabla  \right]\bar{B}(\bar{r})+... \\
\end{align}</math>
 
im stationären Fall gilt wieder:
 
<math>\left[ \int_{{}}^{{}}{{}}{{d}^{3}}r\acute{\ }\bar{j}(\bar{r}\acute{\ }) \right]=0</math>
( keine Monopole)
Also:
 
<math>\begin{align}
& \bar{F}=\int_{{}}^{{}}{{}}{{d}^{3}}r\acute{\ }\bar{j}(\bar{r}\acute{\ })\times \left[ \left( \bar{r}\acute{\ } \right){{\nabla }_{r}} \right]\bar{B}(\bar{r})-\int_{{}}^{{}}{{}}{{d}^{3}}r\acute{\ }\bar{j}(\bar{r}\acute{\ })\times \left[ \left( {\bar{r}} \right){{\nabla }_{r}} \right]\bar{B}(\bar{r}) \\
& \int_{{}}^{{}}{{}}{{d}^{3}}r\acute{\ }\bar{j}(\bar{r}\acute{\ })\times \left[ \left( {\bar{r}} \right){{\nabla }_{r}} \right]\bar{B}(\bar{r})=0,da\int_{{}}^{{}}{{}}{{d}^{3}}r\acute{\ }\bar{j}(\bar{r}\acute{\ })=0 \\
& \Rightarrow \bar{F}=\int_{{}}^{{}}{{}}{{d}^{3}}r\acute{\ }\bar{j}(\bar{r}\acute{\ })\times \left[ \left( \bar{r}\acute{\ } \right){{\nabla }_{r}} \right]\bar{B}(\bar{r}) \\
& \left[ \left( \bar{r}\acute{\ } \right){{\nabla }_{r}} \right]\bar{B}(\bar{r})={{\nabla }_{r}}\left[ \left( \bar{r}\acute{\ } \right)\cdot \bar{B}(\bar{r}) \right]-\bar{r}\acute{\ }\times \left[ {{\nabla }_{r}}\times \bar{B}(\bar{r}) \right] \\
\end{align}</math>
 
Man fordert:
 
<math>\left[ {{\nabla }_{r}}\times \bar{B}(\bar{r}) \right]=0</math>
 
( Das externe Feld soll keine Stromwirbel im Bereich von
<math>\bar{j}(\bar{r}\acute{\ })</math>
haben:
 
<math>\begin{align}
& \bar{F}=\int_{{}}^{{}}{{}}{{d}^{3}}r\acute{\ }\bar{j}(\bar{r}\acute{\ })\times {{\nabla }_{r}}\left[ \left( \bar{r}\acute{\ } \right)\cdot \bar{B}(\bar{r}) \right] \\
& \bar{j}(\bar{r}\acute{\ })\times {{\nabla }_{r}}\left[ \left( \bar{r}\acute{\ } \right)\cdot \bar{B}(\bar{r}) \right]=-{{\nabla }_{r}}\times \left[ \left( \left( \bar{r}\acute{\ } \right)\cdot \bar{B}(\bar{r}) \right)\bar{j}(\bar{r}\acute{\ }) \right]+\left[ \left( \bar{r}\acute{\ } \right)\cdot \bar{B}(\bar{r}) \right]{{\nabla }_{r}}\times \bar{j}(\bar{r}\acute{\ }) \\
& {{\nabla }_{r}}\times \bar{j}(\bar{r}\acute{\ })=0 \\
& \Rightarrow \bar{F}=-\int_{{}}^{{}}{{}}{{d}^{3}}r\acute{\ }{{\nabla }_{r}}\times \left[ \left( \left( \bar{r}\acute{\ } \right)\cdot \bar{B}(\bar{r}) \right)\bar{j}(\bar{r}\acute{\ }) \right]=-{{\nabla }_{r}}\times \left( \bar{m}\times \bar{B}(\bar{r}) \right) \\
& \bar{F}=-{{\nabla }_{r}}\times \left( \bar{m}\times \bar{B}(\bar{r}) \right)=\left( \bar{m}\cdot {{\nabla }_{r}} \right)\bar{B}(\bar{r})=-{{\nabla }_{r}}\left( -\bar{m}\cdot \bar{B}(\bar{r}) \right) \\
\end{align}</math>
 
( Vergl. S. 34)
 
# <u>'''Die Maxwell-Gleichungen'''</u>
 
Ziel: Beschreibung der Dynamik der Felder
 
Methode: Erweiterung der elektrostatischen und magnetostatischen Feldgleichungen derart, dass allgemeine Invarianz- Prinzipien erfüllt sind !
 
Invarianz- Prinzipien sind / können sein:
 
<u>'''3.1 TCP- Invarianz'''</u>
 
Zeitumkehr T: t -> t´=-t
Ladungsumkehr / Konjugation :  C :  Q  Q´= - Q
Paritätsumkehr P :  r - >  r´= -r ( für den Ortsvektor)
 
<u>'''Die Zeitumkehr- Transformation'''</u>
 
<math>\begin{align}
& {{T}_{g}}:=\left\{ T-in\operatorname{var}iante\ ObservableA:TA=A \right\} \\
& =\left\{ \bar{r},d\bar{r},a:=\frac{{{d}^{2}}\bar{r}}{d{{t}^{2}}},m,q,\rho :=\begin{matrix}
\lim  \\
\Delta V\to 0  \\
\end{matrix}\frac{\Delta q}{\Delta V},\bar{F}=m\bar{a},\bar{E}=\frac{{\bar{F}}}{q},\Phi ... \right\} \\
\end{align}</math>
Diese Observablen sind "gerade" unter T
 
Daneben gibt es auch Observablen, die "ungerade" unter T sind:
 
<math>{{T}_{u}}:=\left\{ A:TA=-A \right\}=\left\{ \bar{v}:=\frac{d\bar{r}}{dt},\bar{j}=\rho \bar{v},\bar{B},\bar{A} \right\}</math>
 
Denn:
 
<math>\begin{align}
& \bar{F}=q\bar{v}\times \bar{B} \\
& \bar{F}\in {{T}_{g}},\bar{v}\in {{T}_{u}},q\in {{T}_{g}}\Rightarrow \bar{B}\in {{T}_{u}} \\
& \bar{B}=\nabla \times \bar{A},\nabla \in {{T}_{g}} \\
\end{align}</math>
 
Somit folgt jedoch vollständige T- Invarianz der elektromagnetischen Grundgleichungen:
 
<math>\begin{align}
& T:\left\{ {{\nabla }_{r}}\times \bar{E}=0 \right\}\to \left\{ {{\nabla }_{r}}\times \bar{E}=0 \right\} \\
& T:\left\{ {{\varepsilon }_{0}}{{\nabla }_{r}}\cdot \bar{E}=\rho  \right\}\to \left\{ {{\varepsilon }_{0}}{{\nabla }_{r}}\cdot \bar{E}=\rho  \right\} \\
& T:\left\{ {{\nabla }_{r}}\cdot \bar{B}=0 \right\}\to \left\{ -{{\nabla }_{r}}\cdot \bar{B}=0 \right\}\Leftrightarrow \left\{ {{\nabla }_{r}}\cdot \bar{B}=0 \right\} \\
& T:\left\{ \nabla \times \bar{B}={{\mu }_{0}}\bar{j} \right\}\to \left\{ -\nabla \times \bar{B}=-{{\mu }_{0}}\bar{j} \right\} \\
&  \\
\end{align}</math>
 
Kontinuitätsgleichung:
 
<math>T:\left\{ \frac{\partial }{\partial t}\rho +{{\nabla }_{r}}\cdot \bar{j}=0 \right\}\to \left\{ -\frac{\partial }{\partial t}\rho -{{\nabla }_{r}}\cdot \bar{j}=0 \right\}</math>
 
Die Gleichungen sind FORMINVARIANT !
 
'''Ladungsumkehr ( Konjugation)'''
 
<math>\begin{align}
& {{C}_{g}}:=\left\{ C-in\operatorname{var}iante\ ObservableA:CA=A \right\} \\
& {{C}_{g}}=\left\{ \bar{F},m,\bar{r},\bar{v},\bar{a} \right\} \\
\end{align}</math>
 
sind gerade unter C
'''Ungerade unter c sind:'''
 
<math>\begin{align}
& {{C}_{u}}:=\left\{ A:CA=-A \right\}=\left\{ \bar{E}=\frac{1}{q}\bar{F},\bar{B},\bar{j},\rho  \right\} \\
& \bar{F}=q\bar{v}\times \bar{B} \\
\end{align}</math>
 
* C- Invarianz der Elektro- Magnetostatik:
 
<math>\begin{align}
& C:\left\{ {{\nabla }_{r}}\times \bar{E}=0 \right\}\to \left\{ -{{\nabla }_{r}}\times \bar{E}=0 \right\} \\
& C:\left\{ {{\varepsilon }_{0}}{{\nabla }_{r}}\cdot \bar{E}=\rho  \right\}\to \left\{ -{{\varepsilon }_{0}}{{\nabla }_{r}}\cdot \bar{E}=-\rho  \right\} \\
& C:\left\{ {{\nabla }_{r}}\cdot \bar{B}=0 \right\}\to \left\{ -{{\nabla }_{r}}\cdot \bar{B}=0 \right\} \\
& C:\left\{ \nabla \times \bar{B}={{\mu }_{0}}\bar{j} \right\}\to \left\{ -\nabla \times \bar{B}=-{{\mu }_{0}}\bar{j} \right\} \\
\end{align}</math>
 
<math>C:\left\{ \frac{\partial }{\partial t}\rho +{{\nabla }_{r}}\cdot \bar{j}=0 \right\}\to \left\{ -\frac{\partial }{\partial t}\rho -{{\nabla }_{r}}\cdot \bar{j}=0 \right\}</math>
 
<u>'''Paritätsumkehr: Räumliche Spiegelung/ Inversion'''</u>
 
Vertauschung: rechts <-> links
 
 
Man unterscheidet:
 
<math>P\bar{r}=-\bar{r}</math>
-> polarer Vektor
und
 
<math>P\left( \bar{a}\times \bar{b} \right)=\left( -\bar{a}\times -\bar{b} \right)=\left( \bar{a}\times \bar{b} \right)</math>
P- invariant = " axialer Vektor", sogenannter Pseudovektor !!
 
 
 
Seien:
 
<math>\bar{a},\bar{b}</math>
polar,
<math>\bar{w},\bar{\sigma }</math>
axial
Dann ist
 
<math>\begin{align}
& \bar{a}\times \bar{w}\quad polar \\
& \bar{a}\times \bar{b},\bar{w}\times \bar{\sigma }\quad axial \\
& \bar{a}\bar{b}\ skalar:P(\bar{a}\bar{b})=\bar{a}\bar{b} \\
& \bar{w}\bar{\sigma }\ pseudoskalarP(\bar{w}\bar{\sigma })=-\bar{w}\bar{\sigma } \\
\end{align}</math>
 
<math>\begin{align}
& {{C}_{g}}:=\left\{ C-in\operatorname{var}iante\ ObservableA:CA=A \right\} \\
& {{C}_{g}}=\left\{ \bar{F},m,\bar{r},\bar{v},\bar{a} \right\} \\
\end{align}</math>
 
Wegen
 
<math>\begin{align}
& \bar{F}=q\bar{v}\times \bar{B} \\
& \bar{F}\in {{P}_{u}} \\
& q\in {{P}_{g}} \\
& \bar{v}\in {{P}_{u}} \\
\end{align}</math>
 
ungerade Parität dagegen:
 
<math>{{P}_{u}}=\left\{ polareVektoren,\bar{r},d\bar{r},\bar{v},\bar{a},\bar{F},\bar{E}=\frac{1}{q}\bar{F},\bar{j}=\rho \bar{v},\bar{A},Pseudoskalare\quad \nabla \cdot \bar{B} \right\}</math>
 
Wegen
 
<math>\begin{align}
& \bar{B}=\nabla \times \bar{A} \\
& \nabla \in {{P}_{u}} \\
& \bar{B}\in {{P}_{g}} \\
\end{align}</math>
 
P- Invarianz der Elektro- / Magnetostatik:
 
<math>\begin{align}
& P:\left\{ {{\nabla }_{r}}\times \bar{E}=0 \right\}\to \left\{ {{\nabla }_{r}}\times \bar{E}=0 \right\} \\
& P:\left\{ {{\varepsilon }_{0}}{{\nabla }_{r}}\cdot \bar{E}=\rho  \right\}\to \left\{ {{\varepsilon }_{0}}{{\nabla }_{r}}\cdot \bar{E}=\rho  \right\} \\
& P:\left\{ {{\nabla }_{r}}\cdot \bar{B}=0 \right\}\to \left\{ -{{\nabla }_{r}}\cdot \bar{B}=0 \right\} \\
& P:\left\{ \nabla \times \bar{B}={{\mu }_{0}}\bar{j} \right\}\to \left\{ -\nabla \times \bar{B}=-{{\mu }_{0}}\bar{j} \right\} \\
\end{align}</math>
 
<math>P:\left\{ \frac{\partial }{\partial t}\rho +{{\nabla }_{r}}\cdot \bar{j}=0 \right\}\to \left\{ \frac{\partial }{\partial t}\rho +{{\nabla }_{r}}\cdot \bar{j}=0 \right\}</math>
 
Nebenbemerkung: Gäbe es magnetische Ladungen, dann wären sie pseudoskalare
Außerdem ( Weinberg e.a.) : Schwache Wechselwirkung verletzt die Paritätserhaltung!
 
<u>'''3.2 Maxwell- Gleichungen im Vakuum'''</u>
 
Die Forderungen an dynamische Gleichungen für zeitartige Felder
<math>\bar{E}(\bar{r},t),\bar{B}(\bar{r},t)</math>
lauten:
1) im quasistatischen Grenzfall sollen die statischen MWGl herauskommen:
 
<math>\begin{align}
& {{\nabla }_{r}}\times \bar{E}=0 \\
& {{\varepsilon }_{0}}{{\nabla }_{r}}\cdot \bar{E}-\rho =0 \\
& {{\nabla }_{r}}\cdot \bar{B}=0 \\
& \nabla \times \bar{B}-{{\mu }_{0}}\bar{j}=0 \\
\end{align}</math>
 
2) die Gleichungen sollen linear in
<math>\bar{E}(\bar{r},t),\bar{B}(\bar{r},t)</math>
sein, um das Superpositionsprinzip zu erfüllen !
Die Gleichungen sollen 1. Ordnung in t sein ( um das Kausalitätsprinzip zu erfüllen !)
 
Die linke Seite der Maxwellgleichungen ( oben) soll zur Zeit t=0 den Zustand für t> 0 vollständig festlegen !!
 
Somit sind
 
<math>\begin{align}
& {{\nabla }_{r}}\times \bar{E}={{a}_{1}}\dot{\bar{E}}+{{b}_{1}}\dot{\bar{B}} \\
& \nabla \times \bar{B}-{{\mu }_{0}}\bar{j}={{a}_{2}}\dot{\bar{E}}+{{b}_{2}}\dot{\bar{B}} \\
& {{\varepsilon }_{0}}{{\nabla }_{r}}\cdot \bar{E}-\rho =0 \\
& {{\nabla }_{r}}\cdot \bar{B}=0 \\
\end{align}</math>
 
Dies sind 6 Vektorgleichungen, die
<math>\bar{E}(\bar{r},t),\bar{B}(\bar{r},t)</math>
für t> 0 festlegen und 2 skalare Gleichungen
 
3) Wir fordern TCP- Invarianz:
 
<math>\begin{align}
& {{T}_{g}}oder\ {{P}_{g}}\Rightarrow {{a}_{1}}=0 \\
& {{T}_{u}}oder\ {{P}_{u}}\Rightarrow {{b}_{2}}=0 \\
\end{align}</math>
 
Also bleibt:
 
<math>\begin{align}
& {{\nabla }_{r}}\times \bar{E}={{b}_{1}}\dot{\bar{B}} \\
& \nabla \times \bar{B}-{{\mu }_{0}}\bar{j}={{a}_{2}}\dot{\bar{E}} \\
& {{\varepsilon }_{0}}{{\nabla }_{r}}\cdot \bar{E}-\rho =0 \\
& {{\nabla }_{r}}\cdot \bar{B}=0 \\
\end{align}</math>
 
4) Ladungserhaltung:
 
<math>\begin{align}
& 0=\frac{\partial }{\partial t}\left( {{\varepsilon }_{0}}{{\nabla }_{r}}\cdot \bar{E}-\rho  \right)={{\varepsilon }_{0}}{{\nabla }_{r}}\cdot \dot{\bar{E}}-\dot{\rho }=\frac{{{\varepsilon }_{0}}}{{{a}_{2}}}\nabla \cdot \left( \nabla \times \bar{B}-{{\mu }_{0}}\bar{j} \right)-\dot{\rho } \\
& \frac{{{\varepsilon }_{0}}}{{{a}_{2}}}\nabla \cdot \nabla \times \bar{B}=0 \\
& \Rightarrow \frac{{{\varepsilon }_{0}}}{{{a}_{2}}}\nabla \cdot \left( {{\mu }_{0}}\bar{j} \right)-\dot{\rho }=0 \\
& \Rightarrow {{a}_{2}}={{\varepsilon }_{0}}{{\mu }_{0}} \\
\end{align}</math>
 
Unter Verwendung der Kontinuitätsgleichung !
Somit ( vergl. S. 32, §2.3  folgt die Verschiebungsstromdichte
<math>{{\varepsilon }_{0}}\dot{\bar{E}}</math>
 
5) Lorentzkraft
 
<math>\bar{F}=q\bar{v}\times \bar{B}</math>
soll aus einem Extremalprinzip, ergo dem Hamiltonschen Prinzip ableitbar sein.
Suche also eine Lagrange- Funktion
 
<math>L(\bar{r},\bar{v},t)</math>
so dass die Lagrangegleichung
 
<math>\frac{d}{dt}\left( \frac{\partial L(\bar{r},\bar{v},t)}{\partial {{v}_{k}}} \right)-\frac{\partial L(\bar{r},\bar{v},t)}{\partial {{x}_{k}}}=0</math>
 
die nichtrelativistische Bewegungsgleichung
 
<math>m\ddot{\bar{r}}=q\left[ \bar{E}(\bar{r},t)+\bar{v}\times \bar{B}(\bar{r},t) \right]</math>
 
ergibt !
 
Lösung:
 
<math>L=\frac{m}{2}{{v}^{2}}+q\left[ \bar{v}\bar{A}(\bar{r},t)-\Phi (\bar{r},t) \right]</math>
 
Tatsächlich gilt
 
<math>{{p}_{k}}=\frac{\partial L(\bar{r},\bar{v},t)}{\partial {{v}_{k}}}=m{{v}_{k}}+q{{A}_{k}}(\bar{r},t)</math>
= kanonischer Impuls
 
<math>\frac{d}{dt}\frac{\partial L(\bar{r},\bar{v},t)}{\partial {{v}_{k}}}=m{{\ddot{x}}_{k}}+q\frac{d}{dt}{{A}_{k}}(\bar{r},t)</math>
 
Dabei ist die Zeitableitung von A als totales Differenzial entlang einer Bahn
<math>\bar{r}</math>
zu sehen !
 
<math>\begin{align}
& \frac{d}{dt}\frac{\partial L(\bar{r},\bar{v},t)}{\partial {{v}_{k}}}=m{{{\ddot{x}}}_{k}}+q\left( \frac{\partial }{\partial t}{{A}_{k}}(\bar{r},t)+\frac{\partial {{A}_{k}}(\bar{r},t)}{\partial {{x}_{l}}}\frac{\partial {{x}_{l}}}{\partial t} \right)=m{{{\ddot{x}}}_{k}}+q\left( \frac{\partial }{\partial t}+\bar{v}\cdot \nabla  \right){{A}_{k}}(\bar{r},t) \\
& \frac{\partial L(\bar{r},\bar{v},t)}{\partial {{x}_{k}}}=q\left[ \frac{\partial }{\partial {{x}_{k}}}\left( \bar{v}\bar{A} \right)-\frac{\partial }{\partial {{x}_{k}}}\Phi  \right] \\
& \Rightarrow 0=\frac{d}{dt}\frac{\partial L(\bar{r},\bar{v},t)}{\partial {{v}_{k}}}-\frac{\partial L(\bar{r},\bar{v},t)}{\partial {{x}_{k}}}=m{{{\ddot{x}}}_{k}}+q\left( \frac{\partial }{\partial t}+\bar{v}\cdot \nabla  \right){{A}_{k}}(\bar{r},t)-q\left[ \frac{\partial }{\partial {{x}_{k}}}\left( \bar{v}\bar{A} \right)-\frac{\partial }{\partial {{x}_{k}}}\Phi  \right] \\
& =m{{{\ddot{x}}}_{k}}+q\frac{\partial }{\partial t}{{A}_{k}}(\bar{r},t)+q\left[ \left( \bar{v}\cdot \nabla  \right){{A}_{k}}(\bar{r},t)-\frac{\partial }{\partial {{x}_{k}}}\left( \bar{v}\bar{A} \right) \right]+q\frac{\partial }{\partial {{x}_{k}}}\Phi  \\
& \left[ \left( \bar{v}\cdot \nabla  \right){{A}_{k}}(\bar{r},t)-\frac{\partial }{\partial {{x}_{k}}}\left( \bar{v}\bar{A} \right) \right]=-{{\left[ \bar{v}\times \left( \nabla \times \bar{A} \right) \right]}_{k}} \\
& \Rightarrow 0=m\ddot{\bar{r}}+q\frac{\partial }{\partial t}A(\bar{r},t)-q\left[ \bar{v}\times \left( \nabla \times \bar{A} \right) \right]+q\nabla \Phi =m\ddot{\bar{r}}+q\left[ \frac{\partial }{\partial t}A(\bar{r},t)+\nabla \Phi -\left[ \bar{v}\times \left( \nabla \times \bar{A} \right) \right] \right] \\
\end{align}</math>
 
Vergleich mit der Lorentzkraft liefert:
 
<math>\begin{align}
& \bar{E}(\bar{r},t)=-\frac{\partial }{\partial t}A(\bar{r},t)-\nabla \Phi  \\
& \bar{B}(\bar{r},t)=\nabla \times A(\bar{r},t) \\
\end{align}</math>
 
und:
 
<math>\begin{align}
& \nabla \times \bar{E}(\bar{r},t)=-\frac{\partial }{\partial t}\nabla \times A(\bar{r},t)-\nabla \times \nabla \Phi  \\
& \nabla \times A(\bar{r},t)=\bar{B}(\bar{r},t) \\
& \nabla \times \nabla \Phi =0 \\
& \Rightarrow {{b}_{1}}=-1 \\
\end{align}</math>
 
<u>'''Vollständige ( zeitabhängige) Maxwellgleichungen im Vakuum'''</u>
 
mit den neuen Feldgrößen
 
<math>\bar{D}(\bar{r},t):={{\varepsilon }_{0}}\bar{E}(\bar{r},t)</math>
dielektrische Verschiebung
und
 
<math>\bar{H}(\bar{r},t):=\frac{1}{{{\mu }_{0}}}\bar{B}(\bar{r},t)</math>
, Magnetfeld
ergibt sich:
 
<math>\begin{align}
& {{\nabla }_{r}}\times \bar{E}+\dot{\bar{B}}=0 \\
& {{\nabla }_{r}}\cdot \bar{B}=0 \\
& {{\nabla }_{r}}\cdot \bar{D}=\rho  \\
& {{\nabla }_{r}}\times \bar{H}-\dot{\bar{D}}=\bar{j} \\
\end{align}</math>
 
Dabei sind
 
<math>\begin{align}
& {{\nabla }_{r}}\times \bar{E}+\dot{\bar{B}}=0 \\
& {{\nabla }_{r}}\cdot \bar{B}=0 \\
\end{align}</math>
die homogenen Gleichungen, die die Wechselwirkung einer Punktladung mit gegebenen Feldern
<math>\bar{E},\bar{B}</math>
beschreiben
und
 
<math>\begin{align}
& {{\nabla }_{r}}\cdot \bar{D}=\rho  \\
& {{\nabla }_{r}}\times \bar{H}-\dot{\bar{D}}=\bar{j} \\
\end{align}</math>
die inhomogenen Gleichungen, die Erzeugung der Felder
<math>\bar{D},\bar{H}</math>
durch gegebene Ladungen und Ströme
 
Im Gauß- System:
 
<math>\begin{align}
& {{\nabla }_{r}}\times \bar{E}+\frac{1}{c}\dot{\bar{B}}=0 \\
& {{\nabla }_{r}}\cdot \bar{B}=0 \\
& {{\nabla }_{r}}\cdot \bar{E}=4\pi \rho  \\
& {{\nabla }_{r}}\times \bar{B}-\dot{\bar{E}}=\frac{4\pi }{c}\bar{j} \\
\end{align}</math>
 
Mit
 
<math>\begin{align}
& \bar{E}=-\frac{1}{c}\frac{\partial }{\partial t}\bar{A}-\nabla \Phi  \\
& \bar{B}=\nabla \times \bar{A} \\
& \bar{D}=\bar{E} \\
& \bar{H}=\bar{B} \\
\end{align}</math>
 
im Vakuum !
 
<u>'''Induktionsgesetz :'''</u>
 
Die Maxwellgleichung
 
<math>{{\nabla }_{r}}\times \bar{E}=-\dot{\bar{B}}</math>
wird über eine ortsfeste Fläche F ( nicht geschlossen) mit Rand
<math>\partial F</math>
integriert:
 
<math>\begin{align}
& \int_{F}^{{}}{d\bar{f}}\left( {{\nabla }_{r}}\times \bar{E} \right)=-\int_{F}^{{}}{d\bar{f}}\dot{\bar{B}} \\
& \Rightarrow \oint\limits_{\partial F}{{}}d\bar{s}\bar{E}=-\frac{\partial }{\partial t}\int_{F}^{{}}{d\bar{f}}\bar{B} \\
\end{align}</math>
 
Wobei Differenziation und Integration genau dann vertauscht werden kann, wenn die Variablen unabhängig sind, also die Fläche ortsfest !
 
Damit folgt die integrale Form dieser Maxwellgleichung
 
<math>\begin{align}
& \oint\limits_{\partial F}{{}}d\bar{s}\bar{E}=-\frac{\partial }{\partial t}\Phi (t) \\
& \Phi (t)=\int_{F}^{{}}{d\bar{f}}\bar{B}=\oint\limits_{\partial F}{{}}d\bar{s}\cdot \bar{A} \\
\end{align}</math>
 
Der magnetische Fluß !
 
Der magnetische Fluß
<math>\Phi (t)</math>
hängt nur vom Rand
<math>\partial F</math>
der Fläche ab !
 
Seien F und F´ zwei Flächen mit dem selben Rand, die das Volumen V einschließen :
 
 
<math>\begin{align}
& \int_{F}^{{}}{d\bar{f}}\bar{B}-\int_{F\acute{\ }}^{{}}{d\bar{f}}\bar{B}=\oint\limits_{\partial V}{{}}d\bar{f}\bar{B}=\int_{V}^{{}}{{{d}^{3}}r}\nabla \cdot \bar{B}=0 \\
& \nabla \cdot \bar{B}=0 \\
\end{align}</math>
 
Die Potenzialdifferenz bei einem Umlauf um
<math>\partial F</math>
beträgt:
 
<math>\Delta \Phi :=-\oint\limits_{\partial F}{{}}d\bar{s}\bar{E}</math>
Dies entspricht einer induzierten Spannung ( als Wirbelfeld)
Somit folgt das
 
Faradaysche Induktionsgesetz:
 
<math>\Delta \Phi =\frac{\partial }{\partial t}{{\Phi }_{mag}}</math>
 
mit dem magnetischen Fluß
 
<math>{{\Phi }_{mag}}</math>
 
<u>'''Die Lenzsche Regel:'''</u>
 
 
<math>\begin{align}
& \dot{\bar{B}}\to \bar{E} \\
& \nabla \times \bar{E}=-\bar{B} \\
\end{align}</math>
induziert
 
<math>\bar{E}\to \bar{j}\tilde{\ }\bar{E}</math>
Ladungsverschiebung/- Bewegung
 
<math>\begin{align}
& \bar{j}\to \bar{H} \\
& {{\nabla }_{r}}\times \bar{H}=\bar{j} \\
\end{align}</math>
erzeugt
Also:
<math>\bar{H}</math>
ist
<math>\dot{\bar{B}}</math>
entgegengerichtet !
 
<u>'''Zusammenfassung'''</u>
 
 
<math>\oint\limits_{\partial F}{{}}d\bar{s}\bar{E}=-\frac{\partial }{\partial t}\Phi (t)</math>
Zirkulation des elektrischen Feldes entlang einer geschlossenen Linie ist gleich der zeitlichen Abnahme des eingeschlossenen magnetischen Flusses:
<math>\Phi (t)=\int_{F}^{{}}{d\bar{f}}\bar{B}=\oint\limits_{\partial F}{{}}d\bar{s}\cdot \bar{A}</math>
 
<math>\oint\limits_{\partial V}{{}}d\bar{f}\bar{B}=0</math>
Der Nettofluss des magnetischen Feldes durch eine geschlossene Oberfläche ist NULL
 
<math>\oint\limits_{\partial V}{{}}d\bar{f}\bar{E}=\frac{Q}{{{\varepsilon }_{0}}}</math>
Der Fluß des elektrischen Feldes durch
<math>\partial V</math>
ist gleich der eingeschlossenen Ladung
<math>\frac{Q}{{{\varepsilon }_{0}}}</math>
 
<math>\oint\limits_{\partial F}{{}}d\bar{s}\cdot \bar{H}=\int_{F}^{{}}{{}}d\bar{f}\cdot \dot{\bar{D}}+I</math>
Die Zirkulation des magnetischen Feldes entlang einer eingeschlossenen Linie ist gleich der Summe aus dem dielektrischen Verschiebungsstrom
<math>\int_{F}^{{}}{{}}d\bar{f}\cdot \dot{\bar{D}}</math>
und dem Konvektionsstrom
<math>I=\int_{F}^{{}}{{}}d\bar{f}\cdot \bar{j}</math>
 
 
 
<u>'''3.4 Energiebilanz'''</u>
 
Die Maxwell- Gleichungen enthalten die Kontinuitätsgleichung für die elektrische Ladung
 
<math>\begin{align}
& \dot{\rho }+\nabla \cdot \bar{j}=0 \\
& \dot{\rho }+\nabla \cdot \bar{j}=\nabla \cdot \left( \dot{\bar{D}}+\bar{j} \right)=\nabla \cdot \left( \nabla \times \bar{H} \right)=0 \\
\end{align}</math>
 
'''Frage:'''
 
Enthalten die Maxwell- Gleichungen weitere Erhaltungssätze für extensive physikalische Observablen, wie Energie, Impuls, Drehimpuls.
( Extensiv: Additiv bei Systemzusammensetzung)
 
<u>'''Energietransport durch das elektromagnetische Feld:'''</u>
 
<math>\begin{align}
& {{\nabla }_{r}}\times \bar{E}+\dot{\bar{B}}=0\left. {} \right|\cdot \bar{H} \\
& \nabla \times \bar{H}-\dot{\bar{D}}=\bar{j}\left. {} \right|\cdot \bar{E} \\
& \Rightarrow \bar{H}\cdot \left( \nabla \times \bar{E} \right)-\bar{E}\cdot \left( \nabla \times \bar{H} \right)+\bar{H}\cdot \frac{\partial }{\partial t}\bar{B}+\bar{E}\cdot \frac{\partial }{\partial t}\bar{D}=-\bar{j}\cdot \bar{E} \\
& \bar{H}\cdot \left( \nabla \times \bar{E} \right)-\bar{E}\cdot \left( \nabla \times \bar{H} \right)=\nabla \cdot \left( \bar{E}\times \bar{H} \right) \\
& \bar{H}\cdot \frac{\partial }{\partial t}\bar{B}=\frac{1}{{{\mu }_{0}}}\bar{B}\frac{\partial }{\partial t}\bar{B}=\frac{\partial }{\partial t}\left( \frac{1}{2{{\mu }_{0}}}{{{\bar{B}}}^{2}} \right) \\
& \bar{E}\cdot \frac{\partial }{\partial t}\bar{D}={{\varepsilon }_{0}}\bar{E}\cdot \frac{\partial }{\partial t}\bar{E}=\frac{\partial }{\partial t}\left( \frac{{{\varepsilon }_{0}}}{2}{{{\bar{E}}}^{2}} \right) \\
\end{align}</math>
 
Also:
 
<math>\frac{\partial }{\partial t}w+\nabla \cdot \bar{S}=-\bar{j}\cdot \bar{E}</math>
 
Als Kontinuitätsgleichung ( Bilanzgleichung) für den Energietransport
 
mit
 
<math>w:=\frac{\partial }{\partial t}\left( \frac{1}{2{{\mu }_{0}}}{{{\bar{B}}}^{2}} \right)+\frac{\partial }{\partial t}\left( \frac{{{\varepsilon }_{0}}}{2}{{{\bar{E}}}^{2}} \right)=\frac{1}{2}\left( \bar{E}\cdot \bar{D}+\bar{B}\cdot \bar{H} \right)</math>
 
Als Energiedichte des elektromagnetischen Feldes
Remember:
 
Elektrostatik:
 
<math>\frac{1}{2}\bar{E}\cdot \bar{D}</math>
 
Magnetostatik:
 
<math>\frac{1}{2}\bar{B}\cdot \bar{H}</math>
 
<math>\bar{S}:=\bar{E}\times \bar{H}</math>
als Energiestromdichte des elektromagnetischen Feldes ( Poynting- Vektor)
 
<math>\sigma =-\bar{j}\cdot \bar{E}</math>
als Quelldichte der Feldenergie ( Leistungsdichte)
 
<math>\bar{j}\cdot \bar{E}>0</math>
bedingt die Abnahme der Feldenergie bei
<math>(\bar{r},t)</math>
 
<math>\bar{j}\cdot \bar{E}<0</math>
bedingt die Zunahme der Feldenergie bei
<math>(\bar{r},t)</math>
 
Beispiel: Beschleunigung von Teilchen durch die Felder
<math>\bar{E},\bar{B}</math>
:
 
Kraft auf die Ladung q:
<math>\bar{F}=q\left( \bar{E}+\bar{v}\times \bar{B} \right)</math>
 
Kraftdichte:
<math>\bar{f}=\rho \left( \bar{E}+\bar{v}\times \bar{B} \right)</math>
 
Als Leistungsdichte der Felder auf die Ladungsdichte
<math>\rho </math>
folgt:
 
<math>\begin{align}
& \bar{f}\bar{v}=\rho \bar{v}\left( \bar{E}+\bar{v}\times \bar{B} \right)=\rho \bar{v}\bar{E}+\rho \bar{v}\left( \bar{v}\times \bar{B} \right) \\
& \rho \bar{v}\left( \bar{v}\times \bar{B} \right)=0 \\
& \Rightarrow \bar{f}\bar{v}=\rho \bar{v}\bar{E}=\bar{j}\bar{E} \\
\end{align}</math>
 
Das Magnetfeld leistet keine Arbeit, da die Kraft senkrecht auf die Geschwindigkeit steht
 
Es verbleibt die Kraftdichte, die vom Feld auf Ladungen übertragen wird ( sogenannte Verlustdichte der Feldenergie)
 
Also ist die Feldenergie keine Erhaltungsgröße !!
 
<u>'''Beispiel: '''</u> Ohmsches Gesetz:
 
<math>\sigma \cdot \bar{E}=\bar{j}</math>
mit der konstanten LEITFÄHIGKEIT
<math>\sigma >0</math>
( nicht wie oben Oberflächenladungsdichte)
 
Das Ohmsche Gesetz ist ein phänomenologisches MATERIALGESETZ.
Es gilt in Metallen und Halbleitern für hinreichend kleine Felder
<math>\bar{E}</math>
 
Die Energiebilanz lautet:
 
<math>\frac{\partial }{\partial t}w+\nabla \cdot \bar{S}=-\sigma \cdot {{\bar{E}}^{2}}<0</math>
 
Das heißt: Es gibt stets den VERLUST von Feldenergie !
Eine Konsequenz des 2. Hauptsatz der Thermodynamik
Im Gegensatz zur Elektrodynamik ist das Ohmsche Gesetz also nicht zeitumkehrinvariant !
 
Das bedeutet:
 
<math>\begin{align}
& t\to -t \\
& \bar{j}\to -\bar{j} \\
& aber \\
& \bar{E}\to \bar{E} \\
\end{align}</math>
 
<math>\sigma \cdot {{\bar{E}}^{2}}>0</math>
wird dann als Joulsche Wärme im Leiter dissipiert
 
<u>'''2. Beispiel:'''</u>
 
Antennenstrahlung ( offenes System)
 
<math>\bar{j}</math>
in der metallischen Antenne ist dem Wechselfeld
<math>\bar{E}</math>
außerhalb entgegengesetzt.
 
<math>\Rightarrow \bar{j}\bar{E}<0</math>
 
<math>\Rightarrow </math>
Energiegewinn des Feldes
 
<u>'''3.5 Impulsbilanz'''</u>
 
Aus den Maxwell Gleichungen folgt eine weitere Bilanzgleichung für den Impulstransport durch das elektromagnetische Feld:
 
<math>\begin{align}
& \frac{\partial }{\partial t}\left( \bar{D}\times \bar{B} \right)=\dot{\bar{D}}\times \bar{B}+\bar{D}\times \dot{\bar{B}} \\
& \dot{\bar{D}}=\nabla \times \bar{H}-\bar{j} \\
& \dot{\bar{B}}=-\nabla \times \bar{E} \\
& \Rightarrow \frac{\partial }{\partial t}\left( \bar{D}\times \bar{B} \right)=-\frac{1}{{{\mu }_{0}}}\bar{B}\times \left( \nabla \times \bar{B} \right)-\bar{j}\times \bar{B}-{{\varepsilon }_{o}}\bar{E}\times \left( \nabla \times \bar{E} \right) \\
\end{align}</math>
 
Mittels
 
<math>\begin{align}
& \bar{B}\times \left( \nabla \times \bar{B} \right)=\frac{1}{2}\nabla \left( \bar{B}\cdot \bar{B} \right)-\left( \bar{B}\cdot \nabla  \right)\bar{B} \\
& \bar{B}\times \left( \nabla \times \bar{B} \right)=\nabla \cdot \left\{ \left( 1 \right)\frac{1}{2}\left( \bar{B}\cdot \bar{B} \right)-\bar{B}\otimes \bar{B} \right\}+\bar{B}\left( \nabla \cdot \bar{B} \right)=\nabla \cdot \left\{ \left( 1 \right)\frac{1}{2}\left( \bar{B}\cdot \bar{B} \right)-\bar{B}\otimes \bar{B} \right\} \\
& \bar{B}\left( \nabla \cdot \bar{B} \right)=0 \\
\end{align}</math>
 
Dabei bezeichnet
<math>\left( 1 \right)</math>
den Einheitstensor 1. Stufe und
<math>\bar{B}\otimes \bar{B}</math>
das Tensorprodukt (dyadisches Produkt).
Außerdem ist
<math>\nabla \cdot \left\{ \left( 1 \right)\frac{1}{2}\left( \bar{B}\cdot \bar{B} \right)-\bar{B}\otimes \bar{B} \right\}</math>
die Divergenz eines Tensors
<math>\left( T \right)</math>
zweiter Stufe.
In Komponenten gilt:
<math>{{\left( \nabla \cdot T \right)}_{\beta }}:={{\partial }_{\alpha }}{{T}_{\alpha }}_{\beta }</math>
 
Analog:
 
<math>\begin{align}
& \bar{E}\times \left( \nabla \times \bar{E} \right)=\nabla \cdot \left\{ \left( 1 \right)\frac{1}{2}\left( \bar{E}\cdot \bar{E} \right)-\bar{E}\otimes \bar{E} \right\}+\bar{E}\left( \nabla \cdot \bar{E} \right)=\nabla \cdot \left\{ \left( 1 \right)\frac{1}{2}\left( \bar{E}\cdot \bar{E} \right)-\bar{E}\otimes \bar{E} \right\}+\bar{E}\frac{\rho }{{{\varepsilon }_{0}}} \\
& \Rightarrow \frac{\partial }{\partial t}\left( \bar{D}\times \bar{B} \right)+\nabla \cdot \left\{ \left( 1 \right)\frac{1}{2}\left( {{\varepsilon }_{0}}{{E}^{2}}+\frac{1}{{{\mu }_{0}}}{{B}^{2}} \right)-{{\varepsilon }_{0}}\bar{E}\otimes \bar{E}-\frac{1}{{{\mu }_{0}}}\bar{B}\otimes \bar{B} \right\}=-\left( \bar{E}\rho +\bar{j}\times \bar{B} \right) \\
\end{align}</math>
 
Dabei beschreibt
 
<math>\left( \bar{E}\rho +\bar{j}\times \bar{B} \right)</math>
den Kraftdichtefluß, der von den Feldern auf  Ströme und Ladungen übertragen wird
 
Als Bilanzgleichung für den Impulstransport ergibt sich:
 
<math>\begin{align}
& \frac{\partial }{\partial t}\bar{g}+\nabla \cdot \left( {\bar{\bar{T}}} \right)=-\left( \bar{E}\rho +\bar{j}\times \bar{B} \right) \\
& \bar{g}:=\left( \bar{D}\times \bar{B} \right) \\
& \left\{ \left( 1 \right)\frac{1}{2}\left( {{\varepsilon }_{0}}{{E}^{2}}+\frac{1}{{{\mu }_{0}}}{{B}^{2}} \right)-{{\varepsilon }_{0}}\bar{E}\otimes \bar{E}-\frac{1}{{{\mu }_{0}}}\bar{B}\otimes \bar{B} \right\}:=\left( {\bar{\bar{T}}} \right) \\
\end{align}</math>
 
Dabei ist
 
<math>\bar{g}:=\left( \bar{D}\times \bar{B} \right)</math>
die Impulsdichte des Feldes.
Nach Newton gilt:
 
<math>\begin{align}
& \frac{d}{dt}\bar{p}=\bar{F} \\
& \Rightarrow \frac{d}{dt}\bar{g}=\bar{f} \\
\end{align}</math>
 
Es ergibt sich
 
<math>\left\{ \left( 1 \right)\frac{1}{2}\left( \bar{E}\cdot \bar{D}+\bar{B}\cdot \bar{H} \right)-\bar{E}\otimes \bar{D}-\bar{B}\otimes \bar{H} \right\}:=\left( {\bar{\bar{T}}} \right)</math>
 
Als der
IMPULSSTROMDICHTE- Tensor des Feldes ( Maxwellscher Spannungstensor)
 
in Komponenten:
 
<math>{{T}_{\alpha \beta }}=\left\{ {{\delta }_{\alpha \beta }}\frac{1}{2}\left( \bar{E}\cdot \bar{D}+\bar{B}\cdot \bar{H} \right)-{{{\bar{E}}}_{\alpha }}{{{\bar{D}}}_{\beta }}-{{{\bar{B}}}_{\alpha }}{{{\bar{H}}}_{\beta }} \right\}</math>
 
Dies ist die Stromrichtung der
<math>\beta </math>
- Komponente der Impulsdichte in
<math>\alpha </math>
- Richtung.
Eine Impulsdichte, die in eine feste Richtung weist wird somit entlang einer anderen Richtung transportiert !
 
<math>tr\left( {\bar{\bar{T}}} \right)={{T}_{\alpha \alpha }}=w</math>
Energiedichte
Außerdem ist T symmetrisch:
 
<math>{{T}_{\alpha \beta }}={{T}_{\beta \alpha }}</math>
 
Die komponentenweise Darstellung der Bilanzgleichung
 
<math>\frac{\partial }{\partial t}{{g}_{\beta }}+\frac{\partial }{\partial {{x}_{\alpha }}}{{T}_{\alpha \beta }}=-{{f}_{\beta }}</math>
 
beschriebt den Impulsaustausch zwischen Feld und geladenen Teilchen.
 
'''Bemerkung:'''
Eine analoge Bilanzgleichung gibt es für die Drehimpulsdichte des Feldes. Sie beschreibt den Drehimpulsaustausch zwischen Feld und geladenen Teilchen !
 
<u>'''3.6 Eichinvarianz'''</u>
 
Die Felder
<math>\bar{E},\bar{B}</math>
werden durch die Potenziale
<math>\Phi \left( \bar{r},t \right),\bar{A}\left( \bar{r},t \right)</math>
dargestellt.:
 
<math>\begin{align}
& \bar{E}=-\nabla \Phi \left( \bar{r},t \right)-\frac{\partial }{\partial t}\bar{A}\left( \bar{r},t \right) \\
& \bar{B}=\nabla \times \bar{A}\left( \bar{r},t \right) \\
\end{align}</math>
 
Dabei drängt sich die Frage auf, welche die allgemeinste Transformation
 
<math>\begin{align}
& \Phi \left( \bar{r},t \right)\to \Phi \acute{\ }\left( \bar{r},t \right) \\
& \bar{A}\left( \bar{r},t \right)\to \bar{A}\acute{\ }\left( \bar{r},t \right) \\
\end{align}</math>
 
ist, welche die Felder E und B unverändert läßt.
 
Also:
 
<math>\begin{align}
& \bar{E}=-\nabla \Phi \left( \bar{r},t \right)-\frac{\partial }{\partial t}\bar{A}\left( \bar{r},t \right)=-\nabla \Phi \acute{\ }\left( \bar{r},t \right)-\frac{\partial }{\partial t}\bar{A}\acute{\ }\left( \bar{r},t \right) \\
& \bar{B}=\nabla \times \bar{A}\left( \bar{r},t \right)=\nabla \times \bar{A}\acute{\ }\left( \bar{r},t \right) \\
& \Rightarrow \bar{A}\acute{\ }\left( \bar{r},t \right)=\bar{A}\left( \bar{r},t \right)+\nabla G\left( \bar{r},t \right) \\
& \Rightarrow -\nabla \Phi \left( \bar{r},t \right)-\frac{\partial }{\partial t}\bar{A}\left( \bar{r},t \right)=-\nabla \Phi \acute{\ }\left( \bar{r},t \right)-\frac{\partial }{\partial t}\left( \bar{A}\left( \bar{r},t \right)+\nabla G\left( \bar{r},t \right) \right) \\
& \Rightarrow \nabla \left( \Phi \acute{\ }\left( \bar{r},t \right)-\Phi \left( \bar{r},t \right)+\frac{\partial }{\partial t}G\left( \bar{r},t \right) \right)=0 \\
& \Rightarrow \left( \Phi \acute{\ }\left( \bar{r},t \right)-\Phi \left( \bar{r},t \right)+\frac{\partial }{\partial t}G\left( \bar{r},t \right) \right)=g(t)(r-unabh\ddot{a}ngig) \\
\end{align}</math>
 
Mit
 
<math>\begin{align}
& F\left( \bar{r},t \right):=G\left( \bar{r},t \right)-\int_{to}^{t}{dt\acute{\ }g(t\acute{\ })} \\
& \Rightarrow \bar{A}\acute{\ }\left( \bar{r},t \right)=\bar{A}\left( \bar{r},t \right)+\nabla F\left( \bar{r},t \right) \\
& \Phi \acute{\ }\left( \bar{r},t \right)=\Phi \left( \bar{r},t \right)-\frac{\partial }{\partial t}F\left( \bar{r},t \right) \\
\end{align}</math>
 
mit eine völlig beliebigen Eichfunktion
<math>F\left( \bar{r},t \right)</math>
.
Alle physikalischen Aussagen müssen invariant sein ! Aber nicht nur
<math>\bar{E},\bar{B}</math>
sondern auch
<math>\Phi \left( \bar{r},t \right),\bar{A}\left( \bar{r},t \right)</math>
sind physikalisch relevant.
So muss auch
<math>\oint\limits_{\partial F}{d\bar{s}}\bar{A}\left( \bar{r},t \right)=\int_{F}^{{}}{d\bar{f}\bar{B}\left( \bar{r},t \right)=\Phi \left( \bar{r},t \right)}</math>
erfüllt sein.
 
Dies ist gewährleistet, wenn die Maxwellgleichungen  erfüllt sind.
Durch
 
<math>\begin{align}
& \bar{E}=-\nabla \Phi \left( \bar{r},t \right)-\frac{\partial }{\partial t}\bar{A}\left( \bar{r},t \right) \\
& \bar{B}=\nabla \times \bar{A}\left( \bar{r},t \right) \\
\end{align}</math>
 
sind die '''homogenen '''Maxwellgleichungen bereits erfüllt:
 
<math>\begin{align}
& \nabla \times \bar{E}=-\nabla \times \nabla \Phi \left( \bar{r},t \right)-\frac{\partial }{\partial t}\nabla \times \bar{A}\left( \bar{r},t \right)=-\frac{\partial }{\partial t}\bar{B} \\
& \nabla \cdot \bar{B}=\nabla \cdot \left( \nabla \times \bar{A}\left( \bar{r},t \right) \right)=0 \\
\end{align}</math>
 
Auch die Umkehrung gilt:
 
<math>\begin{align}
& \nabla \cdot \bar{B}=0 \\
& \Rightarrow \exists \bar{A}\left( \bar{r},t \right)\Rightarrow \nabla \times \bar{A}\left( \bar{r},t \right)=\bar{B} \\
& \nabla \times \bar{E}=-\frac{\partial }{\partial t}\bar{B}=-\nabla \times \frac{\partial }{\partial t}\bar{A}\left( \bar{r},t \right)\Rightarrow \nabla \times \left( \bar{E}+\frac{\partial }{\partial t}\bar{A}\left( \bar{r},t \right) \right)=0 \\
& \Rightarrow \exists \Phi \left( \bar{r},t \right)\Rightarrow \bar{E}+\frac{\partial }{\partial t}\bar{A}\left( \bar{r},t \right)=-\nabla \Phi \left( \bar{r},t \right) \\
\end{align}</math>
 
Wähle nun eine Eichung derart, dass die inhomogenen Maxwellgleichungen besonders einfach werden
 
Ziel: Entkopplung der DGLs für
<math>\bar{A}\left( \bar{r},t \right),\Phi \left( \bar{r},t \right)</math>
:
 
# <u>'''Lorentz- Eichung:'''</u>
 
<math>\nabla \cdot \bar{A}\left( \bar{r},t \right)+{{\varepsilon }_{0}}{{\mu }_{0}}\frac{\partial }{\partial t}\Phi \left( \bar{r},t \right)=0</math>
 
Genau dadurch werden die Feldgleichungen entkoppelt:
1)
<math>\begin{align}
& -\nabla \cdot \bar{E}=\nabla \cdot \left( \nabla \Phi \left( \bar{r},t \right)+\frac{\partial }{\partial t}\bar{A}\left( \bar{r},t \right) \right)=-\frac{\rho }{{{\varepsilon }_{0}}} \\
& \Delta \Phi \left( \bar{r},t \right)+\frac{\partial }{\partial t}\nabla \cdot \bar{A}\left( \bar{r},t \right)=-\frac{\rho }{{{\varepsilon }_{0}}} \\
\end{align}</math>
 
Was mit Hilfe der Lorentzeichung wird zu
 
<math>\Delta \Phi \left( \bar{r},t \right)-{{\varepsilon }_{0}}{{\mu }_{0}}\frac{{{\partial }^{2}}}{\partial {{t}^{2}}}\Phi \left( \bar{r},t \right)=-\frac{\rho }{{{\varepsilon }_{0}}}</math>
 
'''Für A:'''
2)
<math>\begin{align}
& \frac{1}{{{\mu }_{0}}}\nabla \times \bar{B}-{{\varepsilon }_{0}}\frac{\partial }{\partial t}\bar{E}=\bar{j} \\
& \Rightarrow \nabla \times \left( \nabla \times \bar{A}\left( \bar{r},t \right) \right)+{{\varepsilon }_{0}}{{\mu }_{0}}\frac{\partial }{\partial t}\left( \nabla \Phi \left( \bar{r},t \right)+\frac{\partial }{\partial t}\bar{A}\left( \bar{r},t \right) \right)={{\mu }_{0}}\bar{j} \\
& \nabla \times \left( \nabla \times \bar{A}\left( \bar{r},t \right) \right)=+\nabla \left( \nabla \cdot \bar{A}\left( \bar{r},t \right) \right)-\Delta \bar{A}\left( \bar{r},t \right) \\
& \Rightarrow \Delta \bar{A}\left( \bar{r},t \right)-{{\varepsilon }_{0}}{{\mu }_{0}}\frac{{{\partial }^{2}}}{\partial {{t}^{2}}}\bar{A}\left( \bar{r},t \right)-\nabla \left( \nabla \cdot \bar{A}\left( \bar{r},t \right)+{{\varepsilon }_{0}}{{\mu }_{0}}\frac{\partial }{\partial t}\Phi \left( \bar{r},t \right) \right)=-{{\mu }_{0}}\bar{j} \\
\end{align}</math>
 
Was mit der Lorentz- Eichung
 
<math>\nabla \cdot \bar{A}\left( \bar{r},t \right)+{{\varepsilon }_{0}}{{\mu }_{0}}\frac{\partial }{\partial t}\Phi \left( \bar{r},t \right)=0</math>
 
wird zu
 
<math>\Delta \bar{A}\left( \bar{r},t \right)-{{\varepsilon }_{0}}{{\mu }_{0}}\frac{{{\partial }^{2}}}{\partial {{t}^{2}}}\bar{A}\left( \bar{r},t \right)=-{{\mu }_{0}}\bar{j}</math>
 
Dies kann in Viererschreibweise mit dem dÁlembertschen Operator # mit
 
<math>\#:=\Delta -\frac{1}{{{c}^{2}}}\frac{{{\partial }^{2}}}{\partial {{t}^{2}}}</math>
 
zusammengefasst werden:
 
<math>\begin{align}
& \#\Phi \left( \bar{r},t \right)=-\frac{\rho }{{{\varepsilon }_{0}}} \\
& \#\bar{A}\left( \bar{r},t \right)=-{{\mu }_{0}}\bar{j} \\
\end{align}</math>
 
Dies sind die inhomogenen Wellengleichungen für die Potenziale ( entkoppelt mittels Lorentz- Eichung)
Es ergibt sich im SI- System:
 
<math>\frac{1}{\sqrt{{{\varepsilon }_{0}}{{\mu }_{0}}}}:=c=2,994\cdot {{10}^{8}}\frac{m}{s}</math>
als Lichtgeschwindigkeit
 
Dies ist einfach die ermittelte Ausbreitungsgeschwindigkeit der elektromagnetischen Wellen im Vakuum !
 
<u>'''Coulomb- Eichung'''</u>
 
( sogenannte Strahlungseichung):
 
<math>\nabla \cdot \bar{A}\left( \bar{r},t \right)=0</math>
 
Vergleiche Kapitel 2.3 ( Magnetostatik):
Für
 
<math>\begin{align}
& \dot{\bar{D}}=0 \\
& \Rightarrow \nabla \times \bar{B}=\nabla \left( \nabla \cdot \bar{A} \right)-\Delta \bar{A}={{\mu }_{0}}\bar{j} \\
\end{align}</math>
 
(Poissongleichung der Magnetostatik)
 
<u>'''Zerlegung in longitudinale und transversale Anteile :'''</u>
 
Allgemein kann man
 
<math>\bar{E}=-\nabla \Phi \left( \bar{r},t \right)-\frac{\partial }{\partial t}\bar{A}\left( \bar{r},t \right)</math>
 
in ein wirbelfreies Longitudinalfeld:
 
<math>{{\bar{E}}_{l}}:=-\nabla \Phi \left( \bar{r},t \right)</math>
 
und ein quellenfreies Transversalfeld
 
<math>{{\bar{E}}_{t}}=-\frac{\partial }{\partial t}\bar{A}\left( \bar{r},t \right)</math>
 
zerlegen.
 
Tatsächlich gilt:
 
<math>\nabla \times {{\bar{E}}_{l}}:=-\nabla \times \left( \nabla \Phi \left( \bar{r},t \right) \right)=0</math>
 
<math>\nabla \cdot {{\bar{E}}_{t}}=-\frac{\partial }{\partial t}\nabla \cdot \bar{A}\left( \bar{r},t \right)=0</math>
 
Da
<math>\bar{B}</math>
quellenfrei ist, ist B auch immer transversal:
 
<math>\nabla \cdot \bar{B}:=\nabla \cdot \left( \nabla \times \bar{A} \right)=0</math>
 
Also:
 
<math>\Phi \left( \bar{r},t \right)</math>
ergibt die longitudinalen Felder und
 
<math>\bar{A}\left( \bar{r},t \right)</math>
die transversalen Felder.
 
Merke: Felder , die Rotation eines Vektorfeldes sind ( Vektorpotenzials) sind grundsätzlich transversaler Natur. (Divergenz verschwindet). Divergenzfelder ( als Gradienten eines Skalars) sind immer longitudinal ! ( Rotation verschwindet).
 
<u>'''Zerlegung der Stromdichte:'''</u>
 
<math>\bar{j}={{\bar{j}}_{l}}+{{\bar{j}}_{t}}</math>
 
mit
 
<math>\nabla \times {{\bar{j}}_{l}}=0</math>
 
<math>\nabla \cdot {{\bar{j}}_{t}}=0</math>
 
Mit
 
<math>\begin{align}
& \frac{\partial }{\partial t}\rho +\nabla \cdot {{{\bar{j}}}_{l}}+\nabla \cdot {{{\bar{j}}}_{t}}=0 \\
& \rho ={{\varepsilon }_{0}}\nabla \cdot {{{\bar{E}}}_{l}} \\
& \nabla \cdot {{{\bar{j}}}_{t}}=0 \\
& \Rightarrow \nabla \cdot \left( {{{\bar{j}}}_{l}}+{{\varepsilon }_{0}}\frac{\partial }{\partial t}{{{\bar{E}}}_{l}} \right)=0 \\
\end{align}</math>
 
Außerdem gilt nach der Definition von longitudinal:
 
<math>\nabla \times \left( {{{\bar{j}}}_{l}}+{{\varepsilon }_{0}}\frac{\partial }{\partial t}{{{\bar{E}}}_{l}} \right)=0</math>
 
Also:
 
<math>\left( {{{\bar{j}}}_{l}}+{{\varepsilon }_{0}}\frac{\partial }{\partial t}{{{\bar{E}}}_{l}} \right)=const</math>
 
Da beide Felder aber für r-> 0 verschwinden folgt:
 
<math>\left( {{{\bar{j}}}_{l}}+{{\varepsilon }_{0}}\frac{\partial }{\partial t}{{{\bar{E}}}_{l}} \right)=0</math>
 
Also:
 
<math>{{\bar{j}}_{l}}={{\varepsilon }_{0}}\nabla \frac{\partial \Phi }{\partial t}</math>
 
Also:
Die Feldgleichungen
 
<math>\begin{align}
& \Delta \Phi +\frac{\partial }{\partial t}\nabla \cdot \bar{A}=-\frac{\rho }{{{\varepsilon }_{0}}} \\
& \nabla \cdot \bar{A}=0 \\
& \Rightarrow \Delta \Phi =-\frac{\rho }{{{\varepsilon }_{0}}} \\
\end{align}</math>
 
und
 
<math>\begin{align}
& \Delta \bar{A}\left( \bar{r},t \right)-{{\varepsilon }_{0}}{{\mu }_{0}}\frac{{{\partial }^{2}}}{\partial {{t}^{2}}}\bar{A}\left( \bar{r},t \right)-\nabla \left( \nabla \cdot \bar{A}\left( \bar{r},t \right)+{{\varepsilon }_{0}}{{\mu }_{0}}\frac{\partial }{\partial t}\Phi \left( \bar{r},t \right) \right)=-{{\mu }_{0}}\bar{j} \\
& \nabla \cdot \bar{A}\left( \bar{r},t \right)=0 \\
& \nabla {{\varepsilon }_{0}}\frac{\partial }{\partial t}\Phi \left( \bar{r},t \right)={{{\bar{j}}}_{l}} \\
\end{align}</math>
 
erhalten dann die Form:
 
<math>\Delta \Phi =-\frac{\rho }{{{\varepsilon }_{0}}}</math>
 
und
 
<math>\begin{align}
& \#\bar{A}\left( \bar{r},t \right)=-{{\mu }_{0}}{{{\bar{j}}}_{t}} \\
&  \\
\end{align}</math>
 
In der Coulomb- Eichung !
Also.
 
<math>\Delta \Phi =-\frac{\rho }{{{\varepsilon }_{0}}}</math>
: longitudinale Felder entsprechend der Elektrostatik
 
<math>\#\bar{A}\left( \bar{r},t \right)=-{{\mu }_{0}}{{\bar{j}}_{t}}</math>
als transversale Felder entsprechend elektromagnetischen Wellen.
 
Das bedeutet : Die Coulombeichung ist zweckmäßig bei Strahlungsproblemen !
 
Sie liefert eine Poissongleichung für
<math>\Phi </math>
und eine Wellengleichung für
<math>\bar{A}\left( \bar{r},t \right)</math>
.
<u>'''4. Elektromagnetische Wellen'''</u>
 
Im statischen Fall sind die Felder
<math>\bar{E},\bar{B}</math>
entkoppelt.
Im dynamischen Fall jedoch sind
<math>\bar{E},\bar{B}</math>
über den Verschiebungsstrom
 
<math>\frac{1}{{{\mu }_{0}}}\nabla \times \bar{B}-\bar{j}={{\varepsilon }_{0}}\dot{\bar{E}}</math>
 
 
und über das Induktionsgesetz
 
<math>\nabla \times \bar{E}=-\dot{\bar{B}}</math>
gekoppelt !
 
 
Dies bestimmt die Ausbreitung von elektromagnetischen Wellen !
 
<u>'''4.1 Freie Wellenausbreitung im Vakuum'''</u>
 
Betrachte einen Raumbereich ohne Quellen:
 
<math>\rho =0</math>
 
<math>\bar{j}=0</math>
 
Damit:
 
<math>\#\Phi =-\frac{1}{{{\varepsilon }_{0}}}\rho =0\Rightarrow \#\Phi =0</math>
 
<math>\#\bar{A}=-{{\mu }_{0}}\bar{j}=0\Rightarrow \#\bar{A}=0</math>
 
Dies sind die homogenen Wellengleichungen in Lorentz- Eichung
 
Wegen
 
<math>\begin{align}
& \bar{E}=-\nabla \Phi \left( \bar{r},t \right)-\frac{\partial }{\partial t}\bar{A}\left( \bar{r},t \right) \\
& \bar{B}=\nabla \times \bar{A}\left( \bar{r},t \right) \\
\end{align}</math>
 
gilt auch
 
<math>\begin{align}
& \#\bar{E}=0 \\
& \#\bar{B}=0 \\
\end{align}</math>
 
Dies folgt auch direkt aus
 
<math>\begin{align}
& \nabla \times \bar{B}={{\varepsilon }_{0}}{{\mu }_{0}}\dot{\bar{E}} \\
& \nabla \times \bar{E}=-\dot{\bar{B}} \\
& \Rightarrow mit\quad \nabla \cdot \bar{E}=0 \\
& \left( \Delta -{{\varepsilon }_{0}}{{\mu }_{0}}\frac{{{\partial }^{2}}}{\partial {{t}^{2}}} \right)\bar{E}=0 \\
\end{align}</math>
 
<u>'''Allgemeine Lösung '''</u>von
<math>u(\bar{r},t)=0</math>
:
 
<math>u(\bar{r},t)=F(\bar{k}\bar{r}-\varpi t)</math>
 
mit einer beliebigen , zweifach diffbaren Funktion
<math>F(\phi )</math>
und
<math>\varpi =c\left| {\bar{k}} \right|</math>
( dÁlembertsche Lösung)
Beweis:
 
<math>\#F(\bar{k}\bar{r}-\varpi t)=\left( {{{\bar{k}}}^{2}}-\frac{{{\varpi }^{2}}}{{{c}^{2}}} \right)F\acute{\ }\acute{\ }\left( \phi  \right)=0</math>
 
Nebenbemerkung:
<math>F(\phi )</math>
muss nicht periodisch in
<math>\phi </math>
sein !
Gegenbeispiel sind solitäre Lösungen / solitäre Wellen = Solitonen :
 
 
Der Wellenvektor
<math>\bar{k}</math>
zeigt in Ausbreitungsrichtung:
 
 
Es gilt:
<math>\nabla \phi (\bar{r},t)=\bar{k}</math>
 
Die markierten Flächen sind sogenannte Phasenflächen. Dies sind Flächen konstanter Phase:
 
<math>\bar{k}\bar{r}-\varpi t=\phi (\bar{r},t)=const!</math>
 
Somit ergibt sich für ebene Wellen die Bedingung:
 
<math>\bar{k}\left( \bar{r}-\frac{1}{{{k}^{2}}}\bar{k}\left( \varpi t+\phi  \right) \right)=0</math>
 
Die  Ausbreitung der Orte konstanter Phase folgt der Bedingung:
 
<math>\bar{r}(t)=\frac{1}{{{k}^{2}}}\bar{k}\left( \varpi t+\phi  \right)</math>
 
Somit ergibt sich die Phasengeschwindigkeit
 
<math>\begin{align}
& {{v}_{ph}}=\left. \frac{d\bar{r}(t)}{dt} \right|_{\phi =const}^{{}}=\frac{{\bar{k}}}{{{k}^{2}}}\varpi =c\frac{{\bar{k}}}{k} \\
& \frac{{\bar{k}}}{k}:=\bar{n} \\
\end{align}</math>
 
spezielle Lösung:  Harmonische Ebene Welle
 
<math>u(\bar{r},t)=\tilde{u}(\bar{k}){{e}^{i(\bar{k}\bar{r}-\varpi t)}}</math>
 
mit der komplexen Amplitude
 
<math>\tilde{u}(\bar{k})</math>
 
Die lineare Superposition der Wellen ist wegen der Linearität möglich und lautet formal für die allgemeine Dispersionsrelation
<math>\varpi (\bar{k})</math>
 
<math>u(\bar{r},t)=\int_{{}}^{{}}{{{d}^{3}}k}\tilde{u}(\bar{k}){{e}^{i(\bar{k}\bar{r}-\varpi (\bar{k})t)}}</math>
 
Literatur: Vergleiche FK Brillouin, L. Wave propagation and group velocity
 
Sei
 
<math>\tilde{u}(\bar{k})</math>
um
<math>{{\bar{k}}_{0}}</math>
herum lokalisiert:
 
So ergibt sich ein '''Wellenpaket ''', welches im Ortsraum lokalisiert ist !
 
Denn: Die Taylorentwicklung der Phase um
<math>{{\bar{k}}_{0}}</math>
ergibt
 
<math>\begin{align}
& \varpi (\bar{k})\approx \varpi ({{{\bar{k}}}_{0}})+\left( \bar{k}-{{{\bar{k}}}_{0}} \right){{\nabla }_{k}}\varpi (\bar{k}){{\left. {} \right|}_{\bar{k}={{{\bar{k}}}_{0}}}}+\frac{1}{2!}{{\left( \bar{k}-{{{\bar{k}}}_{0}} \right)}^{2}}{{\left( {{\nabla }_{k}} \right)}^{2}}\varpi (\bar{k}){{\left. {} \right|}_{\bar{k}={{{\bar{k}}}_{0}}}}+... \\
& {{\nabla }_{k}}\varpi (\bar{k}){{\left. {} \right|}_{\bar{k}={{{\bar{k}}}_{0}}}}={{{\bar{v}}}_{g}} \\
& \varpi (\bar{k})\approx \varpi ({{{\bar{k}}}_{0}})+\left( \bar{k}-{{{\bar{k}}}_{0}} \right){{{\bar{v}}}_{g}} \\
\end{align}</math>
 
Diese lineare Näherung ergibt nun gerade
 
<math>\begin{align}
& u(\bar{r},t)={{e}^{i({{{\bar{k}}}_{0}}\bar{r}-{{\varpi }_{0}}t)}}\int_{{}}^{{}}{{{d}^{3}}\tilde{k}}\tilde{u}({{{\bar{k}}}_{0}}+\tilde{\bar{k}}){{e}^{i\tilde{\bar{k}}(\bar{r}-{{{\bar{v}}}_{g}}t)}} \\
& \tilde{\bar{k}}=\bar{k}-{{{\bar{k}}}_{0}} \\
\end{align}</math>
 
Dies ist zu interpretieren als
 
<math>{{e}^{i({{{\bar{k}}}_{0}}\bar{r}-{{\varpi }_{0}}t)}}</math>
eine Trägerwelle mit der Phasengschwindigkeit
<math>{{\bar{v}}_{ph}}=\frac{{{\varpi }_{0}}}{{{k}_{0}}}</math>
 
<math>\int_{{}}^{{}}{{{d}^{3}}\tilde{k}}\tilde{u}({{\bar{k}}_{0}}+\tilde{\bar{k}}){{e}^{i\tilde{\bar{k}}(\bar{r}-{{{\bar{v}}}_{g}}t)}}</math>
als Einhüllende, deren Maximum sich mit der Gruppengeschwindigkeit
 
<math>{{\bar{v}}_{g}}={{\nabla }_{k}}\varpi \left( {\bar{k}} \right)</math>
bewegt:
 
 
Wir erhalten die Dispersionsrelation
<math>\varpi \left( {\bar{k}} \right)</math>
 
elektromagnetische Wellen im Vakuum:
<math>\varpi \left( {\bar{k}} \right)=c\left| {\bar{k}} \right|\Rightarrow {{\bar{v}}_{g}}=c\frac{{\bar{k}}}{\left| {\bar{k}} \right|}={{\bar{v}}_{ph}}=\frac{1}{\sqrt{{{\varepsilon }_{0}}{{\mu }_{0}}}}\bar{n}</math>
 
es gibt also keine Dispersion ( kein zerfließen!)
 
Im Gegensatz zu elektromagentischen Wellen in dispersiven Medien oder quantenmechanischen Materiewellen im Vakuum !
 
<u>'''Polarisation'''</u>
 
Betrachte eine elektromagnetische Welle:
 
<math>\begin{align}
& \bar{E}(\bar{r},t)={{{\bar{E}}}_{0}}{{e}^{i(\bar{k}\bar{r}-\varpi t)}} \\
& \bar{B}(\bar{r},t)={{{\bar{B}}}_{0}}{{e}^{i(\bar{k}\bar{r}-\varpi t)}} \\
\end{align}</math>
 
Allgemein gilt:
 
<math>\bar{E}(\bar{r},t)</math>
heißt transversal, wenn
<math>\nabla \cdot \bar{E}(\bar{r},t)=0</math>
( quellenfrei)
 
<math>\Rightarrow i\bar{k}\cdot \bar{E}(\bar{r},t)=0\Rightarrow \bar{k}\bot \bar{E}(\bar{r},t)</math>
 
<math>\bar{E}(\bar{r},t)</math>
heißt longitudinal, wenn
<math>\nabla \times \bar{E}(\bar{r},t)=0</math>
( wirbelfrei)
 
<math>\Rightarrow i\bar{k}\times \bar{E}(\bar{r},t)=0\Rightarrow \bar{k}||\bar{E}(\bar{r},t)</math>
 
Für
<math>\rho =0</math>
ist wegen
<math>\nabla \cdot \bar{E}(\bar{r},t)=0</math>
das elektrische Feld transversal.
Wegen
<math>\nabla \cdot \bar{B}(\bar{r},t)=0</math>
ist das magnetische Feld stets transversal !
 
Weiter folgt aus:
 
<math>\nabla \times \bar{E}(\bar{r},t)+\dot{\bar{B}}=0</math>
 
dass die transversale Komponente des elektrischen Feldes durch die zeitliche Änderung des Magnetfeldes gegeben ist !
 
<math>\begin{align}
& \nabla \times \bar{E}(\bar{r},t)+\dot{\bar{B}}=0 \\
& \Rightarrow \left( i\bar{k}\times {{{\bar{E}}}_{0}}-i\varpi {{{\bar{B}}}_{0}} \right){{e}^{i\left( \bar{k}\bar{r}-\varpi t \right)}}=0 \\
& \varpi =c\left| {\bar{k}} \right| \\
& \Rightarrow {{{\bar{B}}}_{0}}=\frac{1}{c}\frac{{\bar{k}}}{\left| {\bar{k}} \right|}\times {{{\bar{E}}}_{0}}:=\frac{1}{c}\bar{n}\times {{{\bar{E}}}_{0}} \\
\end{align}</math>
 
Folglich bilden
<math>\bar{k},{{\bar{E}}_{0}},{{\bar{B}}_{0}}</math>
ein Rechtssystem !
 
Die Richtung von
<math>\operatorname{Re}\left\{ {{{\bar{E}}}_{0}},{{{\bar{B}}}_{0}} \right\}</math>
legt die Polarisation fest:
 
Sei
<math>\bar{k}||{{\bar{e}}_{3}}</math>
- Achse, also:
 
<math>\begin{align}
& {{{\bar{E}}}_{0}}={{E}_{01}}{{{\bar{e}}}_{1}}+{{E}_{02}}{{{\bar{e}}}_{2}} \\
& {{E}_{0i}}={{a}_{i}}{{e}^{i{{\delta }_{i}}}}\in C \\
& {{a}_{i}},{{\delta }_{i}}\in R \\
& i=1,2 \\
\end{align}</math>
 
Das physikalische Feld ergibt sich zu
<math>\begin{align}
& {{{\bar{E}}}_{1}}(\bar{r},t)=\operatorname{Re}\left\{ {{a}_{1}}{{e}^{i\left( {{\delta }_{1}}+\bar{k}\bar{r}-\varpi t \right)}} \right\}={{a}_{1}}\cos \left( \phi +{{\delta }_{1}} \right) \\
& \phi :=\bar{k}\bar{r}-\varpi t \\
\end{align}</math>
 
und
 
<math>{{\bar{E}}_{2}}(\bar{r},t)=\operatorname{Re}\left\{ {{a}_{2}}{{e}^{i\left( {{\delta }_{2}}+\phi  \right)}} \right\}={{a}_{2}}\cos \left( \phi +{{\delta }_{2}} \right)</math>
 
Aus
 
<math>\begin{align}
& \frac{{{{\bar{E}}}_{1}}}{{{a}_{1}}}(\bar{r},t)=\cos \phi \cos {{\delta }_{1}}-\sin \phi \sin {{\delta }_{1}} \\
& \frac{{{{\bar{E}}}_{2}}}{a2}(\bar{r},t)=\cos \phi \cos {{\delta }_{2}}-\sin \phi \sin {{\delta }_{2}} \\
\end{align}</math>
 
Kann
<math>\phi </math>
und somit
<math>\left( \bar{r},t \right)</math>
eliminiert werden:
 
<math>\begin{align}
& \frac{{{{\bar{E}}}_{1}}}{{{a}_{1}}}\sin {{\delta }_{2}}-\frac{{{{\bar{E}}}_{2}}}{a2}\sin {{\delta }_{1}}=\cos \phi \sin \left( {{\delta }_{2}}-{{\delta }_{1}} \right) \\
& \frac{{{{\bar{E}}}_{1}}}{{{a}_{1}}}\cos {{\delta }_{2}}-\frac{{{{\bar{E}}}_{2}}}{a2}\cos {{\delta }_{1}}=\sin \phi \sin \left( {{\delta }_{2}}-{{\delta }_{1}} \right) \\
& \Rightarrow {{1}^{2}}+{{2}^{2}}\Rightarrow {{\left( \frac{{{{\bar{E}}}_{1}}}{{{a}_{1}}} \right)}^{2}}+{{\left( \frac{{{{\bar{E}}}_{2}}}{a2} \right)}^{2}}-2\frac{{{{\bar{E}}}_{1}}}{{{a}_{1}}}\frac{{{{\bar{E}}}_{2}}}{a2}\cos \left( {{\delta }_{2}}-{{\delta }_{1}} \right)={{\sin }^{2}}\left( {{\delta }_{2}}-{{\delta }_{1}} \right) \\
\end{align}</math>
 
Dies ist jedoch eine Ellipsengleichung für
<math>{{\bar{E}}_{1}},{{\bar{E}}_{2}}</math>
:
 
 
Der Feldvektor
<math>\bar{E}(\bar{r},t)</math>
läuft als Funktion von
<math>\phi </math>
auf einer Ellipse senkrecht zu
<math>\bar{k}</math>
um die Achse der Ausbreitungsrichtung. Man spricht von elliptischer Polarisation:
 
 
Dabei entspricht die Darstellung dem Ortsvektor
<math>\bar{r}</math>
für eine feste Zeit t oder der vorhergehenden zeit -t für einen festen Ort
<math>\bar{r}</math>
.
 
<u>'''Spezialfälle:'''</u>
 
<u>'''Linear polarisierte Welle:'''</u>
 
<math>\begin{align}
& {{\delta }_{1}}={{\delta }_{2}}+n\pi \Rightarrow \sin \left( {{\delta }_{2}}-{{\delta }_{1}} \right)=0,\cos \left( {{\delta }_{2}}-{{\delta }_{1}} \right)=\pm 1 \\
& \Rightarrow \frac{{{{\bar{E}}}_{1}}}{{{a}_{1}}}\pm \frac{{{{\bar{E}}}_{2}}}{a2}=0 \\
\end{align}</math>
 
Dies ist jedoch eine Geradengleichung:
 
<math>\bar{E}(\bar{r},t)={{\bar{E}}_{0}}\cos \phi (\bar{r},t)</math>
 
mit reeller Amplitude
 
<math>{{\bar{E}}_{0}}</math>
 
<u>'''Zirkular polarisierte Welle'''</u>
 
<math>\begin{align}
& a1=a2=a \\
& {{\delta }_{1}}={{\delta }_{2}}+\left( 2n+1 \right)\frac{\pi }{2}\Rightarrow \sin \left( {{\delta }_{2}}-{{\delta }_{1}} \right)=\pm 1,\cos \left( {{\delta }_{2}}-{{\delta }_{1}} \right)=0 \\
& \Rightarrow {{{\bar{E}}}_{1}}^{2}+{{{\bar{E}}}_{2}}^{2}={{a}^{2}} \\
\end{align}</math>
 
Dies entspricht der Überlagerung zweier linear polarisierter Wellen, die um
<math>\frac{\pi }{2}</math>
phasenverschoben sind !
Der Feldvektor des elektrischen Feldes läuft auf einem Kreis um
 
<math>\bar{E}(\bar{r},t)=a\left( \begin{matrix}
\cos \phi  \\
\pm \sin \phi  \\
\end{matrix} \right)</math>
 
Je nach Vorzeichen spricht man von links- bzw. rechtszirkular polarisiertem Licht:
 
Dabei läuft
<math>\bar{B}(\bar{r},t)</math>
dem
<math>\bar{E}(\bar{r},t)</math>
- Vektor um
<math>\frac{\pi }{2}</math>
verschoben nach bzw. voraus !
 
<u>'''Energiedichte der elektromagnetischen Welle:'''</u>
 
<math>{{\bar{E}}_{0}}(\bar{r},t)</math>
reell:
<math>\begin{align}
& \bar{E}(\bar{r},t)={{{\bar{E}}}_{0}}\cos \left( \bar{k}\bar{r}-\varpi t \right) \\
& \bar{B}(\bar{r},t)={{{\bar{B}}}_{0}}\cos \left( \bar{k}\bar{r}-\varpi t \right) \\
\end{align}</math>
 
mit
 
<math>{{\bar{B}}_{0}}=\frac{1}{c}\bar{n}\times {{\bar{E}}_{0}}</math>
 
Die Energiedichte ergibt sich gemäß
 
<math>w=\frac{{{\varepsilon }_{0}}}{2}{{\bar{E}}^{2}}+\frac{1}{2{{\mu }_{0}}}{{\bar{B}}^{2}}=\frac{{{\varepsilon }_{0}}}{2}{{\bar{E}}^{2}}+\frac{1}{2{{\mu }_{0}}{{c}^{2}}}{{\bar{E}}^{2}}=2\frac{{{\varepsilon }_{0}}}{2}{{\bar{E}}^{2}}</math>
 
Für die Energiestromdichte gilt:
 
<math>\begin{align}
& \bar{S}=\frac{1}{{{\mu }_{0}}}\bar{E}\times \bar{B} \\
& \bar{S}=\frac{1}{c{{\mu }_{0}}}\bar{E}\times \left( \bar{n}\times \bar{E} \right)=\sqrt{\frac{{{\varepsilon }_{0}}}{{{\mu }_{0}}}}{{{\bar{E}}}^{2}}\bar{n}=c{{\varepsilon }_{0}}{{{\bar{E}}}^{2}}\bar{n}=cw\bar{n} \\
\end{align}</math>
 
Also:
Die Energie wird mit Lichtgeschwindigkeit in Richtung
<math>\bar{n}=\frac{{\bar{k}}}{\left| {\bar{k}} \right|}</math>
transportiert
Für ine Kugelwelle:
<math>\bar{E}(\bar{r},t)=\frac{1}{r}{{\bar{E}}_{0}}\cos \left( \bar{k}\bar{r}-\varpi t \right)</math>
verteilt sich die Energie auf eine Kugelschale:
 
für die Energie in einer Kugelschale mit dem Radius r und der Dicke dr gilt:
 
<math>W(r)=4\pi {{r}^{2}}dr{{\varepsilon }_{0}}{{\bar{E}}^{2}}(\bar{r},t)</math>
 
Dabei kann der Exponent der Feldfunktion zeitlich gemittelt werden ( sinus²) und es ergibt sich ein Faktor 1/2:
 
<math>W(r)=4\pi {{r}^{2}}dr{{\varepsilon }_{0}}{{\bar{E}}^{2}}(\bar{r},t)=2\pi {{r}^{2}}dr{{\varepsilon }_{0}}\frac{{{{\bar{E}}}_{0}}^{2}}{{{r}^{2}}}=const.</math>
 
*# <u>'''Retardierte Potenziale'''</u>
 
<u>'''Aufgabe'''</u>
Lösung der inhomogenen Wellengleichungen in Lorentz- Eichung:
 
<math>\begin{align}
& \#\Phi \left( \bar{r},t \right)=-\frac{\rho }{{{\varepsilon }_{0}}} \\
& \#\bar{A}\left( \bar{r},t \right)=-{{\mu }_{0}}\bar{j} \\
\end{align}</math>
 
zu vorgegebenen erzeugenden Quellen
<math>\rho \left( \bar{r},t \right),\bar{j}\left( \bar{r},t \right)</math>
und Randbedingungen
<math>\Phi \left( \bar{r},t \right),\bar{A}\left( \bar{r},t \right)\to 0f\ddot{u}r\quad \bar{r}\to \infty </math>
 
<u>'''Methode: Greensche Funktion verwenden:'''</u>
 
<math>G\left( \bar{r}-\bar{r}\acute{\ },t-t\acute{\ } \right)</math>
 
'''In der Elektrodynamik:'''
 
<math>\#u\left( \bar{r},t \right)=-f\left( \bar{r},t \right)</math>
 
mit
 
<math>\begin{align}
& u\left( \bar{r},t \right):=\Phi \left( \bar{r},t \right),\bar{A}\left( \bar{r},t \right) \\
& f\left( \bar{r},t \right)=\frac{\rho }{{{\varepsilon }_{0}}},{{\mu }_{0}}\bar{j} \\
\end{align}</math>
 
Fourier- Trafo:
 
<math>\begin{align}
& {{{\hat{\#}}}^{-1}}:=-\hat{G} \\
& \Rightarrow \hat{u}\left( \bar{k},\omega  \right)=\hat{G}\hat{f}\left( \bar{k},\omega  \right) \\
\end{align}</math>
 
Rück- Trafo:
es folgt schließlich:
 
<math>u\left( \bar{r},t \right)=\int_{{{R}^{3}}}^{{}}{{{d}^{3}}r\acute{\ }\int_{-\infty }^{\infty }{dt\acute{\ }}}G\left( \bar{r}-\bar{r}\acute{\ },t-t\acute{\ } \right)f\left( \bar{r}\acute{\ },t\acute{\ } \right)</math>
 
mit
 
<math>\#G\left( \bar{r}-\bar{r}\acute{\ },t-t\acute{\ } \right)=-\delta \left( \bar{r}-\bar{r}\acute{\ } \right)\delta \left( t-t\acute{\ } \right)</math>
 
'''Vergleiche: Elektrostatik:'''
 
<math>\Delta \Phi \left( {\bar{r}} \right)=-\frac{1}{{{\varepsilon }_{0}}}\rho \left( {\bar{r}} \right)</math>
 
Fourier- Trafo:
 
<math>\begin{align}
& {{\Delta }^{-1}}:=-\hat{G} \\
& \Rightarrow \hat{\Phi }\left( {\bar{k}} \right)=\hat{G}\hat{\rho } \\
& \hat{G}=\frac{1}{{{\varepsilon }_{0}}{{k}^{2}}} \\
\end{align}</math>
 
Rück- Trafo:
es folgt schließlich:
 
<math>\Phi \left( {\bar{r}} \right)=\int_{{{R}^{3}}}^{{}}{{{d}^{3}}r\acute{\ }}G\left( \bar{r}-\bar{r}\acute{\ } \right)\rho \left( \bar{r}\acute{\ } \right)</math>
 
mit
 
<math>\begin{align}
& G\left( \bar{r}-\bar{r}\acute{\ } \right)=\frac{1}{4\pi {{\varepsilon }_{0}}}\frac{1}{\left| \bar{r}-\bar{r}\acute{\ } \right|} \\
& \Delta G\left( \bar{r}-\bar{r}\acute{\ } \right)=-\frac{1}{{{\varepsilon }_{0}}}\delta \left( \bar{r}-\bar{r}\acute{\ } \right) \\
\end{align}</math>
 
'''Kausalitätsbedingung:'''
 
<math>G\left( \bar{r}-\bar{r}\acute{\ },t-t\acute{\ } \right)=0</math>
 
für t<t´
 
Somit kann
 
<math>u\left( \bar{r},t \right)</math>
nur von
<math>f\left( \bar{r}\acute{\ },t\acute{\ } \right)</math>
mit t´ < t  beeinflusst werden
 
<u>'''Fourier- Transformation:'''</u>
 
<math>\begin{align}
& f\left( \bar{r},t \right)=\frac{1}{{{\left( 2\pi  \right)}^{2}}}\int_{{{R}^{3}}}^{{}}{{{d}^{3}}q\int_{-\infty }^{\infty }{d\omega }}\hat{f}\left( \bar{q},\omega  \right){{e}^{i\left( \bar{q}\bar{r}-\omega t \right)}} \\
& \hat{f}\left( \bar{q},\omega  \right)=\frac{1}{{{\left( 2\pi  \right)}^{2}}}\int_{{{R}^{3}}}^{{}}{{{d}^{3}}r\int_{-\infty }^{\infty }{dt}}f\left( \bar{r},t \right){{e}^{-i\left( \bar{q}\bar{r}-\omega t \right)}} \\
\end{align}</math>
 
Ebenso:
 
<math>\begin{align}
& u\left( \bar{r},t \right)=\frac{1}{{{\left( 2\pi  \right)}^{2}}}\int_{{{R}^{3}}}^{{}}{{{d}^{3}}q\int_{-\infty }^{\infty }{d\omega }}\hat{u}\left( \bar{q},\omega  \right){{e}^{i\left( \bar{q}\bar{r}-\omega t \right)}} \\
& \Rightarrow \#u\left( \bar{r},t \right)=\frac{1}{{{\left( 2\pi  \right)}^{2}}}\int_{{{R}^{3}}}^{{}}{{{d}^{3}}q\int_{-\infty }^{\infty }{d\omega }}\hat{u}\left( \bar{q},\omega  \right)\#{{e}^{i\left( \bar{q}\bar{r}-\omega t \right)}} \\
& \#{{e}^{i\left( \bar{q}\bar{r}-\omega t \right)}}=-\left( {{q}^{2}}-\frac{{{\omega }^{2}}}{{{c}^{2}}} \right){{e}^{i\left( \bar{q}\bar{r}-\omega t \right)}} \\
\end{align}</math>
 
Aber es gilt:
 
<math>\begin{align}
& \#u\left( \bar{r},t \right)=-\frac{1}{{{\left( 2\pi  \right)}^{2}}}\int_{{{R}^{3}}}^{{}}{{{d}^{3}}q\int_{-\infty }^{\infty }{d\omega }}\hat{f}\left( \bar{q},\omega  \right){{e}^{i\left( \bar{q}\bar{r}-\omega t \right)}} \\
& \Rightarrow \left( {{q}^{2}}-\frac{{{\omega }^{2}}}{{{c}^{2}}} \right)\hat{u}\left( \bar{q},\omega  \right)=\hat{f}\left( \bar{q},\omega  \right) \\
& \Rightarrow \hat{u}\left( \bar{q},\omega  \right)=\frac{\hat{f}\left( \bar{q},\omega  \right)}{\left( {{q}^{2}}-\frac{{{\omega }^{2}}}{{{c}^{2}}} \right)} \\
& \Rightarrow \hat{G}=\frac{1}{\left( {{q}^{2}}-\frac{{{\omega }^{2}}}{{{c}^{2}}} \right)} \\
\end{align}</math>
 
'''Rücktransformation:'''
 
<math>\begin{align}
& u\left( \bar{r},t \right)=\frac{1}{{{\left( 2\pi  \right)}^{4}}}\int_{{{R}^{3}}}^{{}}{{{d}^{3}}q\int_{-\infty }^{\infty }{d\omega }}\frac{{{e}^{i\left( \bar{q}\bar{r}-\omega t \right)}}}{\left( {{q}^{2}}-\frac{{{\omega }^{2}}}{{{c}^{2}}} \right)}\int_{{{R}^{3}}}^{{}}{{{d}^{3}}r\acute{\ }\int_{-\infty }^{\infty }{dt}}\acute{\ }f\left( \bar{r}\acute{\ },t\acute{\ } \right){{e}^{-i\left( \bar{q}\bar{r}-\omega t \right)}} \\
& u\left( \bar{r},t \right)=\int_{{{R}^{3}}}^{{}}{{{d}^{3}}r\acute{\ }\int_{-\infty }^{\infty }{dt}}\acute{\ }\left\{ \frac{1}{{{\left( 2\pi  \right)}^{4}}}\int_{{{R}^{3}}}^{{}}{{{d}^{3}}q\int_{-\infty }^{\infty }{d\omega }}\frac{{{e}^{i\bar{q}\left( \bar{r}-\bar{r}\acute{\ } \right)-i\omega \left( t-t\acute{\ } \right)}}}{\left( {{q}^{2}}-\frac{{{\omega }^{2}}}{{{c}^{2}}} \right)} \right\}f\left( \bar{r}\acute{\ },t\acute{\ } \right) \\
& \Rightarrow \frac{1}{{{\left( 2\pi  \right)}^{4}}}\int_{{{R}^{3}}}^{{}}{{{d}^{3}}q\int_{-\infty }^{\infty }{d\omega }}\frac{{{e}^{i\bar{q}\left( \bar{r}-\bar{r}\acute{\ } \right)-i\omega \left( t-t\acute{\ } \right)}}}{\left( {{q}^{2}}-\frac{{{\omega }^{2}}}{{{c}^{2}}} \right)}=G\left( \bar{r}-\bar{r}\acute{\ },t-t\acute{\ } \right) \\
\end{align}</math>
 
Dieses Integral hat jedoch 2 Polstellen im Integrationsbereich. Es kann nur durch Anwendung des Residuensatz (komplexe Integration) gelöst werden.
 
<u>'''Berechnung der Greens- Funktion durch komplexe Integration'''</u>
 
für
<math>\omega =\pm cq</math>
gibt es Polstellen.
Die Greensche Funktion wird eindeutig, indem der Integrationsweg um die Pole herum festgelegt wird:
 
 
Der obere Integrationsweg wird durch
<math>\tau <0</math>
charakterisiert, der untere Integrationsweg durch
<math>\tau >0</math>
.
Dabei:
<math>\tau =t-t\acute{\ }</math>
 
'''Das Integral über den Halbkreis:'''
 
'''Oberer Halbkreis:'''
<math>\tau <0</math>
 
<math>\begin{align}
& \omega =R\cdot {{e}^{i\phi }}\quad 0\le \phi \le \pi  \\
& d\omega =R\cdot {{e}^{i\phi }}id\phi  \\
& \left| {{e}^{-i\omega \tau }} \right|={{e}^{R\sin \phi \tau }} \\
& \sin \phi >0 \\
& \tau <0 \\
& \Rightarrow \begin{matrix}
\lim  \\
R\to \infty  \\
\end{matrix}{{e}^{R\sin \phi \tau }}=0 \\
\end{align}</math>
 
'''Unterer Halbkreis:'''
<math>\tau >0</math>
 
<math>\begin{align}
& \omega =R\cdot {{e}^{i\phi }}\quad \pi \le \phi \le 2\pi  \\
& d\omega =R\cdot {{e}^{i\phi }}id\phi  \\
& \left| {{e}^{-i\omega \tau }} \right|={{e}^{R\sin \phi \tau }} \\
& \sin \phi <0 \\
& \tau >0 \\
& \Rightarrow \begin{matrix}
\lim  \\
R\to \infty  \\
\end{matrix}{{e}^{R\sin \phi \tau }}=0 \\
\end{align}</math>
 
Somit verschwinden die Beiträge aus den Kreisbögen und wir können für das problematische Integral schreiben:
 
<math>\Gamma (\bar{q},\tau ):=\int_{-\infty }^{\infty }{d\omega }\frac{{{e}^{-i\omega \tau }}}{\left( {{q}^{2}}-\frac{{{\omega }^{2}}}{{{c}^{2}}} \right)}=\oint\limits_{C}{d\omega }\frac{{{e}^{-i\omega \tau }}}{\left( {{q}^{2}}-\frac{{{\omega }^{2}}}{{{c}^{2}}} \right)}=2\pi i\sum\limits_{Pole}^{{}}{{}}\operatorname{Re}s\frac{{{e}^{-i\omega \tau }}}{\left( {{q}^{2}}-\frac{{{\omega }^{2}}}{{{c}^{2}}} \right)}</math>
 
( Residuensatz)
 
Für
<math>\tau <0</math>
liegen jedoch gar keine Pole im Integrationsgebiet C
 
<math>\begin{align}
& \Rightarrow \Gamma (\bar{q},\tau )=0 \\
& \Rightarrow G\left( \bar{r}-\bar{r}\acute{\ },t-t\acute{\ } \right)=0:=G\left( \bar{s},\tau  \right)=0 \\
\end{align}</math>
 
für t<t´
 
Dies ist die Kausalitätsbedingung.
 
Für
<math>\tau >0</math>
:
 
<math>\Gamma (\bar{q},\tau )=-2\pi i\sum\limits_{\omega =\pm cq}^{{}}{{}}\operatorname{Re}s\frac{{{e}^{-i\omega \tau }}}{\frac{1}{{{c}^{2}}}\left( \omega -cq \right)\left( \omega +cq \right)}</math>
 
Das Minuszeichen kommt daher, dass der Umlauf im mathematisch negativen Sinn erfolgt:
 
<math>\oint\limits_{C}{dz}f(z)=2\pi i\sum\limits_{Pole}^{{}}{{}}\operatorname{Re}sf(z)</math>
,
 
falls das Ringintegral gegen den Uhrzeigersinn durchlaufen wird. Hier jedoch wird es im Uhrzeigersinn durchlaufen !
 
<math>\Gamma (\bar{q},\tau )=2\pi i{{c}^{2}}\left( \frac{{{e}^{-icq\tau }}}{2cq}+\frac{{{e}^{icq\tau }}}{-2cq} \right)</math>
 
<math>G(\bar{s},\tau )=\frac{c}{{{\left( 2\pi  \right)}^{3}}}\int_{{{R}^{3}}}^{{}}{{}}{{d}^{3}}q{{e}^{i\bar{q}\bar{s}}}\left( \frac{{{e}^{-icq\tau }}-{{e}^{icq\tau }}}{-2iq} \right)</math>
 
Die Auswertung der Greensfunktion muss in Kugelkoordinaten erfolgen:
 
<math>\begin{align}
& {{d}^{3}}q={{q}^{2}}dq\sin \vartheta d\vartheta d\phi  \\
& \bar{q}\bar{s}=qs\cos \vartheta  \\
& G(\bar{s},\tau )=\frac{c}{{{\left( 2\pi  \right)}^{3}}}\int\limits_{0}^{\infty }{{}}dqq\left( \frac{{{e}^{-icq\tau }}-{{e}^{icq\tau }}}{-2i} \right)\int\limits_{-1}^{1}{{}}d\cos \vartheta {{e}^{iqs\cos \vartheta }}\int\limits_{0}^{2\pi }{{}}d\phi  \\
& \int\limits_{-1}^{1}{{}}d\cos \vartheta {{e}^{iqs\cos \vartheta }}=\frac{{{e}^{iqs}}-{{e}^{-iqs}}}{iqs} \\
& \xi :=cq \\
& \Rightarrow G(\bar{s},\tau )=\frac{c}{2{{\left( 2\pi  \right)}^{2}}s}\int\limits_{0}^{\infty }{{}}d\xi \left\{ {{e}^{i\left( \tau -\frac{s}{c} \right)\xi }}+{{e}^{-i\left( \tau -\frac{s}{c} \right)\xi }}-{{e}^{i\left( \tau +\frac{s}{c} \right)\xi }}-{{e}^{-i\left( \tau +\frac{s}{c} \right)\xi }} \right\} \\
& \Rightarrow G(\bar{s},\tau )=\frac{c}{4\pi s}\int\limits_{0}^{\infty }{{}}d\xi \left\{ \delta \left( \tau -\frac{s}{c} \right)-\delta \left( \tau +\frac{s}{c} \right) \right\} \\
& \delta \left( \tau +\frac{s}{c} \right)=0\quad f\ddot{u}r\ \tau >0 \\
\end{align}</math>
 
Also lautet das Ergebnis:
 
<math>G(\bar{r}-\bar{r}\acute{\ },t-t\acute{\ })=\left\{ \begin{matrix}
\frac{1}{4\pi \left| \bar{r}-\bar{r}\acute{\ } \right|}\delta \left( t-t\acute{\ }-\frac{\left| \bar{r}-\bar{r}\acute{\ } \right|}{c} \right)  \\
0\quad \quad \quad \quad \quad t<t\acute{\ }  \\
\end{matrix} \right.\ t>t\acute{\ }</math>
 
Retardierte Greensfunktion (kausal)
 
<u>'''Physikalische Interpretation'''</u>
 
<math>G(\bar{r}-\bar{r}\acute{\ },t-t\acute{\ })</math>
ist das Potenzial
<math>\Phi (\bar{r},t)</math>
, das von einer punktförmigen Ladungsdichte
 
<math>\frac{\rho }{{{\varepsilon }_{0}}}=\delta \left( \bar{r}-\bar{r}\acute{\ } \right)\delta \left( t-t\acute{\ } \right)</math>
 
am Punkt
<math>\bar{r}\acute{\ }</math>
zur Zeit t´ erzeugt wird.
 
'''Die Eigenschaften:'''
 
* Kausalität
* Ausbreitung der Punktstörung als KUGELWELLE mit der Phasengeschwindigkeit c:
* <math>\left| \bar{r}-\bar{r}\acute{\ } \right|=c\left( t-t\acute{\ } \right)</math>
*
 
'''Nebenbemerkung:'''
 
Für den Integrationsweg
 
'''Oberer Halbkreis:'''
<math>\tau <0</math>
 
'''Unterer Halbkreis:'''
<math>\tau >0</math>
 
erhält man die avancierte Greensfunktion ( =0 für t > t´).
Diese beschreibt eigentlich eine einlaufende Kugelwelle, welche sich an
<math>\bar{r}\acute{\ }</math>
zur zeit t´ zusammenzieht !
 
Mit
 
<math>G(\bar{r},t)=\int_{{}}^{{}}{{{d}^{3}}r\acute{\ }}\int_{-\infty }^{t}{dt\acute{\ }}\frac{1}{4\pi \left| \bar{r}-\bar{r}\acute{\ } \right|}\delta \left( t-t\acute{\ }-\frac{\left| \bar{r}-\bar{r}\acute{\ } \right|}{c} \right)f\left( \bar{r}\acute{\ },t\acute{\ } \right)=\int_{{}}^{{}}{{{d}^{3}}r\acute{\ }}\frac{1}{4\pi \left| \bar{r}-\bar{r}\acute{\ } \right|}f\left( \bar{r}\acute{\ },t-\frac{\left| \bar{r}-\bar{r}\acute{\ } \right|}{c} \right)</math>
 
folgt dann für die retardierten Potenziale für beliebige Ladungs- und Stromverteilungen
 
<math>\rho \left( \bar{r},t \right),\bar{j}\left( \bar{r},t \right)</math>
 
<math>\begin{align}
& \Phi \left( \bar{r},t \right)=\frac{1}{4\pi {{\varepsilon }_{0}}}\int_{{}}^{{}}{{}}{{d}^{3}}r\acute{\ }\frac{\rho \left( \bar{r}\acute{\ },t-\frac{\left| \bar{r}-\bar{r}\acute{\ } \right|}{c} \right)}{\left| \bar{r}-\bar{r}\acute{\ } \right|} \\
& \bar{A}\left( \bar{r},t \right)=\frac{{{\mu }_{\acute{\ }0}}}{4\pi }\int_{{}}^{{}}{{}}{{d}^{3}}r\acute{\ }\frac{\bar{j}\left( \bar{r}\acute{\ },t-\frac{\left| \bar{r}-\bar{r}\acute{\ } \right|}{c} \right)}{\left| \bar{r}-\bar{r}\acute{\ } \right|} \\
\end{align}</math>
 
Die retardierten Potenziale
<math>\Phi \left( \bar{r},t \right),\bar{A}\left( \bar{r},t \right)</math>
sind bestimmt durch
<math>\bar{r}\acute{\ }</math>
zu retardierten Zeiten
<math>t\acute{\ }=t-\frac{\left| \bar{r}-\bar{r}\acute{\ } \right|}{c}</math>
.
Dies berücksichtigt die endliche Ausbreitungsgeschwindigkeit von elektromagnetischen Wellen mit Lichtgeschwindigkeit c.
 
## <u>'''Multipolstrahlung'''</u>
 
<u>'''Ziel:'''</u>
 
<u>'''Die '''</u>retardierten Potenziale sollen für räumlich lokalisierte und zeitabhängige Ladungs- und Stromverteilungen analog zu den statischen Multipolentwicklungen für große Abstände von der Quelle, also r>>r´ entwickelt werden.
 
<u>'''Voraussetzung: Lorentz- Eichung'''</u>
 
<math>\dot{\Phi }\left( \bar{r},t \right)+{{c}^{2}}\nabla \cdot \bar{A}\left( \bar{r},t \right)=0</math>
 
Somit kann aus
<math>\bar{A}\left( \bar{r},t \right)</math>
dann
<math>\Phi \left( \bar{r},t \right)</math>
und somit auch
<math>\bar{E}\left( \bar{r},t \right)</math>
 
<math>\bar{B}\left( \bar{r},t \right)</math>
berechnet werden.
 
# <u>'''Näherung:'''</u>
<u>'''r>>a ( '''</u>Ausdehnung der Quelle)
 
Mit
 
<math>\frac{1}{\left| \bar{r}-\bar{r}\acute{\ } \right|}=\frac{1}{r}+\frac{1}{{{r}^{3}}}\left( \bar{r}\cdot \bar{r}\acute{\ } \right)+...</math>
 
folgt:
 
<math>\bar{A}\left( \bar{r},t \right)\approx \frac{{{\mu }_{\acute{\ }0}}}{4\pi r}\int_{{}}^{{}}{{}}{{d}^{3}}r\acute{\ }\bar{j}\left( \bar{r}\acute{\ },t-\frac{\left| \bar{r}-\bar{r}\acute{\ } \right|}{c} \right)+\frac{{{\mu }_{\acute{\ }0}}}{4\pi {{r}^{3}}}\int_{{}}^{{}}{{}}{{d}^{3}}r\acute{\ }\bar{j}\left( \bar{r}\acute{\ },t-\frac{\left| \bar{r}-\bar{r}\acute{\ } \right|}{c} \right)\left( \bar{r}\cdot \bar{r}\acute{\ } \right)</math>
 
Das heißt, es werden nur Terme bis zur zweiten Ordnung berücksichtigt !
 
# <u>'''Näherung'''</u>
 
<math>\begin{align}
& t-\frac{\left| \bar{r}-\bar{r}\acute{\ } \right|}{c}\approx t-\frac{r}{c}+\frac{\bar{r}\cdot \bar{r}\acute{\ }}{cr}+.... \\
& t-\frac{r}{c}:=\tau  \\
\end{align}</math>
 
Diese Näherung sollte gut sein, falls
<math>\tau >>\frac{\bar{r}\cdot \bar{r}\acute{\ }}{cr}\approx \frac{a}{c}</math>
 
Also: Die Retardierung zum Aufpunkt r sollte wesentlich größer sein als die relative Retardierung der einzelnen Punkte der Quelle untereinander !
 
a~ Ausdehnung der Quelle
 
<math>\tau </math>
ist etwa die charakteristisch zeit für die Änderung von
<math>\bar{j}</math>
:
 
Beispielsweise: harmonische Erregung:
 
<math>\begin{align}
& \bar{j}\tilde{\ }{{e}^{i\omega t}} \\
& \omega \tau =!=2\pi \Rightarrow \tau =\frac{2\pi }{\omega }=\frac{2\pi }{ck}=\frac{\lambda }{c} \\
& \Rightarrow a<<\lambda  \\
\end{align}</math>
 
Die Ausdehnung der Quelle müsste also deutlich kleiner sein als die Wellenlänge des abgestrahlten Lichtes !
 
Dann gilt:
 
<math>\bar{j}\left( \bar{r}\acute{\ },t-\frac{\left| \bar{r}-\bar{r}\acute{\ } \right|}{c} \right)\approx \bar{j}\left( \bar{r}\acute{\ },t-\frac{r}{c} \right)+\frac{\bar{r}\cdot \bar{r}\acute{\ }}{cr}\frac{\partial \bar{j}\left( \bar{r}\acute{\ },t-\frac{r}{c} \right)}{\partial \left( t-\frac{r}{c} \right)}=\bar{j}\left( \bar{r}\acute{\ },\tau  \right)+\frac{\bar{r}\cdot \bar{r}\acute{\ }}{cr}\frac{\partial \ \bar{j}\left( \bar{r}\acute{\ },\tau  \right)}{\partial \tau }</math>
 
Also folgt für das Vektorpotenzial:
 
 
Die niedrigste or5dnung verschwindet nicht, da im Gegensatz zu Paragraph § 2.4 die Divergenz des Stromes nicht verschwindet:
 
<math>\nabla \cdot \bar{j}\ne 0</math>
:
 
Mit:
 
<math>{{\nabla }_{r\acute{\ }}}\left( {{x}_{k}}\acute{\ }\bar{j}\left( \bar{r}\acute{\ },\tau  \right) \right)={{x}_{k}}\acute{\ }\left( {{\nabla }_{r\acute{\ }}}\bar{j}\left( \bar{r}\acute{\ },\tau  \right) \right)+{{j}_{k}}</math>
 
mit der Kontinuitäätsgleichung:
 
<math>\begin{align}
& {{\nabla }_{r\acute{\ }}}\bar{j}\left( \bar{r}\acute{\ },\tau  \right)=-\dot{\rho }\left( \bar{r}\acute{\ },\tau  \right) \\
& \Rightarrow {{\nabla }_{r\acute{\ }}}\left( {{x}_{k}}\acute{\ }\bar{j}\left( \bar{r}\acute{\ },\tau  \right) \right)={{j}_{k}}-{{x}_{k}}\acute{\ }\dot{\rho }\left( \bar{r}\acute{\ },\tau  \right) \\
\end{align}</math>
 
und wegen
 
<math>\int_{{}}^{{}}{{}}{{d}^{3}}r\acute{\ }{{\nabla }_{r\acute{\ }}}\left( {{x}_{k}}\acute{\ }\bar{j}\left( \bar{r}\acute{\ },\tau  \right) \right)=0</math>
(Gauß)
 
folgt dann:
 
<math>\begin{align}
& \int_{{}}^{{}}{{}}{{d}^{3}}r\acute{\ }{{\nabla }_{r\acute{\ }}}\left( {{x}_{k}}\acute{\ }\bar{j}\left( \bar{r}\acute{\ },\tau  \right) \right)=0=\int_{{}}^{{}}{{}}{{d}^{3}}r\acute{\ }\left( {{j}_{k}}-{{x}_{k}}\acute{\ }\dot{\rho }\left( \bar{r}\acute{\ },\tau  \right) \right) \\
& \Rightarrow \int_{{}}^{{}}{{{d}^{3}}r\acute{\ }\bar{j}\left( \bar{r}\acute{\ },\tau  \right)=\int_{{}}^{{}}{{}}{{d}^{3}}r\acute{\ }\bar{r}\acute{\ }\dot{\rho }\left( \bar{r}\acute{\ },\tau  \right)=:\dot{\bar{p}}\left( \tau  \right)} \\
\end{align}</math>
 
mit dem elektrischen Dipolmoment:
 
<math>\bar{p}\left( \tau  \right)=\int_{{}}^{{}}{{}}{{d}^{3}}r\acute{\ }\bar{r}\acute{\ }\rho \left( \bar{r}\acute{\ },\tau  \right)</math>
 
Somit für die erste Ordnung:
 
<math>{{\bar{A}}^{(1)}}\left( \bar{r},t \right)\approx \frac{{{\mu }_{\acute{\ }0}}}{4\pi r}\dot{\bar{p}}\left( t-\frac{r}{c} \right)</math>
 
<u>'''Elektrische Dipolstrahlung'''</u>
 
<u>'''Interpretation: Hertzscher Dipol ( H hertz, 1857-1894)'''</u>
 
<math>\bar{p}\left( t \right)=\bar{p}\left( {{t}_{0}} \right){{e}^{-i\omega t}}</math>
 
<math>\bar{p}</math>
 
 
<math>\begin{align}
& {{{\bar{A}}}^{(1)}}\left( \bar{r},t \right)\approx \frac{-i\omega {{\mu }_{\acute{\ }0}}}{4\pi }\frac{\bar{p}\left( {{t}_{0}} \right){{e}^{-i\omega \left( t-\frac{r}{c} \right)}}}{r}=\frac{-i\omega {{\mu }_{\acute{\ }0}}}{4\pi }\frac{\bar{p}\left( {{t}_{0}} \right){{e}^{i\left( kr-\omega t \right)}}}{r} \\
& k:=\frac{\omega }{c} \\
\end{align}</math>
 
Die Kugelwelle !
 
<u>'''Bestimmung des skalaren Potenzials mit Hilfe Lorentzeichung:'''</u>
 
<math>\begin{align}
& \dot{\Phi }\left( \bar{r},t \right)+{{c}^{2}}\nabla \cdot \bar{A}\left( \bar{r},t \right)=0 \\
& \Rightarrow \frac{\partial }{\partial t}\Phi \left( \bar{r},t \right)=-\frac{1}{{{\varepsilon }_{0}}{{\mu }_{0}}}\nabla \cdot \bar{A}\left( \bar{r},t \right)=-\frac{1}{4\pi {{\varepsilon }_{0}}}\nabla \left[ \frac{1}{r}\dot{\bar{p}}\left( t-\frac{r}{c} \right) \right] \\
& \Rightarrow \Phi \left( \bar{r},t \right)=-\frac{1}{4\pi {{\varepsilon }_{0}}}\nabla \left[ \frac{1}{r}\bar{p}\left( t-\frac{r}{c} \right) \right]+{{\Phi }_{stat.}}\left( {\bar{r}} \right) \\
& {{\Phi }_{stat.}}\left( {\bar{r}} \right)=0(obda) \\
& \Rightarrow \Phi \left( \bar{r},t \right)=-\frac{1}{4\pi {{\varepsilon }_{0}}}\nabla \left[ \frac{1}{r}\bar{p}\left( t-\frac{r}{c} \right) \right]=\frac{1}{4\pi {{\varepsilon }_{0}}}\left[ \frac{1}{c{{r}^{2}}}\bar{r}\dot{\bar{p}}\left( t-\frac{r}{c} \right)+\frac{1}{{{r}^{3}}}\bar{r}\bar{p}\left( t-\frac{r}{c} \right) \right] \\
& \frac{1}{c{{r}^{2}}}\bar{r}\dot{\bar{p}}\left( t-\frac{r}{c} \right)\tilde{\ }\frac{1}{r} \\
& \frac{1}{{{r}^{3}}}\bar{r}\bar{p}\left( t-\frac{r}{c} \right)\tilde{\ }\frac{1}{{{r}^{2}}} \\
\end{align}</math>
 
<u>'''Grenzfälle:'''</u>
 
<u>'''1) Fernzone / Wellenzone:'''</u>
 
<math>\begin{align}
& r>>\lambda >>\left( a \right)\Leftrightarrow kr>>1\Leftrightarrow \frac{\omega }{c}r>>1 \\
& \Rightarrow \frac{1}{c}\dot{\bar{p}}\tilde{\ }\frac{\omega }{c}\bar{p}>>\frac{{\bar{p}}}{r} \\
\end{align}</math>
 
In der Fernzone ist die Retardierung sehr wichtig !!
 
Es gilt die Näherung
 
<math>\Phi {{\left( \bar{r},t \right)}_{fern}}\approx \frac{1}{4\pi {{\varepsilon }_{0}}}\frac{1}{c{{r}^{2}}}\bar{r}\dot{\bar{p}}\left( t-\frac{r}{c} \right)</math>
 
<u>'''2) Nahzone: ( quasistatischer Bereich):'''</u>
 
<math>\begin{align}
& \lambda >>r>>>\left( a \right) \\
& \Leftrightarrow kr<<1\Leftrightarrow \frac{\omega }{c}r<<11 \\
& \Rightarrow \frac{1}{c}\dot{\bar{p}}\tilde{\ }\frac{\omega }{c}\bar{p}<<\frac{{\bar{p}}}{r} \\
\end{align}</math>
 
Also:
 
<math>\Phi \left( \bar{r},t \right)\approx \frac{1}{4\pi {{\varepsilon }_{0}}}\frac{1}{{{r}^{3}}}\bar{r}\bar{p}\left( t-\frac{r}{c} \right)</math>
 
Dies kann man noch entwickeln nach
 
<math>\bar{p}\left( t \right)</math>
. dadurch entstehen Terme:
 
<math>\frac{1}{c{{r}^{2}}}\bar{r}\dot{\bar{p}}\left( t \right)-\frac{1}{{{r}^{3}}}\frac{r}{c}\bar{r}\dot{\bar{p}}\left( t \right)</math>
 
Diese kompensieren sich gegenseitig.
Also:
Die Retardierung kompensiert den
<math>\dot{\bar{p}}\left( t \right)</math>
- Term.
 
Wir schreiben:
 
<math>\Phi \left( \bar{r},t \right)\approx \frac{1}{4\pi {{\varepsilon }_{0}}}\frac{1}{{{r}^{3}}}\bar{r}\bar{p}\left( t \right)</math>
 
in guter Näherung ein instantanes Dipolpontenzial ( in der Nahzone ist die Retardierung zu vernachlässigen).
 
<u>'''Berechnung der Felder in Fernfeldnäherung'''</u>
 
 
<math>\Phi {{\left( \bar{r},t \right)}_{fern}}\approx \frac{1}{4\pi {{\varepsilon }_{0}}}\frac{1}{c{{r}^{2}}}\bar{r}\dot{\bar{p}}\left( t-\frac{r}{c} \right)</math>
 
<math>\begin{align}
& \bar{B}\left( \bar{r},t \right)=\nabla \times \bar{A}\left( \bar{r},t \right)\approx \frac{{{\mu }_{\acute{\ }0}}}{4\pi }\nabla \times \frac{1}{r}\dot{\bar{p}}\left( t-\frac{r}{c} \right)=\frac{{{\mu }_{\acute{\ }0}}}{4\pi c}\frac{1}{{{r}^{2}}}\left[ \ddot{\bar{p}}\left( t-\frac{r}{c} \right)\times \bar{r} \right]+O\left( \frac{1}{{{r}^{2}}} \right) \\
& \bar{E}\left( \bar{r},t \right)=-\nabla \Phi \left( \bar{r},t \right)-\dot{\bar{A}}\left( \bar{r},t \right)=\frac{1}{4\pi {{\varepsilon }_{0}}{{c}^{2}}}\frac{1}{{{r}^{3}}}\left[ \ddot{\bar{p}}\left( t-\frac{r}{c} \right)\times \bar{r} \right]\times \bar{r}+O\left( \frac{1}{{{r}^{2}}} \right) \\
\end{align}</math>
 
Es gilt:
 
<math>\begin{align}
& \bar{B}\left( \bar{r},t \right)\times \frac{{\bar{r}}}{r}=\frac{{{\mu }_{0}}}{4\pi c}\frac{1}{{{r}^{3}}}\left[ \ddot{\bar{p}}\left( t-\frac{r}{c} \right)\times \bar{r} \right]\times \bar{r}=\frac{1}{c}\bar{E}\left( \bar{r},t \right) \\
& \frac{{{\mu }_{0}}}{4\pi c}=\frac{{{\mu }_{0}}{{\varepsilon }_{0}}}{4\pi c{{\varepsilon }_{0}}}=\frac{1}{4\pi {{c}^{3}}{{\varepsilon }_{0}}} \\
\end{align}</math>
 
F
Fazit:
 
<math>\bar{r},\bar{E}\left( \bar{r},t \right),\bar{B}\left( \bar{r},t \right)</math>
 
bilden für Dipolstrahlung ein Rechtssystem, r, B und E stehen senkrecht aufeinander !
Allerdings als Ausbreitung einer freien Kugelwelle nur in der Fernzone !!
 
<u>'''Nebenbemerkung:'''</u>
In der Nahzone gilt immer noch wegen
<math>\nabla \cdot \bar{B}\left( \bar{r},t \right)=0</math>
, dass r und B senkrecht stehen.
 
Aber: das elektrische Feld hat neben der senkrechten Komponente , die zu r senkrecht steht ( transversale Komponente) noch longitudinale Anteile ( E- parallel, die zu r parallel sind).
 
<u>'''Poynting- Vektor ( Energiestromdichte)'''</u>
 
<math>\begin{align}
& \bar{S}=\bar{E}\times \bar{H}=\frac{-1}{{{\mu }_{0}}}\bar{B}\times \bar{E}=\frac{-c}{{{\mu }_{0}}r}\bar{B}\times \left( \bar{B}\times \bar{r} \right)=\frac{-c}{{{\mu }_{0}}r}\left[ \left( \bar{B}\cdot \bar{r} \right)\bar{B}-{{B}^{2}}\bar{r} \right] \\
& \left( \bar{B}\cdot \bar{r} \right)=0 \\
& \Rightarrow \bar{S}=\frac{c}{{{\mu }_{0}}r}{{B}^{2}}\bar{r} \\
\end{align}</math>
 
<math>\bar{B}\left( \bar{r},t \right)=\nabla \times \bar{A}\left( \bar{r},t \right)\approx \frac{{{\mu }_{\acute{\ }0}}}{4\pi }\nabla \times \frac{1}{r}\dot{\bar{p}}\left( t-\frac{r}{c} \right)=\frac{{{\mu }_{\acute{\ }0}}}{4\pi c}\frac{1}{{{r}^{2}}}\left[ \ddot{\bar{p}}\left( t-\frac{r}{c} \right)\times \bar{r} \right]+O\left( \frac{1}{{{r}^{2}}} \right)</math>
 
Also:
entspricht
 
<math>l=1,m=0</math>
 
 
 
Abstrahl- Charakteristik des Hertzschen Dipols:
 
<math>\begin{align}
& \bar{p}(t)={{{\bar{p}}}_{0}}{{e}^{-i\omega t}} \\
& {{\left| {\ddot{\bar{p}}} \right|}^{2}}={{{\bar{p}}}_{0}}^{2}{{\omega }^{4}} \\
\end{align}</math>
 
Stark Richtungs- und stark frequenzabhängig !! höhere Frequenzen werden mit 4. Potenz besser abgestrahlt !
Nebenbemerkung:
Die gemachte Rechnung ist eine Näherung für eine lineare Antenne
 
<u>'''Magnetische Dipol- und Quadrupolstrahlung'''</u>
 
Die niedrigste Ordnung der Mutipolentwicklung von
<math>\bar{A}(\bar{r})=\frac{{{\mu }_{0}}}{4\pi }\int_{{{R}^{3}}}^{{}}{{}}{{d}^{3}}r\acute{\ }\frac{\bar{j}(\bar{r}\acute{\ })}{\left| \bar{r}-\bar{r}\acute{\ } \right|}</math>
(mit der Coulomb- Eichung
<math>\nabla \cdot \bar{A}(\bar{r})=0</math>
)
 
mit den Randbedingungen
<math>\bar{A}(\bar{r})\to 0</math>
für r-> unendlich  verschwindet für eine quellenfreie Stromdichte:
 
Taylorentwicklung nach
<math>\frac{1}{\left| \bar{r}-\bar{r}\acute{\ } \right|}</math>
von analog zum elektrischen Fall:
Die Stromverteilung
<math>\bar{j}(\bar{r}\acute{\ })</math>
sei stationär für
<math>r>>r\acute{\ }</math>
 
<math>\frac{1}{\left| \bar{r}-\bar{r}\acute{\ } \right|}=\frac{1}{r}+\frac{1}{{{r}^{3}}}\left( \bar{r}\cdot \bar{r}\acute{\ } \right)+...</math>
 
<math>\bar{A}(\bar{r})=\frac{{{\mu }_{0}}}{4\pi r}\int_{{{R}^{3}}}^{{}}{{}}{{d}^{3}}r\acute{\ }\bar{j}(\bar{r}\acute{\ })+\frac{{{\mu }_{0}}}{4\pi {{r}^{3}}}\int_{{{R}^{3}}}^{{}}{{}}{{d}^{3}}r\acute{\ }\bar{j}(\bar{r}\acute{\ })\left( \bar{r}\cdot \bar{r}\acute{\ } \right)+...</math>
 
'''Monopol- Term'''
 
'''Mit'''
 
<math>{{\nabla }_{r\acute{\ }}}\cdot \left[ {{x}_{k}}\acute{\ }\bar{j}(\bar{r}\acute{\ }) \right]={{x}_{k}}\acute{\ }\left( {{\nabla }_{r\acute{\ }}}\cdot \bar{j}(\bar{r}\acute{\ }) \right)+\bar{j}(\bar{r}\acute{\ })\cdot \left( {{\nabla }_{r\acute{\ }}}{{x}_{k}}\acute{\ } \right)</math>
 
Im stationären Fall folgt aus der Kontinuitätsgleichung:
 
<math>{{\nabla }_{r\acute{\ }}}\cdot \bar{j}(\bar{r}\acute{\ })=0</math>
 
<math>{{\nabla }_{r\acute{\ }}}\cdot \left[ {{x}_{k}}\acute{\ }\bar{j}(\bar{r}\acute{\ }) \right]=\bar{j}(\bar{r}\acute{\ })\cdot \left( {{\nabla }_{r\acute{\ }}}{{x}_{k}}\acute{\ } \right)={{j}_{l}}{{\delta }_{kl}}={{j}_{k}}</math>
 
Mit
<math>{{\nabla }_{r\acute{\ }}}\cdot \left[ {{x}_{k}}\acute{\ }\bar{j}(\bar{r}\acute{\ }) \right]={{j}_{k}}</math>
folgt dann:
 
<math>\int_{{}}^{{}}{{{d}^{3}}r\acute{\ }}{{j}_{k}}(\bar{r}\acute{\ })=\int_{{}}^{{}}{{{d}^{3}}r\acute{\ }}{{\nabla }_{r\acute{\ }}}\cdot \left[ {{x}_{k}}\acute{\ }\bar{j}(\bar{r}\acute{\ }) \right]=\oint\limits_{S\infty }{d\bar{f}}\left[ {{x}_{k}}\acute{\ }\bar{j}(\bar{r}\acute{\ }) \right]=0</math>
 
Somit verschwindet der Monopolterm in der Theorie.
 
Also: Falls
 
<math>\bar{j}(\bar{r}\acute{\ },\tau )</math>
quellenfrei und damit divergenzfrei, so verschwindet die niedrigste Ordnung der Entwicklung von A:
Mit Hilfe der Kontinuitätsgleichung:
 
<math>\begin{align}
& {{\nabla }_{r\acute{\ }}}\cdot \bar{j}(\bar{r}\acute{\ },\tau )=-\frac{\partial }{\partial \tau }\rho (\bar{r}\acute{\ },\tau )=0 \\
& \Rightarrow \dot{\bar{p}}(\tau )=\int_{{}}^{{}}{{}}{{d}^{3}}r\acute{\ }\bar{r}\acute{\ }\dot{\rho }=0 \\
& \Rightarrow {{A}^{(1)}}=\frac{{{\mu }_{0}}}{4\pi r}\dot{\bar{p}}(\tau )\equiv 0 \\
\end{align}</math>
 
Im Herztschen Dipol existiert keine Ausstrahlung ( In der Hertzschen Dipol- Näherung)
 
<u>'''Beispiel:  '''</u>geschlossene Leiterschleife ( sogenannte Rahmenantenne):
 
Mit
 
<math>I(t)={{I}_{0}}{{e}^{-i\omega t}}</math>
 
<u>'''2. Ordnung:'''</u>
 
<math>{{\bar{A}}^{(2)}}\left( \bar{r},t \right)=\frac{{{\mu }_{0}}}{4\pi {{r}^{3}}}\int_{{}}^{{}}{{{d}^{3}}r\acute{\ }\left( \bar{r}\cdot \bar{r}\acute{\ } \right)\left( 1+\frac{r}{c}\frac{\partial }{\partial \tau } \right)\bar{j}(\bar{r}\acute{\ },\tau )}</math>
 
Mit
 
<math>\begin{align}
& \left( \bar{r}\bar{r}\acute{\ } \right)\bar{j}\left( \bar{r}\acute{\ },\tau  \right)=\frac{1}{2}\left( \bar{r}\acute{\ }\times \bar{j} \right)\times \bar{r}+\frac{1}{2}\left[ \left( \bar{r}\bar{r}\acute{\ } \right)\bar{j}+\left( \bar{r}\bar{j} \right)\bar{r}\acute{\ } \right] \\
& und \\
& {{\nabla }_{r\acute{\ }}}\left[ {{x}_{k}}\acute{\ }\left( \bar{r}\bar{r}\acute{\ } \right)\bar{j} \right]=\left[ \left( \bar{r}\bar{r}\acute{\ } \right){{j}_{k}}+{{x}_{k}}\acute{\ }\left( \bar{r}\bar{j} \right)+{{x}_{k\acute{\ }}}\left( \bar{r}\bar{r}\acute{\ } \right){{\nabla }_{r\acute{\ }}}\cdot \bar{j} \right] \\
& {{\nabla }_{r\acute{\ }}}\cdot \bar{j}=-\frac{\partial }{\partial \tau }\rho \left( \bar{r}\acute{\ },\tau  \right) \\
\end{align}</math>
 
Kontinuitätsgleichung
Dann folgt integriert:
Der zweite Term rechts kann durch den Tensor des elektrischen Quadrupolmoments ausgedrückt werden ( vergl. S. 15, Elektrostatik):
 
<math>\bar{\bar{Q}}\left( \tau  \right)=\int_{{}}^{{}}{{{d}^{3}}r\acute{\ }\rho \left( \bar{r}\acute{\ },\tau  \right)}\left( 3\bar{r}\acute{\ }\otimes \bar{r}\acute{\ }-r{{\acute{\ }}^{2}}\bar{\bar{1}} \right)=:\tilde{\bar{\bar{Q}}}-\frac{1}{3}\left( tr\left( {\tilde{\bar{\bar{Q}}}} \right) \right)\bar{\bar{1}}</math>
 
Falls
 
<math>\tilde{Q}\left( \tau  \right)</math>
oszilliert ( sogenannter "breathing mode"), gibt
 
<math>\frac{1}{3}\left( tr\left( {\tilde{\bar{\bar{Q}}}} \right) \right)\bar{\bar{1}}</math>
 
keinen Beitrag zu
 
<math>\bar{E},\bar{B}</math>
 
* verschwindet durch Eichtrafo innerhalb der Klasse der Lorentz- Eichungen
 
<u>'''->'''</u>
<math>\bar{\bar{Q}}\left( \tau  \right)\cdot \bar{r}=3\int_{{}}^{{}}{{{d}^{3}}r\acute{\ }\rho \left( \bar{r}\acute{\ },\tau  \right)}\bar{r}\acute{\ }\left( \bar{r}\acute{\ }\cdot \bar{r} \right)</math>
 
'''Also:'''
 
<math>\begin{align}
& {{{\bar{A}}}^{(2)}}\left( \bar{r},t \right)=\frac{{{\mu }_{0}}}{4\pi {{r}^{3}}}\left( 1+\frac{r}{c}\frac{\partial }{\partial \tau } \right)\left[ \bar{m}\left( \tau  \right)\times \bar{r}+\frac{1}{6}\dot{\bar{\bar{Q}}}\left( \tau  \right)\cdot \bar{r} \right] \\
& =\frac{{{\mu }_{0}}}{4\pi }\left( \frac{1}{{{r}^{3}}}\bar{m}\times \bar{r}+\frac{1}{c{{r}^{2}}}\dot{\bar{m}}\times \bar{r}+\frac{1}{6{{r}^{3}}}\dot{\bar{\bar{Q}}}\left( \tau  \right)\cdot \bar{r}+\frac{1}{6c{{r}^{2}}}\ddot{\bar{\bar{Q}}}\left( \tau  \right)\cdot \bar{r} \right) \\
\end{align}</math>
 
Mit der magnetischen Dipolstrahlung
 
<math>\frac{1}{{{r}^{3}}}\bar{m}\times \bar{r}+\frac{1}{c{{r}^{2}}}\dot{\bar{m}}\times \bar{r}</math>
 
und elektrischer Quadrupolstrahlung
 
<math>\frac{1}{6{{r}^{3}}}\dot{\bar{\bar{Q}}}\left( \tau  \right)\cdot \bar{r}+\frac{1}{6c{{r}^{2}}}\ddot{\bar{\bar{Q}}}\left( \tau  \right)\cdot \bar{r}</math>
 
Die magnetische Dipolstrahlung kann mit Hilfe
 
<math>\nabla \times \frac{1}{r}\bar{m}\left( t-\frac{r}{c} \right)=\frac{1}{{{r}^{3}}}\bar{m}\left( t-\frac{r}{c} \right)\times \bar{r}+\frac{1}{c{{r}^{2}}}\dot{\bar{m}}\left( t-\frac{r}{c} \right)\times \bar{r}</math>
 
schreiben als:
 
Die magnetische Dipolstrahlung
 
'''Skalares Potenzial aus der Lorentz- Eichung'''
 
<math>\begin{align}
& \frac{\partial }{\partial t}\Phi \left( \bar{r},t \right)=-{{c}^{2}}\nabla \cdot \bar{A}\left( \bar{r},t \right)=-\frac{{{\mu }_{0}}{{c}^{2}}}{4\pi }\nabla \cdot \left( \nabla \times \frac{1}{r}\bar{m} \right)\equiv 0 \\
& \Rightarrow \Phi \left( \bar{r},t \right)=\Phi \left( {\bar{r}} \right)=!=0 \\
\end{align}</math>
 
O.B.d.A.: Es existiere kein statisches Potenzial/ es wird auf Null gesetzt
 
'''Berechnung der Felder in Fernfeldnäherung:'''
das elektrische Feld ergibt sich wie für die elektrische Dipolstrahlung
 
<u>'''Nebenbemerkung'''</u>
 
Für Systeme von Teilchen mit gleicher spezifischer Ladung
<math>\frac{q}{m}</math>
 
ist
 
<math>\bar{p}\tilde{\ }\bar{R}</math>
(Schwerpunkt)
und
 
<math>\bar{m}\tilde{\ }\bar{L}</math>
( Gesamtdrehimpuls)
 
<math>\Rightarrow \dot{\bar{p}}=\dot{\bar{m}}=0</math>
 
In diesem Fall ( vier gleiche Ladungen etc...) ist nur elektrische Quadrupolstrahlung möglich
 
vergleiche ART: durch die unipolarität der Masse existiert nur Gravitations- Quadrupolstrahlung
 
<u>'''4.4 Wellenoptik und Beugung'''</u>
 
Betrachte Ausbreitung elektromagnetischer Wellen bei gegebenen lokalisierten Quellen
<math>\rho \left( \bar{r},t \right)</math>
und
<math>\bar{j}\left( \bar{r},t \right)</math>
und bei vorgegebenen Leitern
<math>{{L}_{\alpha }}</math>
im Vakuum:
 
 
 
<u>'''Ziel'''</u>
 
ist die Berechnung des Wellenfeldes im Außenraum V
 
Anwendung: Radiowellen
<math>\lambda =1-{{10}^{4}}</math>
m
Radar
Optik
<math>\lambda =400-800nm</math>
-> Beugung
 
<u>'''Rückführung auf Randwertaufgabe'''</u>
 
Lösung der inhomogenen Wellengleichungen in Lorentzeichung ( Potenzialgleichungen) ( vergleiche dazu 1.6 in der Elektrostatik)
 
<math>\begin{align}
& \#\Phi \left( \bar{r},t \right)=-\frac{\rho }{{{\varepsilon }_{0}}} \\
& \#\bar{A}\left( \bar{r},t \right)=-{{\mu }_{0}}\bar{j} \\
\end{align}</math>
 
Zu vorgegebenen Ladungen und Strömen.
Gleichzeitig haben wir Randbedingungen auf
<math>{{L}_{\alpha }}</math>
 
und schließlich die Kausalitätsbedingung ( Ausstrahlungsbedingung) -> Retardierung, § 4.2
 
'''Annahme:'''
 
<math>\begin{align}
& \rho \left( \bar{r},t \right)=\rho \left( {\bar{r}} \right){{e}^{-i\omega t}} \\
& \bar{j}\left( \bar{r},t \right)=\bar{j}\left( {\bar{r}} \right){{e}^{-i\omega t}} \\
\end{align}</math>
 
Dies sollte wegen Fourier- Zerlegung bei periodischer Erregung beliebiger Art grundsätzlich möglich sein.
 
<math>\begin{align}
& \Phi \left( \bar{r},t \right)=\Phi \left( {\bar{r}} \right){{e}^{-i\omega t}} \\
& \bar{A}\left( \bar{r},t \right)=\bar{A}\left( {\bar{r}} \right){{e}^{-i\omega t}} \\
\end{align}</math>
 
eingesetzt in die Wellengleichung
 
<math>\begin{align}
& \#\Phi \left( \bar{r},t \right)=-\frac{\rho }{{{\varepsilon }_{0}}}=\left( \Delta -\frac{1}{{{c}^{2}}}\frac{{{\partial }^{2}}}{\partial {{t}^{2}}} \right)\Phi \left( \bar{r},t \right) \\
& \Rightarrow \left( \Delta +{{k}^{2}} \right)\Phi \left( {\bar{r}} \right)=-\frac{\rho \left( {\bar{r}} \right)}{{{\varepsilon }_{0}}} \\
& k:=\frac{\omega }{c} \\
\end{align}</math>
 
Mit der Greenschen Funktion der Wellengleichung
<math>\#\Phi \left( \bar{r},t \right)=-\frac{\rho }{{{\varepsilon }_{0}}}</math>
:
 
<math>\#G\left( \bar{r}-\bar{r}\acute{\ },t-t\acute{\ } \right)=-\delta \left( \bar{r}-\bar{r}\acute{\ } \right)\delta \left( t-t\acute{\ } \right)</math>
 
Haben wir formal sofort die allgemeine Lösung:
 
<math>\begin{align}
& \Phi \left( \bar{r},t \right)=\int_{{}}^{{}}{{{d}^{3}}r\acute{\ }\int_{-\infty }^{t}{dt\acute{\ }}}\frac{\rho \left( \bar{r}\acute{\ },t\acute{\ } \right)}{{{\varepsilon }_{0}}}G\left( \bar{r}-\bar{r}\acute{\ },t-t\acute{\ } \right)=\int_{{}}^{{}}{{{d}^{3}}r\acute{\ }\int_{-\infty }^{t}{dt\acute{\ }}}\frac{\rho \left( \bar{r}\acute{\ } \right)}{{{\varepsilon }_{0}}}{{e}^{-i\omega t\acute{\ }}}G\left( \bar{r}-\bar{r}\acute{\ },t-t\acute{\ } \right) \\
& t-t\acute{\ }:=\tau  \\
& \Rightarrow \int_{-\infty }^{t}{dt\acute{\ }}{{e}^{-i\omega t\acute{\ }}}G\left( \bar{r}-\bar{r}\acute{\ },t-t\acute{\ } \right)=\int_{-\infty }^{t}{dt\acute{\ }}{{e}^{-i\omega t\acute{\ }}}G\left( \bar{r}-\bar{r}\acute{\ },\tau  \right) \\
& =\left[ \int_{0}^{\infty }{d\tau }{{e}^{i\omega \tau }}G\left( \bar{r}-\bar{r}\acute{\ },\tau  \right) \right]{{e}^{-i\omega t}}:=\tilde{G}\left( \bar{r}-\bar{r}\acute{\ } \right){{e}^{-i\omega t}} \\
& \int_{0}^{\infty }{d\tau }{{e}^{i\omega \tau }}G\left( \bar{r}-\bar{r}\acute{\ },\tau  \right):=\tilde{G}\left( \bar{r}-\bar{r}\acute{\ } \right) \\
\end{align}</math>
 
Somit kann die periodische Zeitabhängigkeit absepariert werden:
 
<math>\begin{align}
& \Phi \left( {\bar{r}} \right)=\int_{{}}^{{}}{{{d}^{3}}r\acute{\ }}\tilde{G}\left( \bar{r}-\bar{r}\acute{\ } \right)\frac{\rho \left( \bar{r}\acute{\ } \right)}{{{\varepsilon }_{0}}} \\
& mit \\
& \left( \Delta +{{k}^{2}} \right)\tilde{G}\left( \bar{r}-\bar{r}\acute{\ } \right)=-\delta \left( \bar{r}-\bar{r}\acute{\ } \right) \\
\end{align}</math>
 
Problem:
Die Randbedingungen für
<math>\Phi \left( {\bar{r}} \right),\bar{A}</math>
sind im stationären Fall nicht bekannt, sondern müssen selbstkonsistent bestimmt werden.
Man kann das Problem jedoch mit Hilfe des Greenschen Satzes umformulieren:
 
<u>'''Skalare Kirchhoff- Identität'''</u>
 
( eine notwendige, nicht hinreichende Bedingung für Lösung):
 
Skalar: Wir beschreiben keine Polarisationseffekte !!! Alle Polarisationseffekte sind vernachlässigt. Das hat man unter dem Begriff Skalar an dieser Stelle zu verstehen !
 
Weiter: Greenscher Satz:
 
<math>\int_{\partial V}^{{}}{d\bar{f}}\left( \phi \nabla \Psi -\Psi \nabla \phi  \right)=\int_{V}^{{}}{{{d}^{3}}r}\left( \phi \Delta \Psi -\Psi \Delta \phi  \right)</math>
 
Setze:
 
<math>\begin{align}
& \Psi (\bar{r})=\tilde{G}\left( \bar{r}-\bar{r}\acute{\ } \right) \\
& \phi (\bar{r})=\Phi \left( {\bar{r}} \right) \\
\end{align}</math>
 
Dabei sei das Potenzial als Lösung angenommen:
 
<math>\begin{align}
& \int_{\partial V}^{{}}{d\bar{f}}\left( \Phi \left( {\bar{r}} \right)\nabla \tilde{G}\left( \bar{r}-\bar{r}\acute{\ } \right)-\tilde{G}\left( \bar{r}-\bar{r}\acute{\ } \right)\nabla \Phi \left( {\bar{r}} \right) \right)=\int_{V}^{{}}{{{d}^{3}}r}\left( \Phi \left( {\bar{r}} \right)\Delta \tilde{G}\left( \bar{r}-\bar{r}\acute{\ } \right)-\tilde{G}\left( \bar{r}-\bar{r}\acute{\ } \right)\Delta \Phi \left( {\bar{r}} \right) \right) \\
& \Delta \tilde{G}\left( \bar{r}-\bar{r}\acute{\ } \right)=-\delta \left( \bar{r}-\bar{r}\acute{\ } \right)-{{k}^{2}}\tilde{G}\left( \bar{r}-\bar{r}\acute{\ } \right) \\
& \Delta \Phi \left( {\bar{r}} \right)=\frac{-\rho }{{{\varepsilon }_{0}}}-{{k}^{2}}\Phi \left( {\bar{r}} \right) \\
& \Rightarrow \int_{V}^{{}}{{{d}^{3}}r}\left( \Phi \left( {\bar{r}} \right)\Delta \tilde{G}\left( \bar{r}-\bar{r}\acute{\ } \right)-\tilde{G}\left( \bar{r}-\bar{r}\acute{\ } \right)\Delta \Phi \left( {\bar{r}} \right) \right)=-\Phi \left( \bar{r}\acute{\ } \right) \\
& \int_{\partial V}^{{}}{d\bar{f}}\left( \tilde{G}\left( \bar{r}-\bar{r}\acute{\ } \right)\nabla \Phi \left( {\bar{r}} \right)-\Phi \left( {\bar{r}} \right)\nabla \tilde{G}\left( \bar{r}-\bar{r}\acute{\ } \right) \right)=\Phi \left( \bar{r}\acute{\ } \right) \\
\end{align}</math>
 
Also:
 
<math>\begin{align}
& \Phi \left( \bar{r}\acute{\ } \right)=\int_{\partial V}^{{}}{d\bar{f}}\left( \tilde{G}\left( \bar{r}-\bar{r}\acute{\ } \right)\nabla \Phi \left( {\bar{r}} \right)-\Phi \left( {\bar{r}} \right)\nabla \tilde{G}\left( \bar{r}-\bar{r}\acute{\ } \right) \right) \\
& \bar{r}\acute{\ }\in V \\
\end{align}</math>
 
Dabei ist
<math>\Phi \left( \bar{r}\acute{\ } \right)</math>
im inneren von V durch
<math>\Phi </math>
und
<math>\nabla \Phi </math>
auf dem Rand festgelegt, falls die Greensfunktion
 
<math>\tilde{G}\left( \bar{r}-\bar{r}\acute{\ } \right)</math>
bekannt ist
 
<u>'''Freier Raum: Greensfunktion des unendlichen Raumes:'''</u>
 
Randbedingung
<math>\begin{matrix}
\lim  \\
r\to \infty  \\
\end{matrix}\tilde{G}\left( \bar{r}-\bar{r}\acute{\ } \right)=0</math>
 
* Retardierte Potenziale ( Vergl. § 4.2):
 
<math>G\left( \bar{r}-\bar{r}\acute{\ },\tau  \right)=\left\{ \begin{matrix}
\frac{1}{4\pi \left| \bar{r}-\bar{r}\acute{\ } \right|}\delta \left( \tau -\frac{\left| \bar{r}-\bar{r}\acute{\ } \right|}{c} \right)\quad \tau >0  \\
0\quad \quad \quad \quad \tau <0  \\
\end{matrix} \right.</math>
 
Somit:
 
<math>\begin{align}
& \tilde{G}\left( \bar{r}-\bar{r}\acute{\ } \right)=\int_{0}^{\infty }{d}\tau G\left( \bar{r}-\bar{r}\acute{\ },\tau  \right){{e}^{i\omega \tau }}=\frac{{{e}^{ik\left| \bar{r}-\bar{r}\acute{\ } \right|}}}{4\pi \left| \bar{r}-\bar{r}\acute{\ } \right|} \\
& k:=\frac{\omega }{c} \\
\end{align}</math>
 
Es folgt für das Potenzial:
 
<math>\begin{align}
& \Phi \left( \bar{r},t \right)=\int_{{}}^{{}}{{{d}^{3}}r\acute{\ }\tilde{G}\left( \bar{r}-\bar{r}\acute{\ } \right){{e}^{-i\omega t}}\frac{\rho \left( \bar{r}\acute{\ } \right)}{{{\varepsilon }_{0}}}}=\int_{{}}^{{}}{{{d}^{3}}r\acute{\ }\frac{{{e}^{ik\left| \bar{r}-\bar{r}\acute{\ } \right|}}}{4\pi \left| \bar{r}-\bar{r}\acute{\ } \right|}{{e}^{-i\omega t}}\frac{\rho \left( \bar{r}\acute{\ } \right)}{{{\varepsilon }_{0}}}} \\
& \Phi \left( \bar{r},t \right)=\int_{{}}^{{}}{{{d}^{3}}r\acute{\ }\frac{{{e}^{i\left( k\left| \bar{r}-\bar{r}\acute{\ } \right|-\omega t \right)}}}{4\pi \left| \bar{r}-\bar{r}\acute{\ } \right|}\frac{\rho \left( \bar{r}\acute{\ } \right)}{{{\varepsilon }_{0}}}} \\
\end{align}</math>
 
beschreibt eine Überlagerung auslaufender Kugelwellen-> Lösung als Entwicklung in Kugelwellen.
( Ausstrahlbedingung, Konsequenz der Kausalität).
 
Mit
 
<math>\bar{R}:=\bar{r}-\bar{r}\acute{\ }</math>
 
lautet die Kirchhoff- Identität:
 
<math>\begin{align}
& \Phi \left( \bar{r}\acute{\ },t \right)=\frac{1}{4\pi }\int_{\partial V}^{{}}{d{{{\bar{f}}}_{R}}}\left[ \frac{{{e}^{ikR}}}{R}{{\nabla }_{r}}\Phi \left( {\bar{r}} \right)-\Phi \left( {\bar{r}} \right){{\nabla }_{r}}\frac{{{e}^{ikR}}}{R} \right] \\
& {{\nabla }_{r}}\frac{{{e}^{ikR}}}{R}=\frac{{{e}^{ikR}}}{R}\left( ik-\frac{1}{R} \right)\frac{\bar{r}-\bar{r}\acute{\ }}{\left| \bar{r}-\bar{r}\acute{\ } \right|} \\
\end{align}</math>
 
Dazu eine Grafik:
 
 
Mittels
<math>d\bar{f}\frac{\bar{r}-\bar{r}\acute{\ }}{\left| \bar{r}-\bar{r}\acute{\ } \right|}=df\cos \vartheta </math>
 
und über Beschränkung auf Fernzone von
<math>\partial V</math>
, also R >> 1/k gilt:
 
 
<math>\Phi \left( \bar{r}\acute{\ },t \right)=\frac{1}{4\pi }\int_{\partial V}^{{}}{d{{f}_{R}}}\left[ \frac{\partial }{\partial n}\Phi \left( {\bar{r}} \right)-ik\Phi \left( {\bar{r}} \right)\cos \vartheta  \right]\frac{{{e}^{ikR}}}{R}</math>
 
Mit der richtungsabhängigen Amplitude
<math>\left[ \frac{\partial }{\partial n}\Phi \left( {\bar{r}} \right)-ik\Phi \left( {\bar{r}} \right)\cos \vartheta  \right]</math>
und der Kugelwelle
<math>\frac{{{e}^{ikR}}}{R}</math>
.
Beides zusammen ergeben sogenannte Sekundärwellen.
 
Insgesamt ist dies die exakte ( mathematische) Formulierung des Huygensschen Prinzips
( jeder Punkt an der Oberfläche des Hindernisses ist Ausgangspunkt einer Kugelwelle).
deren phasengerechte Überlagerung ergibt dann das Wellenfeld in r´
 
<u>'''b) Greensfunktion zu Randbedingungen'''</u>
 
<math>\tilde{G}\left( \bar{r}-\bar{r}\acute{\ } \right){{\left. {} \right|}_{\begin{smallmatrix}
\bar{r}\in \partial V \\
\bar{r}\acute{\ }\in V
\end{smallmatrix}}}=0</math>
 
<math>\Rightarrow \Phi \left( \bar{r}\acute{\ } \right)=-\int_{\partial V}^{{}}{d\bar{f}\Phi \left( {\bar{r}} \right){{\nabla }_{r}}\tilde{G}\left( \bar{r}-\bar{r}\acute{\ } \right)}</math>
 
Die neue Greensfunktion unterscheidet sich von der alten nur durch eine additive  Lösung g der homogenen Wellengleichung:
 
<math>\begin{align}
& \tilde{G}\left( {\bar{R}} \right)=g\left( {\bar{R}} \right)+\frac{1}{4\pi }\frac{{{e}^{ikR}}}{R} \\
& \left( \Delta +{{k}^{2}} \right)g=0 \\
\end{align}</math>
 
Mit Randbedingung
 
<math>g{{\left. {} \right|}_{\partial V}}=-\frac{1}{4\pi }{{\left. \frac{{{e}^{ikR}}}{R} \right|}_{\partial V}}</math>
 
Beispiel für die Konstruktion von
<math>\tilde{G}\left( {\bar{R}} \right)</math>
:
 
'''Ebener Schirm:'''
 
Spiegelladungsmethode:
 
Hinter dem Schirm wird die Halbkugel im UNENDLICHEN geschlossen.
 
Hinter dem ebenen Schirm wenden wir die Spiegelladungsmethode an:
 
<math>\begin{align}
& \tilde{G}\left( \bar{r}-\bar{r}\acute{\ } \right)=\frac{1}{4\pi }\left( \frac{{{e}^{ik\left| \bar{r}-\bar{r}\acute{\ } \right|}}}{\left| \bar{r}-\bar{r}\acute{\ } \right|}-\frac{{{e}^{ik\left| \bar{r}-\bar{r}\acute{\ }\acute{\ } \right|}}}{\left| \bar{r}-\bar{r}\acute{\ }\acute{\ } \right|} \right) \\
& \Rightarrow {{\nabla }_{r}}\tilde{G}\left( \bar{r}-\bar{r}\acute{\ } \right)=\frac{1}{4\pi }\left( {{\nabla }_{r}}\frac{{{e}^{ik\left| \bar{r}-\bar{r}\acute{\ } \right|}}}{\left| \bar{r}-\bar{r}\acute{\ } \right|}-{{\nabla }_{r}}\frac{{{e}^{ik\left| \bar{r}-\bar{r}\acute{\ }\acute{\ } \right|}}}{\left| \bar{r}-\bar{r}\acute{\ }\acute{\ } \right|} \right):=\frac{1}{4\pi }\left( {{\nabla }_{r}}\frac{{{e}^{ikR}}}{R}-{{\nabla }_{r}}\frac{{{e}^{ikR\acute{\ }\acute{\ }}}}{R\acute{\ }\acute{\ }} \right) \\
\end{align}</math>
 
Dieser Gradient wurde einige Seiten vorher bereits gelöst:
 
<math>\begin{align}
& {{\nabla }_{r}}\tilde{G}\left( \bar{r}-\bar{r}\acute{\ } \right)=\frac{1}{4\pi }\left( {{\nabla }_{r}}\frac{{{e}^{ik\left| \bar{r}-\bar{r}\acute{\ } \right|}}}{\left| \bar{r}-\bar{r}\acute{\ } \right|}-{{\nabla }_{r}}\frac{{{e}^{ik\left| \bar{r}-\bar{r}\acute{\ }\acute{\ } \right|}}}{\left| \bar{r}-\bar{r}\acute{\ }\acute{\ } \right|} \right):=\frac{1}{4\pi }\left( {{\nabla }_{r}}\frac{{{e}^{ikR}}}{R}-{{\nabla }_{r}}\frac{{{e}^{ikR\acute{\ }\acute{\ }}}}{R\acute{\ }\acute{\ }} \right) \\
& =\frac{1}{4\pi }\left( \frac{{{e}^{ikR}}}{R}\left( ik-\frac{1}{R} \right)\frac{\bar{r}-\bar{r}\acute{\ }}{\left| \bar{r}-\bar{r}\acute{\ } \right|}-\frac{{{e}^{ikR\acute{\ }\acute{\ }}}}{R\acute{\ }\acute{\ }}\left( ik-\frac{1}{R\acute{\ }\acute{\ }} \right)\frac{\bar{r}-\bar{r}\acute{\ }\acute{\ }}{\left| \bar{r}-\bar{r}\acute{\ }\acute{\ } \right|} \right) \\
\end{align}</math>
 
Mit
 
<math>\begin{align}
& R=R\acute{\ }\acute{\ } \\
& d\bar{f}\cdot \frac{\bar{r}-\bar{r}\acute{\ }}{\left| \bar{r}-\bar{r}\acute{\ } \right|}=-d\bar{f}\cdot \frac{\bar{r}-\bar{r}\acute{\ }\acute{\ }}{\left| \bar{r}-\bar{r}\acute{\ }\acute{\ } \right|}=+df\cos \vartheta  \\
& \Rightarrow d\bar{f}\cdot {{\nabla }_{r}}\tilde{G}=df\frac{1}{2\pi }\frac{{{e}^{ikR}}}{R}\left( ik-\frac{1}{R} \right)\cos \vartheta  \\
\end{align}</math>
 
Für
<math>\lambda <<R</math>
( Fernzone):
 
 
<math>\Phi \left( \bar{r}\acute{\ } \right)=-\int_{\partial V}^{{}}{d\bar{f}\Phi \left( {\bar{r}} \right){{\nabla }_{r}}\tilde{G}\left( \bar{r}-\bar{r}\acute{\ } \right)}=-\frac{i}{\lambda }\int_{F}^{{}}{df\Phi \left( {\bar{r}} \right)\frac{{{e}^{ik\left| \bar{r}-\bar{r}\acute{\ } \right|}}}{\left| \bar{r}-\bar{r}\acute{\ } \right|}}\cos \vartheta </math>
 
Zur Konstruktion der Lösung müssen die Randwerte
<math>{{\left. \Phi \left( {\bar{r}} \right) \right|}_{F}}</math>
erraten werden.
 
<u>'''Kirchhoffsche Näherung'''</u>
 
Beugung an Blenden B in einem ebenen Schirm:
 
Annahme:
 
<math>{{\left. \Phi \left( {\bar{r}} \right) \right|}_{S}}=0</math>
Das Potenzial verschwindet auf dem Schirm ( leitender Schirm)
 
<math>{{\left. \Phi \left( {\bar{r}} \right) \right|}_{B}}=\frac{{{e}^{ik{{R}^{Q}}}}}{{{R}^{Q}}}</math>
freie einfallende Welle -> Kugelwellen in der Blende
 
Ro rage dabei zum Schwerpunkt der Blende
 
<math>\begin{align}
& \Phi \left( \bar{r}\acute{\ } \right)=-\frac{i}{\lambda }\int_{B}^{{}}{df\frac{{{e}^{ik\left| R+{{R}^{Q}} \right|}}}{R{{R}^{Q}}}}\cos \vartheta  \\
& \cos \vartheta \approx const. \\
\end{align}</math>
 
Der Winkel ist näherungsweise konstant für kleine Blenden:
 
<math>\lambda <<d</math>
 
<math>\begin{align}
& \bar{R}=\bar{r}-\bar{r}\acute{\ } \\
& {{{\bar{R}}}^{Q}}=\bar{r}-{{{\bar{r}}}^{Q}} \\
& df={{d}^{2}}r \\
\end{align}</math>
 
Somit:
 
<math>\begin{align}
& \Phi \left( \bar{r}\acute{\ } \right)=-\frac{i}{\lambda }\frac{\cos {{\vartheta }_{0}}}{{{R}_{0}}{{R}_{0}}^{Q}}\int_{B}^{{}}{df}{{e}^{ik\left| R+{{R}^{Q}} \right|}} \\
& \cos \vartheta \approx const. \\
\end{align}</math>
 
im schnell oszillierenden Exponenten darf man R und RQ nicht so ohne weiteres durch Ro / RQo ersetzen !
 
* typisches Näherungsverfahren in Fernfeldoptik
 
<u>'''Grenzfälle'''</u>
 
# <u>'''Fraunhofersche Beugung ( Fernzone:'''</u>
# <math>\lambda <<d<<R</math>
# )
 
Setze
<math>\bar{R}={{\bar{R}}_{0}}+\bar{s}</math>
 
<math>{{R}^{2}}\approx {{R}_{0}}^{2}+2{{\bar{R}}_{0}}\bar{s}</math>
 
<math>\begin{align}
& \Rightarrow R\approx {{R}_{0}}+\bar{\alpha }\bar{s} \\
& \bar{\alpha }:=\frac{{{{\bar{R}}}_{0}}}{{{R}_{0}}} \\
\end{align}</math>
 
Analog:
 
<math>\begin{align}
& \Rightarrow {{R}^{Q}}\approx {{R}_{0}}^{Q}+{{{\bar{\alpha }}}_{0}}\bar{s} \\
& {{{\bar{\alpha }}}_{0}}:=\frac{{{{\bar{R}}}_{0}}^{Q}}{{{R}_{0}}^{Q}} \\
\end{align}</math>
 
<math>\Rightarrow \Phi \left( \bar{r}\acute{\ } \right)\approx -\frac{i}{\lambda }{{e}^{ik\left( {{R}_{0}}+{{R}_{0}}^{Q} \right)}}\frac{\cos {{\vartheta }_{0}}}{{{R}_{0}}{{R}_{0}}^{Q}}\int_{B}^{{}}{{{d}^{2}}s}{{e}^{ik\left( \bar{\alpha }+{{{\bar{\alpha }}}_{0}} \right)\bar{s}}}</math>
 
<u>'''Fresnelsche Beugung ( Mittelzone:'''</u>
<math>\lambda <<R\approx d</math>
 
hier:
<math>{{R}^{2}}={{R}_{0}}^{2}+2{{\bar{R}}_{0}}\bar{s}+{{s}^{2}}</math>
nicht genähert !!
 
'''Beispiel: Fraunhofersche Beugung am Spalt ( eindimensional):'''
 
 
Bei senkrechtem Einfall gilt:
<math>{{\bar{\alpha }}_{0}}\bar{s}=0</math>
 
<math>\begin{align}
& \Rightarrow \Phi \left( \bar{r}\acute{\ } \right)=C\int_{-d/2}^{d/2}{d{{s}_{1}}}{{e}^{ik\alpha {{s}_{1}}}} \\
& \alpha :=\sin {{\vartheta }_{0}} \\
& \bar{\alpha }\bar{s}={{s}_{1}}\sin {{\vartheta }_{0}} \\
& \Rightarrow \Phi \left( \bar{r}\acute{\ } \right)=\frac{C}{ik\alpha }\left( {{e}^{ik\alpha \frac{d}{2}}}-{{e}^{-ik\alpha \frac{d}{2}}} \right) \\
& \Phi \left( \bar{r}\acute{\ } \right)=Cd\frac{\sin \left( k\alpha \frac{d}{2} \right)}{k\alpha \frac{d}{2}} \\
\end{align}</math>
 
Die Spaltfunktion, Fouriertransformierte der Rechteckfunktion ( Blende)
 
 
Wir finden Beugungsminima bei den Nullstellen des Sinus ( Außer in der Mitte), also
 
<math>\sin {{\vartheta }_{0}}=n\cdot \frac{\lambda }{d}</math>
 
ebenso ( als ÜBUNG !!!)
können dann Beugung am Gitter und an kreisförmigen Blenden berechnet werden.
 
<u>'''Einwurf:                1. Der holografische Prozess'''</u>
 
*# <u>'''Aufzeichnung und Rekonstruktion'''</u>
 
Lichtintensität einer Lichtwelle:
 
<math>I(x,y)=|O(x,y)|{}^\text{2}=O(x,y)\bullet O*(x,y)</math>
 
* Phaseninformationen gehen verloren
* Idee: Phaseninfo durch Interferenz aufzeichnen
* Lösung mittels eines Zweistufenprozesses: Aufzeichnung und Rekonstruktion
* Kohärenz erforderlich
* monochromatisches Licht
* unpolarisiertes Licht
 
<u>'''1. Schritt: Die Aufzeichnungsphase'''</u>
 
* Problem: Speichern komplexer Funktionen in einem reellen Medium
* Überlagerung der Objektwelle
 
<math>O(x,y)=|O(x,y)|\bullet \exp (i{{\varphi }_{O}}(x,y))</math>
 
* Mit einer Referenzwelle
 
<math>R(x,y)=|R(x,y)|\bullet \exp (i{{\varphi }_{R}}(x,y))</math>
 
* Auch in diesem Fall werden nur Intensitäten gespeichert. Doch diese sind nun:
 
<math>I(x,y)=|O(x,y)+R(x,y)|{}^\text{2}\ =\quad |O|{}^\text{2}\ +\ |R|{}^\text{2}\ +\ OR*\ +\ O*R</math>
 
<math>I(x,y)=|O|{}^\text{2}\ +\ |R|{}^\text{2}\ +2RO\cos [{{\varphi }_{R}}(x,y)-{{\varphi }_{O}}(x,y)]</math>
 
* Diese Intensitätsverteilung kann verstanden werden als " Hologrammfunktion" oder "Aperturefunktion"
* Planare Wellen: Fraunhofer Hologramme
* Divergierende Wellen: Fresnelhologramme
* Im obigen Bild dargestellt: Trägerfrequenzholografie
* Eigentliche Holografie: ohne Trägerfrequenz: Referenzstrahl, in den auch das Objekt gestellt wird.
* Dabei überlagern sich jedoch mehrere Ordnungen.
* Generell: verschiedenste Aufzeichnungstechniken:
* Trägerfrequenzholografie ( wie oben)
* Denisyukhologramm
 
<u>'''2. Schritt: Rekonstruktionsphase'''</u>
 
* Gleiche Wellenlänge wie bei Aufzeichnung rekonstruiert das Objekt
* Ansonsten: Verzerrung
* Beugung durch Hologrammstrukturen vergleichbar mit Gitter
* Motivation: Gittergeister an Gitterspektrographen
* Das Gitter kann als leeres Hologramm verstanden werden ( Überlagerung zweier ebener Wellen)
* Die Hologrammfunktion / Aperturefunktion ( bei optischen Hologrammen eine Intensitätsfunktion -> reell) moduliert dabei die einfallende Rekonstruktionswelle:
 
 
<math>O\acute{\ }=R\cdot I(x,y)=R\cdot (|O|{}^\text{2}+|R|{}^\text{2})+O\cdot |R|{}^\text{2}+R\cdot R\cdot O*</math>
 
* Zu beachten: komplexe Funktionen
 
<u>'''Fresnel- und Fourier- Hologramme'''</u>
 
* Wesentlich für Fourier- Hologramme: Aufzeichnung mittels ebener Wellen
* Linse
* Objekt in weiter Entfernung
* Wesentlich für Fresnelhologramme: Die Objektwelle ist eine Kugelwelle. Das Objekt muss sich also in der Nähe der Hologrammebene befinden.
 
* Fouriernäherung des Beugungsintegrals
* Fresnel- Näherung des Beugungsintegrals
 
*# <u>'''Grundlagen der Beugung'''</u>
 
* Das Beugungsintegral beschreibt die Lichterregung in der Beobachtungsebene.
* Keine Berücksichtigung der Polarisation
* Voraussetzung: kohärente Beleuchtung
 
* Die einfallende Welle wird mit der Aperturefunktion A(x1, y1) ( z.B. Amplitudentransparenz einer Blende oder Phasenaddition durch ein Phasenobjekt) multipliziert und dann über den gesamten Raum integriert.
 
* Fällt das Licht durch ein Hologramm, so muss in die Gleichungen die entsprechende Funktion der Lichttransmission, die das Hologramm beschreibt, gesetzt werden ( Hologramm-/ Aperturefunktion).
 
* Ausgangspunkt:
Helmholtz- Gleichung
<math>(\nabla {}^\text{2}+k{}^\text{2})\,U(\bar{r})=0</math>
 
mit     
<math>U(\bar{r})=\frac{{{e}^{i\bar{k}\bar{r}o1}}}{ro1}</math>
 
* lauter Kugelwellen in x1/y1
 
<math>O(xo,yo)\tilde{\ }\int\limits_{-\infty }^{\infty }{\int\limits_{-\infty }^{\infty }{A(x1,y1)\bullet U(\bar{r})dx1dy1}}</math>
 
<math>\approx \tilde{\ }\frac{1}{z}\int\limits_{-\infty }^{\infty }{\int\limits_{-\infty }^{\infty }{{{e}^{ikr}}\bullet A(x1,y1)}}dx1dy1</math>
 
* Komposition des Objekts durch Interferenz aufgrund von Phasenunterschieden im Raum hinter der Blende
 
'''Reihenentwicklung des Abstandes als Näherung:'''
 
<math>r=\sqrt{(xo-x1){}^\text{2}+(yo-y1){}^\text{2}+z{}^\text{2}}</math>
 
<math>\approx z\left[ 1+\frac{(xo-x1){}^\text{2}+(yo-y1){}^\text{2}}{2z{}^\text{2}} \right]</math>
 
'''Fresnel- Näherung:'''
* Die Hologrammfunktion / Aperturfunktion kodiert das Fresnelsche Beugungsbild
 
<math>O(xo,yo)\approx \tilde{\ }\frac{{{e}^{ikz}}}{z}\int\limits_{-\infty }^{\infty }{\int\limits_{-\infty }^{\infty }{A(x1,y1)}}\bullet {{e}^{\frac{i\pi }{\lambda z}\left[ (xo-x1){}^\text{2}+(yo-y1){}^\text{2} \right]}}dx1dy1</math>
 
'''Fraunhofer- Näherung:'''
* Aufzeichnung allgemein mit Linse
* Zur Aufzeichnung: ebene Welle erforderlich
 
* Das Integral entspricht einer Fouriertransformation der Hologrammfunktion/ Aperturefunktion
 
'''Aufzeichnung:'''
 
 
<u>'''1.3 Beispiele: Einfach- und Doppelspalt'''</u>
 
'''Hintergrund'''
* Laufstreckenunterschiede von kohärenten Lichtstrahlen bestimmen Interferenzerscheinungen
 
Fernfeldnäherungen/ Fouriernäherungen:
 
* Für schmalen Doppelspalt gilt:
 
<math>d\varphi (P)=k\cdot ds=k(r2-r1)\approx k\cdot \sin \theta \cdot a=2\pi \sin \theta \cdot \frac{a}{\lambda }</math>
 
<math>\Rightarrow \sin \theta \cdot a=m\lambda </math>
als Maximabedingung
 
Sofort ersichtlich:
* Variation des Spaltabstands variiert Phase
* Variation der Spaltbreite variiert Amplitude
 
* Breiter Spalt: Interferenz der Strahlen untereinander
* 1. Strahl <-> n/2 +1  ,    2. Stahl <-> N/2 + 2
 
<math>\sin \theta \cdot b=m\lambda </math>
als Minimabedingung
 
<u>'''Der Einfachspalt:'''</u>
 
Amplitudenverteilung bei Einzelspalt:
 
<math>O\tilde{\ }\text{sinc(}\frac{k}{2}\cdot b\cdot \sin \theta )</math>
entspricht Feldverteilung des E-Feldes:
<math>E\tilde{\ }\text{sinc(}\frac{k}{2}\cdot b\cdot \sin \theta )</math>
 
 
 
<math>I(\theta )={{I}_{o}}\cdot \text{sinc }\!\!{}^\text{2}\!\!\text{ (}\frac{k}{2}\cdot b\cdot \sin \theta )</math>
 
<u>'''2. Doppelspalt mit endlicher Spaltbreite/ Minimalgitter:'''</u>
 
 
* Lösung Gesamtproblem: Reihe von Beugungsintegralen
 
Amplitudenverteilung bei Einzelspalt:
 
<math>E\tilde{\ }\text{sinc(}\frac{k}{2}\cdot b\cdot \sin \theta )</math>
 
* Entspricht den Beugungserscheinungen an einer Periode
 
Amplitudenverteilung bei mehreren schmalen Spalten/ Vielstrahlinterferenz:
 
<math>E\tilde{\ }\left\{ \frac{\sin (\frac{Nka}{2}\sin (\theta ))}{\sin (\frac{ka}{2}\sin (\theta ))} \right\}</math>
 
<math>I(\theta )={{I}_{o}}\text{sin}{{\text{c}}^{\text{2}}}\text{(}\frac{k}{2}\cdot b\cdot \sin \theta )\cdot {{\left\{ \frac{\sin (\frac{Nka}{2}\sin (\theta ))}{\sin (\frac{ka}{2}\sin (\theta ))} \right\}}^{2}}</math>
 
* Der Abstand der Spalte ist immer größer als die Breite: a>b
* Die Interferenzstreifen aus Spaltbreite modulieren mit niedriger Frequenz
* Interferenzstreifen aus Spaltanzahl modulieren mit hoher Frequenz
* Für schmale Spalte: Kammfunktion
 
<u>'''5. Materie in elektrischen und magnetischen Feldern'''</u>
 
<u>'''5.1 Polarisation'''</u>
 
Materie enthält mikroskopische elektrisch geladene Bausteine
# '''freie Ladungsträger'''
 
Elektronen in Metallen, Elektronen + Löcher in Halbleitern
* Beschleunigung in äußeren Feldern, E- Felder, B- Felder über Ohmsches Gesetz und Lorentz-kraft
 
<math>\bar{K}=q\left[ \bar{E}+\left( \bar{v}\times \bar{B} \right) \right]</math>
 
* elektrische Ströme -> Beschreibung der Materialeigenschaften durch die elektrische Leitfähigkeit
*
* <math>\sigma </math>
*
 
# '''gebundene Ladungen  ( In Isolatoren)'''
 
* '''Polarisierung im '''<u>'''E- '''</u>'''Feld'''
 
# '''Für '''<u>'''E '''</u>=0 vorhandene mikroskopische Dipole <u>p</u> werden zur Minimierung der potenziellen Energie
Wel.=-<u>p</u> <u>E</u>
vorzugsweise  ( entgegen der zufälligen thermischen Bewegung) parallel zu <u>E </u>orientiert ( z.B. bei polarisierten Molekülen, Wasser etc... gut zu beobachten !)
 
# Nicht- polare Atome oder Moleküle werden dann durch <u>E </u> durch Verschiebung der Ladungswolken polarisiert. Es entstehen induzierte elektrische Dipole, die zu <u>E</u> parallel ausgerichtet sind:
 
<math>\bar{p}=\int_{{}}^{{}}{{{d}^{3}}r}\rho \left( {\bar{r}} \right)\bar{r}\ne 0</math>
nach Einschalten des Feldes.
Es werden in den Atomen/ Molekülen positive und negative Ladungen getrennt !
 
 
 
<u>'''Makroskopische räumliche Mittelung'''</u>
 
Netto- Ladungen entstehen dadurch an den Grenzflächen
 
 
Dies erzeugt im Inneren ein Polarisationsgegenfeld
<math>\begin{align}
& \bar{E}\acute{\ }=\bar{E}+{{{\bar{E}}}_{p}} \\
& {{\varepsilon }_{0}}\nabla \cdot \bar{E}\acute{\ }={{\varepsilon }_{0}}\nabla \cdot \bar{E}+{{\rho }_{P}} \\
\end{align}</math>
gemäß
<math>{{\varepsilon }_{0}}\nabla \cdot {{\bar{E}}_{p}}={{\rho }_{P}}</math>
 
 
Das resultierende Gesamtfeld lautet:
 
<math>\begin{align}
& \bar{E}\acute{\ }=\bar{E}+{{{\bar{E}}}_{p}} \\
& {{\varepsilon }_{0}}\nabla \cdot \bar{E}\acute{\ }={{\varepsilon }_{0}}\nabla \cdot \bar{E}+{{\rho }_{P}} \\
\end{align}</math>
 
Mit der freien Ladungsdichte
 
<math>{{\varepsilon }_{0}}\nabla \cdot \bar{E}=\rho </math>
 
Also:
 
<math>{{\varepsilon }_{0}}\nabla \cdot \bar{E}\acute{\ }=\rho +{{\rho }_{P}}</math>
 
Die Polarisation selbst bestimmt sich nach
 
<math>\bar{P}\left( \bar{r},t \right):=-{{\varepsilon }_{0}}{{\bar{E}}_{p}}\left( \bar{r},t \right)</math>
 
ein makroskopisches lokales Feld, dessen Quelle Polarisationsladungen sind.
 
Somit:
 
<math>\begin{align}
& \nabla \cdot \left( {{\varepsilon }_{0}}\bar{E}\acute{\ }+\bar{P} \right)=\rho  \\
& \nabla \cdot \bar{P}=-{{\rho }_{P}} \\
\end{align}</math>
 
Als Dielektrische Verschiebung bezeichnen wir
 
<math>\bar{D}(\bar{r},t)=\left( {{\varepsilon }_{0}}\bar{E}\acute{\ }+\bar{P} \right)</math>
 
Dies ist die effektive makroskopische Feldgröße, als dessen Quellen nur noch die freien Ladungen ( ohne Polarisationsladungen) auftreten:
 
<math>\nabla \cdot \bar{D}=\rho </math>
 
Wir bezeichnen mit
 
<math>\bar{P}\left( \bar{r},t \right)d\bar{f}=d{{Q}_{P}}</math>
 
die Polarisationsladung, die beim Übergang vom unpolarisierten zum polarisierten Zustand  durch die Fläche df verschoben wird:
 
 
 
Denn ( bei Betrachtung eines Volumens V, das durch df begrenzt ist):
 
<math>\oint_{\partial V}{{}}\bar{P}\left( \bar{r},t \right)d\bar{f}=\int_{V}^{{}}{{{d}^{3}}r\nabla \cdot \bar{P}\left( \bar{r},t \right)=-\int_{V}^{{}}{{}}{{d}^{3}}r}{{\rho }_{P}}</math>
 
= Polarisationsladung, die V verläßt !
 
<u>'''Zusammenhang mikroskopische elektrische Dipole / makroskopische Größen:'''</u>
 
<math>{{\rho }_{m}}\left( \bar{r},t \right)=\sum\limits_{i}{{}}{{q}_{i}}\delta \left( \bar{r}-{{{\bar{r}}}_{i}}(t) \right)</math>
( mikroskopische Ladungsdichte)
 
<math>{{\bar{P}}_{m}}\left( \bar{r},t \right)=\sum\limits_{i}{{}}{{\bar{p}}_{i}}\delta \left( \bar{r}-{{{\bar{r}}}_{i}}(t) \right)</math>
( mikroskopische Dipoldichte) mit:
 
<math>\int_{V}^{{}}{{{d}^{3}}r{{{\bar{P}}}_{m}}\left( \bar{r},t \right)=\sum\limits_{i}{{}}{{{\bar{p}}}_{i}}}</math>
 
Mittelung über ein kleines makroskopisches Volumen
<math>\Delta V:</math>
 
<math>{{\left( \Delta V \right)}^{\frac{1}{3}}}<<</math>
Längenskala der makroskopischen Dichtevariation
 
Somit:
 
<math>\rho \left( \bar{r},t \right)=\frac{1}{\Delta V}\int_{\Delta V}^{{}}{{}}{{d}^{3}}s{{\rho }_{m}}\left( \bar{r}+\bar{s},t \right)</math>
( makroskopische Ladungsdichte)
 
<math>\bar{P}\left( \bar{r},t \right)=\frac{1}{\Delta V}\int_{\Delta V}^{{}}{{}}{{d}^{3}}s{{\bar{P}}_{m}}\left( \bar{r}+\bar{s},t \right)</math>
 
Also: Die makroskopische Dipoldichte ist GLEICH DER POLARISATION !!
 
'''Beweis:'''
 
Betrachten wir das mikroskopische retardierte Potenzial:
 
<math>{{\Phi }_{m}}\left( \bar{r},t \right)=\frac{1}{4\pi {{\varepsilon }_{0}}}\int_{{{R}^{3}}}^{{}}{{}}{{d}^{3}}r\acute{\ }\frac{{{\rho }_{m}}\left( \bar{r}\acute{\ },t-\frac{\left| \bar{r}-\bar{r}\acute{\ } \right|}{c} \right)}{\left| \bar{r}-\bar{r}\acute{\ } \right|}</math>
 
wobei unter dem Integral die mikroskopische Ladungsdichte einzusetzen ist !
 
Das makroskopisch gemittelte Potenzial folgt dann gemäß
 
<math>\begin{align}
& \Phi \left( \bar{r},t \right)=\frac{1}{\Delta V}\int_{\Delta V}^{{}}{{}}{{d}^{3}}s{{\Phi }_{m}}\left( \bar{r}+\bar{s},t \right)=\frac{1}{4\pi {{\varepsilon }_{0}}}\frac{1}{\Delta V}\int_{\Delta V}^{{}}{{}}{{d}^{3}}s\int_{{{R}^{3}}}^{{}}{{}}{{d}^{3}}r\acute{\ }\frac{{{\rho }_{m}}\left( \bar{r}\acute{\ },t-\frac{\left| \bar{r}+\bar{s}-\bar{r}\acute{\ } \right|}{c} \right)}{\left| \bar{r}+\bar{s}-\bar{r}\acute{\ } \right|} \\
& \bar{r}\acute{\ }\acute{\ }:=\bar{r}\acute{\ }-\bar{s} \\
& \Phi \left( \bar{r},t \right)=\frac{1}{4\pi {{\varepsilon }_{0}}}\frac{1}{\Delta V}\int_{\Delta V}^{{}}{{}}{{d}^{3}}s\int_{{{R}^{3}}}^{{}}{{}}{{d}^{3}}r\acute{\ }\acute{\ }\frac{{{\rho }_{m}}\left( \bar{r}\acute{\ }\acute{\ }+\bar{s},t-\frac{\left| \bar{r}-\bar{r}\acute{\ }\acute{\ } \right|}{c} \right)}{\left| \bar{r}-\bar{r}\acute{\ }\acute{\ } \right|} \\
& =\frac{1}{4\pi {{\varepsilon }_{0}}}\int_{{{R}^{3}}}^{{}}{{}}{{d}^{3}}r\acute{\ }\acute{\ }\frac{1}{\left| \bar{r}-\bar{r}\acute{\ }\acute{\ } \right|}\frac{1}{\Delta V}\int_{\Delta V}^{{}}{{}}{{d}^{3}}s{{\rho }_{m}}\left( \bar{r}\acute{\ }\acute{\ }+\bar{s},t-\frac{\left| \bar{r}-\bar{r}\acute{\ }\acute{\ } \right|}{c} \right) \\
\end{align}</math>
 
Wobei
 
<math>\frac{1}{\Delta V}\int_{\Delta V}^{{}}{{}}{{d}^{3}}s{{\rho }_{m}}\left( \bar{r}\acute{\ }\acute{\ }+\bar{s},t-\frac{\left| \bar{r}-\bar{r}\acute{\ }\acute{\ } \right|}{c} \right)=\rho \left( \bar{r}\acute{\ }\acute{\ }+\bar{s},t-\frac{\left| \bar{r}-\bar{r}\acute{\ }\acute{\ } \right|}{c} \right)</math>
 
Die makroskopische Ladungsdichte ist !
 
<math>\begin{align}
& \Rightarrow \Phi \left( \bar{r},t \right)=\frac{1}{4\pi {{\varepsilon }_{0}}}\int_{{{R}^{3}}}^{{}}{{}}{{d}^{3}}r\acute{\ }\acute{\ }\frac{1}{\left| \bar{r}-\bar{r}\acute{\ }\acute{\ } \right|}\frac{1}{\Delta V}\int_{\Delta V}^{{}}{{}}{{d}^{3}}s{{\rho }_{m}}\left( \bar{r}\acute{\ }\acute{\ }+\bar{s},t-\frac{\left| \bar{r}-\bar{r}\acute{\ }\acute{\ } \right|}{c} \right) \\
& =\frac{1}{4\pi {{\varepsilon }_{0}}}\int_{{{R}^{3}}}^{{}}{{}}{{d}^{3}}r\acute{\ }\acute{\ }\frac{1}{\left| \bar{r}-\bar{r}\acute{\ }\acute{\ } \right|}\rho \left( \bar{r}\acute{\ }\acute{\ }+\bar{s},t-\frac{\left| \bar{r}-\bar{r}\acute{\ }\acute{\ } \right|}{c} \right) \\
\end{align}</math>
 
'''Analog:'''
 
Das mikroskopische Potenzial der elektrischen Dipole
 
<math>{{\bar{p}}_{i}}</math>
:
 
<math>{{\Phi }_{m}}\left( \bar{r},t \right)=-\frac{1}{4\pi {{\varepsilon }_{0}}}{{\nabla }_{r}}\left\{ \sum\limits_{i}{{}}\frac{1}{\left| \bar{r}-{{{\bar{r}}}_{i}} \right|}{{{\bar{p}}}_{i}}\left( t-\frac{\left| \bar{r}-{{{\bar{r}}}_{i}} \right|}{c} \right) \right\}</math>
 
mit dem mikroskopischen Dipolmoment
 
<math>{{\bar{p}}_{i}}\left( t-\frac{\left| \bar{r}-{{{\bar{r}}}_{i}} \right|}{c} \right)</math>
 
Analog:
 
<math>{{\Phi }_{m}}\left( \bar{r},t \right)=-\frac{1}{4\pi {{\varepsilon }_{0}}}\int_{{{R}^{3}}}^{{}}{{}}{{d}^{3}}r\acute{\ }{{\nabla }_{r}}\left\{ \frac{1}{\left| \bar{r}-\bar{r}\acute{\ } \right|}{{{\bar{P}}}_{m}}\left( \bar{r}\acute{\ },t-\frac{\left| \bar{r}-\bar{r}\acute{\ } \right|}{c} \right) \right\}</math>
 
mit der mikroskopischen Dipoldichte
 
<math>{{\bar{P}}_{m}}\left( \bar{r}\acute{\ },t-\frac{\left| \bar{r}-\bar{r}\acute{\ } \right|}{c} \right)</math>
 
Somit ergibt sich für das makroskopisch gemittelte elektrische Potenzial:
 
<math>\begin{align}
& \Phi \left( \bar{r},t \right)=\frac{1}{\Delta V}\int_{\Delta V}^{{}}{{}}{{d}^{3}}s{{\Phi }_{m}}\left( \bar{r}+\bar{s},t \right) \\
& =-\frac{1}{4\pi {{\varepsilon }_{0}}}\frac{1}{\Delta V}\int_{\Delta V}^{{}}{{}}{{d}^{3}}s\int_{{{R}^{3}}}^{{}}{{}}{{d}^{3}}r\acute{\ }{{\nabla }_{r}}\left\{ \frac{1}{\left| \bar{r}+\bar{s}-\bar{r}\acute{\ } \right|}{{{\bar{P}}}_{m}}\left( \bar{r}\acute{\ },t-\frac{\left| \bar{r}+\bar{s}-\bar{r}\acute{\ } \right|}{c} \right) \right\} \\
& =-\frac{1}{4\pi {{\varepsilon }_{0}}}\int_{{{R}^{3}}}^{{}}{{}}{{d}^{3}}r\acute{\ }\acute{\ }{{\nabla }_{r}}\left\{ \frac{1}{\left| \bar{r}-\bar{r}\acute{\ }\acute{\ } \right|}\bar{P}\left( \bar{r}\acute{\ }\acute{\ },t-\frac{\left| \bar{r}-\bar{r}\acute{\ }\acute{\ } \right|}{c} \right) \right\} \\
\end{align}</math>
 
'''Umformung:'''
 
<math>{{\nabla }_{r}}\left\{ \frac{1}{\left| \bar{r}-\bar{r}\acute{\ } \right|}\bar{P}\left( \bar{r}\acute{\ },t-\frac{\left| \bar{r}-\bar{r}\acute{\ } \right|}{c} \right) \right\}=-{{\nabla }_{r\acute{\ }}}\left\{ \frac{1}{\left| \bar{r}-\bar{r}\acute{\ } \right|}\bar{P}\left( \bar{r}\acute{\ },t-\frac{\left| \bar{r}-\bar{r}\acute{\ } \right|}{c} \right) \right\}+Korrektur</math>
 
Dabei haben wir das Problem , dass beim Übergang zur gestrichenen Ableitung hier auch nach dem Argument r´ von P abgeleitet wird. Also müssen wir dies wieder abziehen:
 
<math>\begin{align}
& {{\nabla }_{r}}\left\{ \frac{1}{\left| \bar{r}-\bar{r}\acute{\ } \right|}\bar{P}\left( \bar{r}\acute{\ },t-\frac{\left| \bar{r}-\bar{r}\acute{\ } \right|}{c} \right) \right\}=-{{\nabla }_{r\acute{\ }}}\left\{ \frac{1}{\left| \bar{r}-\bar{r}\acute{\ } \right|}\bar{P}\left( \bar{r}\acute{\ },t-\frac{\left| \bar{r}-\bar{r}\acute{\ } \right|}{c} \right) \right\}+\frac{1}{\left| \bar{r}-\bar{r}\acute{\ } \right|}{{\nabla }_{r\acute{\ }}}\bar{P}\left( \bar{r}\acute{\ },t-\frac{\left| \bar{r}-\bar{r}\acute{\ } \right|}{c} \right) \\
& t\acute{\ }=t-\frac{\left| \bar{r}-\bar{r}\acute{\ } \right|}{c} \\
& {{\nabla }_{r}}\left\{ \frac{1}{\left| \bar{r}-\bar{r}\acute{\ } \right|}\bar{P}\left( \bar{r}\acute{\ },t-\frac{\left| \bar{r}-\bar{r}\acute{\ } \right|}{c} \right) \right\}=-{{\nabla }_{r\acute{\ }}}\left\{ \frac{1}{\left| \bar{r}-\bar{r}\acute{\ } \right|}\bar{P}\left( \bar{r}\acute{\ },t-\frac{\left| \bar{r}-\bar{r}\acute{\ } \right|}{c} \right) \right\}+\frac{1}{\left| \bar{r}-\bar{r}\acute{\ } \right|}{{\nabla }_{r\acute{\ }}}\bar{P}\left( \bar{r}\acute{\ },t\acute{\ } \right) \\
\end{align}</math>
 
Also folgt für das Potenzial:
Dies ist das makroskopische Potenzial einer Polarisationsladungsdichte
 
<math>{{\rho }_{p}}\left( \bar{r}\acute{\ },t\acute{\ } \right)=\left( -{{\nabla }_{r\acute{\ }}}\bar{P}\left( \bar{r}\acute{\ },t\acute{\ } \right) \right)</math>
 
Damit können wir die makroskopische Dipoldichte
<math>\bar{P}</math>
mit der durch
<math>\bar{P}:=-{{\varepsilon }_{0}}{{\bar{E}}_{p}}</math>
bzw.
 
<math>\nabla \cdot \bar{P}=-{{\rho }_{p}}</math>
definierten Polarisation identifizieren.
 
<u>'''5.2 Magnetisierung'''</u>
 
Mirkoskopische Ursache für den Magnetismus der Materie sind mikroskopische Kreisströme bzw. mikroskopische magnetische Dipolmomente
<math>\bar{m}</math>
:
 
a) Für
<math>\bar{B}=0</math>
vorhandene, permanente magnetische Momente
<math>\bar{m}</math>
werden zur Minimierung der potenziellen Energie
<math>{{W}_{mag.}}=-\bar{m}\bar{B}</math>
vorzugsweise ( entgegen der thermischen Bewegung) parallel zum äußeren B- Feld orientiert. Beispiel: Spin- Bahn- Momente von Elektronen
* paramagnetisches Verhalten
 
# durch B können nach dem Faradayschen Induktionsgesetz Kreisströme freier oder gebundener Ladungen induziert werden. Wegen der Lenzschen Regel ist die induzierte Magnetisierung antiparallel zum äußeren B- Feld.
* diamagnetisches Verhalten !
 
<u>'''Makroskopisch gemittelte Felder'''</u>
 
mikroskopische magnetische Dipoldichte:
Wie bei Polarisationsdichte:
 
<math>\begin{align}
& {{{\bar{M}}}_{m}}\left( \bar{r},t \right)=\sum\limits_{i}{{}}{{{\bar{m}}}_{i}}(t)\delta \left( \bar{r}-{{{\bar{r}}}_{i}} \right) \\
& {{{\bar{P}}}_{m}}\left( \bar{r},t \right)=\sum\limits_{i}{{}}{{{\bar{p}}}_{i}}(t)\delta \left( \bar{r}-{{{\bar{r}}}_{i}} \right)\quad el.Dipoldichte \\
\end{align}</math>
 
Mittelung über ein kleines, makroskopisches Volumen
<math>\Delta V</math>
:
 
<math>\bar{M}\left( \bar{r},t \right)=\frac{1}{\Delta V}\int_{\Delta V}^{{}}{{}}{{d}^{3}}s{{\bar{M}}_{m}}\left( \bar{r}+\bar{s},t \right)</math>
 
'''makroskopische magnetische Dipoldichte:= Magnetisierung'''
 
<u>'''Ziel:'''</u>
Zusammenhang zwischen der magnetischen Dipoldichte
<math>\bar{M}\left( \bar{r},t \right)</math>
und den effektiven Feldern
<math>\bar{B}</math>
in der Materie finden.
Hierzu zeige man, dass eine Magnetisierungsstromdichte
<math>{{\bar{j}}_{M}}</math>
 
als Quelle der Felder eingeführt werden kann:
 
<math>\nabla \times {{\bar{B}}_{M}}={{\mu }_{0}}{{\bar{j}}_{M}}</math>
 
bzw.
 
<math>\nabla \times \bar{M}={{\bar{j}}_{M}}</math>
 
'''effektive Gesamtinduktion ( im stationären Fall):'''
 
<math>\begin{align}
& \bar{B}\acute{\ }=\bar{B}+{{{\bar{B}}}_{M}} \\
& \Rightarrow \nabla \times \left( \frac{1}{{{\mu }_{0}}}\bar{B}\acute{\ } \right)=\nabla \times \left( \frac{1}{{{\mu }_{0}}}\bar{B} \right)+{{{\bar{j}}}_{M}}=\bar{j}+{{{\bar{j}}}_{M}} \\
\end{align}</math>
 
Also: Erzeugung des B- Feldes ( Differenz aus effektiver Gesamtinduktion und Magnetisierung) durch den sogenannte freien Strom <u>j :</u>
 
<math>\begin{align}
& \bar{B}\acute{\ }=\bar{B}+{{{\bar{B}}}_{M}} \\
& \nabla \times \left( \frac{1}{{{\mu }_{0}}}\bar{B}\acute{\ }-\bar{M} \right)=\bar{j} \\
& \bar{H}=\frac{1}{{{\mu }_{0}}}\left( \frac{1}{{{\mu }_{0}}}\bar{B}\acute{\ }-{{{\bar{B}}}_{M}} \right)=\frac{{\bar{B}}}{{{\mu }_{0}}} \\
\end{align}</math>
 
Betrachten wir das Vektorpotenzial der mikroskopischen elektrischen und magnetischen Dipole:
 
<math>\begin{align}
& {{{\bar{A}}}_{m}}\left( \bar{r},t \right)=\frac{{{\mu }_{0}}}{4\pi }\sum\limits_{i}{{}}\left[ \frac{1}{\left| \bar{r}-{{{\bar{r}}}_{i}} \right|}{{{\dot{\bar{p}}}}_{i}}\left( t-\frac{\left| \bar{r}-{{{\bar{r}}}_{i}} \right|}{c} \right)+\nabla \times \left( \frac{1}{\left| \bar{r}-{{{\bar{r}}}_{i}} \right|}{{{\bar{m}}}_{i}}\left( t-\frac{\left| \bar{r}-{{{\bar{r}}}_{i}} \right|}{c} \right) \right) \right] \\
& {{{\bar{p}}}_{i}}\left( t-\frac{\left| \bar{r}-{{{\bar{r}}}_{i}} \right|}{c} \right)\quad elektrDipolmoment \\
& {{{\bar{m}}}_{i}}\left( t-\frac{\left| \bar{r}-{{{\bar{r}}}_{i}} \right|}{c} \right)\quad magnetDipolmoment \\
& \Rightarrow {{{\bar{A}}}_{m}}\left( \bar{r},t \right)=\frac{{{\mu }_{0}}}{4\pi }\int_{{}}^{{}}{{}}{{d}^{3}}r\acute{\ }\left[ \frac{1}{\left| \bar{r}-\bar{r}\acute{\ } \right|}{{{\dot{\bar{p}}}}_{m}}\left( \bar{r}\acute{\ },t-\frac{\left| \bar{r}-\bar{r}\acute{\ } \right|}{c} \right)+{{\nabla }_{r}}\times \left( \frac{1}{\left| \bar{r}-\bar{r}\acute{\ } \right|}{{{\bar{M}}}_{m}}\left( \bar{r}\acute{\ },t-\frac{\left| \bar{r}-\bar{r}\acute{\ } \right|}{c} \right) \right) \right] \\
\end{align}</math>
 
mit der mikroskopischen elektrischen Dipoldichte
 
<math>{{\bar{p}}_{m}}\left( \bar{r}\acute{\ },t-\frac{\left| \bar{r}-\bar{r}\acute{\ } \right|}{c} \right)</math>
 
und der magnetischen Dipoldichte
 
<math>{{\bar{M}}_{m}}\left( \bar{r}\acute{\ },t-\frac{\left| \bar{r}-\bar{r}\acute{\ } \right|}{c} \right)</math>
 
'''Als makroskopisch gemitteltes Potenzial:'''
 
<math>\begin{align}
& \bar{A}\left( \bar{r},t \right)=\frac{1}{\Delta V}\int_{\Delta V}^{{}}{{}}{{d}^{3}}s{{{\bar{A}}}_{m}}\left( \bar{r}+\bar{s},t \right) \\
& =\frac{{{\mu }_{0}}}{4\pi }\frac{1}{\Delta V}\int_{\Delta V}^{{}}{{}}{{d}^{3}}s\int_{{}}^{{}}{{}}{{d}^{3}}r\acute{\ }\left[ \frac{1}{\left| \bar{r}+\bar{s}-\bar{r}\acute{\ } \right|}{{{\dot{\bar{p}}}}_{m}}\left( \bar{r}\acute{\ },t-\frac{\left| \bar{r}+\bar{s}-\bar{r}\acute{\ } \right|}{c} \right)+{{\nabla }_{r}}\times \left( \frac{1}{\left| \bar{r}+\bar{s}-\bar{r}\acute{\ } \right|}{{{\bar{M}}}_{m}}\left( \bar{r}\acute{\ },t-\frac{\left| \bar{r}+\bar{s}-\bar{r}\acute{\ } \right|}{c} \right) \right) \right] \\
& ==\frac{{{\mu }_{0}}}{4\pi }\int_{{}}^{{}}{{}}{{d}^{3}}r\acute{\ }\left[ \frac{1}{\left| \bar{r}-\bar{r}\acute{\ } \right|}\dot{\bar{P}}\left( \bar{r}\acute{\ },t-\frac{\left| \bar{r}-\bar{r}\acute{\ } \right|}{c} \right)+{{\nabla }_{r}}\times \left( \frac{1}{\left| \bar{r}-\bar{r}\acute{\ } \right|}\bar{M}\left( \bar{r}\acute{\ },t-\frac{\left| \bar{r}-\bar{r}\acute{\ } \right|}{c} \right) \right) \right] \\
\end{align}</math>
 
'''Wobei '''nur die makroskopischen Dichten einzusetzen sind ( vergleiche oben)
 
Umformung liefert:
 
<math>\begin{align}
& \bar{A}\left( \bar{r},t \right)=\frac{{{\mu }_{0}}}{4\pi }\int_{{}}^{{}}{{}}{{d}^{3}}r\acute{\ }\left[ \frac{1}{\left| \bar{r}-\bar{r}\acute{\ } \right|}\dot{\bar{P}}\left( \bar{r}\acute{\ },t-\frac{\left| \bar{r}-\bar{r}\acute{\ } \right|}{c} \right)+{{\nabla }_{r}}\times \left( \frac{1}{\left| \bar{r}-\bar{r}\acute{\ } \right|}\bar{M}\left( \bar{r}\acute{\ },t-\frac{\left| \bar{r}-\bar{r}\acute{\ } \right|}{c} \right) \right) \right] \\
& \int_{{}}^{{}}{{}}{{d}^{3}}r\acute{\ }{{\nabla }_{r}}\times \left( \frac{1}{\left| \bar{r}-\bar{r}\acute{\ } \right|}\bar{M}\left( \bar{r}\acute{\ },t-\frac{\left| \bar{r}-\bar{r}\acute{\ } \right|}{c} \right) \right)= \\
& =-\int_{{}}^{{}}{{}}{{d}^{3}}r\acute{\ }{{\nabla }_{r\acute{\ }}}\times \left( \frac{1}{\left| \bar{r}-\bar{r}\acute{\ } \right|}\bar{M}\left( \bar{r}\acute{\ },t-\frac{\left| \bar{r}-\bar{r}\acute{\ } \right|}{c} \right) \right)+\int_{{}}^{{}}{{}}{{d}^{3}}r\acute{\ }\frac{1}{\left| \bar{r}-\bar{r}\acute{\ } \right|}{{\nabla }_{r\acute{\ }}}\times \bar{M}\left( \bar{r}\acute{\ },t\acute{\ } \right) \\
& t\acute{\ }=t-\frac{\left| \bar{r}-\bar{r}\acute{\ } \right|}{c} \\
& \int_{{}}^{{}}{{}}{{d}^{3}}r\acute{\ }{{\nabla }_{r\acute{\ }}}\times \left( \frac{1}{\left| \bar{r}-\bar{r}\acute{\ } \right|}\bar{M}\left( \bar{r}\acute{\ },t-\frac{\left| \bar{r}-\bar{r}\acute{\ } \right|}{c} \right) \right)=0 \\
& \Rightarrow \bar{A}\left( \bar{r},t \right)=\frac{{{\mu }_{0}}}{4\pi }\int_{{}}^{{}}{{}}{{d}^{3}}r\acute{\ }\left[ \frac{1}{\left| \bar{r}-\bar{r}\acute{\ } \right|}\dot{\bar{P}}\left( \bar{r}\acute{\ },t-\frac{\left| \bar{r}-\bar{r}\acute{\ } \right|}{c} \right)+\frac{1}{\left| \bar{r}-\bar{r}\acute{\ } \right|}{{\nabla }_{r\acute{\ }}}\times \bar{M}\left( \bar{r}\acute{\ },t\acute{\ } \right) \right] \\
\end{align}</math>
 
'''Definition'''
 
<math>\begin{align}
& \dot{\bar{P}}={{{\bar{j}}}_{p}} \\
& {{\nabla }_{r\acute{\ }}}\times \bar{M}\left( \bar{r}\acute{\ },t\acute{\ } \right)={{{\bar{j}}}_{M}} \\
\end{align}</math>
 
Ersteres: Polarisationsstromdichte
'''Letzteres: Magnetisierungsstromdichte'''
 
'''Also:'''
 
<math>\bar{A}\left( \bar{r},t \right)=\frac{{{\mu }_{0}}}{4\pi }\int_{{}}^{{}}{{}}{{d}^{3}}r\acute{\ }\frac{1}{\left| \bar{r}-\bar{r}\acute{\ } \right|}\left[ {{{\bar{j}}}_{p}}\left( \bar{r}\acute{\ },t-\frac{\left| \bar{r}-\bar{r}\acute{\ } \right|}{c} \right)+{{{\bar{j}}}_{M}}\left( \bar{r}\acute{\ },t\acute{\ } \right) \right]</math>
 
Das heißt, das makroskopisch gemittelte retardierte Vektorpotenzial wird durch die Polarisations- und Magnetisierungsstromdichten im Medium erzeugt !
 
es gilt der Erhaltungssatz:
 
<math>\begin{align}
& \frac{\partial }{\partial t\acute{\ }}{{\rho }_{p}}=-\nabla \cdot \dot{\bar{P}}=-\nabla \cdot {{{\bar{j}}}_{p}} \\
& \Rightarrow {{{\dot{\rho }}}_{p}}+\nabla \cdot {{{\bar{j}}}_{p}}=0 \\
\end{align}</math>
 
Kontinuitätsgleichung für die Erhaltung der Polarisationsladung !
 
<u>'''5.3 Maxwell- Gleichungen in Materie'''</u>
 
Die vollständigen Potenziale enthalten
* die freie Ladungs- und Stromdichten
* <math>\rho ,\bar{j}</math>
*
* die Polarisations- und Magnetisierungsbeiträge
* <math>{{\rho }_{p}},{{\bar{j}}_{p}},{{\bar{j}}_{m}}</math>
*
 
Somit folgt für die vollständigen Potenziale:
 
<math>\begin{align}
& t\acute{\ }=t-\frac{\left| \bar{r}-\bar{r}\acute{\ } \right|}{c} \\
& \bar{A}\left( \bar{r},t \right)=\frac{{{\mu }_{\acute{\ }0}}}{4\pi }\int_{{}}^{{}}{{}}{{d}^{3}}r\acute{\ }\frac{1}{\left| \bar{r}-\bar{r}\acute{\ } \right|}\left[ \bar{j}\left( \bar{r}\acute{\ },t-\frac{\left| \bar{r}-\bar{r}\acute{\ } \right|}{c} \right)+{{{\bar{j}}}_{P}}\left( \bar{r}\acute{\ },t-\frac{\left| \bar{r}-\bar{r}\acute{\ } \right|}{c} \right)+{{{\bar{j}}}_{M}}\left( \bar{r}\acute{\ },t-\frac{\left| \bar{r}-\bar{r}\acute{\ } \right|}{c} \right) \right] \\
& \Phi \left( \bar{r},t \right)=\frac{1}{4\pi {{\varepsilon }_{0}}}\int_{{}}^{{}}{{}}{{d}^{3}}r\acute{\ }\frac{1}{\left| \bar{r}-\bar{r}\acute{\ } \right|}\left[ \rho \left( \bar{r}\acute{\ },t-\frac{\left| \bar{r}-\bar{r}\acute{\ } \right|}{c} \right)+{{\rho }_{P}}\left( \bar{r}\acute{\ },t-\frac{\left| \bar{r}-\bar{r}\acute{\ } \right|}{c} \right) \right] \\
&  \\
\end{align}</math>
 
Diese Potenziale sind Lösungen der inhomogenen Wellengleichung in Lorentz- Eichung
 
<math>\begin{align}
& \#\bar{A}\left( \bar{r},t \right)=-{{\mu }_{\acute{\ }0}}\left[ \bar{j}+{{{\bar{j}}}_{P}}+{{{\bar{j}}}_{M}} \right] \\
& \#\Phi \left( \bar{r},t \right)=-\frac{1}{{{\varepsilon }_{0}}}\left[ \rho +{{\rho }_{P}} \right] \\
&  \\
\end{align}</math>
 
Für die Felder in Materie folgt:
 
<math>\begin{align}
& \bar{E}=-\nabla \Phi \left( \bar{r},t \right)-\frac{\partial }{\partial t}\bar{A}\left( \bar{r},t \right) \\
& \bar{B}=\nabla \times \bar{A}\left( \bar{r},t \right) \\
\end{align}</math>
 
Daraus folgen die Maxwell- Gleichungen:
 
<math>\begin{align}
& 1)\nabla \times \bar{E}=-\frac{\partial }{\partial t}\nabla \times \bar{A}\left( \bar{r},t \right)=-\frac{\partial }{\partial t}\bar{B} \\
& 2)\nabla \cdot \bar{B}=0 \\
\end{align}</math>
 
* Wie im Vakuum
 
<math>\begin{align}
& 3)\nabla \cdot \bar{E}=-\frac{\partial }{\partial t}\nabla \cdot \bar{A}\left( \bar{r},t \right)-\nabla \cdot \nabla \Phi  \\
& \nabla \cdot \bar{A}\left( \bar{r},t \right)=-\frac{1}{{{c}^{2}}}\frac{\partial }{\partial t}\Phi  \\
& \Rightarrow \nabla \cdot \bar{E}=-\frac{\partial }{\partial t}\nabla \cdot \bar{A}\left( \bar{r},t \right)-\nabla \cdot \nabla \Phi =\frac{1}{{{c}^{2}}}\frac{{{\partial }^{2}}}{\partial {{t}^{2}}}\Phi -\Delta \Phi =-\#\Phi  \\
\end{align}</math>
 
In Lorentz Eichung !
 
<math>\nabla \cdot \bar{E}=-\frac{\partial }{\partial t}\nabla \cdot \bar{A}\left( \bar{r},t \right)-\nabla \cdot \nabla \Phi =\frac{1}{{{c}^{2}}}\frac{{{\partial }^{2}}}{\partial {{t}^{2}}}\Phi -\Delta \Phi =-\#\Phi =\frac{1}{{{\varepsilon }_{0}}}\left( \rho +{{\rho }_{p}} \right)=\frac{1}{{{\varepsilon }_{0}}}\left( \rho -\nabla \cdot \bar{P} \right)</math>
 
per Definition von
<math>{{\rho }_{p}}</math>
.
 
<math>\begin{align}
& \Rightarrow 3)\nabla \cdot \left( {{\varepsilon }_{0}}\bar{E}\left( \bar{r},t \right)+\bar{P}\left( \bar{r},t \right) \right)=\rho \left( \bar{r},t \right) \\
& \left( {{\varepsilon }_{0}}\bar{E}\left( \bar{r},t \right)+\bar{P}\left( \bar{r},t \right) \right):=\bar{D}\left( \bar{r},t \right) \\
& \Rightarrow \nabla \cdot \bar{D}\left( \bar{r},t \right)=\rho \left( \bar{r},t \right) \\
\end{align}</math>
 
Die Dielektrische Verschiebung
 
4) Letzte Gleichung:
 
<math>\begin{align}
& \nabla \times \bar{B}\left( \bar{r},t \right)=\nabla \times \left( \nabla \times \bar{A}\left( \bar{r},t \right) \right)=\nabla \left( \nabla \cdot \bar{A}\left( \bar{r},t \right) \right)-\Delta \bar{A}\left( \bar{r},t \right) \\
& \nabla \cdot \bar{A}\left( \bar{r},t \right)=-\frac{1}{{{c}^{2}}}\frac{\partial }{\partial t}\Phi  \\
& \Rightarrow \nabla \times \bar{B}\left( \bar{r},t \right)=-\Delta \bar{A}\left( \bar{r},t \right)-\frac{1}{{{c}^{2}}}\frac{\partial }{\partial t}\nabla \Phi  \\
& \nabla \Phi =-\bar{E}-\frac{\partial }{\partial t}\bar{A}\left( \bar{r},t \right) \\
& \Rightarrow \nabla \times \bar{B}\left( \bar{r},t \right)=-\Delta \bar{A}\left( \bar{r},t \right)+\frac{1}{{{c}^{2}}}\frac{\partial }{\partial t}\bar{E}+\frac{1}{{{c}^{2}}}\frac{{{\partial }^{2}}}{\partial {{t}^{2}}}\bar{A}\left( \bar{r},t \right)=-\#\bar{A}\left( \bar{r},t \right)+\frac{1}{{{c}^{2}}}\frac{\partial }{\partial t}\bar{E} \\
& ={{\mu }_{0}}\left( \bar{j}+{{{\bar{j}}}_{P}}+{{{\bar{j}}}_{M}} \right)+{{\varepsilon }_{0}}{{\mu }_{0}}\frac{\partial }{\partial t}\bar{E} \\
& {{{\bar{j}}}_{P}}=\dot{\bar{P}} \\
& {{{\bar{j}}}_{M}}=\nabla \times \bar{M} \\
& \Rightarrow \nabla \times \bar{B}\left( \bar{r},t \right)={{\mu }_{0}}\frac{\partial }{\partial t}\left( \bar{P}+{{\varepsilon }_{0}}\bar{E} \right)+{{\mu }_{0}}\nabla \times \bar{M}+{{\mu }_{0}}\bar{j} \\
& \Rightarrow 4) \\
& \Rightarrow \nabla \times \left( \frac{1}{{{\mu }_{0}}}\bar{B}\left( \bar{r},t \right)-\bar{M} \right)=\bar{j}+\frac{\partial }{\partial t}\bar{D} \\
& \left( \frac{1}{{{\mu }_{0}}}\bar{B}\left( \bar{r},t \right)-\bar{M} \right)=H\left( \bar{r},t \right) \\
& \Rightarrow \nabla \times H\left( \bar{r},t \right)=\bar{j}+\frac{\partial }{\partial t}\bar{D} \\
\end{align}</math>
 
Mit dem Magnetfeld
<math>H\left( \bar{r},t \right)</math>
, welches so definiert wurde, dass es nur durch die FREIEN Ströme erzeugt wird:
 
<u>'''Zusammenfassung:'''</u>
 
<math>\begin{align}
& 1)\nabla \times \bar{E}=-\frac{\partial }{\partial t}\nabla \times \bar{A}\left( \bar{r},t \right)=-\frac{\partial }{\partial t}\bar{B} \\
& 2)\nabla \cdot \bar{B}=0 \\
\end{align}</math>
 
<math>3)\nabla \cdot \bar{D}\left( \bar{r},t \right)=\rho \left( \bar{r},t \right)</math>
 
<math>4)\nabla \times H\left( \bar{r},t \right)=\bar{j}+\frac{\partial }{\partial t}\bar{D}</math>
 
<math>4)\nabla \times H\left( \bar{r},t \right)-\frac{\partial }{\partial t}\bar{D}=\bar{j}</math>
 
Dabei beschreibt
 
<math>\begin{align}
& 1)\nabla \times \bar{E}=-\frac{\partial }{\partial t}\nabla \times \bar{A}\left( \bar{r},t \right)=-\frac{\partial }{\partial t}\bar{B} \\
& 2)\nabla \cdot \bar{B}=0 \\
\end{align}</math>
 
die Wechselwirkung der Felder mit Probeladungen und
 
<math>3)\nabla \cdot \bar{D}\left( \bar{r},t \right)=\rho \left( \bar{r},t \right)</math>
 
<math>4)\nabla \times H\left( \bar{r},t \right)-\frac{\partial }{\partial t}\bar{D}=\bar{j}</math>
 
die Erzeugung der Felder durch FREIE Ladungen und Ströme
 
Weiter:
 
<math>\begin{align}
& \bar{D}\left( \bar{r},t \right)={{\varepsilon }_{0}}\bar{E}\left( \bar{r},t \right)+\bar{P}\left( \bar{r},t \right) \\
& \bar{H}\left( \bar{r},t \right)=\frac{1}{{{\mu }_{0}}}\bar{B}\left( \bar{r},t \right)-\bar{M}\left( \bar{r},t \right) \\
\end{align}</math>
 
Im Gauß System ( weil so oft in diesem angegeben, vergl. Jackson):
 
<math>\begin{align}
& 1)\nabla \times \bar{E}+\frac{1}{c}\frac{\partial }{\partial t}\bar{B}=0 \\
& 2)\nabla \cdot \bar{B}=0 \\
\end{align}</math>
 
<math>3)\nabla \cdot \bar{D}\left( \bar{r},t \right)=4\pi \rho \left( \bar{r},t \right)</math>
 
<math>4)\nabla \times H\left( \bar{r},t \right)-\frac{1}{c}\frac{\partial }{\partial t}\bar{D}=\frac{4\pi }{c}\bar{j}</math>
 
die Erzeugung der Felder durch FREIE Ladungen und Ströme
 
Weiter:
 
<math>\begin{align}
& 5)\bar{D}\left( \bar{r},t \right)=\bar{E}\left( \bar{r},t \right)+4\pi \bar{P}\left( \bar{r},t \right) \\
& 6)\bar{H}\left( \bar{r},t \right)=\bar{B}\left( \bar{r},t \right)-4\pi \bar{M}\left( \bar{r},t \right) \\
\end{align}</math>
 
Unsere 6 Feldgleichungen ( wenn man so will, also ( es kann nicht oft genug gezeigt werden):
 
<math>\begin{align}
& 1)\nabla \times \bar{E}+\frac{1}{c}\frac{\partial }{\partial t}\bar{B}=0 \\
& 2)\nabla \cdot \bar{B}=0 \\
\end{align}</math>
 
<math>3)\nabla \cdot \bar{D}\left( \bar{r},t \right)=4\pi \rho \left( \bar{r},t \right)</math>
 
<math>4)\nabla \times H\left( \bar{r},t \right)-\frac{1}{c}\frac{\partial }{\partial t}\bar{D}=\frac{4\pi }{c}\bar{j}</math>
 
<math>\begin{align}
& 5)\bar{D}\left( \bar{r},t \right)=\bar{E}\left( \bar{r},t \right)+4\pi \bar{P}\left( \bar{r},t \right) \\
& 6)\bar{H}\left( \bar{r},t \right)=\bar{B}\left( \bar{r},t \right)-4\pi \bar{M}\left( \bar{r},t \right) \\
\end{align}</math>
 
sind nicht vollständig. Es muss noch der Zusammenhang zwischen Polarisation und E- Feld, bzw. B- Feld und Magnetisierung angegeben werden. Dies sind die sogenannten " Materialgleichungen".
 
<u>'''Einfachster Fall:'''</u>
 
# isotrope Materie:
 
<math>\bar{E}\left( \bar{r},t \right)||\bar{P}\left( \bar{r},t \right)</math>
 
und für paramagnetische Stoffe
<math>\bar{B}\left( \bar{r},t \right)\uparrow \uparrow \bar{M}\left( \bar{r},t \right)</math>
 
für diamagnetische Stoffe:
<math>\bar{B}\left( \bar{r},t \right)\uparrow \downarrow \bar{M}\left( \bar{r},t \right)</math>
,
also ein skalarer Zusammenhang
 
# bei nicht zu hohen Feldern:
 
<math>\bar{E}\tilde{\ }\bar{P}</math>
 
<math>\bar{B}\tilde{\ }\bar{M}</math>
 
also ein linearer Zusammenhang
 
# ohne Gedächtniseffekte, keine nichtlokale Wechselwirkung ( keine Phasenkohärenzen):
 
<math>\bar{E}\left( \bar{r},t \right)\tilde{\ }\bar{P}\left( \bar{r},t \right)</math>
 
<math>\bar{B}\left( \bar{r},t \right)\tilde{\ }\bar{M}\left( \bar{r},t \right)</math>
 
neben der Linearität also ein INSTANTANER, LOKALER Zusammenhang !
 
Dann kann man schreiben:
 
<math>\bar{P}\left( \bar{r},t \right)={{\varepsilon }_{0}}{{\chi }_{e}}\bar{E}\left( \bar{r},t \right)</math>
 
<math>\bar{M}\left( \bar{r},t \right)={{\chi }_{M}}\bar{H}\left( \bar{r},t \right)</math>
 
Mit den Suszeptibilitäten, der elektrischen Suszeptibilität
 
<math>{{\chi }_{e}}</math>
und der magnetischen Suszeptibilität
<math>{{\chi }_{M}}</math>
( Materialkonstanten).
Die Materialkonstanten müssen aus den mikroskopischen Theorien ( z.B. Quantentheorie, Festkörperphysik) abgeleitet werden.
 
<math>\bar{D}\left( \bar{r},t \right)={{\varepsilon }_{0}}\bar{E}\left( \bar{r},t \right)+\bar{P}={{\varepsilon }_{0}}\left( 1+{{\chi }_{e}} \right)\bar{E}\left( \bar{r},t \right)={{\varepsilon }_{0}}\varepsilon \bar{E}\left( \bar{r},t \right)</math>
 
mit
 
<math>\varepsilon =\left( 1+{{\chi }_{e}} \right)</math>
, der relativen Dielektrizitätskonstante ( permittivity)
 
<math>\bar{B}={{\mu }_{0}}\left( \bar{H}+\bar{M} \right)={{\mu }_{0}}\left( 1+{{\chi }_{M}} \right)\bar{H}\left( \bar{r},t \right)={{\mu }_{0}}\mu \bar{H}</math>
 
mit
 
<math>\left( 1+{{\chi }_{M}} \right)=\mu </math>
, der relativen Permeabilität
 
<math>\Rightarrow \bar{M}={{\chi }_{M}}\bar{H}=\frac{1}{{{\mu }_{0}}}\frac{{{\chi }_{M}}}{\mu }\bar{B}=\frac{1}{{{\mu }_{0}}}\frac{{{\chi }_{M}}}{\left( 1+{{\chi }_{M}} \right)}\bar{B}</math>
 
Man sagt:
Ein Stoff ist paramagnetisch für
<math>\frac{{{\chi }_{M}}}{\left( 1+{{\chi }_{M}} \right)}>0</math>
 
diamagnetisch für
<math>\frac{{{\chi }_{M}}}{\left( 1+{{\chi }_{M}} \right)}<0</math>
 
paramagnetisch:
<math>{{\chi }_{M}}>0\Rightarrow \mu >1</math>
 
diamagnetisch
<math>0>{{\chi }_{M}}>-1\Rightarrow 0<\mu <1</math>
 
Bemerkungen
 
<math>\bar{E}\left( \bar{r},t \right)=0\Rightarrow \bar{P}=0</math>
beschreibt kein Ferroelektrikum
 
<math>\bar{B}=0\Rightarrow \bar{M}=0</math>
kein Ferromagnet
 
Es gilt stets
<math>{{\chi }_{e}}>0</math>
( Dielektrischer Effekt, Polarisierbarkeit -> es existiert keine negative Polarisierbarkeit)
 
<math>{{\chi }_{M}}\begin{matrix}
>  \\
<  \\
\end{matrix}0</math>
Para- ODER Diamagnet
 
Ein Term
<math>\tilde{\ }\bar{B}</math>
in
<math>\bar{P}</math>
oder
<math>\tilde{\ }\bar{E}</math>
in
<math>\bar{M}</math>
kann gar nicht auftreten, schon wegen des falschen Raumspiegelverhaltens !
 
<math>\bar{E}</math>
ist polarer Vektor,
<math>\bar{B}</math>
ist axialer Vektor !
 
<math>{{\rho }_{P}}\left( \bar{r},t \right)=-\nabla \cdot \bar{P}\left( \bar{r},t \right)</math>
ist ein Skalar
 
<math>{{\bar{j}}_{M}}=rot\bar{M}</math>
ist ein polarer Vektor.
 
<u>'''Abweichungen'''</u>
 
1)Für anisotrope Kristalle :
<math>\bar{P}\left( \bar{r},t \right)={{\varepsilon }_{0}}{{\bar{\bar{\chi }}}_{e}}\bar{E}</math>
 
drückt den anisotropen Charakter aus mit einem symmetrischen Tensor
<math>{{\bar{\bar{\chi }}}_{e}}</math>
.
 
2) für starke Felder gibt es nichtlineare Effekte, die ebenfalls tensoriellen Charakter der Suszeptibilität bedingen:
 
<math>\bar{P}\left( \bar{r},t \right)={{\varepsilon }_{0}}\left( {{{\bar{\bar{\chi }}}}_{e}}^{(1)}\bar{E}+{{{\bar{\bar{\chi }}}}_{e}}^{(2)}{{{\bar{E}}}^{2}}+{{{\bar{\bar{\chi }}}}_{e}}^{(3)}{{{\bar{E}}}^{3}}+... \right)</math>
 
Anwendung: optische Nichtlinearität,
Beispiel: optische Bistabilität, optische Schalter:
 
 
Für hochfrequente Felder folgt:
 
<math>\bar{P}\left( \bar{r},t \right)={{\varepsilon }_{0}}\int_{{}}^{{}}{{}}{{d}^{3}}r\acute{\ }dt\acute{\ }{{\chi }_{e}}\left( \bar{r},\bar{r}\acute{\ },t,t\acute{\ } \right)\bar{E}\left( \bar{r}\acute{\ },t\acute{\ } \right)</math>
 
( räumliche bzw. zeitliche Dispersion):
 
<math>\hat{\bar{P}}\left( \bar{k},\omega  \right)={{\varepsilon }_{0}}{{\hat{\chi }}_{e}}\left( \bar{k},\omega  \right)\hat{\bar{E}}\left( \bar{k},\omega  \right)</math>
 
<u>'''5.4 Grenzbedingungen für Felder'''</u>
 
_ Frage ist: Wie verhalten sich
<math>\bar{B},\bar{H},\bar{D},\bar{E}</math>
an Grenzflächen, die verschiedene elektrische und magnetische Materialien ( Vakuum/ Materie) trennen ?
 
'''Integration der Maxwell- Gleichungen über ein Volumen V:'''
 
 
<math>\int_{V}^{{}}{{}}{{d}^{3}}r\nabla \cdot \bar{D}\left( \bar{r},t \right)=\int_{V}^{{}}{{}}{{d}^{3}}r\rho \left( \bar{r},t \right)=Q=\oint\limits_{\partial V}{{}}d\bar{f}\cdot \bar{D}\left( \bar{r},t \right)</math>
 
<math>\int_{V}^{{}}{{}}{{d}^{3}}r\nabla \times H\left( \bar{r},t \right)=\int_{V}^{{}}{{}}{{d}^{3}}r\left( \bar{j}+\frac{\partial }{\partial t}\bar{D} \right)</math>
 
'''Bildlich:'''
 
'''Normalkomponenten:'''
'''Betrachte einen Zylinder, '''der senkrecht auf einer Grenzfläche steht.
Nun nimmt man die Maxwellgleichungen in integraler Schreibweise an und läßt den Zylinder unter Berücksichtigung von Integrationssätzen gegen Null- Höhe gehen:
 
also: Für die Normalkomponenten: h -> 0
Während also die Normalkomponente des B- Feldes an der Grenzfläche stetig ist,
springt die Normalkomponente der dielektrischen Verschiebung um die Ladung, die an der Grenzfläche sitzt:
Unter der Annahme, dass die Grenzfläche die freie Flächenladungsdichte
<math>\sigma </math>
trägt:
 
<math>\begin{align}
& \rho \left( \bar{r},t \right)=\sigma \left( x,y,t \right)\delta \left( z \right) \\
& {{{\bar{e}}}_{z}}\equiv \bar{n} \\
& \Rightarrow \begin{matrix}
\lim  \\
h->0  \\
\end{matrix}\int_{V}^{{}}{{}}{{d}^{3}}r\rho \left( \bar{r},t \right)=Q=\int_{F}^{{}}{{}}df\sigma \left( x,y,t \right) \\
& \begin{matrix}
\lim  \\
h->0  \\
\end{matrix}\oint\limits_{\partial V}{{}}d\bar{f}\cdot \bar{D}\left( \bar{r},t \right)=\int_{F}^{{}}{{}}d\bar{f}\left( {{{\bar{D}}}^{(1)}}-{{{\bar{D}}}^{(2)}} \right)=\int_{F}^{{}}{{}}df\bar{n}\left( {{{\bar{D}}}^{(1)}}-{{{\bar{D}}}^{(2)}} \right)=\int_{F}^{{}}{{}}df\sigma \left( x,y,t \right) \\
\end{align}</math>
 
<math>\begin{matrix}
\lim  \\
h->0  \\
\end{matrix}\oint\limits_{\partial V}{{}}d\bar{f}\cdot \bar{B}=\int_{F}^{{}}{{}}d\bar{f}\left( {{{\bar{B}}}^{(1)}}-{{{\bar{B}}}^{(2)}} \right)=\int_{F}^{{}}{{}}df\bar{n}\left( {{{\bar{B}}}^{(1)}}-{{{\bar{B}}}^{(2)}} \right)=0</math>
 
Somit müssen die Integranden übereinstimmen:
 
<math>\bar{n}\left( {{{\bar{B}}}^{(1)}}-{{{\bar{B}}}^{(2)}} \right)=0</math>
 
<math>\bar{n}\left( {{{\bar{D}}}^{(1)}}-{{{\bar{D}}}^{(2)}} \right)=\sigma \left( x,y,t \right)</math>
 
<u>'''Tangentialkomponenten'''</u>
 
<u>'''Anwendung des verallgemeinerten Gaußschen Satz:'''</u>
 
<math>1)\nabla \times \bar{E}+\frac{1}{c}\frac{\partial }{\partial t}\bar{B}=0</math>
 
<math>4)\nabla \times H\left( \bar{r},t \right)-\frac{1}{c}\frac{\partial }{\partial t}\bar{D}=\frac{4\pi }{c}\bar{j}</math>
 
<math>\int_{V}^{{}}{{}}{{d}^{3}}r\nabla \times \bar{E}=-\int_{V}^{{}}{{}}{{d}^{3}}r\frac{\partial }{\partial t}\bar{B}</math>
 
<math>\int_{V}^{{}}{{}}{{d}^{3}}r\nabla \times H\left( \bar{r},t \right)=\int_{V}^{{}}{{}}{{d}^{3}}r\left( \bar{j}+\frac{\partial }{\partial t}\bar{D} \right)</math>
 
Auch hier: h-> 0
 
<math>\begin{align}
& \int_{V}^{{}}{{}}{{d}^{3}}r\nabla \times \bar{E}=\oint\limits_{\partial V}{{}}d\bar{f}\times \bar{E}=-\int_{V}^{{}}{{}}{{d}^{3}}r\frac{\partial }{\partial t}\bar{B} \\
& \int_{V}^{{}}{{}}{{d}^{3}}r\nabla \times H\left( \bar{r},t \right)=\oint\limits_{\partial V}{{}}d\bar{f}\times H\left( \bar{r},t \right)=\int_{V}^{{}}{{}}{{d}^{3}}r\left( \bar{j}+\frac{\partial }{\partial t}\bar{D} \right) \\
& \begin{matrix}
\lim  \\
h->0  \\
\end{matrix}\oint\limits_{\partial V}{{}}d\bar{f}\times \bar{E}=\oint\limits_{\partial V}{{}}df\bar{n}\times \left( {{{\bar{E}}}^{(1)}}-{{{\bar{E}}}^{(2)}} \right) \\
& \begin{matrix}
\lim  \\
h->0  \\
\end{matrix}\oint\limits_{\partial V}{{}}d\bar{f}\times H\left( \bar{r},t \right)=\oint\limits_{\partial V}{{}}df\bar{n}\times \left( H{{\left( \bar{r},t \right)}^{(1)}}-H{{\left( \bar{r},t \right)}^{(2)}} \right) \\
\end{align}</math>
 
In beiden Fällen die Tangentialkomponenten der Felder ! senkrecht auf Flächenvektor und Feld
 
Wegen:
 
<math>\begin{align}
& \begin{matrix}
\lim  \\
h->0  \\
\end{matrix}\oint\limits_{\partial V}{{}}d\bar{f}\times \bar{E}=\oint\limits_{\partial V}{{}}df\bar{n}\times \left( {{{\bar{E}}}^{(1)}}-{{{\bar{E}}}^{(2)}} \right)=-\begin{matrix}
\lim  \\
h->0  \\
\end{matrix}\int_{V}^{{}}{{}}{{d}^{3}}r\frac{\partial }{\partial t}\bar{B} \\
& \begin{matrix}
\lim  \\
h->0  \\
\end{matrix}\oint\limits_{\partial V}{{}}d\bar{f}\times H\left( \bar{r},t \right)=\oint\limits_{\partial V}{{}}df\bar{n}\times \left( H{{\left( \bar{r},t \right)}^{(1)}}-H{{\left( \bar{r},t \right)}^{(2)}} \right)=\begin{matrix}
\lim  \\
h->0  \\
\end{matrix}\int_{V}^{{}}{{}}{{d}^{3}}r\left( \bar{j}+\frac{\partial }{\partial t}\bar{D} \right) \\
\end{align}</math>
 
Annahme: Grenzfläche trägt (freie) Flächenstromdichte
<math>\begin{align}
& {\bar{g}} \\
& \Rightarrow \bar{j}\left( \bar{r},t \right)=\bar{g}\left( x,y,t \right)\delta \left( z \right) \\
\end{align}</math>
 
wie es bei metallen der Fall ist !,
dann:
 
<math>\begin{matrix}
\lim  \\
h->0  \\
\end{matrix}\int_{V}^{{}}{{}}{{d}^{3}}r\bar{j}=\int_{F}^{{}}{{}}df\bar{g}</math>
 
Weiter:
 
<math>\begin{align}
& \begin{matrix}
\lim  \\
h->0  \\
\end{matrix}\int_{V}^{{}}{{}}{{d}^{3}}r\frac{\partial }{\partial t}\bar{B} \\
& \begin{matrix}
\lim  \\
h->0  \\
\end{matrix}\int_{V}^{{}}{{}}{{d}^{3}}r\frac{\partial }{\partial t}\bar{D} \\
\end{align}</math>
 
können für Volumenintegrale mit verschwindendem Volumen nur einen Beitrag liefern, wenn
<math>\frac{\partial }{\partial t}\bar{B},\frac{\partial }{\partial t}\bar{D}</math>
Unendlichkeitsstellen besitzen.
 
Annahme:
 
<math>\bar{B},\bar{D}</math>
und
<math>\frac{\partial }{\partial t}\bar{B},\frac{\partial }{\partial t}\bar{D}</math>
sind beschränkt:
 
<math>\begin{align}
& \begin{matrix}
\lim  \\
h->0  \\
\end{matrix}\int_{V}^{{}}{{}}{{d}^{3}}r\frac{\partial }{\partial t}\bar{B}=0 \\
& \begin{matrix}
\lim  \\
h->0  \\
\end{matrix}\int_{V}^{{}}{{}}{{d}^{3}}r\frac{\partial }{\partial t}\bar{D}=0 \\
& \begin{matrix}
\lim  \\
h->0  \\
\end{matrix}\int_{V}^{{}}{{}}{{d}^{3}}r\left( \bar{j}+\frac{\partial }{\partial t}\bar{D} \right)=\int_{F}^{{}}{{}}df\bar{g}(x,y,t) \\
& \oint\limits_{\partial V}{{}}df\bar{n}\times \left( {{{\bar{E}}}^{(1)}}-{{{\bar{E}}}^{(2)}} \right)=0 \\
& \oint\limits_{\partial V}{{}}df\bar{n}\times \left( H{{\left( \bar{r},t \right)}^{(1)}}-H{{\left( \bar{r},t \right)}^{(2)}} \right)=\int_{F}^{{}}{{}}df\bar{g}(x,y,t) \\
\end{align}</math>
 
Somit haben wir die Grenzbedingungen für die Tangentialkomponenten:
 
<math>\begin{align}
& \bar{n}\times \left( {{{\bar{E}}}^{(1)}}-{{{\bar{E}}}^{(2)}} \right)=0 \\
& \bar{n}\times \left( H{{\left( \bar{r},t \right)}^{(1)}}-H{{\left( \bar{r},t \right)}^{(2)}} \right)=\bar{g}(x,y,t) \\
\end{align}</math>
 
Das heißt:
 
Die Tangentialkomponente des elektrischen Feldes E ist am Grenzübergang stetig
Die Tangentialkomponente des magnetischen Feldes H springt am Grenzübergang um die Flächenstromdichte !
 
Bildlich:
Sitzen Ladungen an einer Grenzfläche, so ist die Normalkomponente von D ( wichtig: Polarisationseffekt -> Polarisation muss irgendwo mit auftauchen) nicht stetig !
Fließen flächenartige Ströme entlang einer Grenzfläche, so ist die Tangentialkomponente von H nicht stetig !
 
<u>'''Zusammenfassung:'''</u>
 
<math>\delta \bar{E}:=\left( {{{\bar{E}}}^{(1)}}-{{{\bar{E}}}^{(2)}} \right)</math>
 
<u>'''Maxwellgleichung Grenzbedingung'''</u>
 
<math>\begin{align}
& 1)\nabla \times \bar{E}=-\frac{\partial }{\partial t}\nabla \times \bar{A}\left( \bar{r},t \right)=-\frac{\partial }{\partial t}\bar{B}\quad \quad \quad \quad \quad \quad \bar{n}\times \delta \bar{E}=0 \\
& 2)\nabla \cdot \bar{B}=0\quad \quad \quad \quad \quad \quad \quad \quad \quad \quad \quad \quad \quad \quad \quad \bar{n}\cdot \delta \bar{B}=0 \\
\end{align}</math>
 
<math>3)\nabla \cdot \bar{D}\left( \bar{r},t \right)=\rho \left( \bar{r},t \right)\quad \quad \quad \quad \quad \quad \quad \quad \quad \quad \quad \bar{n}\cdot \delta \bar{D}\left( \bar{r},t \right)=\sigma </math>
 
<math>4)\nabla \times H\left( \bar{r},t \right)=\bar{j}+\frac{\partial }{\partial t}\bar{D}\quad \quad \quad \quad \quad \quad \quad \quad \quad \quad \bar{n}\times \delta H\left( \bar{r},t \right)=\bar{g}</math>
 
Also: die Tangenzialkomponente von E ist stetig
Die Normalkomponente von D springt um die Flächenladungsdichte ( Flächendivergenz)
Die Tangentialkomponente von H springt ( Flächenrotation) um die Flächenstromdichte
Die Normalkomponente von B ist stetig.
 
'''Beispiele:'''
 
# Grenzfläche zwischen 2 dielektrischen Materialien  mit
 
<math>\begin{align}
& {{\varepsilon }^{(1)}}<{{\varepsilon }^{(2)}} \\
& \sigma =0 \\
\end{align}</math>
 
 
Zuerst zeichne man sich ein derartiges Diagramm hin !
 
<math>\begin{align}
& {{{\bar{E}}}_{t}}^{(1)}={{{\bar{E}}}_{t}}^{(2)} \\
& {{{\bar{D}}}_{n}}^{(1)}={{{\bar{D}}}_{n}}^{(2)} \\
\end{align}</math>
 
letzteres wegen der verschwindenden Flächenladungsdichte !
 
<math>\begin{align}
& {{{\bar{E}}}_{t}}^{(1)}={{{\bar{E}}}_{t}}^{(2)} \\
& {{{\bar{D}}}_{n}}^{(1)}={{{\bar{D}}}_{n}}^{(2)}\Rightarrow {{\varepsilon }_{1}}{{{\bar{E}}}_{n}}^{(1)}={{\varepsilon }_{2}}{{{\bar{E}}}_{n}}^{(2)} \\
& \Rightarrow {{{\bar{E}}}_{n}}^{(2)}=\frac{{{\varepsilon }_{1}}}{{{\varepsilon }_{2}}}{{{\bar{E}}}_{n}}^{(1)} \\
& \tan {{\alpha }_{1}}=\frac{{{{\bar{E}}}_{t}}^{(1)}}{{{{\bar{E}}}_{n}}^{(1)}}=\frac{{{\varepsilon }_{1}}}{{{\varepsilon }_{2}}}\frac{{{{\bar{E}}}_{t}}^{(2)}}{{{{\bar{E}}}_{n}}^{(2)}}=\frac{{{\varepsilon }_{1}}}{{{\varepsilon }_{2}}}\tan {{\alpha }_{2}} \\
\end{align}</math>
 
Dies ist das Brechungsgesetz für die Feldlinien
 
Achtung ! Das Snelliussche Brechungsgesetz müsste man sich für den Verlauf des Energiestroms berechnen
 
# <u>'''Grenzfläche zwischen Vakuum ( Luft) und magnetischem Material'''</u>
 
<u>'''2.1 Sei '''</u>speziell
<math>\bar{B}\bot </math>
Grenzfläche ( z.B. zwischen den Polschuhen eines Ringmagneten mit Luft dazwischen / Material genauso !)):
In diesem Fall (keine Oberflächenströme) ist
<math>\bar{B}</math>
grundsätzlich stetig !
B ist eh immer grundsätzlich stetig ! Wegen der Divergenzgleichung wird B immer ( wie D´) für Normalkomponenten herangezogen.
 
# <u>'''Paramagnetisch:'''</u>
 
<math>\begin{align}
& \frac{1}{{{\mu }_{0}}}\bar{B}=\bar{M}+\bar{H} \\
& \bar{M}\uparrow \uparrow \bar{H} \\
\end{align}</math>
 
 
# <u>'''Paramagnetisch:'''</u>
 
<math>\begin{align}
& \frac{1}{{{\mu }_{0}}}\bar{B}=\bar{M}+\bar{H} \\
& \bar{M}\uparrow \downarrow \bar{H} \\
\end{align}</math>
 
 
<u>'''2.2 Sei '''</u>speziell
<math>\bar{B}||</math>
Grenzfläche ( z.B. lange Spule mit Luft dazwischen / Material genauso !)):
Wir müssen nun Tangentialkomponenten untersuchen. Dazu nimmt man die Rotationsgleichungen ( E und H):
 
In diesem Fall ist
<math>\bar{H}</math>
stetig für
<math>\bar{g}=0</math>
( kein Oberflächenstrom)
 
<u>'''5.5 Mikroskopisches Modell der Polarisierbarkeit'''</u>
 
Ziel: Berechnung der Materialkonstanten
 
<u>'''5.5 Mikroskopisches Modell der Polarisierbarkeit'''</u>
 
Ziel: Berechnung der Materialkonstanten
<math>{{\chi }_{e}}</math>
aus einfachen mikroskopischen Modellen
Methode:  Berechne die induzierte mittlere elektrische Dipoldichte
<math>\bar{P}</math>
für ein gegebenes Feld
<math>\bar{E}</math>
.
 
'''Nebenbemerkung: '''Die Orientierungspolarisation ist nur mittels einer thermodynamischen- statistischen Theorie zu berechnen: Hier: Auseinandersetzung nur mit der " induzierten" Polarisation
 
<u>'''Klassisches Atommodell:'''</u>
 
homogen geladene Kugel mit Radius R und Elektronenladung
<math>{{Q}_{e}}=-Ze<0</math>
 
Außerdem ein punktförmiger Kern mit
<math>{{Q}_{k}}=+Ze>0</math>
am Ort
<math>{{\bar{r}}_{k}}</math>
 
'''Merke:'''
 
Auch diese Berechnungen geschehen, wie im NOTFALL grundsätzlich zu empfehlen, durch Lösen integraler Darstellungen der Maxwellgleichungen
 
Ziel: Berechnung des elektrischen Feldes
<math>{{\bar{E}}_{el.}}\left( {\bar{r}} \right)</math>
der Elektronen nach außen:
 
Gauß- Gesetz
 
 
<math>\int_{V}^{{}}{{}}{{d}^{3}}r\nabla \cdot \bar{D}\left( \bar{r},t \right)=\int_{V}^{{}}{{}}{{d}^{3}}r\rho \left( \bar{r},t \right)=Q=\oint\limits_{\partial V}{{}}d\bar{f}\cdot \bar{D}\left( \bar{r},t \right)</math>
 
Wir müssen aber zurückkehren zu den mikroskopischen Maxwellgleichungen
 
Wichtig ! Integration immer über das Gebiet, in dem die Ladung vorhanden ist, aber ! Betrachtung des elektrischen Feldes an einem gewissen Aufpunkt r! Die Ladung ist eigentlich von r´ abhängig , aber hier homogen verteilt !-> einfache Integration.
 
Auswertung liefert
 
<math>\begin{align}
& {{\varepsilon }_{0}}\oint\limits_{\partial V(r\acute{\ })}{{}}d\bar{f}\cdot \bar{E}\left( \bar{r},t \right)=\int_{V(r\acute{\ })}^{{}}{{}}\frac{Q}{\frac{4}{3}\pi {{R}^{3}}}=\frac{r{{\acute{\ }}^{3}}}{{{R}^{3}}}Q \\
& \Rightarrow 4r{{\acute{\ }}^{2}}\pi {{\varepsilon }_{0}}\left| \bar{E}\left( \bar{r},t \right) \right|=\frac{r{{\acute{\ }}^{3}}}{{{R}^{3}}}Q \\
& \Rightarrow \left| \bar{E}\left( \bar{r},t \right) \right|=\frac{r\acute{\ }}{4\pi {{\varepsilon }_{0}}{{R}^{3}}}Q \\
\end{align}</math>
 
Natürlich nur für
 
<math>r\acute{\ }\le R</math>
 
setzt man
<math>\bar{r}\acute{\ }=\bar{r}-{{\bar{r}}_{e}}</math>
, wobei
<math>{{\bar{r}}_{e}}</math>
das Zentrum der elektrischen Ladung angibt,
 
so gewinnt man das rotationssymmetrische Ergebnis
 
<math>\bar{E}\left( \bar{r},t \right)=\frac{\bar{r}-{{{\bar{r}}}_{e}}}{4\pi {{\varepsilon }_{0}}{{R}^{3}}}{{Q}_{e}}</math>
 
und die Kraft auf den Kern folgt gemäß:
 
<math>{{\bar{F}}_{K}}={{Q}_{K}}\bar{E}\left( {{{\bar{r}}}_{\acute{\ }k}},t \right)=\frac{{{{\bar{r}}}_{k}}-{{{\bar{r}}}_{e}}}{4\pi {{\varepsilon }_{0}}{{R}^{3}}}{{Q}_{e}}{{Q}_{k}}=-\frac{{{Z}^{2}}{{e}^{2}}}{4\pi {{\varepsilon }_{0}}{{R}^{3}}}\left( {{{\bar{r}}}_{k}}-{{{\bar{r}}}_{e}} \right)</math>
 
wegen actio = reactio folgt dann für die Kraft auf die Elektronen:
 
<math>{{\bar{F}}_{e}}=-{{\bar{F}}_{K}}</math>
 
Aufstellen der Bewegungsgleichungen ( inklusive einem äußeren Feld
<math>{{\bar{E}}_{a}}</math>
):
 
<math>\begin{align}
& {{m}_{K}}{{{\ddot{\bar{r}}}}_{k}}={{{\bar{F}}}_{K}}+{{Q}_{K}}{{{\bar{E}}}_{a}}\left( {{{\bar{r}}}_{\acute{\ }k}},t \right)=-\frac{{{Z}^{2}}{{e}^{2}}}{4\pi {{\varepsilon }_{0}}{{R}^{3}}}\left( {{{\bar{r}}}_{k}}-{{{\bar{r}}}_{e}} \right)+{{Q}_{K}}{{{\bar{E}}}_{a}}\left( {{{\bar{r}}}_{\acute{\ }k}},t \right)=-\frac{{{Z}^{2}}{{e}^{2}}}{4\pi {{\varepsilon }_{0}}{{R}^{3}}}\left( {{{\bar{r}}}_{k}}-{{{\bar{r}}}_{e}} \right)+Ze{{{\bar{E}}}_{a}}\left( {{{\bar{r}}}_{\acute{\ }k}},t \right) \\
& Z{{m}_{e}}{{{\ddot{\bar{r}}}}_{e}}=-{{{\bar{F}}}_{K}}+{{Q}_{e}}{{{\bar{E}}}_{a}}\left( {{{\bar{r}}}_{\acute{\ }k}},t \right)=\frac{{{Z}^{2}}{{e}^{2}}}{4\pi {{\varepsilon }_{0}}{{R}^{3}}}\left( {{{\bar{r}}}_{k}}-{{{\bar{r}}}_{e}} \right)+{{Q}_{e}}{{{\bar{E}}}_{a}}\left( {{{\bar{r}}}_{\acute{\ }k}},t \right)=\frac{{{Z}^{2}}{{e}^{2}}}{4\pi {{\varepsilon }_{0}}{{R}^{3}}}\left( {{{\bar{r}}}_{k}}-{{{\bar{r}}}_{e}} \right)-Ze{{{\bar{E}}}_{a}}\left( {{{\bar{r}}}_{\acute{\ }k}},t \right) \\
\end{align}</math>
 
Also folgt für die Relativbewegung:
 
<math>\bar{r}={{\bar{r}}_{k}}-{{\bar{r}}_{e}}</math>
 
als relativer Abstand
 
<math>\begin{align}
& \ddot{\bar{r}}={{{\ddot{\bar{r}}}}_{k}}-{{{\ddot{\bar{r}}}}_{e}}=-\frac{{{Z}^{2}}{{e}^{2}}}{4\pi {{\varepsilon }_{0}}{{R}^{3}}{{m}_{K}}}\left( {{{\bar{r}}}_{k}}-{{{\bar{r}}}_{e}} \right)+\frac{Ze}{{{m}_{K}}}{{{\bar{E}}}_{a}}\left( {{{\bar{r}}}_{\acute{\ }k}},t \right)-\frac{Z{{e}^{2}}}{4\pi {{\varepsilon }_{0}}{{R}^{3}}{{m}_{e}}}\left( {{{\bar{r}}}_{k}}-{{{\bar{r}}}_{e}} \right)+\frac{e}{{{m}_{e}}}{{{\bar{E}}}_{a}}\left( {{{\bar{r}}}_{k}},t \right) \\
& =-\frac{{{Z}^{2}}{{e}^{2}}}{4\pi {{\varepsilon }_{0}}{{R}^{3}}}\left( \frac{1}{{{m}_{K}}}+\frac{1}{Z{{m}_{e}}} \right)\left( {{{\bar{r}}}_{k}}-{{{\bar{r}}}_{e}} \right)+Ze\left( \frac{1}{{{m}_{K}}}+\frac{1}{Z{{m}_{e}}} \right){{{\bar{E}}}_{a}}\left( {{{\bar{r}}}_{k}},t \right) \\
& \left( \frac{1}{{{m}_{K}}}+\frac{1}{Z{{m}_{e}}} \right)\approx \frac{1}{Z{{m}_{e}}} \\
& \left( {{{\bar{r}}}_{k}}-{{{\bar{r}}}_{e}} \right)=\bar{r} \\
& \Rightarrow \ddot{\bar{r}}=-\frac{Z{{e}^{2}}}{4\pi {{\varepsilon }_{0}}{{m}_{e}}{{R}^{3}}}\bar{r}+\frac{e}{{{m}_{e}}}{{{\bar{E}}}_{a}}\left( {{{\bar{r}}}_{k}},t \right) \\
& \frac{Z{{e}^{2}}}{4\pi {{\varepsilon }_{0}}{{m}_{e}}{{R}^{3}}}:={{\omega }_{0}}^{2} \\
& \Rightarrow \ddot{\bar{r}}+{{\omega }_{0}}^{2}\bar{r}=\frac{e}{{{m}_{e}}}{{{\bar{E}}}_{a}}\left( {{{\bar{r}}}_{k}},t \right) \\
\end{align}</math>
 
Also ergibt sich ein harmonischer Oszillator mit quadratischem Potenzial ! was wir schon an der Bestimmung des Potenzials sofort hätten sehen können !
 
Jedenfalls im stationären Zustand gilt:
 
<math>\bar{r}=\frac{e}{{{\omega }_{0}}^{2}{{m}_{e}}}{{\bar{E}}_{a}}\left( {{{\bar{r}}}_{k}},t \right)</math>
 
( Dynamik mit Dämpfung)
 
<math>\Rightarrow {{\chi }_{e}}\left( \omega  \right)</math>
 
Als Ergebnis gewinnen wir ein statisch mikroskopisch elektrisches Dipolmoment, welches sich über p=qd bereits hinschreiben läßt und welches auch übereinstimmt mit Gleichungen von oben zur exakten Berechnung des elektrischen Dipolmoments:
 
<math>\begin{align}
& \bar{p}=Ze\bar{r}=\frac{Z{{e}^{2}}}{{{\omega }_{0}}^{2}{{m}_{e}}}{{{\bar{E}}}_{a}}\left( {{{\bar{r}}}_{k}},t \right)={{\varepsilon }_{0}}\alpha {{{\bar{E}}}_{a}} \\
& \alpha :=\frac{Z{{e}^{2}}}{{{\omega }_{0}}^{2}{{\varepsilon }_{0}}{{m}_{e}}} \\
& \frac{Z{{e}^{2}}}{4\pi {{\varepsilon }_{0}}{{m}_{e}}{{R}^{3}}}:={{\omega }_{0}}^{2} \\
& \Rightarrow \alpha :=\frac{Z{{e}^{2}}}{{{\omega }_{0}}^{2}{{\varepsilon }_{0}}{{m}_{e}}}=4\pi {{R}^{3}}=3{{V}_{Atom}} \\
\end{align}</math>
 
Die Polarisierbarkeit des Atoms, ein mikroskopischer Parameter.
Entsprechend:
 
<math>\begin{align}
& \bar{p}=\int_{V}^{{}}{{}}{{d}^{3}}r\acute{\ }{{\rho }_{e}}(r\acute{\ })\bar{r}\acute{\ }+Ze\int_{V}^{{}}{{}}{{d}^{3}}r\acute{\ }\delta (\bar{r}-\bar{r}\acute{\ }) \\
& Ze\int_{V}^{{}}{{}}{{d}^{3}}r\acute{\ }\delta (\bar{r}-\bar{r}\acute{\ })=Ze\bar{r} \\
& \int_{V}^{{}}{{}}{{d}^{3}}r\acute{\ }{{\rho }_{e}}(r\acute{\ })\bar{r}\acute{\ }=-\frac{Ze}{\frac{4\pi }{3}{{R}^{3}}}\int_{V}^{{}}{{}}{{d}^{3}}r\acute{\ }\bar{r}\acute{\ } \\
& \int_{V}^{{}}{{}}{{d}^{3}}r\acute{\ }\bar{r}\acute{\ }=0 \\
\end{align}</math>
 
wegen Symmetrie
 
<math>\bar{p}=Ze\bar{r}</math>
 
makroskopisch gemittelte Energiedichte:
 
<math>\bar{P}=n\bar{p}={{\varepsilon }_{0}}n\alpha {{\bar{E}}_{a}}</math>
 
mit der mittleren Atomdichte n
 
<u>'''Selbstkonsistente Berechnung des Lokalfeldes Ea:'''</u>
 
Wichtig: Berücksichtigung der Felder, die durch andere elektrische Dipole erzeugt werden:
 
<u>'''Gedankenexperiment'''</u>
 
 
<u>Feld einer homogenen polarisierten Kugel:</u>
 
Ansatz: homogen geladene Kugel:
 
<math>{{\bar{E}}_{0}}\left( {\bar{r}} \right)=\frac{Q}{4\pi {{\varepsilon }_{0}}}\left\{ \begin{matrix}
\frac{{\bar{r}}}{{{a}^{3}}}r\le a  \\
\frac{{\bar{r}}}{{{r}^{3}}}r\ge a  \\
\end{matrix} \right.</math>
 
Also:
 
 
<math>{{\Phi }_{0}}\left( {\bar{r}} \right)=\frac{Q}{4\pi {{\varepsilon }_{0}}}\left\{ \begin{matrix}
c-\frac{{{{\bar{r}}}^{2}}}{2{{a}^{3}}}r\le a  \\
\frac{1}{r}r\ge a  \\
\end{matrix} \right.</math>
 
Bestimmung der Integrationskonstanten:
 
<math>\begin{matrix}
\lim  \\
\varepsilon ->0  \\
\end{matrix}{{\Phi }_{0}}\left( a-\varepsilon  \right)={{\Phi }_{0}}\left( a+\varepsilon  \right)\Rightarrow c=\frac{3}{2a}</math>
 
<u>'''die homogen polarisierte Kugel'''</u>
 
Bei der homogen polarisierten Kugel kann man 2 entgegegengesetzt homogen geladene Kugeln mit Abstand ro annehmen.
 
Dann: ro -> 0
 
 
Bilde:
 
<math>\begin{align}
& {{\Phi }_{0}}\left( {\bar{r}} \right)={{\Phi }_{0}}\left( \bar{r}-\frac{1}{2}{{{\bar{r}}}_{0}} \right)-{{\Phi }_{0}}\left( \bar{r}+\frac{1}{2}{{{\bar{r}}}_{0}} \right) \\
& \approx -{{{\bar{r}}}_{0}}\nabla {{\Phi }_{0}}\left( {\bar{r}} \right) \\
& \nabla {{\Phi }_{0}}\left( {\bar{r}} \right)=-{{{\bar{E}}}_{0}} \\
& \Rightarrow {{\Phi }_{0}}\left( {\bar{r}} \right)\approx {{{\bar{r}}}_{0}}{{{\bar{E}}}_{0}}=\frac{Q}{4\pi {{\varepsilon }_{0}}}\left\{ \begin{matrix}
\frac{{{{\bar{r}}}_{0}}\bar{r}}{{{a}^{3}}}r\le a  \\
\frac{{{{\bar{r}}}_{0}}\bar{r}}{{{r}^{3}}}r\ge a  \\
\end{matrix} \right.=\frac{1}{4\pi {{\varepsilon }_{0}}}\left\{ \begin{matrix}
\frac{\bar{p}\bar{r}}{{{a}^{3}}}r\le a  \\
\frac{\bar{p}\bar{r}}{{{r}^{3}}}r\ge a  \\
\end{matrix} \right. \\
& \bar{p}:=Q{{{\bar{r}}}_{0}} \\
\end{align}</math>
 
Das Dipolmoment der herausgeschnittenen Kugel.
 
Als Näherung wurde taylorentwickelt. Dabei allerdings nur bis zur ersten Ordnung und Nullte Ordnung verschwindet.
Verwendet wurde das Dipolmoment der Kugel. Man kann auf Polarisation ( eigentlich Dipoldichte) umschreiben:
 
<math>\begin{align}
& \bar{P}=\frac{{\bar{p}}}{\frac{4}{3}{{a}^{3}}\pi } \\
& \Rightarrow {{\Phi }_{0}}\left( {\bar{r}} \right)\approx {{{\bar{r}}}_{0}}{{{\bar{E}}}_{0}}=\frac{Q}{4\pi {{\varepsilon }_{0}}}\left\{ \begin{matrix}
\frac{{{{\bar{r}}}_{0}}\bar{r}}{{{a}^{3}}}r\le a  \\
\frac{{{{\bar{r}}}_{0}}\bar{r}}{{{r}^{3}}}r\ge a  \\
\end{matrix} \right.=\frac{1}{{{\varepsilon }_{0}}}\left\{ \begin{matrix}
\frac{\bar{P}\bar{r}}{3}r\le a  \\
\bar{P}\bar{r}\frac{{{a}^{3}}}{{{r}^{3}}}r\ge a  \\
\end{matrix} \right. \\
\end{align}</math>
 
Wir gewinnen innerhalb der Kugel homogene Polarisation und außerhalb ein Dipolpotenzial.
 
<math>{{\bar{E}}_{Kugel}}=-\nabla \Phi =-\frac{1}{{{\varepsilon }_{0}}}\frac{{\bar{P}}}{3}r\le a</math>
 
für das elektrische Feld im Inneren der Kugel ( homogen polarisiert).
 
<u>'''Gesamtes Lokalfeld am Ort des Atoms ergibt sich nach:'''</u>
 
das äußere Feld wird erzeugt durch Atome, die sich außerhalb der Hohlkugel befinden.
Das innere Feld durch Atome im Inneren der Hohlkugel.
Gezeichnet: Lokalfeld einer polarisierten dielektrischen Kugel im homogenen elektrischen Feld
 
 
Das Lokalfeld  im INNEREN des KugelHOHLRAUMS, welcher aus dem Volumen herausgeschnitten wurde:
 
<math>{{\bar{E}}_{a}}\left( {\bar{r}} \right)=\bar{E}-{{\bar{E}}_{KUgel}}</math>
 
<math>\begin{align}
& {{{\bar{E}}}_{a}}\left( {\bar{r}} \right):Lokalfeld \\
& \bar{E}:makroskopisch \\
& {{{\bar{E}}}_{a}}\left( {\bar{r}} \right)=\bar{E}+\frac{1}{3{{\varepsilon }_{0}}}\bar{P} \\
\end{align}</math>
 
Letztes wurde von Lorentz eingeführt als "Korrekturfeld"
 
weil
 
<math>{{\bar{E}}_{a}}+{{\bar{E}}_{Kugel}}=\bar{E}</math>
sein muss
 
Das Lokalfeld am Ort des Atoms mit dem Innenfeld der dielektrischen Kugel ( wieder in den Hohlraum eingesetzt) ergibt das mittlere makroskopische Feld !
 
'''Zusammenhang zwischen P und makroskopischem Feld E:'''
 
<math>\begin{align}
& \bar{P}={{\varepsilon }_{0}}n\alpha {{{\bar{E}}}_{a}}={{\varepsilon }_{0}}n\alpha \left( \bar{E}+\frac{1}{3{{\varepsilon }_{0}}}\bar{P} \right) \\
& \bar{P}={{\varepsilon }_{0}}{{\chi }_{e}}\bar{E} \\
& \Rightarrow {{\chi }_{e}}=\frac{n\alpha }{1-\frac{1}{3}n\alpha } \\
& n\alpha =\frac{{{\chi }_{e}}}{1+\frac{1}{3}{{\chi }_{e}}}=\frac{\varepsilon -1}{1+\frac{\varepsilon -1}{3}}=3\frac{\varepsilon -1}{\varepsilon +2} \\
\end{align}</math>
 
Formel von Clausius - Masotti für polarisierte Kugel
 
<u>'''5.6 Wellenausbreitung in Materie'''</u>
 
Annahme: homogene, isotrope, lineare Medien mit skalaren Materialparametern
<math>\varepsilon ,\mu ,\sigma </math>
:
 
<math>\begin{align}
& \bar{D}=\varepsilon {{\varepsilon }_{0}}\bar{E}\quad \varepsilon >1 \\
& \bar{B}={{\mu }_{0}}\mu \bar{H}\quad i.a.\mu \tilde{\ }1 \\
& \bar{j}=\sigma \bar{E} \\
\end{align}</math>
 
( ohmsches Gesetz)
 
<u>'''Wellen in leitenden Medien ohne Dispersion:'''</u>
 
<u>'''Das heißt:'''</u>
<math>\varepsilon ,\mu ,\sigma </math>
nicht frequenzabhängig !
 
Sei
 
<math>\begin{align}
& \rho =0 \\
& \nabla \times \bar{E}+\dot{\bar{B}}=0 \\
& \nabla \times \bar{B}-{{\mu }_{0}}\mu \varepsilon {{\varepsilon }_{0}}\dot{\bar{E}}={{\mu }_{0}}\mu \bar{j}={{\mu }_{0}}\mu \sigma \bar{E} \\
& \nabla \cdot \bar{E}=0 \\
& \nabla \cdot \bar{B}=0 \\
& \Rightarrow \nabla \times \left( \nabla \times \bar{E} \right)=\nabla \left( \nabla \cdot \bar{E} \right)-\Delta \bar{E}=-\Delta \bar{E}=-\nabla \times \dot{\bar{B}}=-{{\mu }_{0}}\mu \sigma \dot{\bar{E}}-{{\mu }_{0}}\mu \varepsilon {{\varepsilon }_{0}}\ddot{\bar{E}} \\
&  \\
& \Delta \bar{E}={{\mu }_{0}}\mu \sigma \dot{\bar{E}}+{{\mu }_{0}}\mu \varepsilon {{\varepsilon }_{0}}\ddot{\bar{E}} \\
\end{align}</math>
 
Somit erhalten wir die Gleichung einer gedämpften Welle
 
<math>\begin{align}
& \Delta \bar{E}-\frac{1}{{{c}_{m}}^{2}}\left( \frac{\sigma }{\varepsilon {{\varepsilon }_{0}}}\dot{\bar{E}}+\ddot{\bar{E}} \right)=0 \\
& {{c}_{m}}:=\frac{1}{\sqrt{\varepsilon {{\varepsilon }_{0}}\mu {{\mu }_{0}}}}=c\frac{1}{\sqrt{\varepsilon \mu }} \\
\end{align}</math>
 
Für den eindimensionalen Fall: sogenannte Telegraphengleichung. Beschreibt die Drahtwellenausbreitung !
 
<u>'''Spezielle Lösung dieses Problems:'''</u>
 
<u>homogene, ebene Welle:</u>
 
<math>\begin{align}
& \bar{E}(\bar{r},t)={{{\bar{E}}}_{0}}{{e}^{i\left( \bar{k}\bar{r}-\omega t \right)}} \\
& \Rightarrow {{k}^{2}}=\varepsilon \mu \frac{{{\omega }^{2}}}{{{c}^{2}}}\left( 1+i\frac{1}{\omega \tau } \right) \\
\end{align}</math>
 
Dispersionsrelation für den Fall der frequenzunabhängigen Parameter
Durch die Dämpfung
<math>\sigma </math>
ist der Wellenvektor ein komplexer Parameter.
 
<math>k\in C</math>
 
Setze:
 
<math>k=\frac{\omega }{c}\tilde{n}=\frac{\omega }{c}\left( n+i\gamma  \right)</math>
 
mit c: Vakuumlichtgeschwindigkeit
 
<math>\tilde{n}=\left( n+i\gamma  \right)</math>
komplexer Brechungsindex !
Somit:
 
<math>{{k}^{2}}=\frac{{{\omega }^{2}}}{{{c}^{2}}}{{\tilde{n}}^{2}}=\frac{{{\omega }^{2}}}{{{c}^{2}}}\left( {{n}^{2}}-{{\gamma }^{2}}+2in\gamma  \right)=\frac{{{\omega }^{2}}}{{{c}^{2}}}\varepsilon \mu \left( 1+i\frac{1}{\omega \tau } \right)</math>
 
Damit können Real- und Imaginärteil durch Vergleich herangezogen werden, um Gamma und n zu bestimmen:
 
<math>\begin{align}
& {{n}^{2}}-{{\gamma }^{2}}=\varepsilon \mu  \\
& n\gamma =\frac{\varepsilon \mu }{2\omega \tau } \\
\end{align}</math>
 
* Bestimmung von
* <math>n,\gamma </math>
* :
 
o.B.d.A.:
 
<math>\bar{k}||{{\bar{x}}_{3}}</math>
:
 
Ausschreiben der Welle:
 
<math>\begin{align}
& \bar{E}(\bar{r},t)={{{\bar{E}}}_{0}}{{e}^{i\left( \bar{k}\bar{r}-\omega t \right)}} \\
& \bar{E}({{{\bar{x}}}_{3}},t)={{{\bar{E}}}_{0}}{{e}^{-\frac{{{x}_{3}}}{\lambda }}}{{e}^{-i\omega \left( t-\frac{n}{c}{{x}_{3}} \right)}} \\
\end{align}</math>
 
Also eine gedämpfte Welle mit der Phasengeschwindigkeit
<math>\frac{c}{n}</math>
und dem Extinktionskoeffizienten
 
<math>\lambda =\frac{c}{\omega \gamma }</math>
 
'''Lineare Polarisation:'''
 
<math>{{\bar{E}}_{0}}||{{\bar{x}}_{1}}\Rightarrow {{\bar{B}}_{0}}||{{\bar{x}}_{2}}</math>
 
<math>\begin{align}
& {{\left( \nabla \times \bar{E} \right)}_{2}}=\frac{\partial {{E}_{1}}}{\partial {{x}_{3}}}=-{{{\dot{B}}}_{2}} \\
& \Leftrightarrow i\frac{\omega }{c}\left( n+i\gamma  \right){{E}_{1}}=i\omega {{B}_{2}} \\
& \Leftrightarrow {{B}_{2}}=\frac{\left( n+i\gamma  \right)}{c}{{E}_{1}}=\frac{\sqrt{{{n}^{2}}+{{\gamma }^{2}}}}{c}{{e}^{i\phi }}{{E}_{1}} \\
\end{align}</math>
 
Somit existiert eine Phasenverschiebung
<math>\phi </math>
zwischen E und B
 
<u>'''Der Isolator'''</u>
 
<math>\begin{align}
& \sigma =0 \\
& \tau \to \infty  \\
\end{align}</math>
 
Folgen:
 
<math>\gamma =0</math>
keine Dämpfung
 
<math>\phi </math>
=0  keine Phasenverschiebung zwischen E und B
* kommt erst durch die Dämpfung !
* i m Isolator schwingen E und B in Phase !
 
reeller Brechungsindex:
 
<math>n=\sqrt{\varepsilon \mu }\approx \sqrt{\varepsilon }>1</math>
 
* Phasengeschwindigkeit :
* <math>\frac{c}{n}<c</math>
*
 
Nebenbemerkung:
Nur OHNE DISPERSION  ist
<math>\varepsilon </math>
reell
 
<u>'''Metalle'''</u>
 
<math>\tau =\frac{{{\varepsilon }_{0}}\varepsilon }{\sigma }<<\frac{1}{\omega }</math>
für alle Frequenzen bis UV
Somit:
 
<math>\begin{align}
& {{k}^{2}}=\frac{{{\omega }^{2}}}{{{c}^{2}}}\left( {{n}^{2}}-{{\gamma }^{2}}+2in\gamma  \right)\approx \frac{{{\omega }^{2}}}{{{c}^{2}}}\varepsilon \mu \frac{i}{\omega \tau } \\
& \Rightarrow {{n}^{2}}-{{\gamma }^{2}}\approx 0 \\
& n\gamma \approx {{n}^{2}}\approx {{\gamma }^{2}}\approx \frac{\varepsilon \mu }{2\omega \tau }\Rightarrow n=\gamma =\sqrt{\frac{\varepsilon \mu }{2\omega \tau }} \\
& \tan \phi =\frac{\gamma }{n}\approx 1\Rightarrow \phi \approx \frac{\pi }{4} \\
\end{align}</math>
 
Extinktionskoeffizient
 
<math>d<<\frac{c}{\omega \gamma }\tilde{\ }cm</math>
für 100 Hz
( hochfrequente Wellen dringen nicht in Metall ein, Grund: Verschiebungsstrom << Leitungsstrom)
 
<u>'''Dielektrische Dispersion'''</u>
 
Annahme:
<math>\mu =1</math>
 
Betrachte nun zeitliche Dispersion, also
 
<math>\begin{align}
& \hat{\chi }\left( \omega  \right): \\
& \hat{\bar{P}}\left( \omega  \right)={{\varepsilon }_{0}}\hat{\chi }\left( \omega  \right)\hat{\bar{E}}\left( \omega  \right) \\
\end{align}</math>
 
mit:
 
<math>\hat{\chi }\left( \omega  \right)=\frac{1}{\sqrt{2\pi }}\int_{-\infty }^{\infty }{{}}dt\chi \left( t \right){{e}^{i\omega t}}</math>
 
dynamische elektrische Suszeptibilität
 
'''Fourier- Trafo:'''
 
<math>\begin{align}
& \bar{P}\left( \bar{r},t \right)=\frac{1}{\sqrt{2\pi }}\int_{-\infty }^{\infty }{{}}d\omega \hat{\bar{P}}\left( \bar{r},\omega  \right){{e}^{-i\omega t}} \\
& \hat{\bar{E}}\left( \bar{r},\omega  \right)=\frac{1}{\sqrt{2\pi }}\int_{-\infty }^{\infty }{{}}dt\bar{E}\left( \bar{r},t \right){{e}^{+i\omega t}} \\
& \Rightarrow \bar{P}\left( \bar{r},t \right)=\frac{1}{2\pi }\int_{-\infty }^{\infty }{{}}d\omega {{\varepsilon }_{0}}\hat{\chi }\left( \omega  \right)\int_{-\infty }^{\infty }{{}}dt\acute{\ }\bar{E}\left( \bar{r},t\acute{\ } \right){{e}^{+i\omega \left( t\acute{\ }-t \right)}} \\
\end{align}</math>
 
Betrachte:
 
<math>\begin{align}
& \frac{1}{2\pi }\int_{-\infty }^{\infty }{{}}d\omega {{\varepsilon }_{0}}\hat{\chi }\left( \omega  \right)\int_{-\infty }^{\infty }{{}}dt\acute{\ }{{e}^{+i\omega \left( t\acute{\ }-t \right)}}:=\frac{{{\varepsilon }_{0}}}{\sqrt{2\pi }}\chi \left( t-t\acute{\ } \right) \\
& \Rightarrow \bar{P}\left( \bar{r},t \right)=\frac{1}{2\pi }\int_{-\infty }^{\infty }{{}}d\omega {{\varepsilon }_{0}}\hat{\chi }\left( \omega  \right)\int_{-\infty }^{\infty }{{}}dt\acute{\ }\bar{E}\left( \bar{r},t\acute{\ } \right){{e}^{+i\omega \left( t\acute{\ }-t \right)}}=\frac{{{\varepsilon }_{0}}}{\sqrt{2\pi }}\int_{-\infty }^{t}{{}}dt\acute{\ }\chi \left( t-t\acute{\ } \right)\bar{E}\left( \bar{r},t\acute{\ } \right) \\
\end{align}</math>
 
Nachwirkungseffekt: Faltungsintegral -> Berücksichtigung des Nachwirkungseffekts über Faltungsintegral.
 
'''Nebenbemerkung: Kausalität verlangt:'''
 
<math>\begin{align}
& \chi \left( t-t\acute{\ } \right)=0 \\
& f\ddot{u}r \\
& t\acute{\ }>t \\
\end{align}</math>
 
Aus mikroskopischen Modellen folgt i.A. ein komplexes
<math>\hat{\chi }\left( \omega  \right)\in C</math>
 
* Komplexe dielektrische Funktion:
 
<math>\begin{align}
& \varepsilon \left( \omega  \right)=1+\hat{\chi }\left( \omega  \right)=\varepsilon \acute{\ }\left( \omega  \right)+i\varepsilon \acute{\ }\acute{\ }\left( \omega  \right) \\
& \varepsilon \acute{\ },\varepsilon \acute{\ }\acute{\ }\in R \\
\end{align}</math>
 
Aus:
 
<math>\begin{align}
& \varepsilon \left( \omega  \right)=1+\frac{1}{\sqrt{2\pi }}\int_{0}^{\infty }{{}}dt\chi \left( t \right){{e}^{i\omega t}} \\
& \Rightarrow \varepsilon *(\omega )=\varepsilon (-\omega ) \\
& \varepsilon \acute{\ }(\omega )=\varepsilon \acute{\ }(-\omega ) \\
& \varepsilon \acute{\ }\acute{\ }(\omega )=-\varepsilon \acute{\ }\acute{\ }(-\omega ) \\
\end{align}</math>
 
Monochromatische ebene Welle:
 
<math>\begin{align}
& \bar{E}(\bar{r},t)={{{\bar{E}}}_{0}}{{e}^{i\left( \bar{k}\bar{r}-\omega t \right)}} \\
& \Rightarrow {{k}^{2}}=\varepsilon \left( \omega  \right)\frac{{{\omega }^{2}}}{{{c}^{2}}}\left( 1+i\frac{1}{\omega \tau } \right) \\
\end{align}</math>
 
'''Isolator ( dispersives Dielektrikum)'''
 
<math>\begin{align}
& \bar{E}(\bar{r},t)={{{\bar{E}}}_{0}}{{e}^{i\left( \bar{k}\bar{r}-\omega t \right)}} \\
& \Rightarrow {{k}^{2}}=\varepsilon \left( \omega  \right)\frac{{{\omega }^{2}}}{{{c}^{2}}} \\
\end{align}</math>
 
<math>\begin{align}
& \tilde{n}\left( \omega  \right)=n\left( \omega  \right)+i\gamma \left( \omega  \right) \\
& \tilde{n}{{\left( \omega  \right)}^{2}}=\varepsilon \left( \omega  \right)\equiv \varepsilon \acute{\ }+i\varepsilon \acute{\ }\acute{\ } \\
& \varepsilon \acute{\ }\left( \omega  \right)={{n}^{2}}-{{\gamma }^{2}} \\
& \varepsilon \acute{\ }\acute{\ }\left( \omega  \right)=2n\gamma  \\
& \Rightarrow \left. \begin{matrix}
\gamma  \\
n  \\
\end{matrix} \right\}=\frac{1}{\sqrt{2}}{{\left( \sqrt{\varepsilon {{\acute{\ }}^{2}}+\varepsilon \acute{\ }{{\acute{\ }}^{2}}}\mp \varepsilon \acute{\ } \right)}^{\frac{1}{2}}} \\
\end{align}</math>
 
Dabei
 
<math>\left. \begin{matrix}
\gamma  \\
n  \\
\end{matrix} \right\}=\frac{1}{\sqrt{2}}{{\left( \sqrt{\varepsilon {{\acute{\ }}^{2}}+\varepsilon \acute{\ }{{\acute{\ }}^{2}}}\mp \varepsilon \acute{\ } \right)}^{\frac{1}{2}}}</math>
 
Als Absorptionskoeffizient
<math>\gamma </math>
( reeller Brechungsindex n)
 
'''Absorption'''
 
<math>\varepsilon \acute{\ }\acute{\ }=0\Rightarrow \gamma =0,n=\sqrt{\varepsilon \acute{\ }}</math>
 
Absorptionskoeffizient Null, reeller Brechungsindex: Wurzel epsilon
Also: für
<math>\varepsilon \acute{\ }>0</math>
-> ungedämpfte Welle
 
<math>\varepsilon \acute{\ }\acute{\ }>0\Rightarrow \gamma >0</math>
 
* in jedem Fall gedämpfte Welle ( Energiedissipation).
 
Der Frequenzbereich mit
 
<math>\varepsilon \acute{\ }\acute{\ }<<\varepsilon \acute{\ }</math>
heißt Transparenzgebiet der Substanz ( besonders wenig Absorption).
 
'''Dispersion'''
 
<math>\operatorname{Re}k=k\acute{\ }=\frac{\omega }{c}n(\omega )</math>
nichtlineare Dispersion ( nur in erster Näherung ist n(w) linear !)
 
* Definition der Gruppengeschwindigkeit:
 
<math>\begin{align}
& {{v}_{g}}:=\frac{d\omega }{dk\acute{\ }}=\frac{1}{\frac{dk\acute{\ }}{d\omega }}=\frac{c}{\frac{d\left( \omega n \right)}{d\omega }} \\
& {{v}_{g}}=\frac{c}{n+\omega \frac{dn}{d\omega }}\ne \frac{c}{n\left( \omega  \right)}={{v}_{ph.}} \\
\end{align}</math>
 
<u>'''Typische Frequenzabhängigkeit: ( sogenanntes Resonanzverhalten):'''</u>
 
 
<u>'''Normale Dispersion'''</u>
 
<math>\frac{dn}{d\omega }>0</math>
 
Stets im Transparenzgebiet, also wenn
<math>\varepsilon \acute{\ }\acute{\ }\tilde{\ }0</math>
 
<math>{{v}_{g}}<{{v}_{ph.}}</math>
 
'''Anormale Dispersion'''
 
<math>\frac{dn}{d\omega }<0</math>
bei Absorption !
 
<u>'''Beziehung zwischen'''</u>
<math>\varepsilon \acute{\ }\left( \omega  \right)</math>
und
<math>\varepsilon \acute{\ }\acute{\ }\left( \omega  \right)</math>
 
<u>'''Kramers- Kronig- Relation'''</u>
 
* Allgemein gültiger Zusammenhang zwischen  Dispersion
* <math>n\left( \omega  \right)</math>
* und Absorption
* <math>\gamma \left( \omega  \right)</math>
* .
* erlaubt z.B. dann die Berechnung von Dispersionsrelationen aus dem Absorptionsspektrum und auch umgekehrt
* Folgt alleine aus dem Kausalitätsprinzip !
 
<u>'''Beweis ( Funktionenthorie)'''</u>
 
Für kausale Funktion gilt:
 
<math>\begin{align}
& \chi \left( t \right)=\Theta \left( t \right)\chi \left( t \right) \\
& \Theta \left( t \right)=\left\{ \begin{matrix}
\begin{align}
& 0t<0 \\
& 1t\ge 0 \\
\end{align}  \\
{}  \\
\end{matrix} \right. \\
\end{align}</math>
Heavyside
 
'''Fourier- Trafo:'''
 
<math>\hat{\chi }\left( \omega  \right)=\frac{1}{\sqrt{2\pi }}\int_{{}}^{{}}{{}}d\omega \acute{\ }\Theta \left( \omega -\omega \acute{\ } \right)\hat{\chi }\left( \omega \acute{\ } \right)</math>
 
<math>\begin{align}
& \hat{\Theta }\left( \omega  \right):=\begin{matrix}
\lim  \\
\sigma ->0+  \\
\end{matrix}\frac{1}{\sqrt{2\pi }}\int_{0}^{\infty }{dt{{e}^{i\omega t-\sigma t}}}=\begin{matrix}
\lim  \\
\sigma ->0+  \\
\end{matrix}\frac{1}{\sqrt{2\pi }}\frac{1}{i\omega -\sigma } \\
&  \\
\end{align}</math>
 
Mit dem konvergenzerzeugenden Faktor
<math>\sigma </math>
:
Also:
 
<math>\hat{\chi }\left( \omega  \right)=\frac{1}{2\pi i}\begin{matrix}
\lim  \\
\sigma ->0+  \\
\end{matrix}\int_{-\infty }^{\infty }{{}}d\omega \acute{\ }\frac{1}{\omega \acute{\ }-\omega -i\sigma }\hat{\chi }\left( \omega \acute{\ } \right)</math>
 
'''Der Integrand hat einen Pol für'''
 
<math>\omega \acute{\ }=\omega +i\sigma </math>
 
Also:
 
'''Äquivalenter Integrationsweg:'''
 
'''Zerlegung:'''
 
<math>\int_{-\infty }^{\infty }{{}}d\omega \acute{\ }\frac{1}{\omega \acute{\ }-\omega }\hat{\chi }\left( \omega \acute{\ } \right)=\begin{matrix}
\lim  \\
\varepsilon ->{{0}^{+}}  \\
\end{matrix}\left[ \int_{-\infty }^{\omega -\varepsilon }{+\int_{\omega +\varepsilon }^{\infty }{{}}} \right]d\omega \acute{\ }\frac{1}{\omega \acute{\ }-\omega }\hat{\chi }\left( \omega \acute{\ } \right)+\int\limits_{Kreisbogen}{{}}d\omega \acute{\ }\frac{1}{\omega \acute{\ }-\omega }\hat{\chi }\left( \omega \acute{\ } \right)</math>
 
Man sagt:
 
<math>\begin{matrix}
\lim  \\
\varepsilon ->{{0}^{+}}  \\
\end{matrix}\left[ \int_{-\infty }^{\omega -\varepsilon }{+\int_{\omega +\varepsilon }^{\infty }{{}}} \right]d\omega \acute{\ }\frac{1}{\omega \acute{\ }-\omega }\hat{\chi }\left( \omega \acute{\ } \right)=P\int_{-\infty }^{\infty }{{}}d\omega \acute{\ }\frac{1}{\omega \acute{\ }-\omega }\hat{\chi }\left( \omega \acute{\ } \right)</math>
 
= Hauptwertintegral ( principal Value), entsteht nur direkt an der Polstelle !
 
<math>\int\limits_{Kreisbogen}{{}}d\omega \acute{\ }\frac{1}{\omega \acute{\ }-\omega }\hat{\chi }\left( \omega \acute{\ } \right)</math>
 
Integral längs des Halbkreis mit Radius
<math>\varepsilon </math>
um den Pol !
 
<math>\begin{align}
& \int\limits_{Kreisbogen}{{}}ds\frac{f(s)}{s}=f(0)\int\limits_{Kreisbogen}{{}}\frac{ds}{s} \\
& s=\varepsilon {{e}^{i\phi }}\Rightarrow ds=isd\phi  \\
& f(0)\int\limits_{Kreisbogen}{{}}\frac{ds}{s}=f(0)i\int\limits_{0}^{\pi }{{}}d\phi =i\pi f(0) \\
\end{align}</math>
 
sogenanntes " Halbes Residuum!"
 
Also:
 
<math>\begin{align}
& \hat{\chi }\left( \omega  \right)=\frac{1}{2\pi i}\begin{matrix}
\lim  \\
\sigma ->0+  \\
\end{matrix}\int_{-\infty }^{\infty }{{}}d\omega \acute{\ }\frac{1}{\omega \acute{\ }-\omega -i\sigma }\hat{\chi }\left( \omega \acute{\ } \right) \\
& =\frac{1}{2\pi i}P\int_{-\infty }^{\infty }{{}}d\omega \acute{\ }\frac{1}{\omega \acute{\ }-\omega }\hat{\chi }\left( \omega \acute{\ } \right)+\frac{1}{2}\hat{\chi }\left( \omega  \right) \\
& \Rightarrow \hat{\chi }\left( \omega  \right)=\frac{1}{\pi i}P\int_{-\infty }^{\infty }{{}}d\omega \acute{\ }\frac{1}{\omega \acute{\ }-\omega }\hat{\chi }\left( \omega \acute{\ } \right) \\
\end{align}</math>
 
Nun: Zerlegung in Re und Im mit
 
<math>\begin{align}
& \operatorname{Re}\hat{\chi }\left( \omega  \right)=\varepsilon \acute{\ }\left( \omega  \right)-1 \\
& \operatorname{Im}\hat{\chi }\left( \omega  \right)=\varepsilon \acute{\ }\acute{\ }\left( \omega  \right) \\
\end{align}</math>
 
Also:
 
<math>\begin{align}
& \operatorname{Re}\hat{\chi }\left( \omega  \right)=\varepsilon \acute{\ }\left( \omega  \right)-1=\frac{1}{\pi }P\int_{-\infty }^{\infty }{{}}d\omega \acute{\ }\frac{1}{\omega \acute{\ }-\omega }\varepsilon \acute{\ }\acute{\ }\left( \omega \acute{\ } \right) \\
& \operatorname{Im}\hat{\chi }\left( \omega  \right)=\varepsilon \acute{\ }\acute{\ }\left( \omega  \right)=-\frac{1}{\pi }P\int_{-\infty }^{\infty }{{}}d\omega \acute{\ }\frac{1}{\omega \acute{\ }-\omega }\left( \varepsilon \acute{\ }\left( \omega \acute{\ } \right)-1 \right) \\
\end{align}</math>
 
Dies ist die Kramers- Kronig- Relation. Sie verknüpft  Real- und Imaginärteil des komplexen Brechungsindex miteinander !
 
Titchmask- Theorem:
 
<math>\hat{\chi }\left( z \right)</math>
sollte regulär sein auf der oberen komplexen z- Halbebene
Somit:
 
<math>\hat{\chi }\left( z \right)\to 0</math>
für
<math>\operatorname{Im}z\to \infty </math>
 
<u>'''Brechung und Reflexion'''</u>
 
Wir haben bereits gesehen, wie man aus den Stetigkeitsbedingungen mit Hilfe der integralen Maxwellgleichungen die Brechungsrelationen für die Feldvektoren herleiten kann. Nun soll dies für Lichtwellen wiederholt / vertieft werden:
 
 
Sogenannte Wellenausbreitung in geschichteten Medien
Transparent ->
<math>{{\varepsilon }_{i}}\in R</math>
 
<math>\begin{align}
& \frac{\omega }{{{c}_{1}}}=\left| {\bar{k}} \right|=\left| \bar{k}\acute{\ } \right|=\frac{\omega \acute{\ }}{{{c}_{1}}} \\
& \left| \bar{k}\acute{\ }\acute{\ } \right|=\frac{\omega \acute{\ }\acute{\ }}{{{c}_{2}}} \\
& {{c}_{i}}=\frac{c}{{{n}_{i}}}=\frac{c}{\sqrt{{{\varepsilon }_{i}}}}\quad i=1,2 \\
& \bar{E}(\bar{r},t)={{{\bar{E}}}_{0}}{{e}^{i\left( \bar{k}\bar{r}-\omega t \right)}} \\
\end{align}</math>
 
Einfallende Welle:
 
<math>\bar{E}(\bar{r},t)={{\bar{E}}_{0}}{{e}^{i\left( \bar{k}\bar{r}-\omega t \right)}}</math>
 
Reflektierte Welle:
 
<math>\bar{E}\acute{\ }(\bar{r},t)={{\bar{E}}_{0}}\acute{\ }{{e}^{i\left( \bar{k}\acute{\ }\bar{r}-\omega \acute{\ }t \right)}}</math>
 
Transmittierte Welle:
 
<math>\bar{E}\acute{\ }\acute{\ }(\bar{r},t)={{\bar{E}}_{0}}\acute{\ }\acute{\ }{{e}^{i\left( \bar{k}\acute{\ }\acute{\ }\bar{r}-\omega \acute{\ }\acute{\ }t \right)}}</math>
 
<u>'''Grenzbedingungen für'''</u>
<math>\bar{E}(\bar{r},t)</math>
. Annahme: linear polarisiert:
 
<math>{{\left. {{E}_{1}}+{{E}_{1}}\acute{\ } \right|}_{{{x}_{3}}=0}}={{\left. {{E}_{1}}\acute{\ }\acute{\ } \right|}_{{{x}_{3}}=0}}</math>
-> Stetigkeit der Tangenzialkomponenten
Diese Bedingungen werden nur an die Amplituden gestellt. Für die Phasen gibt es keine Bedingungen, besser gesagt:
 
Betrachte Situation für r=0
 
<math>\begin{align}
& {{{\bar{E}}}_{01}}{{e}^{i\omega t}}+{{{\bar{E}}}_{01}}\acute{\ }{{e}^{i\omega \acute{\ }t}}={{{\bar{E}}}_{01}}\acute{\ }\acute{\ }{{e}^{i\omega \acute{\ }\acute{\ }t}} \\
& \Rightarrow \omega =\omega \acute{\ }=\omega \acute{\ }\acute{\ } \\
& {{{\bar{E}}}_{01}}+{{{\bar{E}}}_{01}}\acute{\ }={{{\bar{E}}}_{01}}\acute{\ }\acute{\ } \\
\end{align}</math>
 
Das Snelliussche Brechungsgesetz können wir uns nicht als Amplitudenverhältnis anschauen, weil wir sonst wieder nur die Brechung der elektrischen Feldvektoren gewinnen.
Aber: Wenn man ein Verhältnis der Beträge der k- Vektoren ( Ausbreitungsrichtung des Energiestroms) betrachtet, so ergibt sich das richtige Ausbreitungsgesetz:
 
Betrachte für t=0
 
<math>{{E}_{01}}{{e}^{i{{k}_{1}}{{x}_{1}}}}+{{E}_{01}}\acute{\ }{{e}^{ik{{\acute{\ }}_{1}}{{x}_{1}}}}={{E}_{01}}\acute{\ }\acute{\ }{{e}^{i{{k}_{1}}\acute{\ }\acute{\ }{{x}_{1}}}}</math>
 
Also:
 
<math>{{k}_{1}}={{k}_{1}}\acute{\ }={{k}_{1}}\acute{\ }\acute{\ }</math>
 
Aber: ( Siehe Skizze) ! Dies gilt ja genau für die Anteile entlang x^1, also:
muss man den Winkel dazunehmen und man gewinnt:
 
<math>\begin{align}
& \left| {\bar{k}} \right|\sin \gamma =\left| \bar{k}\acute{\ } \right|\sin \gamma \acute{\ }=\left| \bar{k}\acute{\ }\acute{\ } \right|\sin \gamma \acute{\ }\acute{\ } \\
& \left| {\bar{k}} \right|=\frac{\omega }{{{c}_{1}}} \\
& \left| \bar{k}\acute{\ } \right|=\frac{\omega }{{{c}_{1}}} \\
& \left| \bar{k}\acute{\ }\acute{\ } \right|=\frac{\omega }{{{c}_{2}}} \\
\end{align}</math>
 
Somit gewinnen wir Reflexions und Snelliussches Brechungsgesetz:
 
<math>\begin{align}
& \sin \gamma =\sin \gamma \acute{\ } \\
& \frac{\sin \gamma \acute{\ }\acute{\ }}{\sin \gamma }=\frac{{{c}_{2}}}{{{c}_{1}}}=\frac{{{n}_{1}}}{{{n}_{2}}} \\
\end{align}</math>
 
Reflexions- und Brechungsgesetz
 
<u>'''Bestimmung der Amplituden:'''</u>
 
# <u>'''Polarisation von E in der Einfallsebene'''</u>
Stetigkeitsbedingungen: Normalkomponenten sind keine vorhanden -> Nur Tangentialkomponenten:
 
<math>\begin{align}
& {{E}_{01}}={{E}_{01}}\acute{\ }={{E}_{01}}\acute{\ }\acute{\ }=0 \\
& {{E}_{03}}={{E}_{03}}\acute{\ }={{E}_{03}}\acute{\ }\acute{\ }=0 \\
\end{align}</math>
 
Für die Tangentialkomp.:
 
<math>{{E}_{02}}+{{E}_{02}}\acute{\ }={{E}_{02}}\acute{\ }\acute{\ }</math>
 
Mit
 
<math>{{\bar{B}}_{0}}=\frac{c}{\omega }\bar{k}\times {{\bar{E}}_{0}}=\frac{c}{\omega }{{E}_{02}}\left( \begin{matrix}
-{{k}_{3}}  \\
0  \\
{{k}_{1}}  \\
\end{matrix} \right)</math>
 
Somit folgt dann für die Tangentialkomponente von B:
 
<math>{{B}_{01}}+{{B}_{01}}\acute{\ }={{B}_{01}}\acute{\ }\acute{\ }\Rightarrow {{k}_{3}}{{E}_{02}}+{{k}_{3}}\acute{\ }E{{\acute{\ }}_{02}}={{k}_{3}}\acute{\ }\acute{\ }{{E}_{02}}\acute{\ }\acute{\ }</math>
 
mit dem Reflexionsgesetz.
 
<math>{{k}_{3}}=-{{k}_{3}}\acute{\ }</math>
 
<math>\begin{align}
& \Rightarrow {{k}_{3}}\left( {{E}_{02}}-E{{\acute{\ }}_{02}} \right)={{k}_{3}}\acute{\ }\acute{\ }\left( {{E}_{02}}+{{E}_{02}}\acute{\ } \right) \\
& \Rightarrow \frac{E{{\acute{\ }}_{02}}}{{{E}_{02}}}=\frac{{{k}_{3}}-{{k}_{3}}\acute{\ }\acute{\ }}{{{k}_{3}}+{{k}_{3}}\acute{\ }\acute{\ }} \\
& \frac{E\acute{\ }{{\acute{\ }}_{02}}}{{{E}_{02}}}=\frac{2{{k}_{3}}}{{{k}_{3}}+{{k}_{3}}\acute{\ }\acute{\ }} \\
\end{align}</math>
 
Man muss nun  nur
<math>{{k}_{3}}\acute{\ }\acute{\ }</math>
über den Brechungswinkel
<math>\gamma \acute{\ }\acute{\ }</math>
ausdrücken und man gewinnt die Fresnelschen Formeln:
 
<math>\begin{align}
& {{k}_{3}}\acute{\ }\acute{\ }=\left| \bar{k}\acute{\ }\acute{\ } \right|\cos \gamma \acute{\ }\acute{\ }=\left| \bar{k}\acute{\ } \right|\frac{{{n}_{2}}}{{{n}_{1}}}\cos \gamma \acute{\ }\acute{\ } \\
& \frac{{{n}_{2}}}{{{n}_{1}}}=\frac{\sin \gamma }{\sin \gamma \acute{\ }\acute{\ }} \\
& \Rightarrow {{k}_{3}}\acute{\ }\acute{\ }=\left| \bar{k}\acute{\ }\acute{\ } \right|\cos \gamma \acute{\ }\acute{\ }=\left| \bar{k}\acute{\ } \right|\frac{\sin \gamma }{\sin \gamma \acute{\ }\acute{\ }}\cos \gamma \acute{\ }\acute{\ } \\
& {{k}_{3}}=\left| {\bar{k}} \right|\cos \gamma  \\
\end{align}</math>
 
Also können wir dies in die gefundenen Formeln für die Amplitudenverhältnisse einsetzen und erhalten die Brechungsformeln ( Fresnelsche Formeln) nur noch in Abhängigkeit von den Winkeln:
 
Also:
 
<math>\begin{align}
& \frac{E{{\acute{\ }}_{02}}}{{{E}_{02}}}=\frac{\cos \gamma \sin \gamma \acute{\ }\acute{\ }-\sin \gamma \cos \gamma \acute{\ }\acute{\ }}{\cos \gamma \sin \gamma \acute{\ }\acute{\ }+\sin \gamma \cos \gamma \acute{\ }\acute{\ }}=\frac{\sin \left( \gamma \acute{\ }\acute{\ }-\gamma  \right)}{\sin \left( \gamma \acute{\ }\acute{\ }+\gamma  \right)} \\
& \frac{E\acute{\ }{{\acute{\ }}_{02}}}{{{E}_{02}}}=\frac{2{{k}_{3}}}{{{k}_{3}}+{{k}_{3}}\acute{\ }\acute{\ }}=\frac{2\sin \left( \gamma \acute{\ }\acute{\ } \right)\cos \gamma }{\sin \left( \gamma \acute{\ }\acute{\ }+\gamma  \right)} \\
\end{align}</math>
 
<u>'''Intensitätsverhältnisse:'''</u>
 
<u>'''betrachte: Zeitmittel des Poynting- Vektors:'''</u>
 
<math>\left\langle {\bar{S}} \right\rangle =\frac{1}{T}\int_{0}^{T}{{}}dt\left( \bar{E}\times \bar{H} \right)</math>
 
'''Reflexionskoeffizient: ( bei senkrechter Polarisation)'''
 
<math>\begin{align}
& {{R}_{\bot }}={{\left| \frac{E{{\acute{\ }}_{02}}}{{{E}_{02}}} \right|}^{2}}=\frac{{{\sin }^{2}}\left( \gamma \acute{\ }\acute{\ }-\gamma  \right)}{{{\sin }^{2}}\left( \gamma \acute{\ }\acute{\ }+\gamma  \right)} \\
&  \\
\end{align}</math>
 
Transmissionskoeffizient ( bei senkrechter Polarisation)
 
<math>{{T}_{\bot }}={{\left| \frac{E\acute{\ }{{\acute{\ }}_{02}}}{{{E}_{02}}} \right|}^{2}}=\frac{4{{\sin }^{2}}\left( \gamma \acute{\ }\acute{\ } \right){{\cos }^{2}}\gamma }{{{\sin }^{2}}\left( \gamma \acute{\ }\acute{\ }+\gamma  \right)}=1-{{R}_{\bot }}</math>
 
# <u>'''Polarisation von'''</u>
# <math>\bar{E}||</math>
# Einfallsebene:
<u>'''Dadurch:'''</u>
<math>\bar{B}\bot </math>
Einfallsebene
 
* Analoge Argumentation:
 
<math>\begin{align}
& {{B}_{01}}={{B}_{01}}\acute{\ }={{B}_{01}}\acute{\ }\acute{\ }=0 \\
& {{B}_{03}}={{B}_{03}}\acute{\ }={{B}_{03}}\acute{\ }\acute{\ }=0 \\
& {{B}_{02}}+{{B}_{02}}\acute{\ }={{B}_{02}}\acute{\ }\acute{\ } \\
\end{align}</math>
 
usw... ebenfalls Bildung der Verhältnisse  in Abhängigkeit von k -> wie beim Vorgehen in a) weiter rechnen.
k durch Zwischenwinkel ausdrücken:
Zur Übung berechnen, es ergibt sich:
 
<math>\begin{align}
& \frac{E{{\acute{\ }}_{||}}}{{{E}_{||}}}=\frac{\tan \left( \gamma \acute{\ }\acute{\ }-\gamma  \right)}{\tan \left( \gamma \acute{\ }\acute{\ }+\gamma  \right)} \\
& \frac{E\acute{\ }{{\acute{\ }}_{||}}}{{{E}_{||}}}=\frac{2\sin \left( \gamma \acute{\ }\acute{\ } \right)\cos \gamma }{\sin \left( \gamma \acute{\ }\acute{\ }+\gamma  \right)\cos \left( \gamma \acute{\ }\acute{\ }-\gamma  \right)} \\
\end{align}</math>
 
Ebenso:
 
<math>\begin{align}
& {{R}_{||}}={{\left| \frac{E{{\acute{\ }}_{||}}}{{{E}_{||}}} \right|}^{2}}=\frac{{{\tan }^{2}}\left( \gamma \acute{\ }\acute{\ }-\gamma  \right)}{{{\tan }^{2}}\left( \gamma \acute{\ }\acute{\ }+\gamma  \right)}=1-{{T}_{||}} \\
&  \\
\end{align}</math>
 
'''Bemerkung'''
Bei Reflexion und Brechung wird im Allgemeinen die Polarisationsrichtung gedreht. Speziell für den Fall
 
<math>\begin{align}
& \gamma \acute{\ }\acute{\ }+\gamma =\frac{\pi }{2} \\
& ->\tan \left( \gamma \acute{\ }\acute{\ }+\gamma  \right)\to \infty  \\
& {{R}_{||}}=0 \\
\end{align}</math>
 
In diesem Fall kommt es nicht zu Teilpolarisation sondern: die reflektierte Welle wird vollständig polarisiert ( senkrecht zur Einfallsebene)
* Dies ist der Brewsterwinkel:
*
* <math>\begin{align}
*  & \gamma \acute{\ }\acute{\ }+\gamma =\frac{\pi }{2}->\gamma =\left( {{\gamma }_{Brew}} \right) \\
*  & \tan {{\gamma }_{B}}=\sqrt{\frac{{{\varepsilon }_{2}}}{{{\varepsilon }_{1}}}} \\
* \end{align}</math>
*
 
'''Totalreflexion'''
'''Sei'''
 
<math>\begin{align}
& {{\varepsilon }_{2}}<{{\varepsilon }_{1}} \\
& \sin {{\gamma }_{G}}=\sqrt{\frac{{{\varepsilon }_{2}}}{{{\varepsilon }_{1}}}} \\
\end{align}</math>
 
Totalreflexion unter diesem Winkel oder flacher !
 
Grenzwinkel der Totalreflexion ->
<math>\gamma \acute{\ }\acute{\ }=\frac{\pi }{2}</math>
 
<math>\begin{align}
& {{R}_{\bot }}={{R}_{||}}=1 \\
& {{T}_{\bot }}={{T}_{||}}=0 \\
\end{align}</math>
 
<math>\begin{align}
& {{\varepsilon }_{2}}<{{\varepsilon }_{1}} \\
& \gamma >{{\gamma }_{G}}\Rightarrow  \\
\end{align}</math>
 
<math>k\acute{\ }\acute{\ }</math>
wird imaginär -> es dringt kein reeller Strahl mehr ins Medium ein !
 
<u>'''6. Relativistische Formulierung der Elektrodynamik'''</u>
 
<u>'''6.1 Ko- und Kontravariante Schreibweise der Relativitätstheorie'''</u>
 
Grundpostulat der speziellen Relativitätstheorie:
 
Kein Inertialsystem ist gegenüber einem anderen ausgezeichnet ! ( Einstein, 1904).
Die Lichtgeschwindigkeit c ist in jedem Inertialsystem gleich !
* Kugelwellen sind
* -> Lorentz- Invariant, also:
*
* <math>{{r}^{2}}-{{c}^{2}}{{t}^{2}}=r{{\acute{\ }}^{2}}-{{c}^{2}}t{{\acute{\ }}^{2}}</math>
*
 
Für Lorentz- Transformationen !
 
<u>'''Formalisierung:'''</u>
<u>'''Der Raumzeitliche Abstand als'''</u>
 
<math>{{\left( ds \right)}^{2}}:={{\left( cdt \right)}^{2}}-{{\left( d\bar{r} \right)}^{2}}</math>
 
Zwischen 2 Ereignissen bleibt der raumzeitliche Abstand invariant bei Lorentz- Transformationen ! zwischen den Inertialsystemen :
<math>\Sigma \leftrightarrow \Sigma \acute{\ }</math>
 
Ziel: Um dies sofort zu sehen führt man Vierervektoren ein.
Dann schreibt man
<math>{{\left( ds \right)}^{2}}</math>
als Skalarprodukt von Vierervektoren im Minkowski- Raum V und man benutze den Formalismus der '''linearen orthogonalen '''Transformation , unter denen das Skalarprodukt invariant bleibt:
 
In der ko / kontravarianten Schreibweise tritt jeder Vierervektor in 2 möglichen Versionen auf:
 
<u>'''kontravariante Komponenten:'''</u>
 
<math>\begin{align}
& {{x}^{i}} \\
& {{x}^{1}}:=ct \\
& {{x}^{1}},{{x}^{2}},{{x}^{3}} \\
\end{align}</math>
 
als Komponenten des Ortsvektors
<math>\bar{r}</math>
:
 
<u>'''kovariante Komponenten'''</u>
 
<math>\begin{align}
& {{x}_{i}}: \\
& {{x}_{0}}:=ct \\
& {{x}_{\alpha }}=-{{x}^{\alpha }},\alpha =1,2,3 \\
\end{align}</math>
 
kovarianter Vektor
<math>\in \tilde{V}</math>
, dualer Vektorraum zu V !
Merke: Die Räume der kovarianten Vektoren ist dual zur menge der kontravarianten
->
<math>\in \tilde{V}</math>
als Raum der linearen Funktionale l:
<math>V\to R</math>
 
Damit werden dann die Skalarprodukte gebildet !
 
Schreibe
 
<math>{{\left( ds \right)}^{2}}=d{{x}^{0}}d{{x}_{0}}+d{{x}^{1}}d{{x}_{1}}+d{{x}^{2}}d{{x}_{2}}+d{{x}^{3}}d{{x}_{3}}=d{{x}^{i}}d{{x}_{i}}</math>
 
Mit: Summenkonvention !
über je einen ko- und einen kontravarianten Index ( hier i =0,1,2,3) wird summiert !
 
<u>'''Physikalische Anwendung'''</u>
 
Lorentz- Invarianten lassen sich als Skalarprodukt
<math>{{a}^{i}}{{a}_{i}}</math>
schreiben !
 
'''Beispiel: dÁlemebert- Operator:'''
 
<math>\#=\Delta -\frac{1}{{{c}^{2}}}\frac{{{\partial }^{2}}}{\partial {{t}^{2}}}=-\frac{\partial }{\partial {{x}^{i}}}\frac{\partial }{\partial {{x}_{i}}}=-{{\partial }_{i}}{{\partial }^{i}}</math>
 
<u>'''Vierergeschwindigkeit'''</u>
 
<math>\begin{align}
& {{u}^{i}}:=\frac{d{{x}^{i}}}{ds}\Rightarrow {{u}^{i}}{{u}_{i}}=\frac{d{{x}^{i}}d{{x}_{i}}}{{{\left( ds \right)}^{2}}}=1 \\
& mit \\
& ds={{\left( d{{x}^{i}}d{{x}_{i}} \right)}^{\frac{1}{2}}}=c{{\left( 1-{{\beta }^{2}} \right)}^{\frac{1}{2}dt}}=\frac{c}{\gamma }dt \\
& \Rightarrow {{u}^{0}}=\gamma  \\
& {{u}^{\alpha }}=\frac{\gamma }{c}{{v}^{\alpha }} \\
& {{v}^{\alpha }}:=\frac{d{{x}^{\alpha }}}{dt} \\
& \beta :=\frac{v}{c} \\
& \gamma :=\frac{1}{\sqrt{1-{{\beta }^{2}}}} \\
\end{align}</math>
 
'''Physikalische Interpretation'''
 
<math>\begin{align}
& {{u}^{\alpha }}=\frac{1}{c}\frac{d{{x}^{\alpha }}}{d\tau } \\
& d\tau =\frac{dt}{\gamma } \\
\end{align}</math>
 
'''Viererimpuls'''
 
<math>{{p}^{i}}:={{m}_{0}}c{{u}^{i}}</math>
mit der Ruhemasse  m0
 
Also:
 
<math>\begin{align}
& {{p}^{i}}{{p}_{i}}={{m}_{0}}^{2}{{c}^{2}}{{u}^{i}}{{u}_{i}} \\
& {{u}^{i}}{{u}_{i}}=1 \\
& \Rightarrow {{p}^{i}}{{p}_{i}}={{m}_{0}}^{2}{{c}^{2}} \\
& {{p}^{0}}={{m}_{0}}\gamma c=m(v)c=\frac{E}{c} \\
& {{p}^{\alpha }}={{m}_{0}}\gamma {{v}^{\alpha }}=m(v){{v}^{\alpha }} \\
& {{p}^{i}}{{p}_{i}}={{m}_{0}}^{2}{{c}^{2}}{{u}^{i}}{{u}_{i}}\Leftrightarrow {{E}^{2}}={{m}_{0}}^{2}{{c}^{4}}+{{c}^{2}}{{{\bar{p}}}^{2}} \\
\end{align}</math>
 
Mit der Energie
 
<math>E=m(v){{c}^{2}}</math>
 
'''Analoge Definition von Tensoren 2. Stufe:'''
 
<math>\begin{align}
& {{A}^{ik}},{{A}^{i}}_{k},{{A}_{i}}^{k},{{A}_{ik}} \\
& {{A}^{00}}={{A}^{0}}_{0}={{A}_{0}}^{0}={{A}_{00}} \\
& {{A}^{10}}={{A}^{1}}_{0}=-{{A}_{1}}^{0}=-{{A}_{10}} \\
& {{A}^{11}}=-{{A}^{1}}_{1}=-{{A}_{1}}^{1}={{A}_{11}} \\
\end{align}</math>
 
<u>'''Der metrische Tensor'''</u>
 
<math>{{g}^{ik}}:={{\delta }^{ik}}=\left. \left\{ \begin{matrix}
{{\delta }^{i}}_{k}\quad k=0  \\
-{{\delta }^{i}}_{k}\quad k=1,2,3  \\
\end{matrix} \right. \right\}={{g}_{ik}}</math>
 
<math>{{g}^{ik}}={{g}_{ik}}=\left( \begin{matrix}
1 & 0 & 0 & 0  \\
0 & -1 & 0 & 0  \\
0 & 0 & -1 & 0  \\
0 & 0 & 0 & -1  \\
\end{matrix} \right)</math>
 
Mittels der Metrik werden Indices gehoben bzw. gesenkt:
 
<math>{{g}^{ik}}{{a}_{k}}={{a}^{i}}</math>
 
Wichtig fürs Skalarprodukt:
 
<math>d{{s}^{2}}={{g}^{ik}}d{{x}_{i}}d{{x}_{k}}={{g}_{ik}}d{{x}^{i}}d{{x}^{k}}</math>
 
<u>Lorentz- Trafo</u>
 
zwischen Bezugssystemen: Lineare / homogene Trafo
 
die Lorentz- Transformation für
 
<math>\begin{align}
& \left( {{x}^{0}}\begin{matrix}
, & {{x}^{1}}, & {{x}^{2}}, & {{x}^{3}}  \\
\end{matrix} \right)=\left( \begin{matrix}
ct, & x, & y, & z  \\
\end{matrix} \right) \\
& d{{s}^{2}}={{c}^{2}}d{{t}^{2}}-d{{x}^{2}}-d{{y}^{2}}-d{{z}^{2}} \\
\end{align}</math>
 
Nämlich:
 
<math>\begin{align}
& \left( \begin{matrix}
{{x}_{0}}\acute{\ }  \\
{{x}_{1}}\acute{\ }  \\
{{x}_{2}}\acute{\ }  \\
{{x}_{3}}\acute{\ }  \\
\end{matrix} \right)=\left( \begin{matrix}
\frac{1}{\sqrt{1-{{\beta }^{2}}}} & \frac{-\beta }{\sqrt{1-{{\beta }^{2}}}} & 0 & 0  \\
\frac{-\beta }{\sqrt{1-{{\beta }^{2}}}} & \frac{1}{\sqrt{1-{{\beta }^{2}}}} & 0 & 0  \\
0 & 0 & 1 & 0  \\
0 & 0 & 0 & 1  \\
\end{matrix} \right)\left( \begin{matrix}
{{x}_{0}}  \\
{{x}_{1}}  \\
{{x}_{2}}  \\
{{x}_{3}}  \\
\end{matrix} \right) \\
& x{{\acute{\ }}^{i}}={{U}^{i}}_{k}{{x}^{k}} \\
\end{align}</math>
 
Mit
<math>{{U}^{i}}_{k}=\left( \begin{matrix}
\frac{1}{\sqrt{1-{{\beta }^{2}}}} & \frac{-\beta }{\sqrt{1-{{\beta }^{2}}}} & 0 & 0  \\
\frac{-\beta }{\sqrt{1-{{\beta }^{2}}}} & \frac{1}{\sqrt{1-{{\beta }^{2}}}} & 0 & 0  \\
0 & 0 & 1 & 0  \\
0 & 0 & 0 & 1  \\
\end{matrix} \right)</math>
 
für
<math>v||{{x}_{1}}</math>
 
Wesentliche Eigenschaft ( die Viererschreibweise ist so konstruiert worden):
 
U ist orthogonale Trafo:
 
<math>\begin{align}
& {{U}^{i}}_{k}{{U}_{i}}^{l}=\delta _{k}^{l} \\
& \Rightarrow a{{\acute{\ }}^{i}}b{{\acute{\ }}_{i}}={{U}^{i}}_{k}{{U}_{i}}^{l}{{a}^{k}}{{b}_{l}}={{a}^{k}}{{b}_{k}} \\
\end{align}</math>
 
Das Skalarprodukt ist invariant, falls U eine orthogonale Trafo ist
Bzw.
Forderung: Skalarprodukt invariant -> U muss orthogonale Trafo sein !
 
Umkehr- Transformation:
 
<math>{{x}^{i}}={{U}_{k}}^{i}x{{\acute{\ }}^{k}}</math>
 
<u>'''6.2 Transformationsverhalten der Ströme und Felder'''</u>
 
<u>'''Ziel: Ko- / Kontravariante Schreibweise der Elektrodynamik im Vakuum'''</u>
 
Grund: Die klassische Elektrodynamik ist bereits eine Lorentz- invariante Theorie !!
 
Historisch gab die  Maxwellsche Elektrodynamik und nicht die Mechanik den Anstoß zur Relativitätstheorie überhaupt !
 
'''Ladungserhaltung aus Kontinuitätsgleichung:'''
 
<math>\begin{align}
& div\bar{j}+\frac{\partial \rho }{\partial t}=\frac{\partial {{j}_{x}}}{\partial x}+\frac{\partial {{j}_{y}}}{\partial y}+\frac{\partial {{j}_{z}}}{\partial z}+\frac{\partial c\rho }{\partial ct}=0 \\
& 0=\frac{\partial \rho }{\partial t}+\sum\limits_{\alpha =1}^{3}{{}}{{\partial }_{\alpha }}{{j}^{\alpha }} \\
\end{align}</math>
 
Somit gewinnen wir aber ebenfalls wieder einen Lorentz- Skalar, nämlich
 
<math>{{\partial }_{\mu }}{{j}^{\mu }}=0</math>
 
in Viererschreibweise.
Die Vierer- Stromdichte ist
 
<math>\left\{ {{j}^{\mu }} \right\}=\left\{ c\rho ,\bar{j} \right\}</math>
ebenfalls ein kontravarianter Vierer- Vektor . Er heißt Vierer- Stromdichte.
Die Kontinuitätsgleichung ist gleich
<math>{{\partial }_{\mu }}{{j}^{\mu }}=0</math>
 
'''Forderung:'''
Ladungserhaltung soll in allen Inertialsystemen gelten !
->
 
<math>{{j}^{\mu }}=0</math>
muss sich wie ein Vierervektor transformieren, damit  das Skalarprodukt
<math>{{\partial }_{\mu }}{{j}^{\mu }}=0</math>
Lorentz- invariant ist !:
 
<math>\begin{align}
& {{x}^{0}}\acute{\ }=\gamma \left( {{x}^{0}}-\beta {{x}^{1}} \right)\Leftrightarrow t\acute{\ }=\gamma \left( t-\frac{v}{{{c}^{2}}}{{x}^{1}} \right) \\
& {{x}^{1}}\acute{\ }=\gamma \left( {{x}^{1}}-\beta {{x}^{0}} \right)\Leftrightarrow {{x}^{1}}\acute{\ }=\gamma \left( {{x}^{1}}-vt \right) \\
& {{x}^{2}}\acute{\ }={{x}^{2}} \\
& {{x}^{3}}\acute{\ }={{x}^{3}} \\
\end{align}</math>
 
Also gilt für Ladungs- und Stromdichten:
 
<math>\begin{align}
& {{j}^{0}}\acute{\ }=\gamma \left( {{j}^{0}}-\beta {{j}^{1}} \right)\Leftrightarrow \rho \acute{\ }=\gamma \left( \rho -\frac{v}{{{c}^{2}}}{{j}^{1}} \right) \\
& {{j}^{1}}\acute{\ }=\gamma \left( {{j}^{1}}-\beta {{j}^{0}} \right)\Leftrightarrow {{j}^{1}}\acute{\ }=\gamma \left( {{j}^{1}}-v\rho  \right) \\
& {{j}^{2}}\acute{\ }={{j}^{2}} \\
& {{j}^{3}}\acute{\ }={{j}^{3}} \\
\end{align}</math>
 
Merke: Es sollte kein Missverständnis geschehen: Ist ein Vektor in ein Lorentz- invariantes Skalarprodukt verwickelt, so ist es ein Vierervektor. Damit ist klar: Seine Komponenten transfornmieren nach der Lorentz- Trafo.
Dadurch aber ist die Trafo für seine Komponenten, die Beispielsweise Ladungs- und Stromdichten sind, gefunden.
 
<u>'''4- Potenziale:'''</u>
 
<u>Die </u>Potenziale
<math>\Phi ,\bar{A}</math>
sind in der Lorentz- Eichung
<math>\nabla \cdot \bar{A}+\frac{1}{{{c}^{2}}}\frac{\partial }{\partial t}\phi =0</math>
Lösungen von
 
<math>\begin{align}
& \Delta \bar{A}\left( \bar{r},t \right)-\frac{1}{{{c}^{2}}}\frac{{{\partial }^{2}}}{\partial {{t}^{2}}}\bar{A}\left( \bar{r},t \right)=-{{\mu }_{0}}\bar{j} \\
& \#\bar{A}\left( \bar{r},t \right)=-{{\mu }_{0}}\bar{j} \\
& \#=-{{\partial }_{\mu }}{{\partial }^{\mu }} \\
& {{\mu }_{0}}c=\frac{1}{{{\varepsilon }_{0}}c} \\
& \#\bar{A}\left( \bar{r},t \right)=-{{\mu }_{0}}\bar{j}\Leftrightarrow {{\partial }_{\mu }}{{\partial }^{\mu }}c{{A}^{\alpha }}=\frac{1}{{{\varepsilon }_{0}}c}{{j}^{\alpha }} \\
& \alpha =1,2,3 \\
\end{align}</math>
 
<math>\begin{align}
& \Delta \phi \left( \bar{r},t \right)-\frac{1}{{{c}^{2}}}\frac{{{\partial }^{2}}}{\partial {{t}^{2}}}\phi \left( \bar{r},t \right)=-\frac{\rho }{{{\varepsilon }_{0}}}=-{{\mu }_{0}}{{c}^{2}}\rho  \\
& \#\phi \left( \bar{r},t \right)=-\frac{\rho }{{{\varepsilon }_{0}}}\Leftrightarrow {{\partial }_{\mu }}{{\partial }^{\mu }}\phi =\frac{1}{{{\varepsilon }_{0}}c}{{j}^{0}} \\
\end{align}</math>
 
Zusammen:
 
<math>\begin{align}
& -\#{{\Phi }^{\mu }}={{\partial }_{\alpha }}{{\partial }^{\alpha }}{{\Phi }^{\mu }}={{\mu }_{0}}{{j}^{\mu }} \\
& {{\Phi }^{0}}:=\phi  \\
& {{\Phi }^{i}}:=c{{A}^{i}}\quad i=1..3 \\
\end{align}</math>
 
Da
<math>{{j}^{\mu }}</math>
Vierervektoren sind ( wie Vierervektoren transformieren), muss auch
<math>{{\Phi }^{\mu }}</math>
wie ein Vierervektor transformieren.
Denn: Der d´Alembert- Operator ist Lorentz- invariant:
 
<math>{{\partial }_{\alpha }}{{\partial }^{\alpha }}</math>
lorentz- invariant !:
 
<math>\begin{align}
& {{\Phi }^{0}}\acute{\ }=\gamma \left( {{\Phi }^{0}}-\beta {{\Phi }^{1}} \right)\quad bzw.\quad \Phi \acute{\ }=\gamma \left( \Phi -v{{A}^{1}} \right) \\
& {{\Phi }^{1}}\acute{\ }=\gamma \left( {{\Phi }^{1}}-\beta {{\Phi }^{0}} \right)\quad bzw.\quad A{{\acute{\ }}^{1}}=\gamma \left( {{A}^{1}}-\frac{v}{{{c}^{2}}}\Phi  \right),{{A}^{\acute{\ }2}}={{A}^{2}},A{{\acute{\ }}^{3}}={{A}^{3}} \\
\end{align}</math>
 
Nun: Lorentz- Eichung:
 
<math>\nabla \cdot \bar{A}+\frac{1}{{{c}^{2}}}\frac{\partial }{\partial t}\phi =0</math>
 
Lorentz- Eichung <->  Lorentz- Invarianz
<math>{{\partial }_{\mu }}{{\Phi }^{\mu }}=0</math>
( Gegensatz zur Coulomb- Eichung)
 
<math>{{\partial }_{\mu }}{{\Phi }^{\mu }}=0\Leftrightarrow \nabla \cdot \bar{A}+\frac{1}{{{c}^{2}}}\frac{\partial }{\partial t}\phi =0</math>
 
<u>'''Umeichung:'''</u>
 
<math>\begin{align}
& \tilde{\bar{A}}=\bar{A}+\nabla F \\
& \tilde{\phi }=\phi -\frac{\partial }{\partial t}F \\
& \Leftrightarrow  \\
& c{{{\tilde{A}}}^{\alpha }}=c{{A}^{\alpha }}+{{\partial }_{\alpha }}cF=c{{A}^{\alpha }}-{{\partial }^{\alpha }}cF \\
& {{{\tilde{\Phi }}}^{0}}={{\Phi }^{0}}-{{\partial }_{0}}cF={{\Phi }^{0}}-{{\partial }^{0}}cF \\
\end{align}</math>
 
'''Also:'''
 
<math>{{\tilde{\Phi }}^{\mu }}={{\Phi }^{\mu }}-{{\partial }^{\mu }}cF</math>
 
'''Felder E und B:'''
 
<math>\begin{align}
& \bar{E}=-grad\phi -\frac{\partial }{\partial t}\bar{A} \\
& \Rightarrow {{E}^{\alpha }}=-{{\partial }_{\alpha }}\phi -\frac{1}{c}\frac{\partial }{\partial t}c{{A}^{\alpha }}=-{{\partial }_{\alpha }}{{\Phi }^{0}}-{{\partial }_{0}}{{\Phi }^{\alpha }}={{\partial }^{\alpha }}{{\Phi }^{0}}-{{\partial }^{0}}{{\Phi }^{\alpha }} \\
\end{align}</math>
 
<math>\begin{align}
& \bar{B}=\nabla \times \bar{A} \\
& \Rightarrow c{{B}^{1}}={{\partial }_{2}}c{{A}^{3}}-{{\partial }_{3}}c{{A}^{2}}={{\partial }_{2}}{{\Phi }^{3}}-{{\partial }_{3}}{{\Phi }^{2}}={{\partial }^{3}}{{\Phi }^{2}}-{{\partial }^{2}}{{\Phi }^{3}} \\
\end{align}</math>
 
Die anderen Komponenten gewinnt man durch zyklische Vertauschung:
 
<math>\begin{align}
& c{{B}^{2}}={{\partial }^{1}}{{\Phi }^{3}}-{{\partial }^{3}}{{\Phi }^{1}} \\
& c{{B}^{3}}={{\partial }^{2}}{{\Phi }^{1}}-{{\partial }^{1}}{{\Phi }^{2}} \\
\end{align}</math>
 
Diese Gleichungen werden zusammengefasst durch den antisymmetrtischen Feldstärketensor:
 
<math>\begin{align}
& \left\{ {{F}_{\mu \nu }} \right\}=\left\{ {{\partial }_{\mu }}{{\Phi }_{\nu }}-{{\partial }_{\nu }}{{\Phi }_{\mu }} \right\}=\left( \begin{matrix}
0 & \frac{1}{c}{{E}_{x}} & \frac{1}{c}{{E}_{y}} & \frac{1}{c}{{E}_{z}}  \\
-\frac{1}{c}{{E}_{x}} & 0 & -{{B}_{z}} & {{B}_{y}}  \\
-\frac{1}{c}{{E}_{y}} & {{B}_{z}} & 0 & -{{B}_{x}}  \\
-\frac{1}{c}{{E}_{z}} & -{{B}_{y}} & {{B}_{x}} & 0  \\
\end{matrix} \right) \\
& {{F}^{\mu \nu }}=\left\{ {{\partial }^{\mu }}{{\Phi }^{\nu }}-{{\partial }^{\nu }}{{\Phi }^{\mu }} \right\}=\left( \begin{matrix}
0 & -\frac{1}{c}{{E}_{x}} & -\frac{1}{c}{{E}_{y}} & -\frac{1}{c}{{E}_{z}}  \\
\frac{1}{c}{{E}_{x}} & 0 & -{{B}_{z}} & {{B}_{y}}  \\
\frac{1}{c}{{E}_{y}} & {{B}_{z}} & 0 & -{{B}_{x}}  \\
\frac{1}{c}{{E}_{z}} & -{{B}_{y}} & {{B}_{x}} & 0  \\
\end{matrix} \right) \\
& \Leftrightarrow {{F}^{\mu \nu }}=\left\{ {{\partial }^{\mu }}{{\Phi }^{\nu }}-{{\partial }^{\nu }}{{\Phi }^{\mu }} \right\}=\left( \begin{matrix}
0 & -{{E}^{1}} & -{{E}^{2}} & -{{E}^{3}}  \\
{{E}^{1}} & 0 & -c{{B}^{3}} & c{{B}^{2}}  \\
{{E}^{2}} & c{{B}^{3}} & 0 & -c{{B}^{1}}  \\
{{E}^{3}} & -c{{B}^{2}} & c{{B}^{1}} & 0  \\
\end{matrix} \right) \\
\end{align}</math>
 
Wegen der Antisymmetrie hat
<math>{{F}^{\mu \nu }}</math>
nur 6 unabhängige Komponenten !
 
Das bedeutet, die Raum- Raum- Komponenten entsprechen
 
<math>rot\bar{A}=\bar{B}</math>
 
während die Raum- zeit- Komponenten:
 
<math>\bar{E}=-grad\phi -\frac{\partial }{\partial t}\bar{A}</math>
erfüllen.
 
<u>'''Lorentz- Trafo der Felder:'''</u>
 
Der Feldstärketensor ist kovariant und transformiert demnach über die inverse Lorentz- Transformation.
Das heißt: Für die Transformation in ein in x- Richtung mit konstanter Geschwindigkeit
<math>\bar{v}</math>
bewegtes System K´ gilt:
 
<math>{{F}_{{}}}{{\acute{\ }}^{\mu \nu }}={{U}^{\mu }}_{\lambda }{{U}^{\nu }}_{\kappa }{{F}^{\lambda \kappa }}</math>
 
<math>{{U}^{i}}_{k}=\left( \begin{matrix}
\frac{1}{\sqrt{1-{{\beta }^{2}}}} & \frac{-\beta }{\sqrt{1-{{\beta }^{2}}}} & 0 & 0  \\
\frac{-\beta }{\sqrt{1-{{\beta }^{2}}}} & \frac{1}{\sqrt{1-{{\beta }^{2}}}} & 0 & 0  \\
0 & 0 & 1 & 0  \\
0 & 0 & 0 & 1  \\
\end{matrix} \right)</math>
 
Damit läßt sich nun das uns unbekannte Transformationsverhalten der Felder
<math>\bar{E}</math>
und
<math>rot\bar{A}=\bar{B}</math>
berechnen, die auch kovariant transformieren müssen. Dabei sollte keinesfalls die Summation über die Indices auf der rechten Seite vergessen werden !!
 
<math>\begin{align}
& E{{\acute{\ }}^{1}}=F{{\acute{\ }}^{10}}={{U}^{1}}_{\lambda }{{U}^{0}}_{\kappa }{{F}^{\lambda \kappa }}=-\beta \gamma {{U}^{0}}_{\kappa }{{F}^{0\kappa }}+\gamma {{U}^{0}}_{\kappa }{{F}^{1\kappa }}={{\left( \beta \gamma  \right)}^{2}}{{F}^{01}}+{{\gamma }^{2}}{{F}^{10}}= \\
& ={{\gamma }^{2}}\left( 1-{{\beta }^{2}} \right){{F}^{10}}={{E}^{1}} \\
& {{\gamma }^{2}}\left( 1-{{\beta }^{2}} \right)=1 \\
&  \\
& E{{\acute{\ }}^{2}}=F{{\acute{\ }}^{20}}={{U}^{2}}_{\lambda }{{U}^{0}}_{\kappa }{{F}^{\lambda \kappa }}={{U}^{0}}_{\kappa }{{F}^{2\kappa }}=\gamma {{F}^{20}}-\beta \gamma {{F}^{21}}=\gamma \left( {{E}^{2}}-v{{B}^{3}} \right) \\
\end{align}</math>
 
<math>E{{\acute{\ }}^{3}}=F{{\acute{\ }}^{30}}={{U}^{0}}_{\kappa }{{F}^{3\kappa }}=\gamma {{F}^{30}}-\beta \gamma {{F}^{31}}=\gamma \left( {{E}^{3}}+v{{B}^{2}} \right)</math>
 
<math>\begin{align}
& B{{\acute{\ }}^{1}}=\frac{1}{c}F{{\acute{\ }}^{32}}=\frac{1}{c}{{U}^{3}}_{\lambda }{{U}^{2}}_{\kappa }{{F}^{\lambda \kappa }}=\frac{1}{c}{{F}^{32}}={{B}^{1}} \\
& B{{\acute{\ }}^{2}}=\frac{1}{c}F{{\acute{\ }}^{13}}=\frac{1}{c}{{U}^{1}}_{\lambda }{{U}^{3}}_{\kappa }{{F}^{\lambda \kappa }}=\frac{1}{c}{{U}^{1}}_{\kappa }{{F}^{\kappa 3}}=-\frac{\beta \gamma }{c}{{F}^{03}}+\frac{\gamma }{c}{{F}^{13}}=\gamma \left( {{B}^{2}}+\frac{v}{{{c}^{2}}}{{E}^{3}} \right) \\
\end{align}</math>
 
<math>B{{\acute{\ }}^{3}}=\gamma \left( {{B}^{3}}-\frac{v}{{{c}^{2}}}{{E}^{2}} \right)</math>
 
'''Zusammenfassung'''
 
<math>\begin{align}
& {{E}^{1}}\acute{\ }={{E}^{1}} \\
& {{E}^{2}}\acute{\ }=\frac{1}{\sqrt{1-{{\beta }^{2}}}}\left( {{E}^{2}}-v{{B}^{3}} \right) \\
& {{E}^{3}}\acute{\ }=\frac{1}{\sqrt{1-{{\beta }^{2}}}}\left( {{E}^{3}}+v{{B}^{2}} \right) \\
& {{B}^{1}}\acute{\ }={{B}^{1}} \\
& {{B}^{2}}\acute{\ }=\frac{1}{\sqrt{1-{{\beta }^{2}}}}\left( {{B}^{2}}+\frac{v}{{{c}^{2}}}{{E}^{3}} \right) \\
& {{B}^{3}}\acute{\ }=\frac{1}{\sqrt{1-{{\beta }^{2}}}}\left( {{B}^{3}}-\frac{v}{{{c}^{2}}}{{E}^{2}} \right) \\
\end{align}</math>
 
Elektrische und magnetische Felder werden beim Übergang zwischen verschiedenen Inertialsystemen ineinander transformiert !
 
<u>'''Umeichung:'''</u>
 
<math>{{\tilde{\Phi }}^{\mu }}={{\Phi }^{\mu }}+{{\partial }^{\mu }}\phi </math>
 
Somit:
 
<math>\begin{align}
& {{{\tilde{F}}}^{\mu \nu }}={{\partial }^{\mu }}{{{\tilde{\Phi }}}^{\nu }}-{{\partial }^{\nu }}{{{\tilde{\Phi }}}^{\mu }}={{\partial }^{\mu }}\left( {{\Phi }^{\nu }}+{{\partial }^{\nu }}\phi  \right)-{{\partial }^{\nu }}\left( {{\Phi }^{\mu }}+{{\partial }^{\mu }}\phi  \right) \\
& ={{\partial }^{\mu }}{{\Phi }^{\nu }}-{{\partial }^{\nu }}{{\Phi }^{\mu }}+{{\partial }^{\mu }}{{\partial }^{\nu }}\phi -{{\partial }^{\nu }}{{\partial }^{\mu }}\phi ={{F}^{\mu \nu }} \\
\end{align}</math>
 
<u>'''Homogene Maxwell- Gleichungen'''</u>
 
<math>\begin{align}
& \nabla \cdot \bar{B}={{\partial }_{1}}{{B}^{1}}+{{\partial }_{2}}{{B}^{2}}+{{\partial }_{3}}{{B}^{3}}=0 \\
& \Rightarrow {{\partial }_{1}}{{F}^{32}}+{{\partial }_{2}}{{F}^{13}}+{{\partial }_{3}}{{F}^{21}}=0 \\
&  \\
\end{align}</math>
 
Mit
 
<math>\begin{align}
& {{\partial }_{1}}=-{{\partial }^{1}} \\
& {{F}^{32}}=-{{F}^{23}} \\
& \Rightarrow {{\partial }^{1}}{{F}^{23}}+{{\partial }^{2}}{{F}^{31}}+{{\partial }^{3}}{{F}^{12}}=0 \\
&  \\
\end{align}</math>
 
+ zyklisch in (123)
 
'''innere Feldgleichung für E- Feld'''
 
<math>\nabla \times \bar{E}=-\frac{\partial }{\partial t}\bar{B}</math>
 
# Komponente
 
<math>{{\partial }_{2}}{{E}^{3}}-{{\partial }_{3}}{{E}^{2}}+\frac{\partial }{\partial t}{{B}^{1}}=0</math>
 
<math>\Rightarrow {{\partial }^{0}}{{F}^{23}}+{{\partial }^{2}}{{F}^{30}}+{{\partial }^{3}}{{F}^{02}}=0</math>
und zyklisch (023)
 
zyklische Permutation 1 -> 2 -> 3 -> 1 und mit
 
<math>{{F}^{ik}}=-{{F}^{ki}}</math>
 
liefert:
 
<math>\begin{align}
& \Rightarrow {{\partial }^{0}}{{F}^{13}}+{{\partial }^{3}}{{F}^{01}}+{{\partial }^{1}}{{F}^{30}}=0\quad zyklisch(013) \\
& \Rightarrow {{\partial }^{0}}{{F}^{12}}+{{\partial }^{1}}{{F}^{20}}+{{\partial }^{2}}{{F}^{01}}=0\quad zyklisch(012) \\
\end{align}</math>
 
'''Zusammenfassung der homogenen Maxwellgleichungen'''
 
<math>{{\varepsilon }^{\kappa \lambda \mu \nu }}{{\partial }_{\lambda }}{{F}_{\mu \nu }}=0</math>
 
<math>{{\varepsilon }_{\kappa \lambda \mu \nu }}{{\partial }^{\lambda }}{{F}^{\mu \nu }}=0</math>
 
Die "4- Rotation" des Feldstärketensors verschwindet !
 
'''Levi- Civita- Tensor:'''
'''+1 für gerade Permutation von 0123'''
'''-1 für ungerade Permutation von 0123'''
'''0, sonst'''
 
'''Bemerkungen'''
 
# Levi- Civita ist vollständig antisymmetrisch ( per Definition).
 
#
# <math>{{\varepsilon }^{\kappa \lambda \mu \nu }}</math>
#  transformiert unter Lorentz- Trafo
 
<math>\begin{align}
& {{\varepsilon }^{\kappa \lambda \mu \nu }}\acute{\ }={{U}^{\kappa }}_{\alpha }{{U}^{\lambda }}_{\beta }{{U}^{\mu }}_{\gamma }{{U}^{\nu }}_{\delta }{{\varepsilon }^{\alpha \beta \gamma \delta }} \\
& =\left| \begin{matrix}
{{U}^{\kappa }}_{0} & {{U}^{\kappa }}_{1} & {{U}^{\kappa }}_{2} & {{U}^{\kappa }}_{3}  \\
{{U}^{\lambda }}_{0} & {{U}^{\lambda }}_{1} & {{U}^{\lambda }}_{2} & {{U}^{\lambda }}_{3}  \\
{{U}^{\mu }}_{0} & {{U}^{\mu }}_{1} & {{U}^{\mu }}_{2} & {{U}^{\mu }}_{3}  \\
{{U}^{\nu }}_{0} & {{U}^{\nu }}_{1} & {{U}^{\nu }}_{2} & {{U}^{\nu }}_{3}  \\
\end{matrix} \right|=\left( \det U \right)\cdot {{\varepsilon }^{\kappa \lambda \mu \nu }} \\
& \left( \det U \right)=\pm 1 \\
\end{align}</math>
 
Damit nun der Levi- Civita- Tensor invariant unter Lorentz- Trafos wird, also
 
<math>{{\varepsilon }^{\kappa \lambda \mu \nu }}\acute{\ }={{\varepsilon }^{\kappa \lambda \mu \nu }}</math>
, muss vereinbart werden, dass die Transformation lautet
 
<math>{{\varepsilon }^{\kappa \lambda \mu \nu }}\acute{\ }=\left( \det U \right){{U}^{\kappa }}_{\alpha }{{U}^{\lambda }}_{\beta }{{U}^{\mu }}_{\gamma }{{U}^{\nu }}_{\delta }{{\varepsilon }^{\alpha \beta \gamma \delta }}</math>
 
Damit ist der Tensor aber ein Pseudotensor !
 
Insgesamt ist die vierdimensionale Schreibweise die gleiche Formalisierung wie im Dreidimensionalen:
 
<math>{{\left( \nabla \times \bar{A} \right)}_{\alpha }}={{\varepsilon }^{\alpha \beta \gamma }}{{\partial }_{\beta }}{{A}_{\gamma }}</math>
 
Mit Pseudovektor
 
<math>{{\left( \nabla \times \bar{A} \right)}_{\alpha }}</math>
 
<u>'''Inhomogene Maxwellgleichungen im Vakuum ( Erregungsgleichungen)'''</u>
 
<math>\begin{align}
& {{\varepsilon }_{0}}\nabla \cdot \bar{E}=\rho  \\
& \Leftrightarrow {{\partial }_{1}}{{E}^{1}}+{{\partial }_{2}}{{E}^{2}}+{{\partial }_{3}}{{E}^{3}}=\frac{1}{{{\varepsilon }_{0}}c}c\rho  \\
& \Leftrightarrow {{\partial }_{1}}{{F}^{10}}+{{\partial }_{2}}{{F}^{20}}+{{\partial }_{3}}{{F}^{30}}=\frac{1}{{{\varepsilon }_{0}}c}{{j}^{0}} \\
& \Leftrightarrow {{\partial }_{\nu }}{{F}^{\nu 0}}=\frac{1}{{{\varepsilon }_{0}}c}{{j}^{0}} \\
& wegen{{\partial }_{0}}{{F}^{00}}=0 \\
& auch{{\partial }_{i}}{{F}^{i0}}=\frac{1}{{{\varepsilon }_{0}}c}{{j}^{0}} \\
\end{align}</math>
 
#
# <math>\nabla \times \bar{B}-\frac{1}{{{c}^{2}}}\frac{\partial }{\partial t}\bar{E}={{\mu }_{0}}\left( \nabla \times \bar{H}-{{\varepsilon }_{0}}\frac{\partial }{\partial t}\bar{E} \right)={{\mu }_{0}}\bar{j}</math>
#
 
# Komponente
 
<math>\begin{align}
& {{\partial }_{2}}{{B}^{3}}-{{\partial }_{3}}{{B}^{2}}={{\mu }_{0}}{{j}^{1}}+{{\varepsilon }_{0}}{{\mu }_{0}}\frac{\partial }{\partial t}{{E}^{1}} \\
& {{\mu }_{0}}c=\frac{1}{{{\varepsilon }_{0}}c} \\
& \Leftrightarrow {{\partial }_{2}}{{F}^{21}}-.{{\partial }_{3}}{{F}^{13}}=\frac{1}{{{\varepsilon }_{0}}c}{{j}^{1}}+.{{\partial }_{0}}{{F}^{10}} \\
& {{\partial }_{2}}{{F}^{21}}+{{\partial }_{3}}{{F}^{31}}+{{\partial }_{0}}{{F}^{01}}=\frac{1}{{{\varepsilon }_{0}}c}{{j}^{1}} \\
& \Leftrightarrow {{\partial }_{\nu }}{{F}^{\nu 1}}=\frac{1}{{{\varepsilon }_{0}}c}{{j}^{1}} \\
& wegen{{\partial }_{1}}{{F}^{11}}=0 \\
\end{align}</math>
 
Dies kann analog für die zweite und dritte Komponente durchgeixt werden. Aus der Nullten Komponente hatten wir die Nullte des Stroms ( Erregungsgleichung des elektrischen Feldes), so dass insgesamt folgt:
 
<math>\begin{align}
& {{\partial }_{\nu }}{{F}^{\mu \nu }}=-\frac{1}{{{\varepsilon }_{0}}c}{{j}^{\mu }} \\
& {{\partial }_{\nu }}{{F}^{\nu \mu }}=\frac{1}{{{\varepsilon }_{0}}c}{{j}^{\mu }} \\
\end{align}</math>
 
Die Viererdivergenz des elektrischen Feldstärketensors !
 
'''Bemerkungen'''
 
# die homogenen Maxwellgleichungen sind durch den Potenzialansatz
 
<math>\left\{ {{F}_{\mu \nu }} \right\}=\left\{ {{\partial }_{\mu }}{{\Phi }_{\nu }}-{{\partial }_{\nu }}{{\Phi }_{\mu }} \right\}=\left( \begin{matrix}
0 & \frac{1}{c}{{E}_{x}} & \frac{1}{c}{{E}_{y}} & \frac{1}{c}{{E}_{z}}  \\
-\frac{1}{c}{{E}_{x}} & 0 & -{{B}_{z}} & {{B}_{y}}  \\
-\frac{1}{c}{{E}_{y}} & {{B}_{z}} & 0 & -{{B}_{x}}  \\
-\frac{1}{c}{{E}_{z}} & -{{B}_{y}} & {{B}_{x}} & 0  \\
\end{matrix} \right)</math>
 
automatisch erfüllt:
 
<math>\begin{align}
& {{\varepsilon }^{\alpha \beta \mu \nu }}{{\partial }_{\beta }}{{F}_{\mu \nu }}={{\varepsilon }^{\alpha \beta \mu \nu }}{{\partial }_{\beta }}{{\partial }_{\mu }}{{\Phi }_{\nu }}-{{\varepsilon }^{\alpha \beta \mu \nu }}{{\partial }_{\beta }}{{\partial }_{\nu }}{{\Phi }_{\mu }} \\
& {{\varepsilon }^{\alpha \beta \mu \nu }}{{\partial }_{\beta }}{{\partial }_{\mu }}{{\Phi }_{\nu }}=0, \\
& da:{{\partial }_{\beta }}{{\partial }_{\mu }}{{\Phi }_{\nu }}\quad symmetrisch \\
& {{\varepsilon }^{\alpha \beta \mu \nu }}\quad antisymmetrisch \\
& {{\varepsilon }^{\alpha \beta \mu \nu }}{{\partial }_{\beta }}{{\partial }_{\nu }}{{\Phi }_{\mu }}=0 \\
\end{align}</math>
 
Aus den inhomogenen Maxwell- Gleichungen
 
<math>{{\partial }_{\beta }}{{F}^{\beta \nu }}={{\partial }_{\beta }}{{\partial }^{\beta }}{{\Phi }^{\nu }}-{{\partial }_{\beta }}{{\partial }^{\nu }}{{\Phi }^{\beta }}=\frac{1}{{{\varepsilon }_{0}}c}{{j}^{\nu }}</math>
 
folgt mit Lorentz- Eichung
 
<math>{{\partial }_{\mu }}{{\Phi }^{\mu }}=0</math>
 
<math>\begin{align}
& {{\partial }_{\beta }}{{\partial }^{\nu }}{{\Phi }^{\beta }}={{\partial }^{\nu }}{{\partial }_{\beta }}{{\Phi }^{\beta }}=0 \\
& also: \\
\end{align}</math>
 
<math>{{\partial }_{\beta }}{{F}^{\beta \nu }}={{\partial }_{\beta }}{{\partial }^{\beta }}{{\Phi }^{\nu }}=\frac{1}{{{\varepsilon }_{0}}c}{{j}^{\nu }}</math>
als inhomogene Wellengleichung
 
'''Die Maxwellgleichungen'''
 
<math>\begin{align}
& {{\varepsilon }^{\alpha \beta \mu \nu }}{{\partial }_{\beta }}{{F}_{\mu \nu }}={{\varepsilon }^{\alpha \beta \mu \nu }}{{\partial }_{\beta }}{{\partial }_{\mu }}{{\Phi }_{\nu }}-{{\varepsilon }^{\alpha \beta \mu \nu }}{{\partial }_{\beta }}{{\partial }_{\nu }}{{\Phi }_{\mu }}=0 \\
& {{\partial }_{\beta }}{{F}^{\beta \nu }}={{\partial }_{\beta }}{{\partial }^{\beta }}{{\Phi }^{\nu }}=\frac{1}{{{\varepsilon }_{0}}c}{{j}^{\nu }} \\
\end{align}</math>
 
sind ihrerseits nun Lorentz- kovariant, da sie durch 4 Pseudovektoren ausgedrückt sind.
Merke: Pseudo - 4- Vektor stört nicht, da rechte Seite gleich Null !!
 
<u>'''Gauß- System:'''</u>
 
<math>{{\partial }_{\beta }}{{F}^{\beta \nu }}=\frac{4\pi }{c}{{j}^{\nu }}</math>
 
<u>'''Relativistisches Hamiltonprinzip'''</u>
 
<u>'''Ziel: '''</u>Formulierung der Elektrodynamik als Lagrange- Feldtheorie
 
Die rel. Dynamik eines Massepunktes kann aus dem Extremalprinzip abgeleitet werden, wenn man Die Punkt 1 und 2 als Anfangs- und Endereignis im 4- Raum sieht  und wenn man die Ränder bei Variation festhält:
 
<math>\begin{align}
& \delta W=0 \\
& W=\int_{1}^{2}{{}}ds \\
\end{align}</math>
 
letzteres: Wirkungsintegral
Wichtig:
<math>{{\left. \delta {{x}^{i}} \right|}_{1,2}}=0</math>
 
Newtonsche Mechanik ist Grenzfall:
 
<math>W=-{{m}_{0}}c\int_{1}^{2}{{}}ds</math>
 
Wechselwirkung eines Massepunktes mit einem 4- Vektor- Feld
 
<math>\begin{align}
& \left( {{\phi }^{i}} \right)({{x}^{j}}) \\
& \Rightarrow  \\
\end{align}</math>
 
<math>W=\int_{1}^{2}{{}}\left\{ -{{m}_{0}}cds-{{\phi }^{i}}d{{x}_{i}} \right\}</math>
 
mit den Lorentz- Invarianten
 
<math>{{m}_{0}}cds</math>
 
und
 
<math>{{\phi }^{i}}d{{x}_{i}}</math>
 
'''Variation:'''
 
<math>\delta W=\int_{1}^{2}{{}}\left\{ -{{m}_{0}}c\delta \left( ds \right)-\delta \left( {{\phi }^{\mu }}d{{x}_{\mu }} \right) \right\}</math>
 
Nun:
 
<math>\begin{align}
& \delta \left( ds \right)=\delta {{\left( d{{x}^{\mu }}d{{x}_{\mu }} \right)}^{\frac{1}{2}}}=\frac{1}{2}\frac{\left( d\delta {{x}^{\mu }} \right)d{{x}_{\mu }}+d{{x}^{\mu }}\left( d\delta {{x}_{\mu }} \right)}{ds} \\
& \left( d\delta {{x}^{\mu }} \right)d{{x}_{\mu }}=d{{x}^{\mu }}\left( d\delta {{x}_{\mu }} \right) \\
& =\frac{d{{x}^{\mu }}}{ds}\left( d\delta {{x}_{\mu }} \right)={{u}^{\mu }}\left( d\delta {{x}_{\mu }} \right) \\
\end{align}</math>
 
Außerdem:
 
<math>\delta \left( {{\phi }^{\mu }}d{{x}_{\mu }} \right)=\delta {{\phi }^{\mu }}d{{x}_{\mu }}+{{\phi }^{\mu }}d\left( \delta {{x}_{\mu }} \right)</math>
 
Somit:
 
<math>\delta W=\int_{1}^{2}{{}}\left\{ -{{m}_{0}}c{{u}^{\mu }}\left( d\delta {{x}_{\mu }} \right)-\delta {{\phi }^{\mu }}d{{x}_{\mu }}-{{\phi }^{\mu }}d\left( \delta {{x}_{\mu }} \right) \right\}</math>
 
Weiter mit partieller Integration:
 
<math>\begin{align}
& \int_{1}^{2}{{}}-{{m}_{0}}c{{u}^{\mu }}d\left( \delta {{x}_{\mu }} \right)=\left[ -{{m}_{0}}c{{u}^{\mu }}\left( \delta {{x}_{\mu }} \right) \right]_{1}^{2}+\int_{1}^{2}{{}}{{m}_{0}}cd{{u}^{\mu }}\left( \delta {{x}_{\mu }} \right) \\
& \left[ -{{m}_{0}}c{{u}^{\mu }}\left( \delta {{x}_{\mu }} \right) \right]_{1}^{2}=0,weil\delta {{x}_{\mu }}_{1}^{2}=0 \\
& \Rightarrow \int_{1}^{2}{{}}-{{m}_{0}}c{{u}^{\mu }}d\left( \delta {{x}_{\mu }} \right)=\int_{1}^{2}{{}}{{m}_{0}}cd{{u}^{\mu }}\left( \delta {{x}_{\mu }} \right)=\int_{1}^{2}{{}}{{m}_{0}}c\frac{d{{u}^{\mu }}}{ds}\left( \delta {{x}_{\mu }} \right)ds \\
\end{align}</math>
 
Weiter:
 
<math>\int_{1}^{2}{{}}-{{\phi }^{\mu }}d\left( \delta {{x}_{\mu }} \right)=-\left[ {{\phi }^{\mu }}\delta {{x}_{\mu }} \right]_{1}^{2}+\int_{1}^{2}{{}}d{{\phi }^{\mu }}\left( \delta {{x}_{\mu }} \right)</math>
 
Mit
 
<math>\begin{align}
& d{{\phi }^{\mu }}={{\partial }^{\nu }}{{\phi }^{\mu }}d{{x}_{\nu }}={{\partial }^{\nu }}{{\phi }^{\mu }}{{u}_{\nu }}ds \\
& \delta {{\phi }^{\mu }}={{\partial }^{\nu }}{{\phi }^{\mu }}\delta {{x}_{\nu }} \\
& \delta {{\phi }^{\mu }}d{{x}_{\mu }}={{\partial }^{\nu }}{{\phi }^{\mu }}\delta {{x}_{\nu }}d{{x}_{\mu }}=i<->k={{\partial }^{\mu }}{{\phi }^{\nu }}\delta {{x}_{\mu }}d{{x}_{\nu }}={{\partial }^{\mu }}{{\phi }^{\nu }}{{u}_{\nu }}\delta {{x}_{\mu }}ds \\
\end{align}</math>
 
Einsetzen in
 
<math>\delta W=\int_{1}^{2}{{}}\left\{ -{{m}_{0}}c{{u}^{\mu }}\left( d\delta {{x}_{\mu }} \right)-\delta {{\phi }^{\mu }}d{{x}_{\mu }}-{{\phi }^{\mu }}d\left( \delta {{x}_{\mu }} \right) \right\}</math>
 
liefert:
 
<math>\delta W=\int_{1}^{2}{{}}\left\{ {{m}_{0}}c\frac{d{{u}^{\mu }}}{ds}-\left( {{\partial }^{\mu }}{{\phi }^{\nu }}-{{\partial }^{\nu }}{{\phi }^{\mu }} \right){{u}_{\nu }} \right\}\delta {{x}_{\mu }}</math>
 
'''Wegen'''
 
<math>\begin{align}
& \delta W=\int_{1}^{2}{{}}\left\{ {{m}_{0}}c\frac{d{{u}^{\mu }}}{ds}-\left( {{\partial }^{\mu }}{{\phi }^{\nu }}-{{\partial }^{\nu }}{{\phi }^{\mu }} \right){{u}_{\nu }} \right\}\delta {{x}_{\mu }}=0 \\
& {{m}_{0}}c\frac{d{{u}^{\mu }}}{ds}=\left( {{\partial }^{\mu }}{{\phi }^{\nu }}-{{\partial }^{\nu }}{{\phi }^{\mu }} \right){{u}_{\nu }}:={{f}^{\mu \nu }}{{u}_{\nu }} \\
& {{f}^{\mu \nu }}=\left( {{\partial }^{\mu }}{{\phi }^{\nu }}-{{\partial }^{\nu }}{{\phi }^{\mu }} \right) \\
\end{align}</math>
 
Dies ist dann die aus dem hamiltonschen Prinzip abgeleitete Bewegungsgleichung eines Massepunktes der Ruhemasse m0 und der Ladung q unter dem Einfluss der Lorentz- Kraft.
 
Man setze:
 
<math>\begin{align}
& {{p}^{\mu }}={{m}_{0}}c{{u}^{\mu }} \\
& {{f}^{\mu \nu }}=\frac{q}{c}{{F}^{\mu \nu }}=\left( {{\partial }^{\mu }}{{\phi }^{\nu }}-{{\partial }^{\nu }}{{\phi }^{\mu }} \right) \\
& {{\phi }^{\mu }}=\frac{q}{c}{{\Phi }^{\mu }} \\
& \frac{d}{ds}{{p}^{\mu }}=\frac{q}{c}{{F}^{\mu \nu }}{{u}_{\nu }}\Leftrightarrow \delta W=\delta \int_{1}^{2}{{}}\left\{ -{{m}_{0}}cds-\frac{q}{c}{{\Phi }^{\mu }}d{{x}_{\mu }} \right\}=0 \\
\end{align}</math>
 
Man bestimmt die Ortskomponenten
<math>\alpha =1,2,3</math>
über
 
<math>\begin{align}
& \frac{d}{dt}\bar{p}=q\left( \bar{E}+\bar{v}\times \bar{B} \right) \\
&  \\
\end{align}</math>
 
überein, denn mit
 
<math>\begin{align}
& {{u}^{0}}=\gamma  \\
& {{u}^{\alpha }}=\frac{\gamma }{c}{{v}^{\alpha }}=-{{u}_{\alpha }} \\
\end{align}</math>
 
folgt dann:
 
<math>\begin{align}
& \frac{d}{dt}{{p}^{1}}=q\left( {{E}^{1}}+{{v}^{2}}{{B}^{3}}-{{v}^{3}}{{B}^{2}} \right) \\
& =q\left( {{F}^{10}}+{{F}^{21}}\frac{1}{c}{{v}^{2}}-{{F}^{13}}\frac{1}{c}{{v}^{3}} \right) \\
& =\frac{q}{\gamma }\left( {{F}^{10}}\gamma +{{F}^{21}}\frac{\gamma }{c}{{v}^{2}}-{{F}^{13}}\frac{\gamma }{c}{{v}^{3}} \right)=\frac{q}{\gamma }{{F}^{1\mu }}{{u}_{\mu }} \\
\end{align}</math>
 
mit
 
<math>ds=\frac{c}{\gamma }dt</math>
:
 
<math>\frac{d}{ds}{{p}^{1}}=\frac{q}{c}{{F}^{1\mu }}{{u}_{\mu }}</math>
 
Die zeitartige Komponente
<math>\mu =0</math>
gibt wegen
<math>{{p}^{0}}=\frac{E}{c}</math>
:
 
<math>\begin{align}
& \frac{d}{ds}\frac{E}{c}=\frac{\gamma }{{{c}^{2}}}\frac{dE}{dt}=\frac{q}{c}\left( {{F}^{01}}{{u}_{1}}+{{F}^{02}}{{u}_{2}}+{{F}^{03}}{{u}_{3}} \right)= \\
& =\frac{q\gamma }{{{c}^{2}}}\left( -{{E}^{1}}{{v}_{1}}-{{E}^{2}}{{v}_{2}}-{{E}^{3}}{{v}_{3}} \right)=\frac{q\gamma }{{{c}^{2}}}\left( {{E}^{1}}{{v}^{1}}+{{E}^{2}}{{v}^{2}}+{{E}^{3}}{{v}^{3}} \right) \\
& \frac{dE}{dt}=q\bar{E}\cdot \bar{v} \\
\end{align}</math>
 
Dies ist die Leistungsbilanz: Die Änderung der inneren Energie ist gleich der reingesteckten Arbeit
 
<u>'''6.4 Eichinvarianz und Ladungserhaltung'''</u>
 
Wirkungsintegral:
 
<math>W=-{{m}_{0}}c\int_{1}^{2}{{}}ds-\frac{q}{c}\int_{1}^{2}{{}}d{{x}_{\mu }}{{\Phi }^{\mu }}</math>
 
Dabei:
 
<math>{{m}_{0}}c\int_{1}^{2}{{}}ds={{W}_{t}}</math>
( Teilchen)
 
<math>-\frac{q}{c}\int_{1}^{2}{{}}d{{x}_{\mu }}{{\Phi }^{\mu }}={{W}_{tf}}</math>
( Teilchen- Feld- Wechselwirkung)
 
Verallgemeinerung auf kontinuierliche Massendichte
<math>m\left( {{x}^{\mu }} \right)</math>
:
Vorsicht: m ist hier Massendichte !!!
 
<math>\begin{align}
& {{W}_{t}}=-c\int_{{}}^{{}}{{}}{{d}^{3}}rm\int_{1}^{2}{{}}ds=-\int_{\Omega }^{{}}{{}}d\Omega m\frac{ds}{dt} \\
& d\Omega :={{d}^{3}}rcdt=d{{x}^{0}}d{{x}^{1}}d{{x}^{2}}d{{x}^{3}} \\
\end{align}</math>
 
dOmega als Volumenelement  im Minkowski- Raum !!!
 
Bemerkungen:
#
# <math>d\Omega </math>
# ist eine Lorentz- Invariante , da das Volumen unter orthogonalen Transformationen
 
<math>{{U}^{\mu }}_{\nu }</math>
erhalten bleibt.
 
2) Aus
<math>d{{m}_{0}}d{{x}^{\mu }}=\frac{\mu }{c}\frac{d{{x}^{\mu }}}{dt}{{d}^{3}}rcdt;{{d}^{3}}rcdt=d\Omega \Rightarrow d{{m}_{0}}d{{x}^{\mu }}=\frac{\mu }{c}\frac{d{{x}^{\mu }}}{dt}d\Omega </math>
 
folgt, dass die Vierer- Massenstromdichte mit Massendichte m=
<math>d{{m}_{0}}d{{x}^{\mu }}=\frac{\mu }{c}\frac{d{{x}^{\mu }}}{dt}{{d}^{3}}rcdt;{{d}^{3}}rcdt=d\Omega \Rightarrow d{{m}_{0}}d{{x}^{\mu }}=\frac{\mu }{c}\frac{d{{x}^{\mu }}}{dt}d\Omega </math>
:
 
<math>{{m}_{0}}\frac{d{{x}^{\mu }}}{dt}\equiv {{g}^{\mu }}</math>
 
ein Vier- Vektor ist, da
<math>d{{m}_{0}},d\Omega </math>
Lorentz- Skalare sind und natürlich
<math>d{{x}^{\mu }}</math>
selbst auch ein Vierervektor
 
#
# <math>{{\mu }^{2}}\frac{d{{x}^{\mu }}d{{x}_{\mu }}}{{{\left( dt \right)}^{2}}}={{g}^{\mu }}{{g}_{\mu }}={{\left( \mu \frac{ds}{dt} \right)}^{2}}</math>
# ist Lorentz - Invariant.
 
Also
<math>{{g}^{\mu }}{{g}_{\mu }}</math>
ist Lorentz- Invariant. Also auch
<math>\left( \mu \frac{ds}{dt} \right)</math>
.
 
Somit ist
<math>{{W}_{t}}</math>
insgesamt Lorentz- Invariant !


----
Page footer 1
----
174


[[Kategorie:Elektrodynamik]]
[[Datei:Elektrodynamik.pdf|thumb|Zum Download der PDF-Version Bild anklicken]]

Aktuelle Version vom 21. August 2011, 16:22 Uhr


Die Abfrage enthält eine leere Bedingung.



Elektrodynamik

Klassische elektrische und magnetische Erscheinungen

  • Elektrodynamik ist relativistisch invariant
  • Feldtheorie (Nahwirkungstheorie, Kontinuumstheorie, endliche Ausbreitungsgeschwindigkeit von Wirkungen)
  • lokale Theorie:
  • quantentheoretische Erweiterung: Quantenelektrodynamik (nicht behandelt)

Vereinheitlichung der elektromagnetischen und schwachen Wechselwirkung zur elektroschwachen WW in den 70- er Jahren (Weinberg)

Starke WW: Quantenchromodynamik (nach dem Vorbild der Quantenelektrodynamik)

GUT (Grand unified): Vereinheitlichung der Elektroschwachen Theorie mit der starken Kernkraft + Gravitationswechselwirkung (nichtlinear, allgemein- relativistisch).

  • grundlegende Theorie:
    • elektrische und magnetische Felder im Vakuum, erzeugt durch lokalisierte Ladungs- und Stromverteilungen
  • elektromagnetische Felder in Materie
    • freie und gebundene Ladungen in Festkörpern/ Gasen, Materie im Allgemeinen
    • Zusammenfassung des Beitrags der mikroskopisch gebundenen Ladungen in phänomenologischen Materialkonstanten: Dielektrizitätskonstante, Permeabilität
    • phänomenologische Theorie elektromagnetische Felder in Materie

(Theorie der Materialkonstanten → Quantentheorie der Festkörper, Flüssigkeiten, Gase)


Stoff der Vorlesung

  • Elektrodynamik im Vakuum
  • Elektrodynamik in materie
  • Relativistische Formulierung


Literatur

  • H. Mitter: Elektrodynamik,. besonders gute relativistisch Formulierung
  • Stumpf, H.: Elektrodynamik Vieweg 1973
  • J. D. Jackson
  • R. Becker, Sauter: Theorie der Elektrizität
  • Landau- Lifschitz Band II und VIII


Datei:Elektrodynamik.pdf
Zum Download der PDF-Version Bild anklicken